You are on page 1of 176

LEX REVIEWS AND SEMINARS INC; That written instrument enacted by direct action of the

NATIONAL BAR REVIEW CENTER people by which the fundamental powers of the
government are established, limited and defined, and by
which those powers are distributed among the several
FOUR HUNDRED AND SEVENTY-NINE (479) departments for their safe and useful exercise for the
QUESTIONS AND ANSWERS IN POLITICAL LAW benefit of the body politic.
AND PUBLIC INTERNATIONAL LAW
(Culled from Significant Laws and Decisions of the It is the basic and paramount law to which all other laws
Supreme Court) must conform and to which all persons, including the
highest officials of the land, must defer. No act shall be
Attorney EDWIN REY SANDOVAL valid, how noble its intention, if it conflicts with the
(As of August 25, 2006) Constitution. The Constitution must ever remain supreme.
All must bow to the mandate of this law. Right or wrong,
PART I the Constitution must be upheld as long as the sovereign
people have not changed it.
A. POLITICAL LAW

That branch of public law which deals with the Classification:


organization and operations of the 1. Written or unwritten
governmental organs of the State and defines the Written Unwritten
relations of the State with the inhabitants of its one whose precepts are integrated into a single,
territory. embodied in one concrete form but are scattered
document or set of in various sources
Scope/Divisions of Political Law: documents
1) Constitutional Law—the study of the maintenance -consists of rules which
have not been
Enacted (conventional) Evolved (Cumulative)
Examples:
formally struck off at a the result of political
a. statutes of
definite time and place evolution, not inaugurated
fundamentalcharacter;
following a conscious or at any specific time but
b. judicial decisions;
deliberate effort taken by a changing by accretion
c. commentaries of publicists;
constituent body or ruler rather than by any
d. customs and traditions;
systematic method
e. certain common law
principles
of the proper balance between authority as
represented by the three inherent powers of the
2. Enacted (conventional) or Evolved (Cumulative)
state and liberty as guaranteed by the Bill of
Rights.
2) Administrative Law-- That branch of public
law which fixes the organization, determines
the competence of administrative authorities who
executes the law, and indicates to the individual
remedies for the violation of his right.
3) Law on Municipal Corporations
3. Rigid Flexible
4) Law of Public Officers
5) Elections Law
Rigid Flexible
Basis:
1) 1987 Constitution
2) 1973 and 1935 Constitution Statute
Constitutions legislation direct from the legislation from the people’s
3) Organic laws made to people representative;
apply to the
Philippines— states general principles; provides the details of the
a) Philippine Bill subject matter of
of 1902 which it treats;
b) Jones Law of intended not merely to intended primarily to meet
1916 meet existing existing conditions
c) Tydings- conditions; only;
McDuffie Law it is the fundamental law of it conforms to the
of 1934 the State Constitution
4) Statutes, executive
orders and decrees, and judicial decisions
5) US Constitution

PHILIPPINE CONSTITUTION
Constitution—it is the document, which serves as the
fundamental law of the State; that body of rules and
maxims in accordance with which the power of
sovereignty are habitually exercised.

1
one that can be amended one that can be changed by vi. it is inoperative, as if it had not been
only by a formal and ordinary legislation passed at all.
usually difficult process
b) MODERN VIEW—Courts simply refuse to
- The Philippine Constitution is written, conventional and recognize the law and determine the rights of the
rigid. It is embodied in one document and can be parties as if the statute had no existence. Certain
amended only by a formal and usually difficult process. legal effects of the statute prior to its declaration
of unconstitutionality may be recognized. Thus, a
Interpretation: public officer who implemented an
1) Verba Legis—whenever possible, the words used unconstitutional law prior to the declaration of
in the Constitution must be given their ordinary unconstitutionality cannot be held liable (Ynot vs.
meaning except where technical terms are IAC).
employed.
2) When there is Ambiguity—ratio legis et anima-- Partial Unconstitutionality
A doubtful provision shall be examined in the light Requisites:
of the history of the times and the conditions and a) The legislature must be willing to retain the valid
circumstances under which the Constitution was portion(s), usually shown by the presence of a
framed. (Civil Liberties Union vs. Executive separability clause in the law—INTENT OF THE
Secretary, 194 SCRA 317) LEGISLATIVE; and
3) Ut magis valeat quam pereat—the b) The valid portion can stand independently as
Constitution has to be interpreted as a whole. law—INDEPENDENCE OF THE PROVISIONS.
(Francisco vs. HR, G.R. No. 160261,
November 10, 2003) If the plain meaning of
the word is not found to be clear, resort to other Distinguish sovereignty from dominion.
aids is available—construe the Constitution from
what “appears upon its face”. The proper Held: Sovereignty is the right to exercise the
interpretation, therefore, depends more on how it functions of a State to the exclusion of any other State. It
was understood by the people adopting it than in is often referred to as the power of imperium, which is
the framers’ understanding thereof. defined as the government authority possessed by the
State. On the other hand, dominion, or dominium, is the
In case of doubt, the provision should be considered capacity of the State to own or acquire property such as
as self-executing; mandatory rather than directory; lands and natural resources. (Separate Opinion,
and prospective rather than retroactive. Kapunan, J., in Isagani Cruz v. Secretary of DENR,
G.R. No. 135385, Dec. 6, 2000, En Banc, See
Self-executing provision—one which is complete in Footnote 86)
itself and becomes operative without the aid of
supplementary or enabling legislation, or that which
supplies a sufficient rule by means of which the right it What is the Doctrine of Constitutional Supremacy?
grants may be enjoyed or protected.
Held: Under the doctrine of constitutional
Essential Qualities of the Written Constitution: supremacy, if a law or contract violates any norm of the
1) Broad; Constitution, that law or contract, whether promulgated
2) Brief; and by the legislative or by the executive branch or entered
3) Definite. into by private persons for private purposes, is null and
void and without any force and effect. Thus, since the
Essential parts of a good written Constitution: Constitution is the fundamental, paramount and supreme
a) Constitution of Liberty—sets forth the law of the nation, it is deemed written in every statute
fundamental civil and political rights of the citizens and contract. (Manila Prince Hotel v. GSIS, 267
and imposes limitations on the powers of the SCRA 408 [1997] [Bellosillo])
government as a means of securing the
enjoyment of those rights. e.g. Bill of Rights What are self-executing and non-self executing
b) Constitution of Government—outlines the provisions of the Constitution?
organization of the government, enumerates its
powers, lays down certain rules relative to its Held: A provision which lays down a general
administration and defines the electorate. e .g. principle, such as those found in Article II of the 1987
Legislative, Executive and Judicial Departments, Constitution, is usually not self-executing. But a provision
Constitutional Commissions which is complete in itself and becomes operative without
c) Constitution of Sovereignty—the provisions the aid of supplementary or enabling legislation, or that
pointing out the mode or procedure in accordance which supplies sufficient rule by means of which the right
with which formal changes in the fundamental law it grants may be enjoyed or protected, is self-executing.
may be brought about. e.g. Art. XVII- Thus a constitutional provision is self-executing if the
Amendments or Revisions nature and extent of the right conferred and the liability
imposed are fixed by the Constitution itself, so that they
Effects of Declaration of Unconstitutionality: can be determined by an examination and construction of
2 Views: its terms, and there is no language indicating that the
a) ORTHODOX VIEW— subject is referred to the legislature for action. (Manila
i. an unconstitutional act is not a law; Prince Hotel v. GSIS, 267 SCRA 408 [1997]
ii. it confers no rights; [Bellosillo])
iii. it imposes no duties;
iv. it affords no protection; Are provisions of the Constitution self-executing or
v. it creates no office; non-self executing? Why?

2
WE, THE SOVEREIGN FILIPINO PEOPLE,
Held: Unless it is expressly provided that a IMPLORING THE AID OF ALMIGHTY GOD, IN
legislative act is necessary to enforce a constitutional ORDER TO BUILD A JUST AND HUMANE SOCIETY
mandate, the presumption now is that all provisions are AND ESTABLISH A GOVERNMENT THAT SHALL
self-executing. If the constitutional provisions are treated EMBODY OUR IDEALS AND ASPIRATIONS,
as requiring legislation instead of self-executing, the PROMOTE THE COMMON GOOD, CONSERVE AND
legislature would have the power to ignore and practically DEVELOP OUR PATRIMONY, AND SECURE TO
nullify the mandate of the fundamental law. This can be OURSELVES AND OUR POSTERITY THE BLESSINGS
cataclysmic. (Manila Prince Hotel v. GSIS, 267 SCRA OF INDEPENDENCE AND DEMOCRACY UNDER THE
408 [1997] [Bellosillo]) RULE OF LAW AND A REGIME OF TRUTH, JUSTICE,
FREEDOM, LOVE, EQUALITY, AND PEACE, DO
What is the “Filipino First” Policy enshrined in the ORDAIN AND PROMULGATE THIS CONSTITUTION.
Constitution?
The Preamble is not a source of power or right for any
Ans.: In the grant of rights, privileges, and department of government. It sets down the origin,
concessions covering the national economy and scope, and purpose of the Constitution. It bears witness
patrimony, the State shall give preference to qualified to the fact that the Constitution is the manifestation of the
Filipinos. (Sec. 10, 2nd par., Art. XII, 1987 sovereign will of the Filipino people.
Constitution) The identification of the Filipino people as the author of
the constitution calls attention to an important principle:
Is the “Filipino First” Policy expressed in Section that the document is not just the work of representatives
10, Article XII of the Constitution a self-executing of the people but of the people themselves who put their
provision? mark approval by ratifying it in a plebiscite.
1) It does not confer rights nor impose duties.
Held: Yes. It is a mandatory, positive command 2) Indicates authorship of the Constitution;
which is complete in itself and which needs no further enumerates the primary aims and aspirations of
guidelines or implementing laws or rules for its the framers; and serves as an aid in the
enforcement. From its very words the provision does not construction of the Constitution.
require any legislation to put it in operation. It is per se
judicially enforceable. When our Constitution mandates ARTICLE I
that [i]n the grant of rights, privileges, and concessions NATIONAL TERRITORY
covering the national economy and patrimony, the State The national territory comprises the Philippine
shall give preference to qualified Filipinos, it means just archipelago, with all the islands and waters
that – qualified Filipinos must be preferred. (Manila embraced therein, and all other territories over
Prince Hotel v. GSIS, G.R. No. 118295, May 2, which the Philippines has sovereignty or
1997, 267 SCRA 408 [Bellosillo]) jurisdiction, consisting of its terrestrial, fluvial and
aerial domains, including its territorial sea, the
Give examples of non-self executing provisions of seabed, the subsoil, the insular shelves, and other
the Constitution. submarines areas. The waters around, between
and connecting the islands of the archipelago,
Held: By its very nature, Article II of the regardless of their breadth and dimensions, form
Constitution is a “declaration of principles and state part of the internal waters of the Philippines.”
policies.” These principles in Article II are not intended to
be self-executing principles ready for enforcement through Two (2) Parts of the National Territory:
the courts. They are used by the judiciary as aids or as 1) The Philippine archipelago with all the islands and
guides in the exercise of its power of judicial review, and waters embraced therein; and
by the legislature in its enactment of laws. As held in the 2) All other territories over which the Philippines has
leading case of Kilosbayan, Incorporated v. Morato (246 sovereignty or jurisdiction.
SCRA 540, 564, July 17, 1995), the principles and state
policies enumerated in Article II and some sections of Do you consider the Spratlys Group of Islands as
Article XII are not “self-executing provisions, the disregard part of Philippine
of which can give rise to a cause of action in courts. They Archipelago? _Spratlys Group of Islands is not part of
do not embody judicially enforceable constitutional rights the Philippine Archipelago because it is too far away from
but guidelines for legislation.” (Tanada v. Angara, 272 the three main islands of the Philippines. It is found,
SCRA 18 [1997], En Banc [Panganiban]) geographically, almost in the middle of the South China
Sea. It is not part of the Philippine Archipelago.
When are acts of persons considered “State action” Historically, when we talk about Philippine Archipelago, we
covered by the Constitution? refer to those islands and waters that were ceded by the
Spain to the United States by virtue of Treaty of Paris in
Held: In constitutional jurisprudence, the act of 1898. And that did not include the Spratlys Group of
persons distinct from the government are considered Islands yet. Under the treaty, the islands that were ceded
“state action” covered by the Constitution (1) when the by Spain were identified—the main islands—Luzon,
activity it engages in is a “public function”; (2) when the Visayas and Mindanao. Clearly, it did not include the
government is so significantly involved with the private Spratlys Group of Islands.
actor as to make the government responsible for his
action; and (3) when the government has approved or Spratlys Group of Islands was only discovered sometime
authorized the action. (Manila Prince Hotel v. GSIS, in the 1950’s by a Filipino, Tomas Cloma. The latter
267 SCRA 408 [1997] [Bellosillo]) waived his rights over the islands in favor of the Philippine
Government. In effect, the government stepped into the
PREAMBLE shoes of the discoverer. By then President Marcos, what
he did the moment Tomas Cloma waived his rights over

3
the Spratlys Group of Islands, is to have the islands 2. Those contemplated under Article I, 1973
immediately occupied by Philippine troops. He then issued Constitution—belonging to the
PD 1596, constituting the Spratlys Group of Islands as a Philippines by historic right or legal title;
regular municipality claiming it the Municipality of PD 1596, June 11, 1978-- constituting the Spratly’s Group
Kalayaan placing it under the Province of Palawan. And of Islands as a regular municipality claiming it the
then he had the elections immediately held in the islands Municipality of Kalayaan, placing it under the Province of
so from that time on until now, we continue to hold Palawan.
elections there. The Philippine exercises not only
jurisdiction but also sovereignty over the Spratlys Group “xxx The waters around, between and connecting
of Islands, yet it is not part of the Philippine Archipelago. the islands of the
Geographically, it is too far away from the Philippine archipelago, regardless of their breadth and
Archipelago. dimensions, form part of the internal waters of the
Philippines.”
On May 20, 1980, the Philippines registered its claim with This second sentence of Article I is not the Archipelago
the UN Secretariat. Doctrine. This is only our restatement/reaffirmation of our
The Philippine claim to the islands is justified by reason of adherence to the Archipelago Doctrine simply because we
history, indispensable need, and effective occupation and are an archipelago consisting of 7,107 islands. It is
control. Thus, in accordance with the international law, essential for our national survival that we adhere to the
the Spratlys Group of islands is subject to the sovereignty archipelago principle.
of the Philippines.
Archipelago Doctrine—merely emphasizes the unity of
Do you consider the Spratlys group of Islands as lands and waters. It is a body of waters interconnected
part of our National Territory? with other natural features. Under the United Nation
Yes. Article I of the Constitution provides: “The national Convention on the Law of Sea (UNCLOS), it consists of
territory comprises the drawing imaginary baseline connecting the outermost
Philippine archipelago, x x x, and all other territories over islands of the archipelago in which all waters, islands is
which the Philippines has sovereignty or jurisdiction, x x considered as one integrated whole. An archipelago is
x.” The Spratlys Group of islands falls under the second defined as group of islands, interconnecting waters and
phrase “and all other territories over which the Philippines other natural features which are so closely interrelated
has sovereignty or jurisdiction”. It is part of our national that such islands, waters and natural features form an
territory because Philippines exercise sovereignty (through intrinsic geographical, economical and political entity, or
election of public officials) over Spratlys Group of Islands. which historically been regarded as such.

What was the basis of the Philippines’ claim over Correlate this doctrine to right of innocent of passage,
the Spratlys? right of arrival under stress and UNCLOS requiring the
Through discovery of Tomas Cloma and occupation designation of archipelagic seaways so that foreign
vessels may pas through an archipelago.
Modes of acquiring territories:
1) Discovery and Occupation—which are terra nullius
(land belonging to no one) Components of National Territory:
Doctrine of Effective Occupation—discovery alone is i. Terrestrial—land mass on which the inhabitants live;
not enough. Mere discovery gives only an inchoate right ii. Fluvial—maritime;
to the discoverer. For title to finally vest, discovery must a. Internal or national waters—bodies of water
be followed by effective occupation in a reasonable within the land mass, among them are:
time and attestation of the same. i. Rivers—which may be:
2) Cession by Treaty. Examples are Treaty of 1. National
Paris, treaty between France and US ceding 2. Boundary—divides the territories of States
Louisiana to the latter and treaty between Russia 3. International—flows thru various States
and US ceding Alaska to the latter; a. Thalweg Doctrine—for boundary rivers, in
3) Prescription—which is a concept under the Civil the absence of an agreement between the
Code. Territory may also be acquired through riparian states, the boundary line is laid on the
continuous and uninterrupted possession over a middle of the main navigable channel.
long period of time. However, in international law, b. Middle of the Bridge Doctrine—where there
there is no rule of thumb as to the length of time is a bridge over a boundary river, the boundary
for acquisition of territory through prescription. In line is the middle or center of the bridge.
this connection, consider the Grotius Doctrine
of immemorial prescription, which speaks of ii. Bays and gulfs—a bay is a well-marked
uninterrupted possession going beyond memory. indentation whose penetration is in such proportion to
4) Conquest or Subjugation (conquistadores)— the width of its mouth as to contain a land-locked
this is no longer recognized inasmuch as the UN waters and constitutes more than a curvature of the
Charter prohibits resort to threat or use of force coast. Also referred to as juridical bay. The area
against the territorial integrity or political must be as large as, or larger than, a semi-circle
independence of any state; and whose diameter is a line drawn across the mouth of
5) Accretion—another concept in the Civil Code. It such indentation, or if the mouth is less than 24 miles
is the increase in the land area of the State, either wide.
through natural means, or artificially, through e.g. Hudson Bay in Canada, one whose waters are
human labor. considered internal because of the existence of a
historic title.
Other territories over which the Philippines has
sovereignty or jurisdiction: iii. Straits—narrow passageways connecting 2 bodies
1. Batanes—(1935 Constitution); of water. If the distance between the 2 opposite coast

4
is not more than 6 miles, they are considered internal 2. To punish infringement of the above laws and
waters. regulations committed within its territory.
In international law, when a strait within a country
has a width of more than six (6) miles, the center lane e. Exclusive Economic Zone—shall not extend beyond
in excess of the three (3) miles on both sides is 200 nautical miles from the archipelagic baselines.
considered international waters.
f. Continental shelf—it is the seabed and subsoil of the
iv. Canals—the most famous is the Suez Canal, submarine areas extending beyond the Philippine
which is neutralized, and the Panama Canal, which is territorial sea throughout the natural prolongation of the
open to everyone in times of war or peace. land territory. It extends up to:
i. The outer edge of the continental margin; or
b. Archipelagic waters—are the waters enclosed by ii. A distance of 200 nautical miles from the
the archipelagic baselines, regardless of their depth or archipelagic baselines, whichever is the farthest.
distance from the coast.
Archipelagic State—a state made up wholly of The continental shelf does not form part of the Philippine
one or two archipelagos. It may include other territory. The
islands. Philippines has the sovereign rights over the continental
shelf for the purpose of exploring it and exploiting its
Straight Archipelagic Baseline—to determine natural resources.
the archipelagic waters, the state shall draw
straight baselines connecting the outermost points g. High Seas—treated as res communes, thus, not
of the outermost islands and drying reefs, territory of any particular State. These are the waters
provided that the ratio of the area of the water to which do not constitute the internal waters, archipelagic
the area of the land, including atolls, is between waters, territorial sea and exclusive economic zones of a
1:1 and 9:1. The length of such baselines shall state. They are beyond the jurisdiction and sovereign
not exceed 100 nautical miles, except up to 3% of rights of States.
the total number of baselines enclosing any
archipelago may exceed that length, up to a Freedom of navigation—refers to the right to sail ship
maximum 125 miles. The baselines drawn should on the high sea, subject to international law and the laws
not depart, to any appreciable extent, from the of the flag of the state.
general configuration of the archipelago. All the
waters within the baselines shall then be iii. Aerial—this refers to the air space above the land
considered internal waters. The breadth of the 12- and waters of the
mile territorial sea, the contiguous zone, the State (See Discussions under International Law)
exclusive economic zone and the continental shelf
shall then be measured from the archipelagic ARTICLE II
baselines.
DECLARATION OF PRINCIPLES AND
Vessels may be allowed innocent passage STATE POLICIES
within the archipelagic waters, but this right may Sec. 1, Article II
be suspended, after publication, in the interest of The Philippines is a democratic and republican
international security. The coastal state may also State. Sovereignty resides in the people and
designate archipelagic sea lanes for continuous, all government authority emanates from
unobstructed transit of vessels. them.
(Relate this to Article XI)
c. Territorial Sea—the belt of the sea located between
the coast and the internal waters of the coastal state on Essential features: Representation and
the other hand, and the high seas on the other, Renovation.
extending up to 12 nautical miles from the low-water Manifestations:
mark, or in the case of archipelagic states, from the 1) Ours is a government of law and not of men
baselines. (Villavicencio vs. Lukban, 39 Phil 778)
2) Rule of the majority. (Plurality in elections)
Baseline—is a line from which the breadth of the 3) Accountability of public officials
territorial sea, the contiguous zone and the exclusive 4) Bill of rights
economic zone is measured in order to determine the
5) Legislature cannot pass irrepealable laws.
maritime boundary of the coastal state.
6) Separation of powers.
Types of baseline:
i. Normal Baseline Method Republicanism
ii. Straight Baseline method What is a republican form of government?
It is a government of the people, by the people, and
d. Contiguous Zone—extends up to 12 nautical miles for the people, a representative government wherein
from the territorial sea; this shall not exceed 24 nautical the powers and duties of government are exercised
miles from the archipelagic baselines. and discharged for the common good and
The coastal state may exercise limited jurisdiction welfare.thus, the supreme power resides on the
over the contiguous zone:
body of people.
1. To prevent infringement of customs, fiscal
immigration or sanitary laws and regulations
within its territory or territorial sea; and Characteristics of a republican form of
government:

5
1) The people do not govern themselves directly the actual exercise of the executive powers is
but through their representatives; vested in a Prime Minister who is chosen by,
2) It is founded upon popular suffrage; and accountable to, Parliament.
3) There is the tripartite system of the 2) In Presidential it embodies interdependence
government, the mutual interdependence of by separation and coordination while in
the three departments of the government. Parliamentary, It embodies
interdependence by integration.
STATE—a community of persons, more or less
numerous, permanently occupying a definite portion Doctrine of Parens Patriae—the government as
of territory, independent of external control, and guardian of the rights of the people may initiate legal
possessing a government to which a great body of actions for and in behalf of particular individual.
inhabitants render habitual obedience. (CIR vs. (Government of the Philippine Islands vs.
Campos Rueda, 42 SCRA 23) Monte de Piedad, 35 SCRA 738; Cabañas vs.
Pilapil, 58 SCRA 94)
State vs. Nation
State is a legal or juristic concept while a Nation is 4) Sovereignty—It is the right to exercise the
an ethnic or racial concept functions of a State to the exclusion of any
other State.
State vs. Government
State possesses a government to which a great While sovereignty has traditionally been deemed
body of inhabitants render habitual obedience while absolute and all-encompassing on the domestic
a Government is merely an instrumentality of the level, it is however subject to restrictions and
State through which the will of the State is limitations voluntarily agreed to by the Philippines,
implemented and realized. expressly or impliedly, as a member of the family of
nations. In its Declaration of Principles and State
Elements of State: Policies, the Constitution adopts the generally
1) People—the inhabitants of the State; the # accepted principles of international law as part of the
of which is capable for self-sufficiency and law of the land, and adheres to the policy of peace,
self-defense; of both sexes for perpetuity. equality, justice, freedom, cooperation and amity,
a. Inhabitants; with all nations. By the doctrine of incorporation, the
b. Citizens; country is bound by generally accepted principles of
c. Electors. international law, which are considered to be
2) Territory—a fixed portion of the surface of automatically part of our own laws.
the earth inhabited by the people of the
State. “Government of Laws and Not of Men.”—
3) Government—the agency or instrumentality sovereignty of the people also includes the concept
through which the will of the State is that government officials have only the authority
formulated, expressed and realized. given them by law and defined by law, and such
authority continues only with the consent of the
De Jure vs. De Facto people.
De Jure has a rightful title but no power or control,
either because the same has been withdrawn from it
or because it has not yet actually entered into the Kinds of Sovereignty:
exercise thereof while a De Facto Actually a) Legal—the power to issue final commands;
exercises the power or control but without legal title. b) Political—the sum total of all the influences
a) De facto proper—government that gets which lie behind the law;
possession and control of, or usurps, by force c) Internal—the supreme power over everything
or by the voice of the majority, the rightful within its territory;
legal government and maintains itself against d) External—also known as independence—
the will of the latter; freedom from external control.
b) Government of Paramount Forces—
established and maintained by the military Characteristics:
forces who invade and occupy a territory of i. Permanence
the enemy in the course of war; ii. Exclusiveness
c) Independent Government—established by iii. Comprehensiveness
the inhabitants of the country who rise in iv. Absoluteness
insurrection against the parent State. v. Indivisibility
vi. Inalienability
Presidential vs. Parliamentary vii. Imprescriptibility
1) In Presidential, there is separation of
legislative and executive powers. The first is Sovereignty, often referred to as Imperium—is the
lodged in the President and the second is State’s authority to govern; it includes passing laws
vested in Congress while in Parliamentary governing a territory, maintaining peace and order
there is fusion of both executive and over it, and defending it against foreign invasion.
legislative powers in Parliament, although
6
It is the government authority possessed by the Discuss the basis of the doctrine of State immunity
State expressed in the concept of sovereignty. from suit.

Dominium—is the capacity of the State to own or Held: The basic postulate enshrined in the
Constitution that “[t]he State may not be sued without its
acquire property such as lands and natural
consent,” reflects nothing less than a recognition of the
resources. (Lee Hong Hok vs. David, No. L- sovereign character of the State and an express
30389, December 27, 1972; Separate Opinion affirmation of the unwritten rule effectively insulating it
of Justice Kapunan in Cruz vs. Secretary of from the jurisdiction of courts. It is based on the very
DENR, G.R. No. 135385, December 2000) essence of sovereignty. As has been aptly observed by
Justice Holmes, a sovereign is exempt from suit, not
because of any formal conception or obsolete theory, but
Effect of Belligerent Occupation—there is no on the logical and practical ground that there can be no
change in sovereignty. However, political laws, legal right as against the authority that makes the law on
except those of treason, are suspended; municipal which the right depends. True, the doctrine, not too
infrequently, is derisively called “the royal prerogative of
laws remain in force unless changed by the
dishonesty” because it grants the state the prerogative to
belligerent occupant. defeat any legitimate claim against it by simply invoking
its non-suability. We have had occasion to explain in its
Principle of Jus Postliminium—at the end of the defense, however, that a continued adherence to the
occupation, when the occupant is ousted from the doctrine of non-suability cannot be deplored, for the loss
territory, the political laws which have been of governmental efficiency and the obstacle to the
suspended shall automatically become effective performance of its multifarious functions would be far
again. (Peralta vs. Director of Prisons, No. greater in severity than the inconvenience that may be
L049, November 12, 1945) caused private parties, if such fundamental principle is to
be abandoned and the availability of judicial remedy is not
to be accordingly restricted. (Department of
Effect of Change of Sovereignty—political laws of
Agriculture v. NLRC, 227 SCRA 693, Nov. 11, 1993
the former sovereign are abrogated unless they are [Vitug])
expressly reenacted by the affirmative act of the new
sovereign. Municipal laws remain in force. Is the rule absolute, i.e., that the State may not be
(Macariola vs. Asuncion, Adm. Case No. 133-J, sued at all? How may consent of the State to be
May31, 1982) sued given?

Effect of Revolutionary Government—it is Held: The rule, in any case, is not really absolute
bound by no constitution. However, it did not for it does not say that the state may not be sued under
repudiate the Covenant or Declaration in the same any circumstances. On the contrary x x x the doctrine
only conveys, “the state may not be sued without its
way it repudiated the Constitution. As the de jure
consent;” its clear import then is that the State may at
government, the revolutionary government could not times be sued. The State's consent may be given either
escape responsibility for the State’s good faith expressly or impliedly. Express consent may be made
compliance with its treaty obligations under through a general law (i.e., Commonwealth Act No. 327,
international law. During the interregnum when no as amended by Presidential Decree No. 1445 [Sections
constitution or Bill of Rights existed, directives and 49-50], which requires that all money claims against the
orders issued by government officers did not exceed government must first be filed with the Commission on
the authority granted them by the revolutionary Audit which must act upon it within sixty days. Rejection
government. The directives or orders should not of the claim will authorize the claimant to elevate the
have also violated the Covenant or the Declaration. matter to the Supreme Court on certiorari and, in effect,
sue the State thereby) or a special law. In this
(Republic vs. Sandiganbayan, G.R. No.
jurisdiction, the general law waiving the immunity of the
104768, July 21, 2003) state from suit is found in Act No. 3083, where the
Philippine government “consents and submits to be sued
Jurisdiction—is the manifestation of sovereignty. upon any money claim involving liability arising from
a) Territorial—power of the State over persons contract, express or implied, which could serve as a basis
and things within its territory subject to its of civil action between the private parties.” Implied
control and protection. consent, on the other hand, is conceded when the State
b) Personal—power of the State over its itself commences litigation, thus opening itself to a
nationals, which may be exercised by the counterclaim or when it enters into a contract. In this
state even if the individual is outside the situation, the government is deemed to have descended
to the level of the other contracting party and to have
territory of the State.
divested itself of its sovereign immunity. (Department of
c) Extraterritorial—power of the State over Agriculture v. NLRC, 227 SCRA 693, Nov. 11, 1993
persons, things or acts beyond its territorial [Vitug])
limits by reason of their effects to its
territory. The rule that when the State enters into a contract
with a private individual or entity, it is deemed to
have descended to the level of that private
individual or entity and, therefore, is deemed to
have tacitly given its consent to be sued, is that
The Doctrine of State Immunity from Suit without any qualification? What is the Restrictive
Doctrine of State Immunity from Suit?

7
Neither does it apply where the public official is
Held: This rule x x x is not x x x without clearly being sued not in his official capacity but in his
qualification. Not all contracts entered into by the personal capacity, although the acts complained of may
government operate as a waiver of its non-suability; have been committed while he occupied a public position.
distinction must still be made between one which is (Amado J. Lansang v. CA, G.R. No. 102667, Feb. 23,
executed in the exercise of its sovereign function and 2000, 2nd Div. [Quisumbing])
another which is done in its proprietary capacity.
2. As early as 1954, this Court has pronounced
In United States of America v. Ruiz (136 SCRA that an officer cannot shelter himself by the plea that he
487), where the questioned transaction dealt with the is a public agent acting under the color of his office when
improvements on the wharves in the naval installation at his acts are wholly without authority. Until recently in
Subic Bay, we held: 1991 (Chavez v. Sandiganbayan, 193 SCRA 282 [1991]),
this doctrine still found application, this Court saying that
“The traditional rule of immunity exempts immunity from suit cannot institutionalize irresponsibility
a State from being sued in the courts of another and non-accountability nor grant a privileged status not
State without its consent or waiver. This rule is a claimed by any other official of the Republic. (Republic
necessary consequence of the principle of v. Sandoval, 220 SCRA 124, March 19, 1993, En
independence and equality of States. However, Banc [Campos, Jr.])
the rules of International Law are not petrified;
they are constantly developing and evolving. And State instances when a suit against the State is
because the activities of states have multiplied, it proper.
has been necessary to distinguish them - between
sovereign and governmental acts (jure imperii) Held: Some instances when a suit against the
and private, commercial and proprietary acts (jure State is proper are:
gestionis). The result is that State immunity now
extends only to acts jure imperii. The restrictive 1) When the Republic is sued by name;
application of State immunity is now the rule in 2) When the suit is against an unincorporated
the United States, the United Kingdom and other government agency;
states in Western Europe. 3) When the suit is on its face against a
government officer but the case is such that
Xxx ultimate liability will belong not to the officer
but to the government.
The restrictive application of State Republic v. Sandoval, 220 SCRA 124, March 19,
immunity is proper only when the proceedings 1993, En Banc [Campos, Jr.])
arise out of commercial transactions of the foreign
sovereign, its commercial activities or economic Hundreds of landless peasants, farmers and
affairs. Stated differently, a State may be said to farmworkers marched in Mendiola on their way to
have descended to the level of an individual and Malacanang protesting against the implementation
can thus be deemed to have tacitly given its of the Comprehensive Agrarian Reform Program of
consent to be sued only when it enters into the government. As the demonstration became
business contracts. It does not apply where the unruly, police and military personnel assigned in
contracts relate to the exercise of its sovereign the area violently dispersed the rallyists causing
functions. In this case the projects are an integral deaths and injuries to several demonstrators, in
part of the naval base which is devoted to the what is now referred to as the infamous “Mendiola
defense of both the United States and the Massacre.” The next day, an indignation rally was
Philippines, indisputably a function of the held where no less than the President herself
government of the highest order; they are not joined. In that rally, she promised to look into the
utilized for nor dedicated to commercial or plight of the victims and their heirs and she
business purposes.” created a Task Force to investigate the cause of the
(Department of Agriculture v. NLRC, 227 SCRA Mendiola massacre. After investigation, the Task
693, Nov. 11, 1993 [Vitug]) Force found that although initially, the police and
military personnel assigned in the area performed
When is a suit against a public official deemed to their functions in accordance with law but when
be a suit against the State? Discuss. later they fired their guns directlty at the
demonstrators, they exceeded their authority.
Held: 1. The doctrine of state immunity from suit Consequently, the Task Force recommended that
applies to complaints filed against public officials for acts the individual police and military officers involved
done in the performance of their duties. The rule is that be prosecuted criminally and for the government to
the suit must be regarded as one against the State where indemnify the victims and/or their heirs. For the
the satisfaction of the judgment against the public official government’s failure to indemnify the victims and
concerned will require the State itself to perform a their heirs, the latter brought an action for
positive act, such as appropriation of the amount damages against the government. The Solicitor
necessary to pay the damages awarded to the plaintiff. General filed a motion to dismiss invoking State
immunity from suit. The plaintiffs opposed the
The rule does not apply where the public official is motion contending that the government has
charged in his official capacity for acts that are unlawful waived its immunity from suit based on the acts
and injurious to the rights of others. Public officials are and pronouncements of the President, as well as
not exempt, in their personal capacity, from liability the recommendation of the Task Force to
arising from acts committed in bad faith. indemnify the victims and/or their heirs. Has the
government waived its immunity from suit in the
Mendiola massacre, and, therefore, should

8
indemnify the heirs and victims of the Mendiola While the Republic in this case is sued by name,
incident? Consequently, is the suit filed against the ultimate liability does not pertain to the government.
the Republic by petitioners in said case really a suit Although the military officers and personnel, then party
against the State? defendants, were discharging their official functions when
the incident occurred, their functions ceased to be official
Held: Petitioners x x x advance the argument the moment they exceeded their authority. Based on the
that the State has impliedly waived its sovereign immunity Commission findings, there was lack of justification by the
from suit. It is their considered view that by the government forces in the use of firearms. Moreover, the
recommendation made by the Commission for the members of the police and military crowd dispersal units
government to indemnify the heirs and victims of the committed a prohibited act under B.P. Blg. 880 as there
Mendiola incident and by the public addresses made by was unnecessary firing by them in dispersing the
then President Aquino in the aftermath of the killings, the marchers.
State has consented to be sued.
Xxx As early as 1954, this Court has pronounced that
This is not a suit against the State with its an officer cannot shelter himself by the plea that he is a
consent. public agent acting under the color of his office when his
acts are wholly without authority. Until recently in 1991
Firstly, the recommendation made by the (Chavez v. Sandiganbayan, 193 SCRA 282 [1991]) , this
Commission regarding indemnification of the heirs of the doctrine still found application, this Court saying that
deceased and the victims of the incident by the immunity from suit cannot institutionalize irresponsibility
government does not in any way mean that liability and non-accountability nor grant a privileged status not
automatically attaches to the State. It is important to claimed by any other official of the Republic. The military
note that A.O. 11 expressly states that the purpose of and police forces were deployed to ensure that the rally
creating the Commission was to have a body that will would be peaceful and orderly as well as to guarantee the
conduct an “investigation of the disorder, deaths and safety of the very people that they are duty-bound to
casualties that took place.” In the exercise of its protect. However, the facts as found by the trial court
functions, A.O. 11 provides guidelines, and what is showed that they fired at the unruly crowd to disperse the
relevant to Our discussion reads: latter.

“1. Its conclusions regarding the existence of While it is true that nothing is better settled than
probable cause for the commission of any offense and the general rule that a sovereign state and its political
of the persons probably guilty of the same shall be subdivisions cannot be sued in the courts except when it
sufficient compliance with the rules on preliminary has given its consent, it cannot be invoked by both the
investigation and the charges arising therefrom may military officers to release them from any liability, and by
be filed directly with the proper court.” the heirs and victims to demand indemnification from the
government. The principle of state immunity from suit
In effect, whatever may be the findings of the does not apply, as in this case, when the relief demanded
Commission, the same shall only serve as the cause of by the suit requires no affirmative official action on the
action in the event that any party decides to litigate part of the State nor the affirmative discharge of any
his/her claim. Therefore, the Commission is merely a obligation which belongs to the State in its political
preliminary venue. The Commission is not the end in capacity, even though the officers or agents who are
itself. Whatever recommendation it makes cannot in any made defendants claim to hold or act only by virtue of a
way bind the State immediately, such recommendation title of the state and as its agents and servants. This
not having become final and executory. This is precisely Court has made it quite clear that even a “high position in
the essence of it being a fact-finding body. the government does not confer a license to persecute or
recklessly injure another.”
Secondly, whatever acts or utterances that then
President Aquino may have done or said, the same are The inescapable conclusion is that the State
not tantamount to the State having waived its immunity cannot be held civilly liable for the deaths that followed
from suit. The President’s act of joining the marchers, the incident. Instead, the liability should fall on the
days after the incident, does not mean that there was an named defendants in the lower court. In line with the
admission by the State of any liability. In fact to borrow ruling of this Court in Shauf v. Court of Appeals (191
the words of petitioner x x x, “it was an act of solidarity by SCRA 713 [1990]), herein public officials, having been
the government with the people.” Moreover, petitioners found to have acted beyond the scope of their authority,
rely on President Aquino’s speech promising that the may be held liable for damages. (Republic v.
government would address the grievances of the rallyists. Sandoval, 220 SCRA 124, March 19, 1993, En Banc
By this alone, it cannot be inferred that the State has [Campos, Jr.])
admitted any liability, much less can it be inferred that it
has consented to the suit. May the Government validly invoke the doctrine of
State immunity from suit if its invocation will serve
Although consent to be sued may be given as an instrument for perpetrating an injustice on a
impliedly, still it cannot be maintained that such consent citizen?
was given considering the circumstances obtaining in the
instant case. Held: To our mind, it would be the apex of
injustice and highly inequitable for us to defeat
Thirdly, the case does not qualify as a suit against petitioners-contractors’ right to be duly compensated for
the State. actual work performed and services rendered, where both
the government and the public have, for years, received
Xxx and accepted benefits from said housing project and
reaped the fruits of petitioners-contractors’ honest toil and
labor.

9
Held: Private respondent Rosalind Ybasco Lopez
Incidentally, respondent likewise argues that the was born on May 16, 1934 in Napier Terrace, Broome,
State may not be sued in the instant case, invoking the Western Australia, to the spouses, Telesforo Ybasco, a
constitutional doctrine of Non-suability of the State, Filipino citizen and native of Daet, Camarines Norte, and
otherwise known as the Royal Prerogative of Dishonesty. Theresa Marquez, an Australian. Historically, this was a
year before the 1935 Constitution took into effect and at
Respondent’s argument is misplaced inasmuch as that time, what served as the Constitution of the
the principle of State immunity finds no application in the Philippines were the principal organic acts by which the
case before us. United States governed the country. These were the
Philippine Bill of July 1, 1902 and the Philippine Autonomy
Under these circumstances, respondent may not Act of August 29, 1916, also known as the Jones Law.
validly invoke the Royal Prerogative of Dishonesty and
conveniently hide under the State’s cloak of invincibility Among others, these laws defined who were
against suit, considering that this principle yields to deemed to be citizens of the Philippine Islands. x x x
certain settled exceptions. True enough, the rule, in any
case, is not absolute for it does not say that the state may Under both organic acts, all inhabitants of the
not be sued under any circumstances. Philippines who were Spanish subjects on April 11, 1899
and resided therein including their children are deemed to
Thus, in Amigable v. Cuenca, this Court, in effect, be Philippine citizens. Private respondent’s father,
shred the protective shroud which shields the state from Telesforo Ybasco, was born on January 5, 1879 in Daet,
suit, reiterating our decree in the landmark case of Camarines Norte, a fact duly evidenced by a certified true
Ministerio v. CFI of Cebu that “the doctrine of copy of an entry in the Registry of Births. Thus, under the
governmental immunity from suit cannot serve as an Philippine Bill of 1902 and the Jones Law, Telesforo
instrument for perpetrating an injustice on a citizen.” It is Ybasco was deemed to be a Philippine citizen. By virtue
just as important, if not more so, that there be fidelity to of the same laws, which were the laws in force at the time
legal norms on the part of officialdom if the rule of law of her birth, Telesforo’s daughter, herein private
were to be maintained. (Citations omitted) respondent Rosalind Ybasco Lopez, is likewise a citizen of
the Philippines.
Although the Amigable and Ministerio cases
generously tackled the issue of the State’s immunity from The signing into law of the 1935 Philippine
suit vis a vis the payment of just compensation for Constitution has established the principle of jus sanguinis
expropriated property, this Court nonetheless finds the as basis for the acquisition of Philippine citizenship x x x.
doctrine enunciated in the aforementioned cases So also, the principle of jus sanguinis, which confers
applicable to the instant controversy, considering that the citizenship by virtue of blood relationship, was
ends of justice would be subverted if we were to uphold, subsequently retained under the 1973 and 1987
in this particular instance, the State’s immunity from suit. Constitutions. Thus, the herein private respondent,
Rosalind Ybasco Lopez, is a Filipino citizen, having been
To be sure, this Court – as the staunch guardian born to a Filipino father. The fact of her being born in
of the citizens’ rights and welfare – cannot sanction an Australia is not tantamount to her losing her Philippine
injustice so patent on its face, and allow itself to be an citizenship. If Australia follows the principle of jus soli,
instrument in the perpetration thereof. Justice and equity then at most, private respondent can also claim Australian
sternly demand that the State’s cloak of invincibility citizenship resulting to her possession of dual citizenship.
against suit be shred in this particular instance, and that (Valles v. COMELEC, 337 SCRA 543, Aug. 9, 2000,
petitioners-contractors be duly compensated – on the En Banc [Purisima])
basis of quantum meruit – for construction done on the
public works housing project. (EPG Construction Co. v. 3. Does a legitimate child born under the 1935
Vigilar, 354 SCRA 566, Mar.16, 2001, 2nd Div. Constitution of a Filipino mother and an alien father
[Buena]) who elected Philippine citizenship fourteen (14) years
after attaining the age of majority become a Filipino?
Citizenship
Held: Under Article IV, Section 1(3) of the 1935
1. What citizenship principle do the Philippines adhere Constitution, the citizenship of a legitimate child born of a
to? Explain, and give illustrative case. Filipino mother and an alien father followed the citizenship
of the father, unless, upon reaching the age of majority,
Held: The Philippine law on citizenship adheres the child elected Philippine citizenship. C.A. No. 625
to the principle of jus sanguinis. Thereunder, a child which was enacted pursuant to Section 1(3), Article IV of
follows the nationality or citizenship of the parents the 1935 Constitution, prescribes the procedure that
regardless of the place of his/her birth, as opposed to the should be followed in order to make a valid election of
doctrine of jus soli which determines nationality or Philippine citizenship. However, the 1935 Constitution
citizenship on the basis of place of birth. (Valles v. and C.A. No. 625 did not prescribe a time period within
COMELEC, 337 SCRA 543, Aug. 9, 2000, En Banc which the election of Philippine citizenship should be
[Purisima]) made. The 1935 Charter only provides that the election
should be made “upon reaching the age of majority.” The
2. Rosalind Ybasco Lopez was born on May 16, 1934 in age of majority then commenced upon reaching twenty-
Napier Terrace, Broome, Western Australia, to the one (21) years. In the opinions of the Secretary of Justice
spouses, Telesforo Ybasco, a Filipino citizen and on cases involving the validity of election of Philippine
native of Daet, Camarines Norte, and Theresa citizenship, this dilemma was resolved by basing the time
Marquez, an Australian. Is she a Filipino citizen and, period on the decisions of this Court prior to the effectivity
therefore, qualified to run for Governor of her of the 1935 Constitution. In these decisions, the proper
province? period for electing Philippine citizenship was, in turn,
based on the pronouncements of the Department of State

10
of the United States Government to the effect that the
election should be made within a “reasonable time” after Considering the reservations made by the parties
attaining the age of majority. The phrase “reasonable on the veracity of some of entries on the birth certificate
time” has been interpreted to mean that the election of respondent and the marriage certificate of his parents,
should be made within three (3) years from reaching the the only conclusions that could be drawn with some
age of majority. degree of certainty from the documents would be that –

The span of fourteen (14) years that lapsed from 1. The parents of FPJ were Allan F. Poe and
the time that person reached the age of majority until he Bessie Kelley;
finally expressed his intention to elect Philippine 2. FPJ was born to them on 20 August 1939;
citizenship is clearly way beyond the contemplation of the 3. Allan F. Poe and Bessie Kelley were married to
requirement of electing “upon reaching the age of each other on 16 September, 1940;
majority.” 4. The father of Allan F. Poe was Lorenzo Poe;
and
Philippine citizenship can never be treated like a 5. At the time of his death on 11 September
commodity that can be claimed when needed and 1954, Lorenzo Poe was 84 years old.
suppressed when convenient. One who is privileged to
elect Philippine citizenship has only an inchoate right to Would the above facts be sufficient or insufficient
such citizenship. As such, he should avail of the right with to establish the fact that FPJ is a natural-born citizen?
fervor, enthusiasm and promptitude. (Re: Application The marriage certificate of Allan F. Poe and Bessie Kelley,
for Admission to the Philippine Bar, Vicente D. the birth certificate of FPJ, and the death certificate of
Ching, Bar Matter No. 914, Oct. 1, 1999, En Banc Lorenzo Pou are documents of public record in the
[Kapunan]) custody of a public officer. The documents have been
submitted in evidence by both contending parties during
4. Is FPJ a natural-born Filipino citizen and, therefore, the proceedings before the COMELEC.
qualified to run for President?
Xxx
Held: The term “natural-born citizens,” is defined
to include “those who are citizens of the Philippines from Being public documents, the death certificate of
birth without having to perform any act to acquire or Lorenzo Pou, the marriage certificate of Allan F. Poe and
perfect their Philippine citizenship.” (Section 2, Article IV, Bessie Kelly, and the birth certificate of FPJ, constitute
1987 Constitution) prima facie proof of their contents. Section 44, Rule 130,
of the Rules of Court provides:
The date, month and year of birth of FPJ
appeared to be 20 August 1939 during the regime of the “Entries in official records. Entries in
1935 Constitution. Through its history, four modes of official records made in the performance of his
acquiring citizenship – naturalization, jus soli, res judicata duty by a public officer of the Philippines, or by a
and jus sanguinis – had been in vogue. Only two, i.e., jus person in the performance of a duty specially
soli and jus sanguinis, could qualify a person to being a enjoined by law, are prima facie evidence of the
“natural-born” citizen of the Philippines. Jus soli, per Roa facts therein stated.”
v. Collector of Customs (23 Phil. 315 [1912]), did not last
long. With the adoption of the 1935 Constitution and the Xxx
reversal of Roa in Tan Chong v. Secretary of Labor
(Supra, which held that jus soli was never applied in the The death certificate of Lorenzo Pou would
Philippines), jus sanguinis or blood relationship would now indicate that he died on 11 September 1954, at the age of
become the primary basis of citizenship by birth. 84 years, in San Carlos, Pangasinan. It could thus be
assumed that Lorenzo Pou was born sometime in the year
Documentary evidence adduced by petitioner 1870 when the Philippines was still a colony of Spain.
would tend to indicate that the earliest established direct Petitioner would argue that Lorenzo Pou was not in the
ascendant of FPJ was his paternal grandfather Lorenzo Philippines during the crucial period of from 1898 to 1902
Pou, married to Marta Reyes, the father of Allan F. Poe. considering that there was no existing record about such
While the records of birth of Lorenzo Pou had not been fact in the Records Management and Archives Office.
presented in evidence, his death certificate, however, Petitioner, however, likewise failed to show that Lorenzo
identified him to be a Filipino, a resident of San Carlos, Pou was at any other place during the same period. In
Pangasinan, and 84 years old at the time of his death on his death certificate, the residence of Lorenzo Pou was
11 September 1954. The certificate of birth of the father stated to be San Carlos, Pangasinan. In the absence of
of FPJ, Allan F. Poe, showed that he was born on 17 May any evidence to the contrary, it should be sound to
1915 to an Espanol father, Lorenzo Pou, and a mestiza conclude, or at least to presume, that the place of
Espanol mother, Marta Reyes. Introduced by petitioner residence of a person at the time of his death was also his
was an “uncertified” copy of a supposed certificate of the residence before death. It would be extremely doubtful if
alleged marriage of Allan F. Poe and Paulita Gomez on 05 the Records Management and Archives Office would have
July 1936. The marriage certificate of Allan F. Poe and had complete records of all residents of the Philippines
Bessie Kelley reflected the date of their marriage to be on from 1898 to 1902.
16 September 1940. In the same certificate, Allan F. Poe
was stated to be twenty-five years old, unmarried, and a Xxx
Filipino citizen, and Bessie Kelley to be twenty-two years
old, unmarried, and an American citizen. The birth Petitioner submits, in any case, that in
certificate of FPJ, would disclose that he was born on 20 establishing filiation (relationship or civil status of the child
August 1939 to Allan F. Poe, a Filipino, twenty-four years to the father [or mother]) or paternity (relationship or civil
old, married to Bessie Kelley, an American citizen, twenty- status of the father to the child) of an illegitimate child,
one years old and married. FPJ evidently being an illegitimate son according the

11
petitioner, the mandatory rules under civil law must be
used. Petitioner would have it that even if Allan F. Poe
were a Filipino citizen, he could not have transmitted his
Xxx citizenship to respondent FPJ, the latter being an
illegitimate child. X x x But the documentary evidence
It should be apparent that the growing trend to introduced by no less than respondent himself, consisting
liberalize the acknowledgment or recognition of of a birth certificate of respondent and a marriage
illegitimate children is an attempt to break away from the certificate of his parents showed that FPJ was born on 20
traditional idea of keeping well apart legitimate and non- August 1939 to a Filipino father and an American mother
legitimate relationships within the family in favor of the who were married to each other a year later, or on 16
greater interest and welfare of the child. The provisions September 1940. Birth to unmarried parents would make
are intended merely to govern the private and personal FPJ an illegitimate child. Petitioner contended that as an
affairs of the family. There is little, if any, to indicate that illegitimate child, FPJ so followed the citizenship of his
the legitimate or illegitimate civil status of the individual mother, Bessie Kelley, an American citizen, basing his
would also affect his political rights or, in general, his stand on the ruling of this Court in Morano v. Vivo (20
relationship to the State. While, indeed, provisions on SCRA 562, Paa v. Chan, 21 SCRA 753), citing Chiongbian
“citizenship” could be found in the Civil Code, such v. de Leon (82 Phil. 771) and Serra v. Republic (91 Phil.
provisions must be taken in the context of private 914, unreported).
relations, the domain of civil law x x x.
On the above score, the disquisition made by
The relevance of “citizenship” or “nationality” to amicus curiae Joaquin G. Bernas, SJ, is most convincing;
Civil Law is best exemplified in Article 15 of the Civil Code he states –
x x x that explains the need to incorporate in the code a
reiteration of the Constitutional provisions on citizenship. “We must analyze these cases and ask
Similarly, citizenship is significant in civil relationships what the lis mota was in each of them. If the
found in different parts of the Civil Code x x x. In pronouncement of the Court on jus sanguinis was
adoption, for instance, an adopted child would be on the lis mota, the pronouncement would be a
considered the child of his adoptive parents and accorded decision constituting doctrine under the rule of
the same rights as their legitimate child but such legal stare decisis. But if the pronouncement was
fiction extended only to define his rights under civil law irrelevant to the lis mota, the pronouncement
(See Ching Leng v. Galang, L-11931, October 1958, would not be a decision but a mere obiter dictum
unreported) and not his political status. which did not establish doctrine. I therefore
invite the Court to look closely into these cases.
Civil law provisions point to an obvious bias
against illegitimacy. This discriminatory attitude may be “First, Morano v. Vivo. The case was not
traced to the Spanish family and property laws, which, about an illegitimate child of a Filipino father. It
while defining proprietary and successional rights of was about a stepson of a Filipino, a stepson who
members of the family, provided distinctions in the rights was the child of a Chinese mother and a Chinese
of legitimate and illegitimate children. X x x father. The issue was whether the stepson
followed the naturalization of the stepfather.
These distinctions between legitimacy and Nothing about jus sanguinis there. The stepson
illegitimacy were codified in the Spanish Civil Code, and did not have the blood of the naturalized father.
the invidious discrimination survived when the Spanish
Civil Code became the primary source of our own Civil “Second, Chiongbian v. de Leon. This
Code. Such distinction, however, remains and should case was not about the illegitimate son of a
remain only in the sphere of civil law and not unduly Filipino father. It was about a legitimate son of a
impede or impinge on the domain of political law. father who had become Filipino by election to
public office before the 1935 Constitution
The proof of filiation or paternity for purposes of pursuant to Article IV, Section 1(2) of the 1935
determining his citizenship status should be deemed Constitution. No one was illegitimate here.
independent from and not inextricably tied up with that
prescribed for civil law purposes. The Civil Code or Family “Third, Serra v. Republic. The case was
Code provisions on proof of filiation or paternity, although not about the illegitimate son of a Filipino father.
good law, do not have preclusive effects on matters alien Serra was an illegitimate child of a Chinese father
to personal and family relations. The ordinary rules on and a Filipino mother. The issue was whether
evidence could well and should govern. For instance, the one who was already a Filipino because of his
matter about pedigree is not necessarily precluded from mother who still needed to be naturalized. There
being applicable by the Civil Code or Family Code is nothing there about invidious jus sanguinis.
provisions.
“Finally, Paa v. Chan (21 SCRA 753). This
Xxx is a more complicated case. The case was about
the citizenship of Quintin Chan who was the son
Thus, the duly notarized declaration made by of Leoncio Chan. Quintin Chan claimed that his
Ruby Kelley Mangahas, sister of Bessie Kelley Poe father, Leoncio, was the illegitimate son of a
submitted as Exhibit 20 before the COMELEC, might be Chinese father and a Filipino mother. Quintin
accepted to prove the acts of Allan F. Poe, recognizing his therefore argued that he got his citizenship from
own paternal relationship with FPJ, i.e., living together Leoncio, his father. But the Supreme Court said
with Bessie Kelley and his children (including respondent that there was no valid proof that Leoncio was in
FPJ) in one house, and as one family x x x. fact the son of a Filipina mother. The Court
therefore concluded that Leoncio was not Filipino.
Xxx If Leoncio was not Filipino, neither was his son

12
Quintin. Quintin therefore was not only not a explicit than it is. Providing neither conditions nor
natural-born Filipino but was not even a Filipino. distinctions, the Constitution states that among the
citizens of the Philippines are “those whose fathers are
“The Court should have stopped there. citizens of the Philippines.” There utterly is no cogent
But instead it followed with an obiter dictum. The justification to prescribe conditions or distinctions where
Court said obiter that even if Leoncio, Quintin’s there clearly are none provided. (Maria Jeanette
father, were Filipino, Quintin would not be Filipino Tecson, et al. v. COMELEC, G.R. No. 161434, March
because Quintin was illegitimate. This statement 3, 2004, En Banc [Vitug])
about Quintin, based on a contrary to fact
assumption, was absolutely unnecessary for the 5. How may Philippine citizenship be renounced? Is the
case. X x x It was obiter dictum, pure and simple, application for an alien certificate of registration, and
simply repeating the obiter dictum in Morano v. the possession of foreign passport, tantamount to acts
Vivo. of renunciation of Philippine citizenship?

Xxx Held: In order that citizenship may be lost by


renunciation, such renunciation must be express.
“Aside from the fact that such a Petitioner’s contention that the application of private
pronouncement would have no textual foundation respondent for an alien certificate of registration, and her
in the Constitution, it would also violate the equal Australian passport, is bereft of merit. This issue was put
protection clause of the Constitution not once but to rest in the case of Aznar v. COMELEC (185 SCRA 703
twice. First, it would make an illegitimate [1990]) and in the more recent case of Mercado v.
distinction between a legitimate child and an Manzano and COMELEC (G.R. No. 135083, 307 SCRA 630,
illegitimate child, and second, it would make an May 26, 1999).
illegitimate distinction between the illegitimate
child of a Filipino father and the illegitimate child In the case of Aznar, the Court ruled that the
of a Filipino mother. mere fact that he is an American did not mean that he is
no longer a Filipino, and that an application for an alien
“The doctrine on constitutionally allowable certificate of registration was not tantamount to
distinctions was established long ago by People v. renunciation of his Philippine citizenship.
Cayat (68 Phil. 12). I would grant that the
distinction between legitimate children and And, in Mercado v. Manzano and COMELEC, it was
illegitimate children rests on real differences. X x held that the fact that respondent Manzano was
x But real differences alone do not justify registered as an American citizen in the Bureau of
invidious distinction. Real differences may justify Immigration and Deportation and was holding an
distinction for one purpose but not for another American passport on April 22, 1997, only a year before
purpose. he filed a certificate of candidacy for vice-mayor of Makati,
were just assertions of his American nationality before the
“x x x What is the relevance of legitimacy termination of his American citizenship.
or illegitimacy to elective public service? What
possible state interest can there be for Thus, the mere fact that private respondent
disqualifying an illegitimate child from becoming a Rosalind Ybasco Lopez was a holder of an Australian
public officer. It was not the fault of the child passport and had an alien certificate of registration are
that his parents had illicit liaison. Why deprive not acts constituting an effective renunciation of
the child of the fullness of political rights for no citizenship and do not militate against her claim of Filipino
fault of his own? To disqualify an illegitimate citizenship. For renunciation to effectively result in the
child from holding an important public office is to loss of citizenship, the same must be express. As held by
punish him for the indiscretion of his parents. this Court in the aforecited case of Aznar, an application
There is neither justice nor rationality in that. for an alien certificate of registration does not amount to
And if there is neither justice nor rationality in the an express renunciation or repudiation of one’s
distinction, then the distinction transgresses the citizenship. The application of the herein private
equal protection clause and must be reprobated.” respondent for an alien certificate of registration, and her
holding of an Australian passport, as in the case of
The other amici curiae, Mr. Justice Mendoza (a Mercado v. Manzano, were mere acts of assertion of her
former member of this Court), Professor Ruben Balane Australian citizenship before she effectively renounced the
and Dean Merlin Magallona, at bottom, have expressed same. Thus, at the most, private respondent had dual
similar views. The thesis of petitioner, unfortunately citizenship – she was an Australian and a Filipino, as well.
hinging solely on pure obiter dicta, should indeed fail.
Moreover, under Commonwealth Act 63, the fact
Where jurisprudence regarded an illegitimate child that a child of Filipino parent/s was born in another
as taking after the citizenship of its mother, it did so for country has not been included as a ground for losing one’s
the benefit of the child. It was to ensure a Filipino Philippine citizenship. Since private respondent did not
nationality for the illegitimate child of an alien father in lose or renounce her Philippine citizenship, petitioner’s
line with the assumption that the mother had custody, claim that respondent must go through the process of
would exercise parental authority and had the duty to repatriation does not hold water. (Valles v. COMELEC,
support her illegitimate child. It was to help the child, not 337 SCRA 543, Aug. 9, 2000, En Banc [Purisima])
to prejudice or discriminate against him.
6. What are the ways of acquiring citizenship? Discuss.
The fact of the matter – perhaps the most
significant consideration – is that the 1935 Constitution, Held: There are two ways of acquiring
the fundamental law prevailing on the day, month and citizenship: (1) by birth, and (2) by naturalization. These
year of birth of respondent FPJ, can never be more ways of acquiring citizenship correspond to the two kinds

13
of citizens: the natural-born citizen, and the naturalized prior to the hearing of his petition for
citizen. A person who at the time of his birth is a citizen naturalization as Philippine citizen.
of a particular country, is a natural-born citizen thereof. (Antonio Bengson III v. HRET, G.R. No. 142840,
May 7, 2001, En Banc [Kapunan])
As defined in the x x x Constitution, natural-born
citizens “are those citizens of the Philippines from birth 9. What are the disqualifications under Section 4, Act
without having to perform any act to acquire or perfect 473, in an application for naturalization?
his Philippine citizenship.”
Held: Section 4, Act 473, provides the following
On the other hand, naturalized citizens are those disqualifications:
who have become Filipino citizens through naturalization,
generally under Commonwealth Act No. 473, otherwise (a) He must not be opposed to organized
known as the Revised Naturalization Law, which repealed government or affiliated with any association
the former Naturalization Law (Act No. 2927), and by or group of persons who uphold and teach
Republic Act No. 530. (Antonio Bengson III v. HRET, doctrines opposing all organized
G.R. No. 142840, May 7, 2001, En Banc [Kapunan]) governments;
(b) He must not be defending or teaching the
7. To be naturalized, what must an applicant prove? necessity or propriety of violence, personal
When and what are the conditions before the decision assault, or assassination for the success and
granting Philippine citizenship becomes executory? predominance of their ideas;
(c) He must not be a polygamist or believer in
Held: To be naturalized, an applicant has to the practice of polygamy;
prove that he possesses all the qualifications and none of (d) He must not have been convicted of any
the disqualifications provided by law to become a Filipino crime involving moral turpitude;
citizen. The decision granting Philippine citizenship (e) He must not be suffering from mental
becomes executory only after two (2) years from its alienation or incurable contagious diseases;
promulgation when the court is satisfied that during the (f) He must have, during the period of his
intervening period, the applicant has (1) not left the residence in the Philippines (or not less than
Philippines; (2) has dedicated himself to a lawful calling or six months before filing his application),
profession; (3) has not been convicted of any offense or mingled socially with the Filipinos, or who
violation of government promulgated rules; or (4) have not evinced a sincere desire to learn
committed any act prejudicial to the interest of the nation and embrace the customs, traditions and
or contrary to any government announced policies ideals of the Filipinos;
(Section 1, R.A. 530). (Antonio Bengson III v. HRET, (g) He must not be a citizen or subject of a
G.R. No. 142840, May 7, 2001, En Banc [Kapunan]) nation with whom the Philippines is at war,
during the period of such war;
8. What qualifications must be possessed by an applicant (h) He must not be a citizen or subject of a
for naturalization? foreign country whose laws do not grant
Filipinos the right to become naturalized
Held: Section 2, Act 473 provides the following citizens or subjects thereof.
qualifications: (Antonio Bengson III v. HRET, G.R. No. 142840,
May 7, 2001, En Banc [Kapunan])
(a) He must be not less than 21 years of age on
the day of the hearing of the petition; 10. Distinguish naturalization from repatriation, and
(b) He must have resided in the Philippines for a discuss the applicable laws in each.
continuous period of not less than ten years;
(c) He must be of good moral character and Held: Naturalization is a mode for both
believes in the principles underlying the acquisition and reacquisition of Philippine citizenship. As a
Philippine Constitution, and must have mode of initially acquiring Philippine citizenship,
conducted himself in a proper and naturalization is governed by Commonwealth Act No. 473,
irreproachable manner during the entire as amended. On the other hand, naturalization as a mode
period of his residence in the Philippines in for reacquiring Philippine citizenship is governed by
his relation with the constituted government Commonwealth Act No. 63 (An Act Providing for the Ways
as well as with the community in which he is in Which Philippine Citizenship May Be Lost or Reacquired
living; [1936]). Under this law, a former Filipino citizen who
(d) He must own real estate in the Philippines wishes to reacquire Philippine citizenship must possess
worth not less than five thousand pesos, certain qualifications and none of the disqualifications
Philippine currency, or must have some mentioned in Section 4 of C.A. 473.
known lucrative trade, profession, or lawful
occupation; Repatriation, on the other hand, may be had
(e) He must be able to speak and write English under various statutes by those who lost their citizenship
or Spanish and any of the principal due to: (1) desertion of the armed forces (Section 4, C.A.
languages; and No. 63); (2) service in the armed forces of the allied
(f) He must have enrolled his minor children of forces in World War II (Section 1, Republic Act No. 965
school age, in any of the public schools or [1953]); (3) service in the Armed Forces of the United
private schools recognized by the Bureau of States at any other time (Sec. 1, Republic Act No. 2630
Private Schools of the Philippines where [1960]); (4) marriage of a Filipino woman to an alien
Philippine history, government and civic are (Sec. 1, Republic Act No. 8171 [1995]); and (5) political
taught or prescribed as part of the school and economic necessity (Ibid).
curriculum, during the entire period of the
residence in the Philippines required of him

14
As distinguished from the lengthy process of Philippine citizenship, he will be restored to his former
naturalization, repatriation simply consists of the taking of status as a natural-born Filipino.
an oath of allegiance to the Republic of the Philippines
and registering said oath in the Local Civil Registry of the In respondent Cruz’s case, he lost his Filipino
place where the person concerned resides or last resided. citizenship when he rendered service in the Armed Forces
of the United States. However, he subsequently
Xxx reacquired Philippine citizenship under R.A. No. 2630 x x
x.
Moreover, repatriation results in the recovery of
the original nationality. This means that a naturalized Having thus taken the required oath of allegiance
Filipino who lost his citizenship will be restored to his prior to the Republic and having registered the same in the Civil
status as a naturalized Filipino citizen. On the other hand, Registry of Mangatarem, Pangasinan in accordance with
if he was originally a natural-born citizen before he lost his the aforecited provision, respondent Cruz is deemed to
Philippine citizenship, he will be restored to his former have recovered his original status as a natural-born
status as a natural-born Filipino. (Antonio Bengson III citizen, a status which he acquired at birth as the son of a
v. HRET, G.R. No. 142840, May 7, 2001, En Banc Filipino father. It bears stressing that the act of
[Kapunan]) repatriation allows him to recover, or return to, his
original status before he lost his Philippine citizenship.
11. How may Filipino citizens who lost their citizenship
reacquire the same? Petitioner’s contention that respondent Cruz is no
longer a natural-born citizen since he had to perform an
Answer: Filipino citizens who have lost their act to regain his citizenship is untenable. [T]he term
citizenship may x x x reacquire the same in the manner “natural-born citizen” was first defined in Article III,
provided by law. Commonwealth Act No. 63 enumerates Section 4 of the 1973 Constitution as follows:
the three modes by which Philippine citizenship may be
reacquired by a former citizen: (1) by naturalization, (2) Section 4. A natural-born citizen is one
by repatriation, and (3) by direct act of Congress . who is a citizen of the Philippines from birth
(Frivaldo v. COMELEC, 257 SCRA 727, June 28, without having to perform any act to acquire or
1996, En Banc [Panganiban]; Antonio Bengson III perfect his Philippine citizenship.
v. HRET, G.R. No. 142840, May 7, 2001, En Banc
[Kapunan]) Two requisites must concur for a person to be
considered as such: (1) a person must be a Filipino citizen
12. Who may validly avail of repatriation under R.A. No. from birth and (2) he does not have to perform any act to
8171? obtain or perfect his Philippine citizenship.

Held: R.A. No. 8171, which has lapsed into law Under the 1973 Constitution definition, there were
on October 23, 1995, is an act providing for the two categories of Filipino citizens which were not
repatriation (a) of Filipino women who have lost their considered natural-born: (1) those who were naturalized
Philippine citizenship by marriage to aliens and (b) of and (2) those born before January 17, 1973 (the date of
natural-born Filipinos who have lost their Philippine effectivity of the 1973 Constitution), of Filipino mothers
citizenship on account of political or economic necessity. who, upon reaching the age of majority, elected Philippine
(Gerardo Angat v. Republic, G.R. No. 132244, Sept. citizenship. Those “naturalized citizens” were not
14, 1999 [Vitug]) considered natural-born obviously because they were not
Filipinos at birth and had to perform an act to acquire
13. Before what agency should application for repatriation Philippine citizenship. Those born of Filipino mothers
under R.A 8171 be filed? before the effectivity of the 1973 Constitution were
likewise not considered natural-born because they also
Held: Under Section 1 of P.D. No. 725, dated had to perform an act to perfect their Philippine
June 5, 1975, amending C.A. No. 63, an application for citizenship.
repatriation could be filed with the Special Committee on
Naturalization chaired by the Solicitor General with the The present Constitution, however, now considers
Undersecretary of Foreign Affairs and the Director of the those born of Filipino mothers before the effectivity of the
National Intelligence Coordinating Agency as the other 1973 Constitution and who elected Philippine citizenship
members. Although the agency was deactivated by virtue upon reaching the majority age as natural-born. After
of President Corazon C. Aquino’s Memorandum of March defining who are natural-born citizens, Section 2 of Article
27, 1987, it was not, however, abrogated. The IV adds a sentence: “Those who elect Philippine
Committee was reactivated on June 8, 1995. Hence, the citizenship in accordance with paragraph (3), Section 1
application should be filed with said Agency, not with the hereof shall be deemed natural-born citizens.”
Regional Trial Court. (Gerardo Angat v. Republic, Consequently, only naturalized Filipinos are considered not
G.R. No. 132244, Sept. 14, 1999 [Vitug]) natural-born citizens. It is apparent from the enumeration
of who are citizens under the present Constitution that
14. May a natural-born Filipino who became an American there are only two classes of citizens: (1) those who are
citizen still be considered a natural-born Filipino upon natural-born and (2) those who are naturalized in
his reacquisition of Philippine citizenship and, accordance with law. A citizen who is not a naturalized
therefore, qualified to run for Congressman? Filipino, i.e., did not have to undergo the process of
naturalization to obtain Philippine citizenship, necessarily
Held: Repatriation results in the recovery of the is a natural-born Filipino. Noteworthy is the absence in
original nationality. This means that a naturalized Filipino the said enumeration of a separate category for persons
who lost his citizenship will be restored to his prior status who, after losing Philippine citizenship, subsequently
as a naturalized Filipino citizen. On the other hand, if he reacquire it. The reason therefore is clear: as to such
was originally a natural-born citizen before he lost his persons, they would either be natural-born or naturalized

15
depending on the reasons for the loss of their citizenship 1) Those born of Filipino fathers and/or mothers
and the mode prescribed by the applicable law for the in foreign countries which follow the principle
reacquisition thereof. As respondent Cruz was not of jus soli;
required by law to go through naturalization proceedings 2) Those born in the Philippines of Filipino
in order to reacquire his citizenship, he is perforce a mothers and alien fathers if by the laws of
natural-born Filipino. As such, he possessed all the their father’s country such children are
necessary qualifications to be elected as member of the citizens of that country;
House of Representatives. (Antonio Bengson III v. 3) Those who marry aliens if by the laws of the
HRET, G.R. No. 142840, May 7, 2001, En Banc latter’s country the former are considered
[Kapunan]) citizens, unless by their act or omission they
are deemed to have renounced Philippine
15. Distinguish Dual Citizenship from Dual Allegiance. citizenship.
(Mercado v. Manzano, G.R. No. 135083, 307 SCRA
Held: Dual citizenship arises when, as a result of 630, May 26, 1999 [Mendoza])
the concurrent application of the different laws of two or
more states, a person is simultaneously considered a 18. The general rule is that res “judicata does” not apply
national by the said states. For instance, such a situation in cases hinging on the issue of citizenship? What is
may arise when a person whose parents are citizens of a the exception to this rule? Discuss.
state which adheres to the principle of jus sanguinis is
born in a state which follows the doctrine of jus soli. Such Held: Petitioner maintains further that when
a person, ipso facto and without any voluntary act on his citizenship is raised as an issue in judicial or administrative
part, is concurrently considered a citizen of both states. proceedings, the resolution or decision thereon is
generally not considered res judicata in any subsequent
Dual allegiance, on the other hand, refers to a proceeding challenging the same; citing the case of Moy
situation in which a person simultaneously owes, by some Ya Lim Yao v. Commissioner of Immigration (41 SCRA 292
positive act, loyalty to two or more states. While dual [1971]). He insists that the same issue of citizenship may
citizenship is involuntary, dual allegiance is the result of be threshed out anew.
an individual’s volition. (Mercado v. Manzano, 307
SCRA 630, May 26, 1999, En Banc [Mendoza]) Petitioner is correct insofar as the general rule is
concerned, i.e., the principle of res judicata generally does
16. What is the main concern of Section 5, Article IV, not apply in cases hinging on the issue of citizenship.
1987 Constitution, on citizenship? Consequently, are However, in the case of Burca v. Republic (51 SCRA 248
persons with mere dual citizenship disqualified to run [1973]), an exception to this general rule was recognized.
for elective local positions under Section 40(d) of the The Court ruled in that case that in order that the doctrine
Local Government Code? of res judicata may be applied in cases of citizenship, the
following must be present:
Held: In including Section 5 in Article IV on
citizenship, the concern of the Constitutional Commission 1) a person’s citizenship be raised as a material
was not with dual citizens per se but with naturalized issue in a controversy where said person is a
citizens who maintain their allegiance to their countries of party;
origin even after their naturalization. Hence, the phrase 2) the Solicitor General or his authorized
“dual citizenship” in R.A. No. 7160, Section 40(d) (Local representative took active part in the
Government Code) must be understood as referring to resolution thereof, and
“dual allegiance.” Consequently, persons with mere dual 3) the finding on citizenship is affirmed by this
citizenship do not fall under this disqualification. Unlike Court.
those with dual allegiance, who must x x x be subject to
strict process with respect to the termination of their Although the general rule was set forth in the
status, for candidates with dual citizenship, it should case of Moy Ya Lim Yao, the case did not foreclose the
suffice if, upon the filing of their certificate of candidacy, weight of prior rulings on citizenship. It elucidated that
they elect Philippine citizenship to terminate their status reliance may somehow be placed on these antecedent
as persons with dual citizenship considering that their official findings, though not really binding, to make the
condition is the unavoidable consequence of conflicting effort easier or simpler. (Valles v. COMELEC, 337
laws of different states. SCRA 543, Aug. 9, 2000, En Banc [Purisima])

By electing Philippine citizenship, such candidates


at the same time forswear allegiance to the other country Civilian Supremacy Clause
of which they are also citizens and thereby terminate their
status as dual citizens. It may be that, from the point of 19. The President issued Letter of Instruction (LOI)
view of the foreign state and of its laws, such an ordering the deployment of members of the Philippine
individual has not effectively renounced his foreign Marines in the metropolis to conduct joint visibility
citizenship. That is of no moment. (Mercado v. patrols with members of the Philippine National Police
Manzano, G.R. No. 135083, 307 SCRA 630, May 26, in various shopping malls. Will this not violate the
1999 [Mendoza]) civilian supremacy clause under Section 3, Article II of
the Constitution? Will this not amount to an "insidious
17. Cite instances when a citizen of the Philippines may incursion" of the military in the task of law
possess dual citizenship considering the citizenship enforcement in violation of Section 5(4), Article XVI of
clause (Article IV) of the Constitution. the Constitution?

Held: Held: The deployment of the Marines does not


constitute a breach of the civilian supremacy clause. The
calling of the marines in this case constitutes permissible

16
use of military assets for civilian law enforcement. x x x. 11. Conduct of nationwide tests for
The limited participation of the Marines is evident in the elementary and high school students;
provisions of the LOI itself, which sufficiently provides the 12. Anti-drug enforcement activities;
metes and bounds of the Marines' authority. It is 13. Sanitary inspections;
noteworthy that the local police forces are the ones in 14. Conduct of census work;
charge of the visibility patrols at all times, the real 15. Administration of the Civil Aeronautics
authority belonging to the PNP. In fact, the Metro Manila Board;
Police Chief is the overall leader of the PNP-Philippine 16. Assistance in installation of weather
Marines joint visibility patrols. Under the LOI, the police forecasting devices;
forces are tasked to brief or orient the soldiers on police 17. Peace and order policy formulation in
patrol procedures. It is their responsibility to direct and local government units.
manage the deployment of the Marines. It is, likewise,
their duty to provide the necessary equipment to the This unquestionably constitutes a gloss on
Marines and render logistical support to these soldiers. In executive power resulting from a systematic, unbroken,
view of the foregoing, it cannot be properly argued that executive practice, long pursued to the knowledge of
military authority is supreme over civilian authority. Congress and, yet, never before questioned. What we
have here is mutual support and cooperation between the
Moreover, the deployment of the Marines to assist military and civilian authorities, not derogation of civilian
the PNP does not unmake the civilian character of the supremacy. (IBP v. Hon. Ronaldo B. Zamora, G.R.
police force. Neither does it amount to an “insidious No. 141284, Aug. 15, 2000, En Banc [Kapunan])
incursion” of the military in the task of law enforcement in
violation of Section 5[4], Article XVI of the Constitution. The Right to a Balanced and Healthful Ecology

In this regard, it is not correct to say that General 20. Is the right to a balanced and healthful ecology any
Angelo Reyes, Chief of Staff of the AFP, by his alleged less important than any of the civil and political rights
involvement in civilian law enforcement, has been virtually enumerated in the Bill of Rights? Explain.
appointed to a civilian post in derogation of the aforecited
provision. The real authority in these operations, as Held: While the right to a balanced and healthful
stated in the LOI, is lodged with the head of a civilian ecology is to be found under the Declaration of Principles
institution, the PNP, and not with the military. Such being and State Policies and not under the Bill of Rights, it does
the case, it does not matter whether the AFP Chief not follow that it is less important than any of the civil and
actually participates in the Task Force Tulungan since he political rights enumerated in the latter. Such a right
does not exercise any authority or control over the same. belongs to a different category of rights altogether for it
Since none of the Marines was incorporated or enlisted as concerns nothing less than self-preservation and self-
members of the PNP, there can be no appointment to a perpetuation, the advancement of which may even be
civilian position to speak of. Hence, the deployment of said to predate all governments and constitutions. As a
the Marines in the joint visibility patrols does not destroy matter of fact, these basic rights need not even be written
the civilian character of the PNP. in the Constitution for they are assumed to exist from the
inception of humankind. If they are now explicitly
Considering the above circumstances, the Marines mentioned in the fundamental charter, it is because of the
render nothing more than assistance required in well-founded fear of its framers that unless the rights to a
conducting the patrols. As such, there can be no balanced and healthful ecology and to health are
“insidious incursion” of the military in civilian affairs nor mandated as state policies by the Constitution itself x x x
can there be a violation of the civilian supremacy clause in the day would not be too far when all else would be lost
the Constitution. not only for the present generation, but also for those to
come – generations which stand to inherit nothing but
It is worth mentioning that military assistance to parched earth incapable of sustaining life. (Oposa v.
civilian authorities in various forms persists in Philippine Factoran, Jr., 224 SCRA 792 [1993][Davide])
jurisdiction. The Philippine experience reveals that it is
not averse to requesting the assistance of the military in 21. The Province of Palawan and the City of Puerto
the implementation and execution of certain traditionally Princesa enacted ordinances prohibiting the catching
“civil” functions. x x x [S]ome of the multifarious activities and/or exportation of live tropical fishes, and imposing
wherein military aid has been rendered, exemplifying the penalties for violations thereof, in order to stop the
activities that bring both the civilian and the military illegal practice of cyanide fishing which destroys the
together in a relationship of cooperation, are: corals and other marine resources. Several fishermen
apprehended for violating the ordinances in question
1. Elections; challenged their constitutionality contending that the
2. Administration of the Philippine National ordinances violated their preferential right as
Red Cross; subsistence and marginal fishermen to the use of our
3. Relief and rescue operations during communal marine resources guaranteed by the
calamities and disasters; Constitution, under Section 7, Article XIII. Will you
4. Amateur sports promotion and sustain the challenge?
development;
5. Development of the culture and the arts; Held: The “preferential right” of subsistence or
6. Conservation of natural resources; marginal fishermen to the use of marine resources is not
7. Implementation of the agrarian reform absolute. In accordance with the Regalian Doctrine,
program; marine resources belong to the State, and, pursuant to
8. Enforcement of customs laws; the first paragraph of Section 2, Article XII of the
9. Composite civilian-military law Constitution, their “exploration, development and
enforcement activities; utilization x x x shall be under the full control and
10. Conduct of licensure examinations; supervision of the State.” Moreover, their mandated

17
protection, development and conservation x x x imply (University of San Agustin, Inc. v. Court of Appeals,
certain restrictions on whatever right of enjoyment there 230 SCRA 761, 774-775, March 7, 1994 [Nocon])
may be in favor of anyone. What must be borne in mind
is the State policy enshrined in the Constitution regarding 24. What are the essential freedoms subsumed in the
the duty of the State to protect and advance the right of term “academic freedom”?
the people to a balanced and healthful ecology in accord
with the rhythm and harmony of nature (Section 16, Held: 1. In Ateneo de Manila University v.
Article II). The ordinances in question are meant Capulong (G.R. No. 99327, 27 May 1993), this Court cited
precisely to protect and conserve our marine resources to with approval the formulation made by Justice Felix
the end that their enjoyment may be guaranteed not only Frankfurter of the essential freedoms subsumed in the
for the present generation, but also for the generations to term “academic freedom” encompassing not only “the
come. The right to a balanced and healthful ecology freedom to determine x x x on academic grounds who
carries with it a correlative duty to refrain from impairing may teach, what may be taught (and) how it shall be
the environment. (Tano v. Gov. Salvador P. Socrates, taught,” but likewise “who may be admitted to study.”
G.R. No. 110249, Aug. 21, 1997) We have thus sanctioned its invocation by a school in
rejecting students who are academically delinquent, or a
Academic Freedom laywoman seeking admission to a seminary, or students
violating “School Rules on Discipline.” (Isabelo, Jr. v.
22. How should the State’s power to regulate educational Perpetual Help College of Rizal, Inc., 227 SCRA
institutions be exercised? 595-597, Nov. 8, 1993, En Banc [Vitug])

Held: Section 4[1], Article XIV of the 2. The essential freedoms subsumed in the term
Constitution recognizes the State’s power to regulate “academic freedom” encompass the freedom to determine
educational institutions: for itself on academic grounds:

The State recognizes the complementary (1) Who may teach,


roles of public and private institutions in the (2) What may be taught,
educational system and shall exercise reasonable (3) How it shall be taught, and
supervision and regulation of all educational (4) Who may be admitted to study.
institutions.
The right of the school to discipline its students is
As may be gleaned from the above provision, at once apparent in the third freedom, i.e., “how it shall
such power to regulate is subject to the requirement of be taught.” A school certainly cannot function in an
reasonableness. Moreover, the Constitution allows merely atmosphere of anarchy.
the regulation and supervision of educational institutions,
not the deprivation of their rights. (Miriam College Thus, there can be no doubt that the
Foundation, Inc. v. Court of Appeals, 348 SCRA establishment of an educational institution
265, 288, Dec. 15, 2000, 1st Div. [Kapunan]) requires rules and regulations necessary for the
maintenance of an orderly educational program
23. Define and discuss the academic freedom of and the creation of an educational environment
institutions of higher learning. conducive to learning. Such rules and regulations
are equally necessary for the protection of the
Held: Academic freedom of educational students, faculty, and property (Angeles v. Sison,
institutions has been defined as the right of the school or 112 SCRA 26, 37 [1982]).
college to decide for itself, its aims and objectives, and
how best to attain them - free from outside coercion or Moreover, the school has an interest in teaching
interference save possibly when the overriding public the student discipline, a necessary, if not indispensable,
welfare calls for some restraint. It has a wide sphere of value in any field of learning. By instilling discipline, the
autonomy certainly extending to the choice of students. school teaches discipline. Accordingly, the right to
Said constitutional provision is not to be construed in a discipline the student likewise finds basis in the freedom
niggardly manner or in a grudging fashion. That would be “what to teach.”
to frustrate its purpose and nullify its intent.
Incidentally, the school not only has the right but
While it is true that an institution of learning has a the duty to develop discipline in its students. The
contractual obligation to afford its students a fair Constitution no less imposes such duty.
opportunity to complete the course they seek to pursue,
since a contract creates reciprocal rights and obligations, [All educational institutions] shall inculcate
the obligation of the school to educate a student would patriotism and nationalism, foster love of
imply a corresponding obligation on the part of the humanity, respect for human rights, appreciation
student to study and obey the rules and regulations of the of the role of national heroes in the historical
school. When a student commits a serious breach of development of the country, teach the rights and
discipline or failed to maintain the required academic duties of citizenship, strengthen ethical and
standard, he forfeits his contractual right. In this spiritual values, develop moral character and
connection, this Court recognizes the expertise of personal discipline, encourage critical and creative
educational institutions in the various fields of learning. thinking, broaden scientific and technological
Thus, they are afforded ample discretion to formulate knowledge, and promote vocational efficiency
reasonable rules and regulations in the admission of (Section 3[2], Article XIV, Constitution).
students, including setting of academic standards. Within
the parameters thereof, they are competent to determine In Angeles v. Sison, we also said that discipline was a
who are entitled to admission and re-admission. means for the school to carry out its responsibility to help

18
its students “grow and develop into mature, responsible, Aside from envisioning a trade policy based on
effective and worthy citizens of the community.” “equality and reciprocity,” the fundamental law
encourages industries that are “competitive in both
Finally, nowhere in the above formulation is the domestic and foreign markets,” thereby demonstrating a
right to discipline more evident than in “who may be clear policy against a sheltered domestic trade
admitted to study.” If a school has the freedom to environment, but one in favor of the gradual development
determine whom to admit, logic dictates that it also has of robust industries that can compete with the best in the
the right to determine whom to exclude or expel, as well foreign markets. (Tanada v. Angara, 272 SCRA 18
as upon whom to impose lesser sanctions such as [1997])
suspension and the withholding of graduation privileges.
27. Is PHILSECO (Philippine Shipyard and Engineering
Thus, in Ateneo de Manila v. Capulong (222 SCRA Corporation), as a shipyard, a public utility and,
643 [1993]), the Court upheld the expulsion of students hence, could be operated only by a corporation at
found guilty of hazing by petitioner therein, holding that: least 60% of whose capital is owned by Filipino
citizens in accordance with Article XII, Section 10 of
No one can be so myopic as to doubt that the Constitution?
the immediate reinstatement of respondent
students who have been investigated and found Held: Petitioner asserts that a shipyard is a
guilty by the Disciplinary Board to have violated public utility pursuant to Section 13 (b) of Commonwealth
petitioner university’s disciplinary rules and Act No. 146. Respondents, on the other hand, contend
standards will certainly undermine the authority of that shipyards are no longer public utilities by express
the administration of the school. This we would provision of Presidential Decree No. 666, which provided
be most loathe to do. incentives to the shipbuilding and ship repair industry.

More importantly, it will seriously impair Indeed, P.D. No. 666 dated March 5, 1975
petitioner university’s academic freedom which explicitly stated that a “shipyard” was not a “public utility.”
has been enshrined in the 1935, 1973 and the xxx
present 1987 Constitution.
(Miriam College Foundation, Inc. v. Court of However, Section 1 of P.D. No. 666 was expressly
Appeals, 348 SCRA 265, Dec. 15, 2000, 1st Div. repealed by Section 20 of Batas Pambansa Blg. 391, the
[Kapunan]) Investment Incentive Policy Act of 1983. Subsequently,
Executive Order No. 226, the Omnibus Investments Code
25. May a university validly revoke a degree or honor it of 1987, was issued and Section 85 thereof expressly
has conferred to a student after the graduation of the repealed B.P. Blg. 391.
latter after finding that such degree or honor was
obtained through fraud? The express repeal of B.P. Blg. 391 by E.O. No.
226 did not revive Section 1 of P.D. No. 666, declassifying
Held: In Garcia v. Faculty Admission Committee, the shipbuilding and ship repair industry as a public utility,
Loyola School of Theology (68 SCRA 277 [1975]), the SC as said executive order did not provide otherwise. When
pointed out that academic freedom of institutions of a law which expressly repeals a prior law is itself repealed,
higher learning is a freedom granted to “institutions of the law first repealed shall not be thereby revived unless
higher learning” which is thus given a “wide sphere of expressly so provided (Administrative Code of 1987, Book
authority certainly extending to the choice of students.” If I, Chapter 5, Section 21). Consequently, when the APT
such institution of higher learning can decide who can and [Asset Privatization Trust] drafted the ASBR [Asset
who cannot study in it, it certainly can also determine on Specific Bidding Rules] sometime in 1993, P.D. No. 666 no
whom it can confer the honor and distinction of being its longer existed in our statute books. While it is true that
graduates. the repeal of a statute does not operate to impair rights
that have become vested or accrued while the statute was
Where it is shown that the conferment of an in force, there are no vested rights of the parties that
honor or distinction was obtained through fraud, a should be protected in the case at bar. The reason is
university has the right to revoke or withdraw the honor simple: said decree was already inexistent when the ASBR
or distinction it has thus conferred. This freedom of a was issued.
university does not terminate upon the “graduation” of a
student, for it is precisely the “graduation” of such a A shipyard such as PHILSECO being a public utility
student that is in question. (UP Board of Regents v. as provided by law, the following provision of the Article
Hon. Court of Appeals and Arokiaswamy William XII of the Constitution applies:
Margaret Celine, G.R. No. 134625, Aug. 31, 1999,
2nd Div. [Mendoza]) “Sec. 11. No franchise, certificate, or any
other form of authorization for the operation of a
Economic Policy public utility shall be granted except to citizens of
the Philippines or to corporations or associations
26. Does the Constitutional policy of a “self-reliant and organized under the laws of the Philippines at
independent national economy” rule out foreign least sixty per centum of whose capital is owned
competition? by such citizens, nor shall such franchise,
certificate, or authorization be exclusive in
Held: The constitutional policy of a “self-reliant character or for a longer period than fifty years.
and independent national economy” does not necessarily Neither shall any such franchise or right be
rule out the entry of foreign investments, goods and granted except under the condition that it shall be
services. It contemplates neither “economic seclusion” subject to amendment, alteration, or repeal by
nor “mendicancy in the international community.” the Congress when the common good so requires.
The State shall encourage equity participation in

19
public utilities by the general public. The Section 1. The Congress shall give the
participation of foreign investors in the governing highest priority to the enactment of measures that
body of any public utility enterprise shall be protect and enhance the right of all the people to
limited to their proportionate share in its capital, human dignity, reduce social, economic and
and all the executive and managing officers of political inequalities, and remove cultural
such corporation or association shall be citizens of inequalities by equitably diffusing wealth and
the Philippines.” political power for the common good.

The progenitor of this constitutional provision, To this end, the State shall regulate the
Article XIV, Section 5 of the 1973 Constitution, required acquisition, ownership, use and disposition of
the same proportion of 60%-40% capitalization. The JVA property and its increments. (Article XIII of the
[Joint Venture Agreement] between NIDC [National Constitution, entitled Social Justice and Human
Investment and Development Corporation] and Kawasaki Rights)
[Kawasaki Heavy Industries, Ltd. of Kobe, Japan] entered
into on January 27, 1977 manifests the intention of the Section 6. The State shall apply the
parties to abide by the constitutional mandate on principles of agrarian reform or stewardship,
capitalization of public utilities. x x x whenever applicable in accordance with law, in
the disposition and utilization of other natural
A joint venture is an association of persons or resources, including lands of the public domain
companies jointly undertaking some commercial under lease or concession, subject to prior rights,
enterprise with all of them generally contributing assets homestead rights of small settlers, and the rights
and sharing risks. x x x. Considered more of a of indigenous communities to their ancestral
partnership, a joint venture is governed by the laws on lands. (Ibid.)
contracts and on partnership. The joint venture created
between NIDC and Kawasaki falls within the purview of an Section 17. The State shall recognize,
“association” pursuant to Section 5 of Article XIV of the respect, and protect the rights of cultural
1973 Constitution and Section 11 of Article XII of the 1987 communities to preserve and develop their
Constitution. Consequently, a joint venture that would cultures, traditions, and institutions. It shall
engage in the business of operating a public utility, such consider these rights in the formulation of
as a shipyard, must observe the proportion of 60%-40% national plans and policies. (Article XIV of the
Filipino-foreign capitalization. (JG Summit Holdings, Constitution, entitled Education, Science,
Inc. v. Court of Appeals, 345 SCRA 143, Nov. 20, Technology, Arts, Culture, and Sports)
2000, 1st Div. [Ynares-Santiago])
Section 12. The Congress may create a
The Rights of Indigenous Cultural consultative body to advise the President on
Communities/Indigenous Peoples policies affecting indigenous cultural communities,
the majority of the members of which shall come
28. Enumerate the Constitutional provisions recognizing from such communities. (Article XVI of the
and protecting the rights and interests of the Constitution, entitled General Provisions)
indigenous peoples. (Separate Opinion, Kapunan, J., in Isagani Cruz v.
Secretary of Environment and Natural Resources,
Held: The framers of the 1987 Constitution, et al., G.R. No. 135385, Dec. 6, 2000, En Banc)
looking back to the long destitution of our less fortunate
brothers, fittingly saw the historic opportunity to actualize 29. Discuss the Indigenous Peoples Rights Act (R.A. No.
the ideals of people empowerment and social justice, and 8371).
to reach out particularly to the marginalized sectors of
society, including the indigenous peoples. They Held: Republic Act No. 8371 is entitled "An Act
incorporated in the fundamental law several provisions to Recognize, Protect and Promote the Rights of
recognizing and protecting the rights and interests of the Indigenous Cultural Communities/Indigenous Peoples,
indigenous peoples, to wit: Creating a National Commission on Indigenous Peoples,
Establishing Implementing Mechanisms, Appropriating
Section 22. The State recognizes and Funds Therefor, and for Other Purposes." It is simply
promotes the rights of indigenous peoples within known as "The Indigenous Peoples Rights Act of 1997" or
the framework of national unity and development. the IPRA.
(Article II of the Constitution, entitled State
Principles and Policies) The IPRA recognizes the existence of the
indigenous cultural communities or indigenous peoples
Section 5. The State, subject to the (ICCs/IPs) as a distinct sector in Philippine society. It
provisions of the Constitution and national grants these people the ownership and possession of their
development policies and programs, shall protect ancestral domains and ancestral lands, and defines the
the rights of indigenous cultural communities to extent of these lands and domains. The ownership given
their ancestral lands to ensure their economic, is the indigenous concept of ownership under customary
social, and cultural well-being. law which traces its origin to native title.

The Congress may provide for the Xxx


applicability of customary laws governing property Within their ancestral domains and ancestral
rights and relations in determining the ownership lands, the ICCs/IPs are given the right to self-governance
and extent of ancestral domains. (Article XII of and empowerment (Sections 13 to 20), social justice and
the Constitution, entitled National Economy and human rights (Sections 21 to 28), the right to preserve
Patrimony) and protect their culture, traditions, institutions and
community intellectual rights, and the right to develop

20
their own sciences and technologies (Sections 29 to 37). ancestral domains. (Separate Opinion, Puno, J., in
(Separate Opinion, Puno, J., in Isagani Cruz v. Isagani Cruz v. Secretary of DENR, et al., G.R. No.
Secretary of DENR, et al., G.R. No. 135385, Dec. 6, 135385, Dec. 6, 2000, En Banc)
2000, En Banc)
31. Define “ancestral domains” and “ancestral lands.” Do
30. Define "indigenous peoples/indigenous cultural they constitute part of the land of the public domain?
communities."
Held: Ancestral domains and ancestral lands are
Held: 1. Drawing inspiration from both our the private property of indigenous peoples and do not
fundamental law and international law, IPRA now employs constitute part of the land of the public domain.
the politically-correct conjunctive term “indigenous
peoples/indigenous cultural communities” as follows: The IPRA grants to ICCs/IPs a distinct kind of
ownership over ancestral domains and ancestral lands.
Section 3. Definition of Terms. - For Ancestral lands are not the same as ancestral domains.
purposes of this Act, the following terms shall These are defined in Section 3(a) and (b) of the
mean: Indigenous Peoples Rights Act x x x.

(i) Indigenous peoples/Indigenous cultural communities. - Ancestral domains are all areas belonging to
refer to a group of people or homogenous societies ICCs/IPs held under a claim of ownership, occupied or
identified by self-ascription and ascription by others, possessed by ICCs/IPs by themselves or through their
who have continuously lived as organized community ancestors, communally or individually since time
on communally bounded and defined territory, and immemorial, continuously until the present, except when
who have, under claims of ownership since time interrupted by war, force majeure or displacement by
immemorial, occupied, possessed and utilized such force, deceit, stealth or as a consequence of government
territories, sharing common bonds of language, projects or any other voluntary dealings with government
customs, traditions, and other distinctive cultural traits, and/or private individuals or corporations. Ancestral
or who have, through resistance to political, social and domains comprise lands, inland waters, coastal areas, and
cultural inroads of colonization, non-indigenous natural resources therein and includes ancestral lands,
religions and cultures, became historically forests, pasture, residential, agricultural, and other lands
differentiated from the majority of Filipinos. individually owned whether alienable or not, hunting
Indigenous peoples shall likewise include peoples who grounds, burial grounds, worship areas, bodies of water,
are regarded as indigenous on account of their descent mineral and other natural resources. They also include
from the populations which inhabited the country at lands which may no longer be exclusively occupied by
the time of conquest or colonization, or at the time of ICCs/IPs but from which they traditionally had access to
inroads of non-indigenous religions and cultures, or the for their subsistence and traditional activities, particularly
establishment of present State boundaries, who retain the home ranges of ICCs/IPs who are still nomadic and/or
some or all of their own social, economic, cultural and shifting cultivators (Section 3[a], IPRA).
political institutions, but who may have been displaced
from their traditional domains or who may have Ancestral lands are lands held by the ICCs/IPs
resettled outside their ancestral domains x x x. under the same conditions as ancestral domains except
(Separate Opinion, Kapunan, J., in Isagani Cruz that these are limited to lands and that these lands are
v. Secretary of Environment and Natural not merely occupied and possessed but are also utilized
Resources, et al., G.R. No. 135385, Dec. 6, 2000, En by the ICCs/IPs under claims of individual or traditional
Banc) group ownership. These lands include but are not limited
to residential lots, rice terraces or paddies, private forests,
2. The IPRA is a law dealing with a specific group swidden farms and tree lots (Section 3[b], IPRA).
of people, i.e., the Indigenous Cultural Communities (Separate Opinion, Puno, J., in Isagani Cruz v.
(ICCs) or the Indigenous Peoples (IPs). The term “ICCs” Secretary of DENR, et al., G.R. No. 135385, Dec. 6,
is used in the 1987 Constitution while that of “IPs” is the 2000, En Banc)
contemporary international language in the International
Labor Organization (ILO) Convention 169 and the United 32. How may ICCs/IPs acquire rights to their ancestral
Nations (UN) Draft Declaration on the Rights of domains and ancestral lands?
Indigenous Peoples.
Held: The rights of the ICCs/IPs to their
Indigenous Cultural Communities or Indigenous ancestral domains and ancestral lands may be acquired in
Peoples refer to a group of people or homogeneous two modes: (1) by native title over both ancestral lands
societies who have continuously lived as an organized and domains; or (2) by torrens title under the Public Land
community on communally bounded and defined territory. Act and the Land Registration Act with respect to
These groups of people have actually occupied, possessed ancestral lands only. (Separate Opinion, Puno, J., in
and utilized their territories under claim of ownership Isagani Cruz v. Secretary of DENR, et al., G.R. No.
since time immemorial. They share common bonds of 135385, Dec. 6, 2000, En Banc)
language, customs, traditions and other distinctive cultural
traits, or, they, by their resistance to political, social and 33. What is the concept of “native title?” What is a
cultural inroads of colonization, non-indigenous religions Certificate of Ancestral Domain Title (CADT)?
and cultures, became historically differentiated from the
Filipino majority. ICCs/IPs also include descendants of Held: Native title refers to ICCs/IPs preconquest
ICCs/IPs who inhabited the country at the time of rights to lands and domains held under a claim of private
conquest or colonization, who retain some or all of their ownership as far back as memory reaches. These lands
own social, economic, cultural and political institutions but are deemed never to have been public lands and are
who may have been displaced from their traditional indisputably presumed to have been held that way since
territories or who may have resettled outside their before the Spanish Conquest. The rights of ICCs/IPs to

21
their ancestral domains (which also include ancestral The theory of the feudal system was that title to
lands) by virtue of native title shall be recognized and all lands was originally held by the King, and while the use
respected (Section 11, IPRA). Formal recognition, when of lands was granted out to others who were permitted to
solicited by ICCs/IPs concerned, shall be embodied in a hold them under certain conditions, the King theoretically
Certificate of Ancestral Domain Title (CADT), which shall retained the title. By fiction of law, the King was regarded
recognize the title of the concerned ICCs/IPs over the as the original proprietor of all lands, and the true and
territories identified and delineated. only source of title, and from him all lands were held. The
theory of jura regalia was therefore nothing more than a
Like a torrens title, a CADT is evidence of private natural fruit of conquest.
ownership of land by native title. Native title, however, is
a right of private ownership peculiarly granted to ICCs/IPs The Regalian theory, however, does not negate
over their ancestral lands and domains. The IPRA native title to lands held in private ownership since time
categorically declares ancestral lands and domains held by immemorial. In the landmark case of Carino v. Insular
native title as never to have been public land. Domains Government (41 Phil. 935, 212 U.S. 449, 53 L. Ed. 594
and lands held under native title are, therefore, [1909]), the United States Supreme Court, reversing the
indisputably presumed to have never been public lands decision of the pre-war Philippine Supreme Court, made
and are private. the following pronouncement:

The concept of native title in the IPRA was taken x x x Every presumption is and ought to
from the 1909 case of Carino v. Insular Government (41 be taken against the Government in a case like the
Phil. 935 [1909], 212 U.S. 449, 53 L. Ed. 594). Carino present. It might, perhaps, be proper and sufficient to
firmly established a concept of private land title that say that when, as far back as testimony or memory goes,
existed irrespective of any royal grant from the State. the land has been held by individuals under a claim of
(Separate Opinion, Puno, J., in Isagani Cruz v. private ownership, it will be presumed to have been held
Secretary of DENR, et al., G.R. No. 135385, Dec. 6, in the same way from before the Spanish conquest, and
2000, En Banc) never to have been public land. x x x (Carino v. Insular
Government, supra note 75, at 941)
34. Distinguish ownership of land under native title and
ownership by acquisitive prescription against the The above ruling institutionalized the recognition
State. of the existence of native title to land, or ownership of
land by Filipinos by virtue of possession under a claim of
Held: Ownership by virtue of native title ownership since time immemorial and independent of any
presupposes that the land has been held by its possessor grant from the Spanish Crown, as an exception to the
and his predecessor-in-interest in the concept of an owner theory of jura regalia.
since time immemorial. The land is not acquired from the
State, that is, Spain or its successor-in-interest, the United Xxx
States and the Philippine Government. There has been no
transfer of title from the State as the land has been Carino was decided by the U.S. Supreme Court in
regarded as private in character as far back as memory 1909, at a time when decisions of the U.S. Court were
goes. In contrast, ownership of land by acquisitive binding as precedent in our jurisdiction (Section 10,
prescription against the State involves a conversion of the Philippine Bill of 1902). We applied the Carino doctrine in
character of the property from alienable public land to the 1946 case of Oh Cho v. Director of Lands (75 Phil. 890
private land, which presupposes a transfer of title from [1946]), where we stated that “[a]ll lands that were not
the State to a private person. (Separate Opinion, acquired from the Government either by purchase or by
Kapunan, J., in Isagani Cruz v. Secretary of DENR, grant, belong to the public domain, but [a]n exception to
G.R. No. 135385, Dec. 6, 2000, En Banc) the rule would be any land that should have been in the
possession of an occupant and of his predecessors in
35. Discuss the concept of “jura regalia” and how it interest since time immemorial, for such possession would
evolved in the Philippines. Does it negate native title justify the presumption that the land had never been part
to lands held in private ownership since time of the public domain or that it had been private property
immemorial? even before the Spanish conquest. (Separate Opinion,
Kapunan, J., in Isagani Cruz v. Secretary of DENR,
Held: Generally, under the concept of jura G.R. No. 135385, Dec. 6, 2000, En Banc)
regalia, private title to land must be traced to some grant,
express or implied, from the Spanish Crown or its 36. Does R.A. 8371, otherwise known as “the Indigenous
successors, the American Colonial government, and People’s Rights Act” infringe upon the State’s
thereafter, the Philippine Republic. The belief that the ownership over the natural resources within the
Spanish Crown is the origin of all land titles in the ancestral domains?
Philippines has persisted because title to land must
emanate from some source for it cannot issue forth from Held: Petitioners posit that IPRA deprives the
nowhere. State of its ownership over mineral lands of the public
domain and other natural resources, as well as the State’s
In its broad sense, the term “jura regalia” refers full control and supervision over the exploration,
to royal grants, or those rights which the King has by development and utilization of natural resources.
virtue of his prerogatives. In Spanish law, it refers to a Specifically, petitioners and the Solicitor General assail
right which the sovereign has over anything in which a Sections 3[a], 5, and 7 of IPRA as violative of Section 2,
subject has a right of property or propriedad. These were Article XII of the Constitution which states, in part, that
rights enjoyed during feudal times by the king as the “[a]ll lands of the public domain, waters, minerals, coal,
sovereign. petroleum, and other mineral oils, all forces of potential
energy, fisheries, forests or timber, wildlife, flora and
fauna, and other natural resources are owned by the

22
State.” (Section 2, Article XII, Constitution) They would therein are undertaken. (Separate Opinion, Kapunan,
have the Court declare as unconstitutional Section 3[a] of J., in Cruz v. Secretary of Environment and Natural
IPRA because the inclusion of natural resources in the Resources, 347 SCRA 128, 284-293, Dec. 6, 2000,
definition of ancestral domains purportedly results in the En Banc [Per Curiam])
abdication of State ownership over these resources.
37. Has the concept of native title to natural resources,
Xxx like native title to land, been recognized in the
Philippines?
Section 3[a] merely defines the coverage of
ancestral domains, and describes the extent, limit and Held: The concept of native title to natural
composition of ancestral domains by setting forth the resources, unlike native title to land, has not been
standards and guidelines in determining whether a recognized in the Philippines. NCIP and Flavier, et al.
particular area is to be considered as part of and within invoke the case of Reavies v. Fianza (40 Phil. 1017
the ancestral domains. In other words, Section 3[a] [1909], 215 US 16, 54 L Ed 72) in support of their thesis
serves only as a yardstick which points out what that native title to natural resources has been upheld in
properties are within the ancestral domains. It does not this jurisdiction. X x x. However, a judicious examination
confer or recognize any right of ownership over the of Reavies reveals that, contrary to the position of NCIP
natural resources to the indigenous peoples. Its purpose and Flavier, et al., the Court did not recognize native title
is definitional and not declarative of a right or title. to natural resources. Rather, it merely upheld the right of
the indigenous peoples to claim ownership of minerals
The specification of what areas belong to the under the Philippine Bill of 1902.
ancestral domains is x x x important to ensure that no
unnecessary encroachment on private properties outside While x x x native title to land or private
the ancestral domains will result during the delineation ownership by Filipinos of land by virtue of time
process. The mere fact that Section 3[a] defines immemorial possession in the concept of an owner was
ancestral domains to include the natural resources found acknowledged and recognized as far back during the
therein does not ipso facto convert the character of such Spanish colonization of the Philippines, there was no
natural resources as private property of the indigenous similar favorable treatment as regards natural resources.
peoples. Similarly, Section 5 in relation to Section 3[a] (Separate Opinion, Kapunan, J., in Cruz v.
cannot be construed as a source of ownership rights of Secretary of Environment and Natural Resources,
indigenous peoples over the natural resources simply 347 SCRA 128, 284-293, Dec. 6, 2000, En Banc [Per
because it recognizes ancestral domains as their “private Curiam])
but community property.”
38. What is the underlying reason for the State’s
The phrase “private but community property” is consistent assertion of ownership and control over
merely descriptive of the indigenous peoples’ concept of natural resources from the Spanish regime up to the
ownership as distinguished from that provided in the Civil present?
Code. x x x. In contrast, the indigenous peoples’ concept
of ownership emphasizes the importance of communal or Held: The unique value of natural resources has
group ownership. By virtue of the communal character of been acknowledged by the State and is the underlying
ownership, the property held in common “cannot be sold, reason for its consistent assertion of ownership and
disposed or destroyed” because it was meant to benefit control over said natural resources from the Spanish
the whole indigenous community and not merely the regime up to the present. Natural resources, especially
individual member. minerals, were considered by Spain as an abundant
source of revenue to finance its battle in wars against
That IPRA is not intended to bestow ownership other nations. Hence, Spain, by asserting its ownership
over natural resources to the indigenous peoples is also over minerals wherever these may be found, whether in
clear from the deliberations of the bicameral conference public or private lands, recognized the separability of title
committee on Section 7 which recites the rights of over lands and that over minerals which may be found
indigenous peoples over their ancestral domains x x x. therein.

Further, Section 7 makes no mention of any right On the other hand, the United States viewed
of ownership of the indigenous peoples over the natural natural resources as a source of wealth for its nationals.
resources. In fact, Section 7[a] merely recognizes the As the owner of natural resources over the Philippines
“right to claim ownership over lands, bodies of water after the latter’s cession from Spain, the United States
traditionally and actually occupied by indigenous peoples, saw it fit to allow both Filipino and American citizens to
sacred places, traditional hunting and fishing grounds, and explore and exploit minerals in public lands, and to grant
all improvements made by them at any time within the patents to private mineral lands. x x x. Although the
domains.” Neither does Section 7[b], which enumerates United States made a distinction between minerals found
certain rights of the indigenous peoples over the natural in public lands and those found in private lands, title in
resources found within their ancestral domains, contain these minerals was in all cases sourced from the State.
any recognition of ownership vis-à-vis the natural The framers of the 1935 Constitution found it necessary to
resources. maintain the State’s ownership over natural resources to
insure their conservation for future generations of
What is evident is that the IPRA protects the Filipinos, to prevent foreign control of the country through
indigenous peoples’ rights and welfare in relation to the economic domination; and to avoid situations whereby the
natural resources found within their ancestral domains, Philippines would become a source of international
including the preservation of the ecological balance conflicts, thereby posing danger to its internal security
therein and the need to ensure that the indigenous and independence. (Separate Opinion, Kapunan, J.,
peoples will not be unduly displaced when the State- in Cruz v. Secretary of Environment and Natural
approved activities involving the natural resources located

23
Resources, 347 SCRA 128, 284-293, Dec. 6, 2000, dismissal or pursuance lies within the full discretion and
En Banc [Per Curiam]) control of the judge. In a criminal case, the manner in
which the prosecution is handled, including the matter of
39. What was the basis for the early Spanish decrees whom to present as witnesses, may lie within the sound
embracing the theory of jura regalia? Is this also the discretion of the government prosecutor; but the court
basis of the declaration in Section 2, Article XII of the decides, based on the evidence proffered, in what manner
1987 Constitution that all lands of the public domain it will dispose of the case. Jurisdiction, once acquired by
are owned by the State? Consequently, did Spain the trial court, is not lost despite a resolution, even by the
acquire title over all lands in the Philippines in the 16 th justice secretary, to withdraw the information or to
century? dismiss the complaint. The prosecution’s motion to
withdraw or to dismiss is not the least binding upon the
Held: Dominium was the basis for the early court. On the contrary, decisional rules require the trial
Spanish decrees embracing the theory of jura regalia. court to make its own evaluation of the merits of the case,
The declaration in Section 2, Article XII of the 1987 because granting such motion is equivalent to effecting a
Constitution that all lands of the public domain are owned disposition of the case itself.
by the State is likewise founded on dominium. If
dominium, not imperium, is the basis of the theory of jura Thus, the PCGG, as the government prosecutor of
regalia, then the lands which Spain acquired in the 16th ill-gotten wealth cases, cannot guarantee the dismissal of
century were limited to non-private lands, because it all such criminal cases against the Marcoses pending in
could only acquire lands which were not yet privately- the courts, for said dismissal is not within its sole power
owned or occupied by the Filipinos. Hence, Spain and discretion. (Chavez v. PCGG, 299 SCRA 744,
acquired title only over lands which were unoccupied and Dec. 9, 1998 [Panganiban])
unclaimed, i.e., public lands. (Separate Opinion,
Kapunan, J., in Isagani Cruz v. Secretary of DENR,
G.R. No. 135385, Dec. 6, 2000, En Banc, See Delegation of Powers
Footnote 86)
42. What are the tests of a valid delegation of power?

The Right of the State to Recover Properties Held: [I]n every case of permissible delegation,
Unlawfully Acquired by Public Officials or there must be a showing that the delegation itself is valid.
Employees It is valid only if the law (a) is complete in itself, setting
forth therein the policy to be executed, carried out, or
40. Does the right of the State to recover properties implemented by the delegate; and (b) fixes a standard –
unlawfully acquired by public officials or employees the limits of which are sufficiently determinate and
which may not be barred by prescription, laches, or determinable – to which the delegate must conform in the
estoppel under Section 15, Article XI of the performance of his functions. A sufficient standard is one
Constitution apply to criminal cases for the recovery of which defines legislative policy, marks its limits, maps out
ill-gotten wealth? its boundaries and specifies the public agency to apply it.
It indicates the circumstances under which the legislative
Held: Section 15, Article XI, 1987 Constitution command is to be effected. (Santiago v. COMELEC,
provides that “[T]he right of the State to recover 270 SCRA 106, March 19, 1997)
properties unlawfully acquired by public officials or
employees, from them or from their nominees as
transferees, shall not be barred by prescription, laches, or The Legislative Department
estoppel.” From the proceedings of the Constitutional
Commission of 1986, however, it was clear that this 43. Discuss the nature of the Party-List system. Is it,
provision applies only to civil actions for recovery of ill- without any qualification, open to all?
gotten wealth, and not to criminal cases. Thus, the
prosecution of offenses arising from, relating or incident Held: 1. The party-list system is a social justice
to, or involving ill-gotten wealth contemplated in Section tool designed not only to give more law to the great
15, Article XI of the Constitution may be barred by masses of our people who have less in life, but also to
prescription. (Presidential Ad Hoc Fact-Finding enable them to become veritable lawmakers themselves,
Committee on Behest Loans, et al. v. Hon. Aniano empowered to participate directly in the enactment of
A. Desierto, et al., G.R. No. 130140, Oct. 25, 1999, laws designed to benefit them. It intends to make the
En Banc [Davide, C.J.]) marginalized and the underrepresented not merely
passive recipients of the State’s benevolence, but active
participants in the mainstream of representative
STRUCTURE OF GOVERNMENT democracy. Thus, allowing all individuals and groups,
including those which now dominate district elections, to
The Doctrine of Separation of Powers have the same opportunity to participate in party-list
elections would desecrate this lofty objective and
41. May the Government, through the PCGG, validly bind mongrelize the social justice mechanism into an atrocious
itself to cause the dismissal of all cases against the veneer for traditional politics. (Ang Bagong Bayani –
Marcos heirs pending before the Sandiganbayan and OFW Labor Party v. COMELEC, G.R. No. 147589,
other courts in a Compromise Agreement entered into June 26, 2001, En Banc [Panganiban])
between the former and the latter?
2. Crucial to the resolution of this case is the
Held: This is a direct encroachment on judicial fundamental social justice principle that those who have
power, particularly in regard to criminal jurisdiction. Well- less in life should have more in law. The party-list system
settled is the doctrine that once a case has been filed is one such tool intended to benefit those who have less
before a court of competent jurisdiction, the matter of its in life. It gives the great masses of our people genuine

24
hope and genuine power. It is a message to the destitute
and the prejudiced, and even to those in the 45. Is the enumeration of marginalized and
underground, that change is possible. It is an invitation underrepresented sectors to be represented under the
for them to come out of their limbo and seize the party-list system in RA 7941 exclusive? Will it be
opportunity. correct to assert that the party-list system is not
exclusive to the marginalized and underrepresented
Clearly, therefore, the Court cannot accept the sectors, but that even the super-rich and
submissions x x x that the party-list system is, without any overrepresented can validly participate in party-list
qualification, open to all. Such position does not only elections?
weaken the electoral chances of the marginalized and
underrepresented; it also prejudices them. It would gut Held: While the enumeration of marginalized and
the substance of the party-list system. Instead of underrepresented sectors is not exclusive, it demonstrates
generating hope, it would create a mirage. Instead of the clear intent of the law that not all sectors can be
enabling the marginalized, it would further weaken them represented under the party-list system. X x x
and aggravate their marginalization. (Ang Bagong
Bayani – OFW Labor Party v. COMELEC, G.R. No. [W]e stress that the party-list system seeks to
147589, June 26, 2001, En Banc [Panganiban]) enable certain Filipino citizens – specifically those
belonging to marginalized and underrepresented sectors,
44. Are political parties – even the major ones – organizations and parties – to be elected to the House of
prohibited from participating in the party-list Representatives. The assertion x x x that the party-list
elections? system is not exclusive to the marginalized and
underrepresented disregards the clear statutory policy.
Held: Under the Constitution and RA 7941, Its claim that even the super-rich and overrepresented
private respondents cannot be disqualified from the party- can participate desecrates the spirit of the party-list
list elections, merely on the ground that they are political system.
parties. Section 5, Article VI of the Constitution, provides
that members of the House of Representatives may “be Indeed, the law crafted to address the peculiar
elected through a party-list system of registered national, disadvantage of Payatas hovel dwellers cannot be
regional, and sectoral parties or organizations. appropriated by the mansion owners of Forbes Park. The
interests of these two sectors are manifestly disparate;
Furthermore, under Sections 7 and 8, Article IX hence, the x x x position to treat them similarly defies
[C] of the Constitution, political parties may be registered reason and common sense. X x x
under the party-list system. X x x
While the business moguls and the mega-rich are,
During the deliberations in the Constitutional numerically speaking, a tiny minority, they are neither
Commission, Comm. Christian S. Monsod pointed out that marginalized nor underrepresented, for the stark reality is
the participants in the party-list system may “be a regional that their economic clout engenders political power more
party, a sectoral party, a national party, UNIDO, awesome than their numerical limitation. Traditionally,
Magsasaka, or a regional party in Mindanao.” x x x. political power does not necessarily emanate from the size
of one’s constituency; indeed, it is likely to arise more
Xxx directly from the number and amount of one’s bank
accounts.
For its part, Section 2 of RA 7941 also provides
for “a party-list system of registered national, regional and It is ironic, therefore, that the marginalized and
sectoral parties or organizations or coalitions thereof, x x underrepresented in our midst are the majority who
x.” Section 3 expressly states that a “party” is “either a wallow in poverty, destitution and infirmity. It was for
political party or a sectoral party or a coalition of parties.” them that the party-list system was enacted – to give
More to the point, the law defines “political party” as “an them not only genuine hope, but genuine power; to give
organized group of citizens advocating an ideology or them opportunity to be elected and to represent the
platform, principles and policies for the general conduct of specific concerns of their constituencies; and simply to
government and which, as the most immediate means of give them a direct vote in Congress and in the larger
securing their adoption, regularly nominates and supports affairs of the State. In its noblest sense, the party-list
certain of its leaders and members as candidates for system truly empowers the masses and ushers a new
public office.” hope for genuine change. Verily, it invites those
marginalized and underrepresented in the past – the farm
Furthermore, Section 11 of RA 7941 leaves no hands, the fisher folk, the urban poor, even those in the
doubt as to the participation of political parties in the underground movement – to come out and participate, as
party-list system. X x x indeed many of them came out and participated during
the last elections. The State cannot now disappoint and
Indubitably, therefore, political parties – even the frustrate them by disabling the desecrating this social
major ones – may participate in the party-list elections. justice vehicle.

That political parties may participate in the party- Because the marginalized and underrepresented
list elections does not mean, however, that any political had not been able to win in the congressional district
party – or any organization or group for that matter – may elections normally dominated by traditional politicians and
do so. The requisite character of these parties or vested groups, 20 percent of the seats in the House of
organizations must be consistent with the purpose of the Representatives were set aside for the party-list system.
party-list system, as laid down in the Constitution and RA In arguing that even those sectors who normally
7941. X x x (Ang Bagong Bayani – OFW Labor Party controlled 80 percent of the seats in the House could
v. COMELEC, G.R. No. 147589, June 26, 2001, En participate in the party-list elections for the remaining 20
Banc [Panganiban]) percent, the OSG and the Comelec disregard the

25
fundamental difference between the congressional district Considering the foregoing statutory requirements,
elections and the party-list elections. it will be shown x x x that Section 5(2), Article VI of the
Constitution is not mandatory. It merely provides a
As earlier noted, the purpose of the party-list ceiling for party-list seats in Congress. (Veterans
provision was to open up the system, in order to enhance Federation Party v. COMELEC, G.R. No. 136781,
the chance of sectoral groups and organizations to gain Oct. 6, 2000, En Banc [Panganiban])
representation in the House of Representatives through
the simplest scheme possible. Logic shows that the 47. What are the inviolable parameters to determine the
system has been opened to those who have never gotten winners in a Philippine-style party-list election?
a foothold within it – those who cannot otherwise win in
regular elections and who therefore need the “simplest Held: To determine the winners in a Philippine-
scheme possible” to do so. Conversely, it would be style party-list election, the Constitution and Republic Act
illogical to open the system to those who have long been No. 7941 mandate at least four inviolable parameters.
within it – those privileged sectors that have long These are:
dominated the congressional district elections.
First, the twenty percent allocation - the
Xxx combined number of all party-list congressmen shall not
exceed twenty percent of the total membership of the
Verily, allowing the non-marginalized and House of Representatives, including those elected under
overrepresented to vie for the remaining seats under the the party list.
party-list system would not only dilute, but also prejudice
the chance of the marginalized and underrepresented, Second, the two percent threshold - only
contrary to the intention of the law to enhance it. The those garnering a minimum of two percent of the total
party-list system is a tool for the benefit of the valid votes cast for the party-list system are "qualified" to
underprivileged; the law could not have given the same have a seat in the House of Representatives.
tool to others, to the prejudice of the intended
beneficiaries. (Ang Bagong Bayani – OFW Labor Third, the three seat limit - each qualified
Party v. COMELEC, G.R. No. 147589, June 26, 2001, party, regardless of the number of votes it actually
En Banc [Panganiban]) obtained, is entitled to a maximum of three seats; that is,
one "qualifying" and two additional seats.
46. Section 5(2), Article VI of the Constitution provides
that “[t]he party-list representatives shall constitute Fourth, proportional representation - the
twenty per centum of the total number of additional seats which a qualified party is entitled to shall
representatives including those under the party-list.” be computed "in proportion to their total number of
Does the Constitution require all such allocated seats votes." (Veterans Federation Party v. COMELEC,
to be filled up all the time and under all G.R. No. 136781 and Companion Cases, Oct. 6,
circumstances? 2000, En Banc [Panganiban])

Held: The Constitution simply states that “[t]he 48. To determine the “total votes cast for the party-list
party-list representatives shall constitute twenty per system,” should the votes tallied for the disqualified
centum of the total number of representatives including candidates be deducted?
those under the party-list.”
Held: The instant Motions for proclamation
Xxx contend that the disqualification of many party-list
organizations has reduced the “total number of votes cast
We rule that a simple reading of Section 5, Article for the party-list elections.” Because of this reduction,
VI of the Constitution, easily conveys the equally simple the two-percent benchmark required by law has now been
message that Congress was vested with the broad power allegedly attained by movants. Hence, they now pray for
to define and prescribe the mechanics of the party-list their proclamation as winners in the last party-list
system of representation. The Constitution explicitly sets elections.
down only the percentage of the total membership in the
House of Representatives reserved for party-list Recall that under Section 11(b) of RA 7941 (the
representatives. Party-List Act), only those parties garnering a minimum of
two percent of the total votes cast for the party-list
In the exercise of its constitutional prerogative, system are entitled to have a seat in the House of
Congress enacted RA 7941. As said earlier, Congress Representatives. The critical question now is this: To
declared therein a policy to promote “proportional determine the “total votes cast for the party-list system,”
representation” in the election of party-list representatives should the votes tallied for the disqualified candidates be
in order to enable Filipinos belonging to the marginalized deducted? Otherwise stated, does the clause “total votes
and underrepresented sectors to contribute legislation cast for the party-list system” include only those ballots
that would benefit them. It however deemed it necessary cast for qualified party-list candidates?
to require parties, organizations and coalitions
participating in the system to obtain at least two percent To answer this question, there is a need to review
of the total votes cast for the party-list system in order to related jurisprudence on the matter, especially Labo v.
be entitled to a party-list seat. Those garnering more Comelec and Grego v. Comelec, which were mentioned in
than this percentage could have “additional seats in our February 18, 2003 Resolution.
proportion to their total number of votes.” Furthermore,
no winning party, organization or coalition can have more Labo and Grego Not Applicable
than three seats in the House of Representatives. X x x
In Labo, the Court declared that “the ineligibility
of a candidate receiving majority votes does not entitle

26
the eligible candidate receiving the next highest number Bayani – OFW Labor Party v. COMELEC, G.R. No.
of votes to be declared elected. A minority or defeated 147589, June 25, 2003, En Banc [Panganiban])
candidate cannot be deemed elected to the office.” In
other words, the votes cast for an ineligible or disqualified 49. State the guidelines for screening Party-List
candidate cannot be considered “stray.” Participants.

However, “this rule would be different if the Held: In this light, the Court finds it appropriate
electorate, fully aware in fact and in law of a candidate’s to lay down the following guidelines, culled from the law
disqualification so as to bring such awareness within the and the Constitution, to assist the Comelec in its work.
realm of notoriety, would nonetheless cast their votes in
favor of the ineligible. In such case, the electorate may First, the political party, sector, organization or
be said to have waived the validity and efficacy of their coalition must represent the marginalized and
votes by notoriously misapplying their franchise or underrepresented groups identified in Section 5 of RA
throwing away their votes, in which case, the eligible 7941. In other words, it must show – through its
candidate obtaining the next higher number of votes may constitution, articles of incorporation, bylaws, history,
be deemed elected.” In short, the votes cast for a platform of government and track record – that it
“notoriously disqualified” candidate may be considered represents and seeks to uplift marginalized and
“stray” and excluded from the canvass. underrepresented sectors. Verily, majority of its
membership should belong to the marginalized and
The foregoing pronouncement was reiterated in underrepresented. And it must demonstrate that in a
Grego, which held that the exception mentioned in Labo conflict of interest, it has chosen or is likely to choose the
v. Comelec “is predicated on the concurrence of two interest of such sectors.
assumptions, namely: 1) the one who obtained the
highest number of votes is disqualified; and 2) the Second, while even major political parties are
electorate is fully aware in fact and in law of a candidate’s expressly allowed by RA 7941 and the Constitution to
disqualification so as to bring such awareness within the participate in the party-list system, they must comply with
realm of notoriety but would nonetheless cast their votes the declared statutory policy of enabling “Filipino citizens
in favor of the ineligible candidate.” belonging to marginalized and underrepresented sectors x
x x to be elected to the House of Representatives.” In
Note, however, that the foregoing other words, while they are not disqualified merely on the
pronouncements (1) referred to regular elections for local ground that they are political parties, they must show,
offices and (2) involved the interpretation of Section 6 of however, that they represent the interests of the
RA 6646. They were not meant to cover party-list marginalized and underrepresented. X x x
elections, which are specifically governed by RA 7941.
Section 10 of this latter law clearly provides that the votes Third, in view of the objections directed against
cast for a party, a sectoral organization or a coalition “not the registration of Ang Buhay Hayaang Yumabong, which
entitled to be voted for shall not be counted”: is allegedly a religious group, the Court notes the express
constitutional provision that the religious sector may not
Xxx be represented in the party-list system. x x x
The language of the law is clear; hence, there is
room, not for interpretation, but merely for application. Furthermore, the Constitution provides that
Likewise, no recourse to extrinsic aids is warranted when “religious denominations and sects shall not be
the language of the law is plain and unambiguous. registered.” (Sec. 2 [5], Article IX [C]) The prohibition
was explained by a member of the Constitutional
Another reason for not applying Labo and Grego Commission in this wise: “[T]he prohibition is on any
is that these cases involve single elective posts, while the religious organization registering as a political party. I do
present controversy pertains to the acquisition of a not see any prohibition here against a priest running as a
number of congressional seats depending on the total candidate. That is not prohibited here; it is the
election results – such that even those garnering second, registration of a religious sect as a political party.”
third, fourth or lesser places could be proclaimed winners
depending on their compliance with other requirements. Fourth, a party or an organization must not be
disqualified under Section 6 of RA 7941, which
RA 7941 is a special statute governing the enumerates the grounds for disqualification as follows:
election of party-list representatives and is the controlling
law in matters pertaining thereto. Since Labo and Section 1) It is a religious sect or denomination,
6 of RA 6646 came into being prior to the enactment of organization or association organized for
RA 7941, the latter is a qualification of the former ruling religious purposes;
and law. On the other hand, Grego and other related 2) It advocates violence or unlawful means to
cases that came after the enactment of RA 7941 should seek its goal;
be construed as inapplicable to the latter. 3) It is a foreign party or organization;
4) It is receiving support from any foreign
Subtracting the votes garnered by these government, foreign political party,
disqualified party-list groups from the total votes cast foundation, organization, whether directly or
under the party-list system will reduce the base figure to through any of its officers or members or
6,523,185. This means that the two-percent threshold indirectly through third parties for partisan
can be more easily attained by the qualified marginalized election purposes;
and under-represented groups. Hence, disregarding the 5) It violates or fails to comply with laws, rules
votes of disqualified party-list participants will increase or regulations relating to elections;
and broaden the number of representatives from these 6) It declares untruthful statements in its
sectors. Doing so will further concretize and give flesh to petition;
the policy declaration in RA 7941 x x x. (Ang Bagong

27
7) It has ceased to exist for at least one (1)
year; or The 1935 Constitution provided in its Article VI on
8) It fails to participate in the last two (2) the Legislative Department:
preceding elections or fails to obtain at least
two per centum (2%) of the votes cast under Sec. 15. The Senators and Members of
the party-list system in the two (2) preceding the House of Representatives shall in all cases
elections for the constituency in which it had except treason, felony, and breach of the peace,
registered.” be privileged from arrest during their attendance
at the sessions of Congress, and in going to and
Note should be taken of paragraph 5, which returning from the same; x x x.
disqualifies a party or group for violation of or failure to
comply with election laws and regulations. These laws Because of the broad coverage of felony and
include Section 2 of RA 7941, which states that the party- breach of the peace, the exemption applied only to civil
list system seeks to “enable Filipino citizens belonging to arrests. A congressman like the accused-appellant,
marginalized and underrepresented sectors, organizations convicted under Title Eleven of the Revised Penal Code
and parties x x x to become members of the House of could not claim parliamentary immunity from arrest. He
Representatives.” A party or organization, therefore, that was subject to the same general laws governing all
does not comply with this policy must be disqualified. persons still to be tried or whose convictions were
pending appeal.
Fifth, the party or organization must not be an
adjunct of, or a project organized or an entity funded or The 1973 Constitution broadened the privilege of
assisted by, the government. By the very nature of the immunity as follows:
party-list system, the party or organization must be a
group of citizens, organized by citizens and operated by Article VIII, Sec. 9. A Member of the
citizens. It must be independent of the government. The Batasang Pambansa shall, in all offenses
participation of the government or its officials in the punishable by not more than six years
affairs of a party-list candidate is not only illegal and imprisonment, be privileged from arrest during his
unfair to other parties, but also deleterious to the attendance at its sessions and in going to and
objective of the law: to enable citizens belonging to returning from the same.
marginalized and underrepresented sectors and
organization to be elected to the House of For offenses punishable by more than six years
Representatives. imprisonment, there was no immunity from arrest. The
restrictive interpretation of immunity and the intent to
Sixth, the party must not only comply with the confine it within carefully defined parameters is illustrated
requirements of the law; its nominees must likewise do by the concluding portion of the provision, to wit:
so. x x x
X x x but the Batasang Pambansa shall
Seventh, not only the candidate party or surrender the member involved to the custody of
organization must represent marginalized and the law within twenty four hours after its
underrepresented sectors; so also must its nominees. To adjournment for a recess or for its next session,
repeat, under Section 2 of RA 7941, the nominees must otherwise such privilege shall cease upon its
be Filipino citizens “who belong to marginalized and failure to do so.
underrepresented sectors, organizations and parties.”
Surely, the interests of the youth cannot be fully The present Constitution adheres to the same
represented by a retiree; neither can those of the urban restrictive rule minus the obligation of Congress to
poor or the working class, by an industrialist. To allow surrender the subject Congressman to the custody of the
otherwise is to betray the State policy to give genuine law. The requirement that he should be attending
representation to the marginalized and underrepresented. sessions or committee meetings has also been removed.
For relatively minor offenses, it is enough that Congress is
Eighth, x x x while lacking a well-defined political in session. (People v. Jalosjos, 324 SCRA 689, Feb.
constituency, the nominee must likewise be able to 3, 2000, En Banc [Ynares-Santiago])
contribute to the formulation and enactment of
appropriate legislation that will benefit the nation as a 51. Accused-appellant Congressman Romeo G. Jalosjos
whole. (Ang Bagong Bayani – OFW Labor Party v. filed a motion before the Court asking that he be
COMELEC, G.R. No. 147589, June 26, 2001, En Banc allowed to fully discharge the duties of a
[Panganiban]) Congressman, including attendance at legislative
sessions and committee meetings despite his having
50. Discuss the history of the constitutional provision been convicted in the first instance of a non-bailable
granting immunity from arrest or detention of offense. He contended that his reelection being an
Members of Congress, and how should it be expression of popular will cannot be rendered inutile
construed? by any ruling, giving priority to any right or interest –
not even the police power of the State. Resolve.
Held: The immunity from arrest or detention of
Senators and members of the House of Representatives x Held: The accused-appellant argues that a
x x arises from a provision of the Constitution. The member of Congress’ function to attend sessions is
history of the provision shows that the privilege has underscored by Section 16(2), Article VI of the
always been granted in a restrictive sense. The provision Constitution which states that –
granting an exemption as a special privilege cannot be
extended beyond the ordinary meaning of its terms. It (2) A majority of each House shall
may not be extended by intendment, implication or constitute a quorum to do business, but a smaller
equitable considerations. number may adjourn from day to day and may

28
compel the attendance of absent Members in such would be a mockery of the aims of the State’s penal
manner, and under such penalties, as such House system.
may provide.
Accused-appellant argues that on several
However, the accused-appellant has not given any occasions, the Regional Trial Court of Makati granted
reason why he should be exempted from the operation of several motions to temporarily leave his cell at the Makati
Section 11, Article VI of the Constitution. The members of City Jail, for official or medical reasons x x x.
Congress cannot compel absent members to attend
sessions if the reason for the absence is a legitimate one. He also calls attention to various instances, after
The confinement of a Congressman charged with a crime his transfer at the New Bilibid Prison in Muntinlupa City,
punishable by imprisonment of more than six years is not when he was likewise allowed/permitted to leave the
merely authorized by law, it has constitutional prison premises x x x.
foundations.
There is no showing that the above privileges are
Accused-appellant’s reliance on the ruling in peculiar to him or to a member of Congress. Emergency
Aguinaldo v. Santos (212 SCRA 768, at 773 [1992]), or compelling temporary leaves from imprisonment are
which states, inter alia, that – allowed to all prisoners, at the discretion of the authorities
or upon court orders.
The Court should never remove a public
officer for acts done prior to his present term of What the accused-appellant seeks is not of an
office. To do otherwise would be to deprive the emergency nature. Allowing accused-appellant to attend
people of their right to elect their officers. When congressional sessions and committee meetings for five
the people have elected a man to office, it must (5) days or more in a week will virtually make him a free
be assumed that they did this with the knowledge man with all the privileges appurtenant to his position.
of his life and character, and that they Such an aberrant situation not only elevates accused-
disregarded or forgave his fault or misconduct, if appellant’s status to that of a special class, it also would
he had been guilty of any. It is not for the Court, be a mockery of the purposes of the correction system. X
by reason of such fault or misconduct, to xx
practically overrule the will of the people.
The accused-appellant avers that his constituents
will not extricate him from his predicament. It can be in the First District of Zamboanga del Norte want their
readily seen x x x that the Aguinaldo case involves the voices to be heard and that since he is treated as bona
administrative removal of a public officer for acts done fide member of the House of Representatives, the latter
prior to his present term of office. It does not apply to urges a co-equal branch of government to respect his
imprisonment arising from the enforcement of criminal mandate. He also claims that the concept of temporary
law. Moreover, in the same way that preventive detention does not necessarily curtail his duty to
suspension is not removal, confinement pending appeal is discharge his mandate and that he has always complied
not removal. He remains a Congressman unless expelled with the conditions/restrictions when he is allowed to
by Congress or, otherwise, disqualified. leave jail.

One rationale behind confinement, whether We remain unpersuaded.


pending appeal or after final conviction, is public self-
defense. Society must protect itself. It also serves as an Xxx
example and warning to others.
When the voters of his district elected the
A person charged with crime is taken into custody accused-appellant to Congress, they did so with full
for purposes of the administration of justice. As stated in awareness of the limitations on his freedom of action.
United States v. Gustilo (19 Phil. 208, 212), it is the injury They did so with the knowledge that he could achieve
to the public which State action in criminal law seeks to only such legislative results which he could accomplish
redress. It is not the injury to the complainant. After within the confines of prison. To give a more drastic
conviction in the Regional Trial Court, the accused may be illustration, if voters elect a person with full knowledge
denied bail and thus subjected to incarceration if there is that he is suffering from a terminal illness, they do so
risk of his absconding. knowing that at any time, he may no longer serve his full
term in office. (People v. Jalosjos, 324 SCRA 689,
The accused-appellant states that the plea of the Feb. 3, 2000, En Banc [Ynares-Santiago])
electorate which voted him into office cannot be
supplanted by unfounded fears that he might escape 52. May the Supreme Court properly inquire into the
eventual punishment if permitted to perform motives of the lawmakers in conducting legislative
congressional duties outside his regular place of investigations? Can it enjoin the Congress or any of
confinement. its regular and special committees from making
inquiries in aid of legislation?
It will be recalled that when a warrant for
accused-appellant’s arrest was issued, he fled and evaded Held: The “allocation of constitutional
capture despite a call from his colleagues in the House of boundaries” is a task that this Court must perform under
Representatives for him to attend the sessions ands to the Constitution. Moreover, as held in a recent case
surrender voluntarily to the authorities. Ironically, it is (Neptali A. Gonzales, et al. v. Hon. Catalino Macaraig, Jr.,
now the same body whose call he initially spurned which et al., G.R. No. 87636, 19 November 1990, 191 SCRA 452,
accused-appellant is invoking to justify his present motion. 463), “[t]he political question doctrine neither interposes
This can not be countenanced because x x x aside from its an obstacle to judicial determination of the rival claims.
being contrary to well-defined Constitutional restraint, it The jurisdiction to delimit constitutional boundaries has
been given to this Court. It cannot abdicate that

29
obligation mandated by the 1987 Constitution, although “the Senate to look into the possible violation of the
said provision by no means does away with the law in the case, particularly with regard to Republic
applicability of the principle in appropriate cases.” Act No. 3019, the Anti-Graft and Corrupt Practices
(Section 1, Article VIII of the 1987 Constitution) Act.”

The Court is thus of the considered view that it On motion of Senator Orlando Mercado, the
has jurisdiction over the present controversy for the matter was referred by the Senate to the Committee
purpose of determining the scope and extent of the power on Accountability of Public Officers (Blue Ribbon
of the Senate Blue Ribbon Committee to conduct inquires Committee). Thereafter, the Senate Blue Ribbon
into private affairs in purported aid of legislation. Committee started its investigation on the matter.
(Bengzon, Jr. v. Senate Blue Ribbon Committee, Petitioners and Ricardo Lopa were subpoenaed by the
203 SCRA 767, Nov. 20, 1991, En Banc [Padilla]) Committee to appear before it and testify on “what
they know” regarding the “sale of the thirty-six (36)
53. Is the power of both houses of Congress to conduct corporations belonging to Benjamin ‘Kokoy’
inquiries in aid of legislation absolute or unlimited? Romualdez.”

Held: The 1987 Constitution expressly At the hearing held on 23 May 1989, Ricardo Lopa
recognizes the power of both houses of Congress to declined to testify on the ground that his testimony
conduct inquiries in aid of legislation (In Arnault v. may “unduly prejudice” the defendants in Civil Case
Nazareno, 87 Phil. 29, this Court held that although there No. 0035 before the Sandiganbayan. Petitioner Jose
was no express provision in the 1935 Constitution giving F.S. Bengzon, Jr. likewise refused to testify invoking
such power to both houses of Congress, it was so his constitutional right to due process, and averring
incidental to the legislative function as to be implied.). that the publicity generated by respondent
Thus, Section 21, Article VI provides x x x. Committee’s inquiry could adversely affect his rights
as well as those of the other petitioners who are his
The power of both houses of Congress to conduct co-defendants in Civil Case No. 0035 before the
inquiries in aid of legislation is not, therefore, absolute or Sandiganbayan.
unlimited. Its exercise is circumscribed by the afore-
quoted provision of the Constitution. Thus, as provided The Senate Blue Ribbon Committee, thereupon,
therein, the investigation must be “in aid of legislation in suspended its inquiry and directed the petitioners to
accordance with its duly published rules of procedure” and file their memorandum on the constitutional issues
that “the rights of persons appearing in or affected by raised, after which, it issued a resolution dated 5 June
such inquiries shall be respected.” It follows then that the 1989 rejecting the petitioners’ plea to be excused
rights of persons under the Bill of Rights must be from testifying, and the Committee voted to pursue
respected, including the right to due process and the right and continue its investigation of the matter. X x x
not to be compelled to testify against one’s self.
Claiming that the Senate Blue Ribbon Committee
The power to conduct formal inquiries or is poised to subpoena and require their attendance
investigations is specifically provided for in Sec. 1 of the and testimony in proceedings before the Committee,
Senate Rules of Procedure Governing Inquiries in Aid of in excess of its jurisdiction and legislative rights, and
Legislation. Such inquiries may refer to the that there is no appeal nor any other plain, speedy
implementation or re-examination of any law or in and adequate remedy in the ordinary course of law,
connection with any proposed legislation or the the petitioners filed the present petition for prohibition
formulation of future legislation. They may also extend to with a prayer for temporary restraining order and/or
any and all matters vested by the Constitution in Congress injunctive relief.
and/or in the Senate alone.
Held: A perusal of the speech of Senator Enrile
As held in Jean L. Arnault v. Leon Nazareno, et al, reveals that he (Senator Enrile) made a statement which
(No. L-3820, July 18, 1950, 87 Phil. 29), the inquiry, to be was published in various newspapers on 2 September
within the jurisdiction of the legislative body making it, 1988 accusing Mr. Ricardo “Baby” Lopa of “having taken
must be material or necessary to the exercise of a power over the FMMC Group of Companies.” X x x
in it vested by the Constitution, such as to legislate or to
expel a member. Verily, the speech of Senator Enrile contained no
suggestion of contemplated legislation; he merely called
Under Sec. 4 of the aforementioned Rules, the upon the Senate to look into a possible violation of Sec. 5
Senate may refer to any committee or committees any of RA No. 3019, otherwise known as “The Anti-Graft and
speech or resolution filed by any Senator which in its Corrupt Practices Act.” In other words, the purpose of the
judgment requires an appropriate inquiry in aid of inquiry to be conducted by respondent Blue Ribbon
legislation. In order therefore to ascertain the character Committee was to find out whether or not the relatives of
or nature of an inquiry, resort must be had to the speech President Aquino, particularly Mr. Ricardo Lopa, had
or resolution under which such an inquiry is proposed to violated the law in connection with the alleged sale of the
be made. (Bengzon, Jr. v. Senate Blue Ribbon 36 or 39 corporations belonging to Benjamin “Kokoy”
Committee, 203 SCRA 767, Nov. 20, 1991, En Banc Romualdez to the Lopa Group. There appears to be,
[Padilla]) therefore, no intended legislation involved.

54. On 13 September 1988, the Senate Minority Floor Xxx


Leader, Hon. Juan Ponce Enrile delivered a speech
“on a matter of personal privilege” before the Senate It appears, therefore, that the contemplated
on the alleged “take-over of SOLOIL Incorporated, the inquiry by respondent Committee is not really “in aid of
flagship on the First Manila Management of legislation” because it is not related to a purpose within
Companies (FMMC) by Ricardo Lopa” and called upon the jurisdiction of Congress, since the aim of the

30
investigation is to find out whether or not the relatives of two distinct bills, H. No. 11197 and S. No. 1630. In
the President or Mr. Ricardo Lopa had violated Section 5 this connection, petitioners point out that although
of RA No. 3019, the “Anti-Graft and Corrupt Practices Art. VI, Sec. 24 was adopted from the American
Act”, a matter that appears more within the province of Federal Constitution, it is notable in two respects: the
the courts rather than of the legislature. Besides, the verb “shall originate” is qualified in the Philippine
Court may take judicial notice that Mr. Ricardo Lopa died Constitution by the word “exclusively” and the phrase
during the pendency of this case. In John T. Watkins v. “as on other bills” in the American version is omitted.
United States, it was held: This means, according to them, that to be considered
as having originated in the House, Republic Act No.
“x x x. The power of Congress to conduct 7716 must retain the essence of H. No. 11197.
inquiries in aid of legislation is inherent in the
legislative process. That power is broad. It Held: This argument will not bear analysis. To
encompasses inquiries concerning the begin with, it is not the law - but the revenue bill - which
administration of existing laws as well as is required by the Constitution to “originate exclusively” in
proposed or possibly needed statutes. It includes the House of Representatives. It is important to
surveys of defects in our social, economic, or emphasize this, because a bill originating in the House
political system for the purpose of enabling may undergo such extensive changes in the Senate that
Congress to remedy them. It comprehends the result may be a rewriting of the whole. The possibility
probes into departments of the Federal of a third version by the conference committee will be
Government to expose corruption, inefficiency or discussed later. At this point, what is important to note is
waste. But broad as is this power of inquiry, it is that, as a result of the Senate action, a distinct bill may be
not unlimited. There is no general authority to produced. To insist that a revenue statute - and not only
expose the private affairs of individuals without the bill which initiated the legislative process culminating
justification in terms of the functions of Congress. in the enactment of the law - must substantially be the
This was freely conceded by the Solicitor General same as the House bill would be to deny the Senate's
in his arguments in this case. Nor is the Congress power not only to “concur with amendments” but also to
a law enforcement or trial agency. These are “propose amendments.” It would be to violate the
functions of the executive and judicial coequality of legislative power of the two houses of
departments of government. No inquiry is an end Congress and in fact make the House superior to the
in itself; it must be related to and in furtherance Senate.
of a legislative task of Congress. Investigations
conducted solely for the personal aggrandizement The contention that the constitutional design is to
of the investigators or to ‘punish’ those limit the Senate's power in respect of revenue bills in
investigated are indefensible.” (italics supplied) order to compensate for the grant to the Senate of the
treaty-ratifying power and thereby equalize its powers and
It cannot be overlooked that when respondent those of the House overlooks the fact that the powers
Committee decided to conduct its investigation of the being compared are different. We are dealing here with
petitioners, the complaint in Civil Case No. 0035 had the legislative power which under the Constitution is
already been filed with the Sandiganbayan. A perusal of vested not only in any particular chamber but in the
that complaint shows that one of its principal causes of Congress of the Philippines, consisting of “a Senate and a
action against herein petitioners, as defendants therein, is House of Representatives.” The exercise of the treaty-
the alleged sale of the 36 (or 39) corporations belonging ratifying power is not the exercise of legislative power. It
to Benjamin “Kokoy” Romualdez. Since the issues in said is the exercise of a check on the executive power. There
complaint had long been joined by the filing of petitioners’ is, therefore, no justification for comparing the legislative
respective answers thereto, the issue sought to be powers of the House and of the Senate on the basis of the
investigated by the respondent Committee is one over possession of a similar non-legislative power by the
which jurisdiction had been acquired by the Senate. The possession of a similar power by the U.S.
Sandiganbayan. In short, the issue has been pre-empted Senate has never been thought of as giving it more
by that court. To allow the respondent Committee to legislative powers than the House of Representatives.
conduct its own investigation of an issue already before
the Sandiganbayan would not only pose the possibility of X x x Given, then, the power of the Senate to
conflicting judgments between a legislative committee and propose amendments, the Senate can propose its own
a judicial tribunal, but if the Committee’s judgment were version even with respect to bills which are required by
to be reached before that of the Sandiganbayan, the the Constitution to originate in the House.
possibility of its influence being made to bear on the
ultimate judgment of the Sandiganbayan can not be It is insisted, however, that S. No. 1630 was
discounted. passed not in substitution of H. No. 11197 but of another
Senate bill (S. No. 1129) earlier filed and that what the
In fine, for the respondent Committee to probe Senate did was merely to “take (H. No. 11197) into
and inquire into the same justiciable controversy already consideration” in enacting S. No. 1630. There is really no
before the Sandiganbayan, would be an encroachment difference between the Senate preserving H. No. 11197
into the exclusive domain of judicial jurisdiction that had up to the enacting clause and then writing its own version
much earlier set in. (Bengzon, Jr. v. Senate Blue following the enacting clause (which, it would seem,
Ribbon Committee, 203 SCRA 767, Nov. 20, 1991, petitioners admit is an amendment by substitution), and,
En Banc [Padilla]) on the other hand, separately presenting a bill of its own
on the same subject matter. In either case the result are
55. Petitioners’ contention is that Republic Act No. 7716 two bills on the same subject.
(The Expanded-VAT Law) did not “originate
exclusively” in the House of Representatives as Indeed, what the Constitution simply means is
required by Art. VI, Sec. 24 of the Constitution, that the initiative for filing revenue, tariff, or tax bills, bills
because it is in fact the result of the consolidation of authorizing an increase of the public debt, private bills and

31
bills of local application must come from the House of that of RA 8189 and that the same is not expressed in
Representatives on the theory that, elected as they are the title of the law. Should the challenge be
from the districts, the members of the House can be sustained?
expected to be more sensitive to the local needs and
problems. On the other hand, the senators, who are Held: Section 44 of RA 8189 is not isolated
elected at large, are expected to approach the same considering that it is related and germane to the subject
problems from the national perspective. Both views are matter stated in the title of the law. The title of RA 8189
thereby made to bear on the enactment of such laws. is "The Voter's Registration Act of 1996" with a subject
matter enunciated in the explanatory note as "AN ACT
Nor does the Constitution prohibit the filing in the PROVIDING FOR A GENERAL REGISTRATION OF VOTERS,
Senate of a substitute bill in anticipation of its receipt of ADOPTING A SYSTEM OF CONTINUING REGISTRATION,
the bill from the House, so long as action by the Senate as PRESCRIBING THE PROCEDURES THEREOF AND
a body is withheld pending receipt of the House bill. The AUTHORIZING THE APPROPRIATION OF FUNDS
Court cannot, therefore, understand the alarm expressed THEREFOR." Section 44, which provides for the
over the fact that on March 1, 1993, eight months before reassignment of election officers, is relevant to the subject
the House passed H. No. 11197, S. No. 1129 had been matter of registration as it seeks to ensure the integrity of
filed in the Senate. After all it does not appear that the the registration process by providing guideline for the
Senate ever considered it. It was only after the Senate COMELEC to follow in the reassignment of election
had received H. No. 11197 on November 23, 1993 that officers. It is not an alien provision but one which is
the process of legislation in respect of it began with the related to the conduct and procedure of continuing
referral to the Senate Committee on Ways and Means of registration of voters. In this regard, it bears stressing
H. No. 11197 and the submission by the Committee on that the Constitution does not require Congress to employ
February 7, 1994 of S. No. 1630. For that matter, if the in the title of an enactment, language of such precision as
question were simply the priority in the time of filing of to mirror, fully index or catalogue, all the contents and the
bills, the fact is that it was in the House that a bill (H. No. minute details therein. (Agripino A. De Guzman, Jr.,
253) to amend the VAT law was first filed on July 22, et al. v. COMELEC, G.R. No. 129118, July 19, 2000,
1992. Several other bills had been filed in the House En Banc [Purisima])
before S. No. 1129 was filed in the Senate, and H. No.
11197 was only a substitute of those earlier bills. 58. Do courts have the power to inquire into allegations
(Tolentino v. Secretary of Finance, 235 SCRA 630, that, in enacting a law, a House of Congress failed to
661-663, Aug. 25, 1994, En Banc [Mendoza]) comply with its own rules?

56. Discuss the objectives of Section 26(1), Article VI of Held: The cases, both here and abroad, in
the 1987 Constitution, that "[e]very bill passed by the varying forms of expression, all deny to the courts the
Congress shall embrace only one subject which shall power to inquire into allegations that, in enacting a law, a
be expressed in the title thereof." House of Congress failed to comply with its own rules, in
the absence of showing that there was a violation of a
Held: The objectives of Section 26(1), Article VI constitutional provision or the right of private individuals.
of the 1987 Constitution are: In Osmena v. Pendatun, it was held: “At any rate, courts
have declared that ‘the rules adopted by deliberative
1) To prevent hodge-podge or log-rolling bodies are subject to revocation, modification or waiver at
legislation; the pleasure of the body adopting them.’ And it has been
2) To prevent surprise or fraud upon the said that ‘Parliamentary rules are merely procedural, and
legislature by means of provisions in bills of with their observance, the courts have no concern. They
which the titles gave no information, and may be waived or disregarded by the legislative body.’
which might therefore be overlooked and Consequently, ‘mere failure to conform to parliamentary
carelessly and unintentionally adopted; and usage will not invalidate that action (taken by a
3) To fairly apprise the people, through such deliberative body) when the requisite number of members
publication of legislative proceedings as is have agreed to a particular measure.’”
usually made, of the subjects of legislation
that are being considered, in order that they It must be realized that each of the three
may have opportunity of being heard thereon departments of our government has its separate sphere
by petition or otherwise if they shall so desire. which the others may not invade without upsetting the
delicate balance on which our constitutional order rests.
Section 26(1) of Article VI of the 1987 Constitution is Due regard for the working of our system of government,
sufficiently complied with where x x x the title is more than mere comity, compels reluctance on the part of
comprehensive enough to embrace the general objective the courts to enter upon an inquiry into an alleged
it seeks to achieve, and if all the parts of the statute are violation of the rules of the House. Courts must
related and germane to the subject matter embodied in accordingly decline the invitation to exercise their power.
the title or so long as the same are not inconsistent with (Arroyo v. De Venecia, 277 SCRA 268, Aug. 14,
or foreign to the general subject and title. (Agripino A. 1997 [Mendoza])
De Guzman, Jr., et al. v. COMELEC, G.R. No.
129118, July 19, 2000, en Banc [Purisima]) 59. What is the Bicameral Conference Committee?
Discuss the nature of its function and its jurisdiction.
57. Section 44 of R.A. No. 8189 (The Voter's Registration
Act of 1996) which provides for automatic transfer to Held: While it is true that a conference
a new station of any Election Officer who has already committee is the mechanism for compromising differences
served for more than four years in a particular city or between the Senate and the House, it is not limited in its
municipality was assailed for being violative of Section jurisdiction to this question. Its broader function is
26(1) of Article VI of the Constitution allegedly described thus:
because it has an isolated and different subject from

32
A conference committee may deal generally with and independent departments requires the judicial
the subject matter or it may be limited to resolving the department to act upon that assurance, and to
precise differences between the two houses. Even where accept, as having passed Congress, all bills
the conference committee is not by rule limited in its authenticated in the manner stated; leaving the
jurisdiction, legislative custom severely limits the freedom court to determine, when the question properly
with which new subject matter can be inserted into the arises, whether the Act, so authenticated, is in
conference bill. But occasionally a conference committee conformity with the Constitution. (Marshall Field
produces unexpected results, results beyond its mandate. & Co. v. Clark, 143 U.S. 649, 672, 36 L. Ed. 294,
These excursions occur even where the rules impose strict 303 [1891])
limitations on conference committee jurisdiction. This is
symptomatic of the authoritarian power of conference To overrule the doctrine now, x x x is to repudiate
committee. (Philippine Judges Association v. Prado, the massive teaching of our cases and overthrow an
227 SCRA 703, Nov. 11, 1993, En Banc [Cruz]) established rule of evidence. (Arroyo v. De Venecia,
277 SCRA 268, Aug. 14, 1997 [Mendoza])
60. Discuss the Enrolled Bill Doctrine.
61. When should the Legislative Journal be regarded as
Held: Under the enrolled bill doctrine, the conclusive upon the courts, and why?
signing of H. Bill No. 7189 by the Speaker of the House
and the President of the Senate and the certification by Held: The Journal is regarded as conclusive with
the secretaries of both Houses of Congress that it was respect to matters that are required by the Constitution to
passed on November 21, 1996 are conclusive of its due be recorded therein. With respect to other matters, in the
enactment. x x x To be sure, there is no claim either absence of evidence to the contrary, the Journals have
here or in the decision in the EVAT cases (Tolentino v. also been accorded conclusive effects. Thus, in United
Secretary of Finance) that the enrolled bill embodies a States v. Pons, this Court spoke of the imperatives of
conclusive presumption. In one case (Astorga v. Villegas) public policy for regarding the Journals as “public
we “went behind” an enrolled bill and consulted the memorials of the most permanent character,” thus: “They
Journal to determine whether certain provisions of a should be public, because all are required to conform to
statute had been approved by the Senate. them; they should be permanent, that rights acquired
today upon the faith of what has been declared to be law
But, where as here there is no evidence to the shall not be destroyed tomorrow, or at some remote
contrary, this Court will respect the certification of the period of time, by facts resting only in the memory of
presiding officers of both Houses that a bill has been duly individuals.” (Arroyo v. De Venecia, 277 SCRA 268,
passed. Under this rule, this Court has refused to 298-299, Aug. 14, 1997 [Mendoza])
determine claims that the three-fourths vote needed to
pass a proposed amendment to the Constitution had not 62. What matters are required to be entered on the
been obtained, because “a duly authenticated bill or Journal?
resolution imports absolute verity and is binding on the Held:
courts.” x x x 1) The yeas and nays on the third and final
reading of a bill (Art. VI, Sec. 26[2]);
This Court has refused to even look into 2) The yeas and nays on any question, at the
allegations that the enrolled bill sent to the President request of one-fifth of the members present
contained provisions which had been “surreptitiously” (Id., Sec. 16[4]);
inserted in the conference committee x x x. (Tolentino v. 3) The yeas and nays upon repassing a bill over
Secretary of Finance) the President’s veto (Id., Sec. 27[1]); and
4) The President’s objection to a bill he had
It has refused to look into charges that an vetoed (Id.).
amendment was made upon the last reading of a bill in (Arroyo v. De Venecia, 277 SCRA 268, 298, Aug. 14,
violation of Art. VI, Sec. 26(2) of the Constitution that 1997 [Mendoza])
“upon the last reading of a bill, no amendment shall be
allowed.” (Philippine Judges Ass’n v. Prado) 63. What are the limitations on the veto power of the
President?
In other cases, this Court has denied claims that
the tenor of a bill was otherwise than as certified by the Held: The act of the Executive in vetoing the
presiding officers of both Houses of Congress. particular provisions is an exercise of a constitutionally
vested power. But even as the Constitution grants the
The enrolled bill doctrine, as a rule of evidence, is power, it also provides limitations to its exercise. The
well-established. It is cited with approval by text writers veto power is not absolute.
here and abroad. The enrolled bill rule rests on the
following considerations: Xxx
The OSG is correct when it states that the
X x x As the President has no authority to Executive must veto a bill in its entirety or not at all. He
approve a bill not passed by Congress, an enrolled or she cannot act like an editor crossing out specific lines,
Act in the custody of the Secretary of State, and provisions, or paragraphs in a bill that he or she dislikes.
having the official attestations of the Speaker of In the exercise of the veto power, it is generally all or
the House of Representatives, of the President of nothing. However, when it comes to appropriation,
the Senate, and of the President of the United revenue or tariff bills, the Administration needs the money
States, carries, on its face, a solemn assurance by to run the machinery of government and it can not veto
the legislative and executive departments of the the entire bill even if it may contain objectionable
government, charged, respectively, with the duty features. The President is, therefore, compelled to
of enacting and executing the laws, that it was approve into law the entire bill, including its undesirable
passed by Congress. The respect due to coequal parts. It is for this reason that the Constitution has wisely

33
provided the “item veto power” to avoid inexpedient riders automatically vest the position in the winning candidate.
being attached to an indispensable appropriation or Section 17 of Article VI of the 1987 Constitution reads:
revenue measure.
The Senate and the House of
The Constitution provides that only a particular Representatives shall have an Electoral Tribunal
item or items may be vetoed. The power to disapprove which shall be the sole judge of all contests
any item or items in an appropriate bill does not grant the relating to the election, returns and qualifications
authority to veto a part of an item and to approve the of their respective Members.
remaining portion of the same item. (Bengzon v.
Drilon, 208 SCRA 133, 143-145, April 15, 1992, En Under the above-stated provision, the electoral
Banc [Gutierrez]) tribunal clearly assumes jurisdiction over all contests
relative to the election, returns and qualifications of
64. Distinguish an “item” from a “provision” in relation to candidates for either the Senate or the House only when
the veto power of the President. the latter become members of either the Senate or the
House of Representatives. A candidate who has not been
Held: The terms item and provision in budgetary proclaimed and who has not taken his oath of office
legislation and practice are concededly different. An item cannot be said to be a member of the House of
in a bill refers to the particulars, the details, the distinct Representatives subject to Section 17 of Article VI of the
and severable parts x x x of the bill. It is an indivisible Constitution. While the proclamation of a winning
sum of money dedicated to a stated purpose . The United candidate in an election is ministerial, B.P. Blg. 881 in
States Supreme Court, in the case of Bengzon v. Secretary conjunction with Sec. 6 of R.A. 6646 allows suspension of
of Justice (299 U.S. 410, 414, 57 Ct 252, 81 L. Ed., 312) proclamation under circumstances mentioned therein.
declared “that an item” of an appropriation bill obviously Thus, petitioner’s contention that “after the conduct of the
means an item which in itself is a specific appropriation of election and (petitioner) has been established the winner
money, not some general provision of law, which happens of the electoral exercise from the moment of election, the
to be put into an appropriation bill. (Bengzon v. Drilon, COMELEC is automatically divested of authority to pass
208 SCRA 133, 143-145, April 15, 1992, En Banc upon the question of qualification” finds no basis in law,
[Gutierrez]) because even after the elections the COMELEC is
empowered by Section 6 (in relation to Section 7) of R.A.
65. May the President veto a law? May she veto a 6646 to continue to hear and decide questions relating to
decision of the SC which has long become final and qualifications of candidates. (Aquino v. COMELEC, 248
executory? SCRA 400, 417-419, Sept. 18, 1995, En Banc
[Kapunan, J.])
Held: We need no lengthy justifications or
citations of authorities to declare that no President may 2. As to the House of Representatives Electoral
veto the provisions of a law enacted thirty-five (35) years Tribunal’s supposed assumption of jurisdiction over the
before his or her term of office. Neither may the issue of petitioner’s qualifications after the May 8, 1995
President set aside or reverse a final and executory elections, suffice it to say that HRET’s jurisdiction as the
judgment of this Court through the exercise of the veto sole judge of all contests relating to the elections, returns
power. (Bengzon v. Drilon, 208 SCRA 133, 143-145, and qualifications of members of Congress begins only
April 15, 1992, En Banc [Gutierrez]) after a candidate has become a member of the House of
Representatives (Art. VI, Sec. 17, 1987 Constitution).
66. A disqualification case was filed against a candidate Petitioner not being a member of the House of
for Congressman before the election with the Representatives, it is obvious that the HRET at this point
COMELEC. The latter failed to resolve that has no jurisdiction over the question. (Romualdez-
disqualification case before the election and that Marcos v. COMELEC, 248 SCRA 300, 340-341, Sept.
candidate won, although he was not yet proclaimed 18, 1995, En Banc [Kapunan, J.])
because of that pending disqualification case. Is the
COMELEC now ousted of jurisdiction to resolve the 67. Will the rule be the same if that candidate wins and
pending disqualification case and, therefore, should was proclaimed winner and already assumed office as
dismiss the case, considering that jurisdiction is now Congressman?
vested with the House of Representatives Electoral
Tribunal (HRET)? Held: While the COMELEC is vested with the
power to declare valid or invalid a certificate of candidacy,
Held: 1. [P]etitioner vigorously contends that its refusal to exercise that power following the
after the May 8, 1995 elections, the COMELEC lost its proclamation and assumption of the position by Farinas is
jurisdiction over the question of petitioner’s qualifications a recognition of the jurisdictional boundaries separating
to run for member of the House of Representatives. He the COMELEC and the Electoral Tribunal of the House of
claims that jurisdiction over the petition for disqualification Representatives (HRET). Under Article VI, Section 17 of
is exclusively lodged with the House of Representatives the Constitution, the HRET has sole and exclusive
Electoral Tribunal (HRET). Given the yet-unresolved jurisdiction over all contests relative to the election,
question of jurisdiction, petitioner avers that the COMELEC returns, and qualifications of members of the House of
committed serious error and grave abuse of discretion in Representatives. Thus, once a winning candidate has
directing the suspension of his proclamation as the been proclaimed, taken his oath, and assumed office as a
winning candidate in the Second Congressional District of member of the House of Representatives, COMELEC’s
Makati City. We disagree. jurisdiction over election contests relating to his election,
returns, and qualifications ends, and the HRET’s own
Petitioner conveniently confuses the distinction jurisdiction begins. Thus, the COMELEC’s decision to
between an unproclaimed candidate to the House of discontinue exercising jurisdiction over the case is
Representatives and a member of the same. Obtaining justifiable, in deference to the HRET’s own jurisdiction and
the highest number of votes in an election does not

34
functions. (Guerrero v. COMELEC, 336 SCRA 458,
July 26, 2000, En Banc [Quisumbing]) 70. Did former President Estrada resign as President or
should be considered resigned as of January 20, 2001
68. Petitioner further argues that the HRET assumes when President Gloria Macapagal Arroyo took her oath
jurisdiction only if there is a valid proclamation of the as the 14th President of the Republic?
winning candidate. He contends that if a candidate
fails to satisfy the statutory requirements to qualify Held: Resignation x x x is a factual question and
him as a candidate, his subsequent proclamation is its elements are beyond quibble: there must be an intent
void ab initio. Where the proclamation is null and to resign and the intent must be coupled by acts of
void, there is no proclamation at all and the mere relinquishment. The validity of a resignation is not
assumption of office by the proclaimed candidate does governed by any formal requirement as to form. It can be
not deprive the COMELEC at all of its power to declare oral. It can be written. It can be express. It can be
such nullity, according to petitioner. implied. As long as the resignation is clear, it must be
given legal effect.
Held: But x x x in an electoral contest where the
validity of the proclamation of a winning candidate who In the cases at bar, the facts show that petitioner
has taken his oath of office and assumed his post as did not write any formal letter of resignation before he
congressman is raised, that issue is best addressed to the evacuated Malacanang Palace in the afternoon of January
HRET. The reason for this ruling is self-evident, for it 20, 2001 after the oath-taking of respondent Arroyo.
avoids duplicity of proceedings and a clash of jurisdiction Consequently, whether or not petitioner resigned has to
between constitutional bodies, with due regard to the be determined from his acts and omissions before, during
people’s mandate. (Guerrero v. COMELEC, 336 SCRA and after January 20, 2001 or by the totality of prior,
458, July 26, 2000, En Banc [Quisumbing]) contemporaneous and posterior facts and circumstantial
evidence bearing a material relevance on the issue.
69. Is there an appeal from a decision of the Senate or
House of Representatives Electoral Tribunal? What Using this totality test, we hold that petitioner
then is the remedy, if any? resigned as President.

Held: The Constitution mandates that the House Xxx


of Representatives Electoral Tribunal and the Senate
Electoral Tribunal shall each, respectively, be the sole In sum, we hold that the resignation of the
judge of all contests relating to the election, returns and petitioner cannot be doubted. It was confirmed by his
qualifications of their respective members. leaving Malacanang. In the press release containing his
final statement, (1) he acknowledged the oath-taking of
The Court has stressed that “x x x so long as the the respondent as President of the Republic albeit with
Constitution grants the HRET the power to be the sole reservation about its legality; (2) he emphasized he was
judge of all contests relating to the election, returns and leaving the Palace, the seat of the presidency, for the
qualifications of members of the House of sake of peace and in order to begin the healing process of
Representatives, any final action taken by the HRET on a our nation. He did not say he was leaving the Palace due
matter within its jurisdiction shall, as a rule, not be to any kind of inability and that he was going to re-
reviewed by this Court. The power granted to the assume the presidency as soon as the disability
Electoral Tribunal x x x excludes the exercise of any disappears; (3) he expressed his gratitude to the people
authority on the part of this Court that would in any wise for the opportunity to serve them. Without doubt, he was
restrict it or curtail it or even affect the same.” referring to the past opportunity given him to serve the
people as President; (4) he assured that he will not shirk
The Court did recognize, of course, its power of from any future challenge that may come ahead on the
judicial review in exceptional cases. In Robles v. HRET, same service of our country. Petitioner’s reference is to a
the Court has explained that while the judgments of the future challenge after occupying the office of the
Tribunal are beyond judicial interference, the Court may president which he has given up; and (5) he called on his
do so, however, but only “in the exercise of this Court’s supporters to join him in the promotion of a constructive
so-called extraordinary jurisdiction x x x upon a national spirit of reconciliation and solidarity. Certainly,
determination that the Tribunal’s decision or resolution the national spirit of reconciliation and solidarity could not
was rendered without or in excess of its jurisdiction, or be attained if he did not give up the presidency. The
with grave abuse of discretion or paraphrasing Morrero v. press release was petitioner’s valedictory, his final act of
Bocar (66 Phil. 429), upon a clear showing of such farewell. His presidency is now in the past tense.
arbitrary and improvident use by the Tribunal of its power (Estrada v. Desierto, G.R. Nos. 146710-15, March
as constitutes a denial of due process of law, or upon a 2, 2001, En Banc [Puno])
demonstration of a very clear unmitigated error,
manifestly constituting such grave abuse of discretion that 71. Discuss our legal history on executive immunity.
there has to be a remedy for such abuse.”
Held: The doctrine of executive immunity in this
The Court does not x x x venture into the perilous jurisdiction emerged as a case law. In the 1910 case of
area of correcting perceived errors of independent Forbes, etc. v. Chuoco Tiaco and Crossfield, the
branches of the Government; it comes in only when it has respondent Tiaco, a Chinese citizen, sued petitioner W.
to vindicate a denial of due process or correct an abuse of Cameron Forbes, Governor-General of the Philippine
discretion so grave or glaring that no less than the Islands, J.E. Harding and C.R. Trowbridge, Chief of Police
Constitution itself calls for remedial action. (Libanan v. and Chief of the Secret Service of the City of Manila,
HRET, 283 SCRA 520, Dec. 22, 1997 [Vitug]) respectively, for damages for allegedly conspiring to
deport him to China. In granting a writ of prohibition, this
Court, speaking thru Mr. Justice Johnson, held:
The Executive Department

35
“The principle of nonliability x x x does late President Ferdinand E. Marcos and the 1973
not mean that the judiciary has no authority to touch the Constitution was born. In 1981, it was amended and one
acts of the Governor-General; that he may, under cover of the amendments involved executive immunity . Section
of his office, do what he will, unimpeded and 17, Article VII stated:
unrestrained. Such a construction would mean that
tyranny, under the guise of the execution of the law, “The President shall be immune from suit
could walk defiantly abroad, destroying rights of person during his tenure. Thereafter, no suit whatsoever
and of property, wholly free from interference of courts shall lie for official acts done by him or by others
or legislatures. This does not mean, either, that a person pursuant to his specific orders during his tenure.
injured by the executive authority by an act unjustifiable
under the law has no remedy, but must submit in The immunities herein provided shall
silence. On the contrary, it means, simply, that the apply to the incumbent President referred to in
Governor-General, like the judges of the courts and the Article XVII of this Constitution.”
members of the Legislature, may not be personally
mulcted in civil damages for the consequences of an act In his second Vicente G. Sinco Professorial Chair Lecture
executed in the performance of his official duties. The entitled, “Presidential Immunity And All The King’s Men:
judiciary has full power to, and will, when the matter is The Law Of Privilege As A Defense To Actions For
properly presented to it and the occasion justly warrants Damages,” (62 Phil. L.J. 113 [1987]) petitioner’s learned
it, declare an act of the Governor-General illegal and void counsel, former Dean of the UP College of Law, Atty.
and place as nearly as possible in status quo any person Pacifico Agabin, brightened the modifications effected by
who has been deprived his liberty or his property by such this constitutional amendment on the existing law on
act. This remedy is assured to every person, however executive privilege. To quote his disquisition:
humble or of whatever country, when his personal or
property rights have been invaded, even by the highest “In the Philippines though, we sought to
authority of the state. The thing which the judiciary can do the American one better by enlarging and fortifying the
not do is mulct the Governor-General personally in absolute immunity concept. First, we extended it to shield
damages which result from the performance of his official the President not only from civil claims but also from
duty, any more than it can a member of the Philippine criminal cases and other claims. Second, we enlarged its
Commission or the Philippine Assembly. Public policy scope so that it would cover even acts of the President
forbids it. outside the scope of official duties. And third, we
broadened its coverage so as to include not only the
Neither does this principle of nonliability President but also other persons, be they government
mean that the chief executive may not be personally officials or private individuals, who acted upon orders of
sued at all in relation to acts which he claims to perform the President. It can be said that at that point most of us
as such official. On the contrary, it clearly appears from were suffering from AIDS (or absolute immunity defense
the discussion heretofore had, particularly that portion syndrome).”
which touched the liability of judges and drew an
analogy between such liability and that of the Governor- The Opposition in the then Batasang Pambansa sought
General, that the latter is liable when he acts in a case so the repeal of this Marcosian concept of executive
plainly outside of his power and authority that he can not immunity in the 1973 Constitution. The move was led by
be said to have exercised discretion in determining then Member of Parliament, now Secretary of Finance,
whether or not he had the right to act. What is held Alberto Romulo, who argued that the after incumbency
here is that he will be protected from personal liability for immunity granted to President Marcos violated the
damages not only when he acts within his authority, but principle that a public office is a public trust. He
also when he is without authority, provided he actually denounced the immunity as a return to the anachronism
used discretion and judgment, that is, the judicial faculty, “the king can do no wrong.” The effort failed.
in determining whether he had authority to act or not.
In other words, he is entitled to protection in determining The 1973 Constitution ceased to exist when
the question of his authority. If he decide wrongly, he is President Marcos was ousted from office by the People
still protected provided the question of his authority was Power revolution in 1986. When the 1987 Constitution
one over which two men, reasonably qualified for that was crafted, its framers did not reenact the executive
position, might honestly differ; but he is not protected if immunity provision of the 1973 Constitution. x x x
the lack of authority to act is so plain that two such men (Estrada v. Desierto, G.R. Nos. 146710-15, March
could not honestly differ over its determination. In such 2, 2001, en Banc [Puno])
case, he acts, not as Governor-General but as a private
individual, and, as such, must answer for the 72. Can former President Estrada still be prosecuted
consequences of his act.” criminally considering that he was not convicted in the
impeachment proceedings against him?
Mr. Justice Johnson underscored the consequences if the
Chief Executive was not granted immunity from suit, viz: Held: We reject his argument that he cannot be
“x x x. Action upon important matters of state delayed; prosecuted for the reason that he must first be convicted
the time and substance of the chief executive spent in in the impeachment proceedings. The impeachment trial
wrangling litigation; disrespect engendered for the person of petitioner Estrada was aborted by the walkout of the
of one of the highest officials of the State and for the prosecutors and by the events that led to his loss of the
office he occupies; a tendency to unrest and disorder; presidency. Indeed, on February 7, 2001, the Senate
resulting in a way, in a distrust as to the integrity of passed Senate Resolution No. 83 “Recognizing that the
government itself.” Impeachment Court is Functus Officio.” Since the
Impeachment Court is now functus officio, it is untenable
Our 1935 Constitution took effect but it did not for petitioner to demand that he should first be impeached
contain any specific provision on executive immunity. and then convicted before he can be prosecuted. The
Then came the tumult of the martial law years under the plea if granted, would put a perpetual bar against his

36
prosecution. Such a submission has nothing to commend and the proclamation of the winning candidates for
itself for it will place him in a better situation than a non- president and vice-president for the entire nation must
sitting President who has not been subjected to remain in the hands of Congress. (Makalintal v.
impeachment proceedings and yet can be the object of a COMELEC, G.R. No. 157013, July 10, 2003, En Banc
criminal prosecution. To be sure, the debates in the [Austria-Martinez])
Constitutional Commission make it clear that when
impeachment proceedings have become moot due to the 74. Discuss the jurisdiction of the Supreme Court to hear
resignation of the President, the proper criminal and civil and decide cases involving the election, returns and
cases may already be filed against him x x x. qualifications of the President and Vice-President.

This is in accord with our ruling in In Re: Held: Petitioners Tecson, et al. x x x and Velez x
Saturnino Bermudez that “incumbent Presidents are x x invoke the provisions of Article VII, Section 4,
immune from suit or from being brought to court during paragraph 7, of the 1987 Constitution in assailing the
the period of their incumbency and tenure” but not jurisdiction of the COMELEC when it took cognizance of
beyond. Considering the peculiar circumstance that the SPA No. 04-003 and in urging the Supreme Court to
impeachment process against the petitioner has been instead take on the petitions they directly instituted before
aborted and thereafter he lost the presidency, petitioner it. X x x
Estrada cannot demand as a condition sine qua non to his
criminal prosecution before the Ombudsman that he be Xxx
convicted in the impeachment proceedings. (Estrada v.
Desierto, G.R. Nos. 146710-15, Mar. 2, 2001, En Ordinary usage would characterize a “contest” in
Banc [Puno]) reference to a post-election scenario. Election
contests consist of either an election protest or a quo
73. Is Section 18.5 of R.A. No. 9189 in relation to Section warranto which, although two distinct remedies, would
4 of the same Act in contravention of Section 4, have one objective in view, i.e., to dislodge the winning
Article VII of the Constitution? candidate from office. A perusal of the phraseology in
Rule 12, Rule 13, and Rule 14 of the “Rules of the
Held: Section 4 of R.A. No. 9189 provides that Presidential Electoral Tribunal,” promulgated by the
the overseas absentee voter may vote for president, vice- Supreme Court en banc on 18 April 1992, would support
president, senators and party-list representatives. this premise x x x.

Xxx The rules categorically speak of the jurisdiction of


Petitioner claims that the provision of Section 18.5 the tribunal over contests relating to the election, returns
of R.A. No. 9189 empowering the COMELEC to order the and qualifications of the “President” or “Vice-President” of
proclamation of winning candidates insofar as it affects the Philippines, and not of “candidates” for President or
the canvass of votes and proclamation of winning Vice-President. A quo warranto proceeding is generally
candidates for president and vice-president, is defined as being an action against a person who usurps,
unconstitutional because it violates the following intrudes into, or unlawfully holds or exercises a public
provisions of paragraph 4, Section 4 of Article VII of the office. In such context, the election contest can only
Constitution x x x which gives to Congress the duty to contemplate a post-election scenario. In Rule 14, only a
canvass the votes and proclaim the winning candidates for registered candidate who would have received either the
president and vice-president. second or third highest number of votes could file an
election protest. This rule again presupposes a post-
Xxx election scenario.
Indeed, the phrase, proclamation of winning
candidates, in Section 18.5 of R.A. No. 9189 is far too It is fair to conclude that the jurisdiction of the
sweeping that it necessarily includes the proclamation of Supreme Court, defined by Section 4, paragraph 7, of the
the winning candidates for the presidency and the vice- 1987 Constitution, would not include cases directly
presidency. brought before it questioning the qualifications of a
candidate for the presidency or vice-presidency before the
Section 18.5 of R.A. No. 9189 appears to be elections are held.
repugnant to Section 4, Article VIII of the Constitution
only insofar as said Section totally disregarded the Accordingly, G.R. No. 161434 x x x and G.R. No.
authority given to Congress by the Constitution to 161634 x x x would have to be dismissed for want of
proclaim the winning candidates for the positions of jurisdiction. (Maria Jeanette Tecson, et al. v.
president and vice-president. COMELEC, G.R. No. 161434, March 3, 2004, En
Banc [Vitug])
In addition, the Court notes that Section 18.4 of
the law x x x clashes with paragraph 4, Section 4, Article 75. State the reason why not all appointments made by
VII of the Constitution which provides that the returns of the President under the 1987 Constitution will require
every election for President and Vice-President shall be confirmation by the Commission on Appointments.
certified by the board of canvassers to Congress.
Held: The aforecited provision (Section 16,
Congress should not have allowed the COMELEC Article VII) of the Constitution has been the subject of
to usurp a power that constitutionally belongs to it or x x several cases on the issue of the restrictive function of the
x to encroach “on the power of Congress to canvass the Commission on Appointments with respect to the
votes for president and vice-president and the power to appointing power of the President. This Court touched
proclaim the winners for the said positions.” The upon the historical antecedent of the said provision in the
provisions of the Constitution as the fundamental law of case of Sarmiento III v. Mison in which it was ratiocinated
the land should be read as part of The Overseas Absentee upon that Section 16 of Article VII of the 1987
Voting Act of 2003 and hence, the canvassing of the votes Constitution requiring confirmation by the Commission on

37
Appointments of certain appointments issued by the
President contemplates a system of checks and balances Held: It is well-settled that only presidential
between the executive and legislative branches of appointees belonging to the first group (enumerated
government. Experience showed that when almost all under the first sentence of Section 16, Article VII of the
presidential appointments required the consent of the 1987 Constitution) require the confirmation by the
Commission on Appointments, as was the case under the Commission on Appointments. The appointments of
1935 Constitution, the commission became a venue of respondent officers who are not within the first category,
“horse trading” and similar malpractices. On the other need not be confirmed by the Commission on
hand, placing absolute power to make appointments in Appointments. As held in the case of Tarrosa v. Singson,
the President with hardly any check by the legislature, as Congress cannot by law expand the power of confirmation
what happened under the 1973 Constitution, leads to of the Commission on Appointments and require
abuse of such power. Thus was perceived the need to confirmation of appointments of other government
establish a “middle ground” between the 1935 and 1973 officials not mentioned in the first sentence of Section 16
Constitutions. The framers of the 1987 Constitution of Article VII of the 1987 Constitution.
deemed it imperative to subject certain high positions in
the government to the power of confirmation of the Consequently, unconstitutional are Sections 26
Commission on Appointments and to allow other positions and 31 of Republic Act 6975 which empower the
within the exclusive appointing power of the President. Commission on Appointments to confirm the
(Manalo v. Sistoza, 312 SCRA 239, Aug. 11, 1999, appointments of public officials whose appointments are
En Banc [Purisima]) not required by the Constitution to be confirmed.
(Manalo v. Sistoza, 312 SCRA 239, Aug. 11, 1999,
76. Who are the officers to be appointed by the President En Banc [Purisima])
under Section 16, Article VII of the 1987 Constitution
whose appointments shall require confirmation by the 78. Will it be correct to argue that since the Philippine
Commission on Appointments, and those whose National Police is akin to the Armed Forces of the
appointments shall no longer require such Philippines, therefore, the appointments of police
confirmation? officers whose rank is equal to that of colonel or naval
captain will require confirmation by the Commission
Held: Conformably, as consistently interpreted on Appointments?
and ruled in the leading case of Sarmiento III v. Mison,
and in the subsequent cases of Bautista v. Salonga, Held: This contention is x x x untenable. The
Quintos-Deles v. Constitutional Commission, and Calderon Philippine National Police is separate and distinct from the
v. Carale, under Section 16, Article VII, of the Armed Forces of the Philippines. The Constitution, no
Constitution, there are four groups of officers of the less, sets forth the distinction. Under Section 4 of Article
government to be appointed by the President: XVI of the 1987 Constitution,

First, the heads of the executive departments, “The Armed Forces of the Philippines
ambassadors, other public ministers and consuls, officers shall be composed of a citizen armed force
of the armed forces from the rank of colonel or naval which shall undergo military training and
captain, and other officers whose appointments are service, as may be provided by law. It shall
vested in him in this Constitution; keep a regular force necessary for the
security of the State.”
Second, all other officers of the Government
whose appointments are not otherwise provided On the other hand, Section 6 of the same Article
for by law; of the Constitution ordains that:

Third, those whom the President may be “The State shall establish and
authorized by law to appoint; maintain one police force, which shall be
national in scope and civilian in character to
Fourth, officers lower in rank whose be administered and controlled by a national
appointments the Congress may by law vest in police commission. The authority of local
the President alone. executives over the police units in their
jurisdiction shall be provided by law.”
It is well-settled that only presidential appointees
belonging to the first group require the confirmation by To so distinguish the police force from the armed
the Commission on Appointments. (Manalo v. Sistoza, forces, Congress enacted Republic Act 6975 x x x.
312 SCRA 239, Aug. 11, 1999, En Banc [Purisima])
Thereunder, the police force is different from and
77. Under Republic Act 6975 (the DILG Act of 1990), the independent of the armed forces and the ranks in the
Director General, Deputy Director General, and other military are not similar to those in the Philippine National
top officials of the Philippine National Police (PNP) Police. Thus, directors and chief superintendents of the
shall be appointed by the President and their PNP x x x do not fall under the first category of
appointments shall require confirmation by the presidential appointees requiring confirmation by the
Commission on Appointments. Respondent Sistoza Commission on Appointments. (Manalo v. Sistoza, 312
was appointed Director General of the PNP but he SCRA 239, Aug. 11, 1999, En Banc [Purisima])
refused to submit his appointment papers to the
Commission on Appointments for confirmation 79. Discuss the nature of an ad-interim appointment. Is it
contending that his appointment shall no longer temporary and, therefore, can be withdrawn or
require confirmation despite the express provision of revoked by the President at her pleasure?
the law requiring such confirmation. Should his
contention be upheld?

38
Held: An ad interim appointment is a permanent Petitioner cites Black’s Law Dictionary which
appointment because it takes effect immediately and can defines the term “ad interim” to mean “in the meantime”
no longer be withdrawn by the President once the or “for the time being.” Hence, petitioner argues that an
appointee has qualified into office. The fact that it is ad interim appointment is undoubtedly temporary in
subject to confirmation by the Commission on character. This argument is not new and was answered
Appointments does not alter its permanent character. by this Court in Pamantasan ng Lungsod ng Maynila v.
The Constitution itself makes an ad interim appointment Intermediate Appellate Court, where we explained that:
permanent in character by making it effective until
disapproved by the Commission on Appointments or until “x x x From the arguments, it is easy to see why the
the next adjournment of Congress. The second petitioner should experience difficulty in understanding
paragraph of Section 16, Article VII of the Constitution the situation. Private respondent had been extended
provides as follows: several ‘ad interim’ appointments which petitioner
mistakenly understands as appointments temporary in
“The President shall have the power to make nature. Perhaps, it is the literal translation of the word
appointments during the recess of the Congress, ‘ad interim’ which creates such belief. The term is
whether voluntary or compulsory, but such defined by Black to mean ‘in the meantime’ or ‘for the
appointments shall be effective only until time being’. Thus, an officer ad interim is one appointed
disapproval by the Commission on Appointments to fill a vacancy, or to discharge the duties of the office
or until the next adjournment of the Congress.” during the absence or temporary incapacity of its regular
incumbent (Black’s Law Dictionary, Revised Fourth
Thus, the ad interim appointment remains effective until Edition, 1978). But such is not the meaning nor the use
such disapproval or next adjournment, signifying that it intended in the context of Philippine law. In referring to
can no longer be withdrawn or revoked by the President. Dr. Esteban’s appointments, the term is not descriptive of
The fear that the President can withdraw or revoke at any the nature of the appointments given to him. Rather, it
time and for any reason an ad interim appointment is is used to denote the manner in which said appointments
utterly without basis. were made, that is, done by the President of the
Pamantasan in the meantime, while the Board of
More than half a century ago, this Court had Regents, which is originally vested by the University
already ruled that an ad interim appointment is Charter with the power of appointment, is unable to act.
permanent in character. In Summers v. Ozaeta, decided X x x.”
on October 25, 1948, we held that:
Thus, the term “ad interim appointment”, as used
“x x x an ad interim appointment is one made in in letters of appointment signed by the President, means a
pursuance of paragraph (4), Section 10, Article VII of the permanent appointment made by the President in the
Constitution, which provides that the ‘President shall have meantime that Congress is in recess. It does not mean a
the power to make appointments during the recess of the temporary appointment that can be withdrawn or revoked
Congress, but such appointments shall be effective only at any time. The term, although not found in the text of
until disapproval by the Commission on Appointments or the Constitution, has acquired a definite legal meaning
until the next adjournment of the Congress.’ It is an under Philippine jurisprudence. The Court had again
appointment permanent in nature, and the circumstance occasion to explain the nature of an ad interim
that it is subject to confirmation by the Commission on appointment in the more recent case of Marohombsar v.
Appointments does not alter its permanent character. An Court of Appeals, where the Court stated:
ad interim appointment is disapproved certainly for a
reason other than that its provisional period has expired. “We have already mentioned that an ad interim
Said appointment is of course distinguishable from an appointment is not descriptive of the nature of the
‘acting’ appointment which is merely temporary, good appointment, that is, it is not indicative of whether the
until another permanent appointment is issued.” appointment is temporary or in an acting capacity,
rather it denotes the manner in which the appointment
The Constitution imposes no condition on the was made. In the instant case, the appointment
effectivity of an ad interim appointment, and thus an ad extended to private respondent by then MSU President
interim appointment takes effect immediately. The Alonto, Jr. was issued without condition nor limitation as
appointee can at once assume office and exercise, as a de to tenure. The permanent status of private
jure officer, all the powers pertaining to the office. In respondent’s appointment as Executive Assistant II was
Pacete v. Secretary of the Commission on Appointments , recognized and attested to by the Civil Service
this Court elaborated on the nature of an ad interim Commission Regional Office No. 12. Petitioner’s
appointment as follows: submission that private respondent’s ad interim
appointment is synonymous with a temporary
“A distinction is thus made between the exercise appointment which could be validly terminated at any
of such presidential prerogative requiring confirmation by time is clearly untenable. Ad interim appointments are
the Commission on Appointments when Congress is in permanent appointment but their terms are only until
session and when it is in recess. In the former, the the Board disapproves them.”
President nominates, and only upon the consent of the
Commission on Appointments may the person thus named An ad interim appointee who has qualified and
assume office. It is not so with reference to ad interim assumed office becomes at that moment a government
appointments. It takes effect at once. The individual employee and therefore part of the civil service. He
chosen may thus qualify and perform his function without enjoys the constitutional protection that “[n]o officer or
loss of time. His title to such office is complete. In the employee in the civil service shall be removed or
language of the Constitution, the appointment is effective suspended except for cause provided by law.” (Section
‘until disapproval by the Commission on Appointments or 2[3], Article IX-B of the Constitution) Thus, an ad interim
until the next adjournment of the Congress.’” appointment becomes complete and irrevocable once the
appointee has qualified into office. The withdrawal or

39
revocation of an ad interim appointment is possible only if the COMELEC requires the Commission on Appointments
it is communicated to the appointee before the moment to first confirm ad interim appointees before the
he qualifies, and any withdrawal or revocation thereafter appointees can assume office will negate the President’s
is tantamount to removal from office. Once an appointee power to make ad interim appointments. This is contrary
has qualified, he acquires a legal right to the office which to the rule on statutory construction to give meaning and
is protected not only by statute but also by the effect to every provision of the law. It will also run
Constitution. He can only be removed for cause, after counter to the clear intent of the framers of the
notice and hearing, consistent with the requirements of Constitution.
due process. (Matibag v. Benipayo, 380 SCRA 49,
April 2, 2002, En Banc [Carpio]) Xxx
The President’s power to extend ad interim
80. How is an ad interim appointment terminated? appointments may indeed briefly put the appointee at the
mercy of both the appointing and confirming powers.
Held: An ad interim appointment can be This situation, however, in only for a short period – from
terminated for two causes specified in the Constitution. the time of issuance of the ad interim appointment until
The first cause is the disapproval of his ad interim the Commission on Appointments gives or withholds its
appointment by the Commission on Appointments. The consent. The Constitution itself sanctions this situation,
second cause is the adjournment of Congress without the as a trade-off against the evil of disruptions in vital
Commission on Appointments acting on his appointment. government services. This is also part of the check-and-
These two causes are resolutory conditions expressly balance under the separation of powers, as a trade-off
imposed by the Constitution on all ad interim against the evil of granting the President absolute and
appointments. These resolutory conditions constitute, in sole power to appoint. The Constitution has wisely
effect, a Sword of Damocles over the heads of ad interim subjected the President’s appointing power to the
appointees. No one, however, can complain because it is checking power of the legislature.
the Constitution itself that places the Sword of Damocles
over the heads of the ad interim appointees. (Matibag v. This situation, however, does not compromise the
Benipayo, 380 SCRA 49, April 2, 2002, En Banc independence of the COMELEC as a constitutional body.
[Carpio]) The vacancies in the COMELEC are precisely staggered to
insure that the majority of its members hold confirmed
appointments, and no one President will appoint all the
81. How is an ad interim appointment distinguished from COMELEC members. x x x The special constitutional
an appointment or designation in an acting or safeguards that insure the independence of the COMELEC
temporary capacity? remain in place (See Sections, 3, 4, 5 and 6, Article IX-A
of the Constitution).
Held: While an ad interim appointment is
permanent and irrevocable except as provided by law, an In fine, we rule that the ad interim appointments
appointment or designation in a temporary or acting extended by the President to Benipayo, Borra and Tuason,
capacity can be withdrawn or revoked at the pleasure of as COMELEC Chairman and Commissioners, respectively,
the appointing power. A temporary or acting appointee do not constitute temporary or acting appointments
does not enjoy any security of tenure, no matter how prohibited by Section 1 (2), Article IX-C of the
briefly. (Matibag v. Benipayo, 380 SCRA 49, April 2, Constitution. (Matibag v. Benipayo, 380 SCRA 49,
2002, En Banc [Carpio]) April 2, 2002, En Banc [Carpio])

82. Benipayo, Tuason and Borra were appointed 83. Discuss the reason why the framers of the 1987
Chairman and Commissioners, respectively, of the Constitution thought it wise to reinstate the 1935
Commission on Elections, by the President when Constitution provision on ad interim appointments of
Congress was not in session. Did their appointment the President.
violate the Sec. 1(2), Art. IX-C of the Constitution that
substantially provides that “No member of the Held: The original draft of Section 16, Article VII
Commission (on Elections) shall be appointed in an of the Constitution – on the nomination of officers subject
acting or temporary capacity? to confirmation by the Commission on Appointments – did
not provide for ad interim appointments. The original
Held: In the instant case, the President did in intention of the framers of the Constitution was to do
fact appoint permanent Commissioners to fill the away with ad interim appointments because the plan was
vacancies in the COMELEC, subject only to confirmation for Congress to remain in session throughout the year
by the Commission on Appointments. Benipayo, Borra except for a brief 30-day compulsory recess. However,
and Tuason were extended permanent appointments because of the need to avoid disruptions in essential
during the recess of Congress. They were not appointed government services, the framers of the Constitution
or designated in a temporary or acting capacity, unlike thought it wise to reinstate the provisions of the 1935
Commissioner Haydee Yorac in Brillantes v. Yorac and Constitution on ad interim appointments. X x x
Solicitor General Felix Bautista in Nacionalista Party v.
Bautista. The ad interim appointments of Benipayo, Borra Xxx
and Tuason are expressly allowed by the Constitution Clearly, the reinstatement in the present
which authorizes the President, during the recess of Constitution of the ad interim appointing power of the
Congress, to make appointments that take effect President was for the purpose of avoiding interruptions in
immediately. vital government services that otherwise would result
from prolonged vacancies in government offices, including
While the Constitution mandates that the the three constitutional commissions. In his concurring
COMELEC “shall be independent,” this provision should be opinion in Guevarra v. Inocentes, decided under the 1935
harmonized with the President’s power to extend ad Constitution, Justice Roberto Concepcion, Jr. explained
interim appointments. To hold that the independence of

40
the rationale behind ad interim appointments in this because the President may then issue new appointments
manner: – not because of implied disapproval of the Commission
deduced from its intention during the session of Congress,
“Now, why is the lifetime of ad interim for, under the Constitution, the Commission may affect
appointments so limited? Because, if they expired before adversely the interim appointments only by action, never
the session of Congress, the evil sought to be avoided – by omission. If the adjournment of Congress were an
interruption in the discharge of essential functions – may implied disapproval of ad interim appointments made prior
take place. Because the same evil would result if the thereto, then the President could no longer appoint those
appointments ceased to be effective during the session of so by-passed by the Commission. But, the fact is that the
Congress and before its adjournment. Upon the other President may reappoint them, thus clearly indicating that
hand, once Congress has adjourned, the evil the reason for said termination of the ad interim
aforementioned may easily be conjured by the issuance of appointments is not the disapproval thereof allegedly
other ad interim appointments or reappointments.” inferred from said omission of the Commission, but the
(Matibag v. Benipayo, 380 SCRA 49, April 2, 2002, circumstance that upon said adjournment of the Congress,
En Banc [Carpio]) the President is free to make ad interim appointments or
reappointments.”
84. The ad interim appointments of Benipayo, Borra and
Tuason as Chairman and Commissioners, respectively, Guevarra was decided under the 1935 Constitution from
of the COMELEC were by-passed by the Commission where the second paragraph of Section 16, Article VII of
on Appointments. However, they were subsequently the present Constitution on ad interim appointments was
reappointed by the President to the same positions. lifted verbatim. The jurisprudence under the 1935
Did their subsequent reappointment violate the Constitution governing ad interim appointments by the
prohibition against reappointment under Section 1(2), President is doubtless applicable to the present
Article IX-C of the 1987 Constitution? Constitution. The established practice under the present
Constitution is that the President can renew the
Held: There is no dispute that an ad interim appointments of by-passed ad interim appointees. This is
appointee disapproved by the Commission on a continuation of the well-recognized practice under the
Appointments can no longer be extended a new 1935 Constitution, interrupted only by the 1973
appointment. The disapproval is a final decision of the Constitution which did not provide for a Commission on
Commission on Appointments in the exercise of its Appointments but vested sole appointing power in the
checking power on the appointing authority of the President.
President. The disapproval is a decision on the merits,
being a refusal by the Commission on Appointments to The prohibition on reappointment in Section 1 (2),
give its consent after deliberating on the qualifications of Article IX-C of the Constitution applies neither to
the appointee. Since the Constitution does not provide for disapproved nor by-passed ad interim appointments. A
any appeal from such decision, the disapproval is final and disapproved ad interim appointment cannot be revived by
binding on the appointee as well as on the appointing another ad interim appointment because the disapproval
power. In this instance, the President can no longer is final under Section 16, Article VII of the Constitution,
renew the appointment not because of the constitutional and not because a reappointment is prohibited under
prohibition on appointment, but because of a final Section 1 (2), Article IX-C of the Constitution. A by-
decision by the Commission on Appointments to withhold passed ad interim appointment cannot be revived by a
its consent to the appointment. new ad interim appointment because there is no final
disapproval under Section 16, Article VII of the
An ad interim appointment that is by-passed Constitution, and such new appointment will not result in
because of lack of time or failure of the Commission on the appointee serving beyond the fixed term of seven
Appointments to organize is another matter. A by-passed years.
appointment is one that has not been finally acted upon
on the merits by the Commission on Appointments at the Xxx
close of the session of Congress. There is no final The framers of the Constitution made it quite
decision by the Commission on Appointments to give or clear that any person who has served any term of office
withhold its consent to the appointment as required by as COMELEC member – whether for a full term of seven
the Constitution. Absent such decision, the President is years, a truncated term of five or three years, or even an
free to renew the ad interim appointment of a by-passed unexpired term for any length of time – can no longer be
appointee. This is recognized in Section 17 of the Rules reappointed to the COMELEC. X x x
of the Commission on Appointments x x x. Hence, under
the Rules of the Commission on Appointments, a by- Xxx
passed appointment can be considered again if the In Visarra v. Miraflor, Justice Angelo Bautista, in his
President renews the appointment. concurring opinion, quoted Nacionalista v. De Vera that a
[r]eappointment is not prohibited when a Commissioner
It is well-settled in this jurisdiction that the has held, office only for, say, three or six years, provided
President can renew the ad interim appointments of by- his term will not exceed nine years in all.” This was the
passed appointees. Justice Roberto Concepcion, Jr. interpretation despite the express provision in the 1935
lucidly explained in his concurring opinion in Guevarra v. Constitution that a COMELEC member “shall hold office for
Inocentes why by-passed ad interim appointees could be a term of nine years and may not be reappointed.”
extended new appointments, thus:
To foreclose this interpretation, the phrase
“In short, an ad interim appointment ceases to be “without reappointment” appears twice in Section 1 (2),
effective upon disapproval by the Commission, because Article IX-C of the present Constitution. The first phrase
the incumbent can not continue holding office over the prohibits reappointment of any person previously
positive objection of the Commission. It ceases, also, appointed for a term of seven years. The second phrase
upon “the next adjournment of the Congress”, simply prohibits reappointment of any person previously

41
appointed for a term of five or three years pursuant to the The ad interim appointments and subsequent
first set of appointees under the Constitution. In either renewals of appointments of Benipayo, Borra and Tuason
case, it does not matter if the person previously appointed do not violate the prohibition on reappointments because
completes his term of office for the intention is to prohibit there were no previous appointments that were confirmed
any reappointment of any kind. by the Commission on Appointments. A reappointment
presupposes a previous confirmed appointment. The
However, an ad interim appointment that has same ad interim appointments and renewal of
lapsed by inaction of the Commission on Appointments appointments will also not breach the seven-year term
does not constitute a term of office. The period from the limit because all the appointments and renewals of
time the ad interim appointment is made to the time it appointments of Benipayo, Borra and Tuason are for a
lapses is neither a fixed term nor an unexpired term. To fixed term expiring on February 2, 2008. Any delay in
hold otherwise would mean that the President by his their confirmation will not extend the expiry date of their
unilateral action could start and complete the running of a terms of office. Consequently, there is no danger
term of office in the COMELEC without the consent of the whatsoever that the renewal of the ad interim
Commission on Appointments. This interpretation renders appointments of these three respondents will result in any
inutile the confirming power of the Commission on of the evils intended to be exorcised by the twin
Appointments. prohibitions in the Constitution. The continuing renewal
of the ad interim appointment of these three respondents,
The phrase “without reappointment” applies only for so long as their terms of office expire on February 2,
to one who has been appointed by the President and 2008, does not violate the prohibition on reappointments
confirmed by the Commission on Appointments, whether in Section 1 (2), Article IX-C of the Constitution.
or not such person completes his term of office. There (Matibag v. Benipayo, 380 SCRA 49, April 2, 2002,
must be a confirmation by the Commission on En Banc [Carpio])
Appointments of the previous appointment before the
prohibition on reappointment can apply. To hold 85. What are the four situations where Section 1(2),
otherwise will lead to absurdities and negate the Article IX-C of the 1987 Constitution which provides
President’s power to make ad interim appointments. that “[t]he Chairman and the Commissioners (of the
COMELEC) shall be appointed x x x for a term of
In the great majority of cases, the Commission on seven years without reappointment” will apply?
Appointments usually fails to act, for lack of time, on the
ad interim appointments first issued to appointees. If Held: Section 1 (2), Article IX-C of the
such ad interim appointments can no longer be renewed, Constitution provides that “[t]he Chairman and the
the President will certainly hesitate to make ad interim Commissioners shall be appointed x x x for a term of
appointments because most of her appointees will seven years without reappointment.” There are four
effectively be disapproved by mere inaction of the situations where this provision will apply. The first
Commission on Appointments. This will nullify the situation is where an ad interim appointee to the
constitutional power of the President to make ad interim COMELEC, after confirmation by the Commission on
appointments, a power intended to avoid disruptions in Appointments, serves his full seven-year term. Such
vital government services. This Court cannot subscribe to person cannot be reappointed to the COMELEC, whether
a proposition that will wreak havoc on vital government as a member or as a chairman, because he will then be
services. actually serving more than seven years. The second
situation is where the appointee, after confirmation,
The prohibition on reappointment is common to serves a part of his term and then resigns before his
the three constitutional commissions. The framers of the seven-year term of office ends. Such person cannot be
present Constitution prohibited reappointments for two reappointed, whether as a member or as a chair, to a
reasons. The first is to prevent a second appointment for vacancy arising from retirement because a reappointment
those who have been previously appointed and confirmed will result in the appointee also serving more than seven
even if they served for less than seven years. The second years. The third situation is where the appointee is
is to insure that the members of the three constitutional confirmed to serve the unexpired term of someone who
commissions do not serve beyond the fixed term of seven died or resigned, and the appointee completes the
years. x x x. unexpired term. Such person cannot be reappointed,
whether as a member or chair, to a vacancy arising from
Xxx retirement because a reappointment will result in the
Plainly, the prohibition on reappointment is intended to appointee also serving more than seven years.
insure that there will be no reappointment of any kind.
On the other hand, the prohibition on temporary or acting The fourth situation is where the appointee has
appointments is intended to prevent any circumvention of previously served a term of less than seven years, and a
the prohibition on reappointment that may result in an vacancy arises from death or resignation. Even if it will
appointee’s total term of office exceeding seven years. not result in his serving more than seven years, a
The evils sought to be avoided by the twin prohibitions reappointment of such person to serve an unexpired term
are very specific – reappointment of any kind and is also prohibited because his situation will be similar to
exceeding one’s term in office beyond the maximum those appointed under the second sentence of Section 1
period of seven years. (2), Article IX-C of the Constitution. This provision refers
to the first appointees under the Constitution whose terms
Not contented with these ironclad twin of office are less than seven years, but are barred from
prohibitions, the framers of the Constitution tightened ever being reappointed under any situation. (Matibag v.
even further the screws on those who might wish to Benipayo, 380 SCRA 49, April 2, 2002, En Banc
extend their terms of office. Thus, the word “designated” [Carpio])
was inserted to plug any loophole that might be exploited
by violators of the Constitution x x x. 86. To what types of appointments is Section 15, Article
VII of the 1987 Constitution (prohibiting the President

42
from making appointments two months before the that the rules are followed, but he himself does not lay
next presidential elections and up to the end of his down such rules, nor does he have the discretion to
term) directed against? modify or replace them. If the rules are not observed, he
may order the work done or re-done to conform to the
Held: Section 15, Article VII is directed against prescribed rules. He cannot prescribe his own manner for
two types of appointments: (1) those made for buying the doing of the act. (Bito-Onon v. Fernandez, 350
votes and (2) those made for partisan considerations. SCRA 732, Jan. 31, 2001, 3rd Div. [Gonzaga-Reyes])
The first refers to those appointments made within two
months preceding the Presidential election and are similar 89. Is the prior recommendation of the Secretary of
to those which are declared election offenses in the Justice a mandatory requirement before the President
Omnibus Election Code; while the second consists of the may validly appoint a provincial prosecutor?
so-called “midnight” appointments. The SC in In Re:
Hon. Mateo A. Valenzuela and Hon. Placido B. Held: This question would x x x pivot on the
Vallarta, (298 SCRA 408, Nov. 9, 1998, En Banc proper understanding of the provision of the Revised
[Narvasa C.J.]) clarified this when it held: Administrative Code of 1987 (Book IV, Title III, Chapter
“Section 15, Article VII has a broader II, Section 9) to the effect that –
scope than the Aytona ruling. It may not unreasonably
be deemed to contemplate not only “midnight” “All provincial and city prosecutors and
appointments – those made obviously for partisan their assistants shall be appointed by the
reasons as shown by their number and the time of their President upon the recommendation of the
making – but also appointments presumed made for the Secretary.”
purpose of influencing the outcome of the Presidential
election.” Petitioners contend that an appointment of a provincial
prosecutor mandatorily requires a prior recommendation
87. Ma. Evelyn S. Abeja was a municipal mayor. She ran of the Secretary of Justice endorsing the intended
for reelection but lost. Before she vacated her office, appointment x x x.
though, she extended permanent appointments to
fourteen new employees of the municipal When the Constitution or the law clothes the
government. The incoming mayor, upon assuming President with the power to appoint a subordinate officer,
office, recalled said appointments contending that such conferment must be understood as necessarily
these were “midnight appointments” and, therefore, carrying with it an ample discretion of whom to appoint.
prohibited under Sec. 15, Art. VII of the 1987 It should be here pertinent to state that the President is
Constitution. Should the act of the new mayor of the head of government whose authority includes the
recalling said appointments on the aforestated ground power of control over all “executive departments, bureaus
be sustained? and offices.” Control means the authority of an
empowered officer to alter or modify, or even nullify or
Held: The records reveal that when the set aside, what a subordinate officer has done in the
petitioner brought the matter of recalling the performance of his duties, as well as to substitute the
appointments of the fourteen (14) private respondents judgment of the latter, as and when the former deems it
before the CSC, the only reason he cited to justify his to be appropriate. Expressed in another way, the
action was that these were “midnight appointments” that President has the power to assume directly the functions
are forbidden under Article VII, Section 15 of the of an executive department, bureau and office. It can
Constitution. However, the CSC ruled, and correctly so, accordingly be inferred therefrom that the President can
that the said prohibition applies only to presidential interfere in the exercise of discretion of officials under him
appointments. In truth and in fact, there is no law that or altogether ignore their recommendations.
prohibits local elective officials from making appointments
during the last days of his or her tenure. (De Rama v. It is the considered view of the Court x x x that
Court of Appeals, 353 SCRA 94, Feb. 28, 2001, En the phrase “upon recommendation of the Secretary,”
Banc [Ynares-Santiago]) found in Section 9, Chapter II, Title III, Book IV, of the
Revised Administrative Code, should be interpreted x x x
88. Distinguish the President’s power of general to be a mere advise, exhortation or indorsement, which is
supervision over local governments from his control essentially persuasive in character and not binding or
power. obligatory upon the party to whom it is made. The
recommendation is here nothing really more than advisory
Held: On many occasions in the past, this Court in nature. The President, being the head of the Executive
has had the opportunity to distinguish the power of Department, could very well disregard or do away with
supervision from the power of control. In Taule v. Santos, the action of the departments, bureaus or offices even in
we held that the Chief Executive wielded no more the exercise of discretionary authority, and in so opting,
authority than that of checking whether a local he cannot be said as having acted beyond the scope of his
government or the officers thereof perform their duties as authority. (Bermudez v. Executive Secretary Ruben
provided by statutory enactments. He cannot interfere Torres, G.R. No. 131429, Aug. 4, 1999, 3rd Div.
with local governments provided that the same or its [Vitug])
officers act within the scope of their authority.
Supervisory power, when contrasted with control, is the 90. Distinguish the President’s power to call out the
power of mere oversight over an inferior body; it does not armed forces as their Commander-in-Chief in order to
include any restraining authority over such body. Officers prevent or suppress lawless violence, invasion or
in control lay down the rules in the doing of an act. If rebellion, from his power to proclaim martial and
they are not followed, it is discretionary on his part to suspend the privilege of the writ of habeas corpus.
order the act undone or redone by his subordinate or he Explain why the former is not subject to judicial
may even decide to do it himself. Supervision does not review while the latter two are.
cover such authority. Supervising officers merely see to it

43
Held: There is a clear textual commitment under investigation beyond the pleadings. The factual necessity
the Constitution to bestow on the President full of calling out the armed forces is not easily quantifiable
discretionary power to call out the armed forces and to and cannot be objectively established since matters
determine the necessity for the exercise of such power. considered for satisfying the same is a combination of
Section 18, Article VII of the Constitution, which embodies several factors which are not always accessible to the
the powers of the President as Commander-in-Chief, courts. Besides the absence of textual standards that the
provides in part: court may use to judge necessity, information necessary
to arrive at such judgment might also prove
The President shall be the Commander-in- unmanageable for the courts. Certain pertinent
Chief of all armed forces of the Philippines and information might be difficult to verify, or wholly
whenever it becomes necessary, he may call out unavailable to the courts. In many instances, the
such armed forces to prevent or suppress lawless evidence upon which the President might decide that
violence, invasion or rebellion. In case of invasion there is a need to call out the armed forces may be of a
or rebellion, when the public safety requires it, he nature not constituting technical proof.
may, for a period not exceeding sixty days,
suspend the privilege of the writ of habeas On the other hand, the President as Commander-
corpus, or place the Philippines or any part in-Chief has a vast intelligence network to gather
thereof under martial law. information, some of which may be classified as highly
confidential or affecting the security of the state. In the
The full discretionary power of the President to exercise of the power to call, on-the-spot decisions may
determine the factual basis for the exercise of the be imperatively necessary in emergency situations to avert
calling out power is also implied and further reinforced in great loss of human lives and mass destruction of
the rest of Section 18, Article VII x x x. property. Indeed, the decision to call out the military to
prevent or suppress lawless violence must be done swiftly
Under the foregoing provisions, Congress may and decisively if it were to have any effect at all. Such a
revoke such proclamations (of martial law) or suspension scenario is not farfetched when we consider the present
(of the privilege of the writ of habeas corpus) and the situation in Mindanao, where the insurgency problem
Court may review the sufficiency of the factual basis could spill over the other parts of the country. The
thereof. However, there is no such equivalent provision determination of the necessity for the calling out power if
dealing with the revocation or review of the President's subjected to unfettered judicial scrutiny could be a
action to call out the armed forces. The distinction places veritable prescription for disaster as such power may be
the calling out power in a different category from the unduly straitjacketed by an injunction or a temporary
power to declare martial law and the power to suspend restraining order every time it is exercised.
the privilege of the writ of habeas corpus, otherwise, the
framers of the Constitution would have simply lumped Thus, it is the unclouded intent of the Constitution
together the three powers and provided for their to vest upon the President, as Commander-in-Chief of the
revocation and review without any qualification. Expressio Armed Forces, full discretion to call forth the military when
unios est exclusio alterius. X x x That the intent of the in his judgment it is necessary to do so in order to prevent
Constitution is exactly what its letter says, i.e., that the or suppress lawless violence, invasion or rebellion. Unless
power to call is fully discretionary to the President, is the petitioner can show that the exercise of such
extant in the deliberation of the Constitutional Commission discretion was gravely abused, the President's exercise of
x x x. judgment deserves to be accorded respect from this
Court. (Integrated Bar of the Philippines v. Hon.
The reason for the difference in the treatment of Ronaldo B. Zamora, G.R. No. 141284, Aug. 15,
the aforementioned powers highlights the intent to grant 2000, En Banc [Kapunan])
the President the widest leeway and broadest discretion in
using the power to call out because it is considered as the 91. By issuing a TRO on the date convicted rapist Leo
lesser and more benign power compared to the power to Echegaray is to be executed by lethal injection, the
suspend the privilege of the writ of habeas corpus and the Supreme Court was criticized on the ground, among
power to impose martial law, both of which involve the others, that it encroached on the power of the
curtailment and suppression of certain basic civil rights President to grant reprieve under Section 19, Article
and individual freedoms, and thus necessitating VII, 1987 Constitution. Justify the SC's act.
safeguards by Congress and review by this Court.
Held: Section 19, Article VII of the 1987
Moreover, under Section 18, Article VII of the Constitution is simply the source of power of the President
Constitution, in the exercise of the power to suspend the to grant reprieves, commutations, and pardons and remit
privilege of the writ of habeas corpus or to impose martial fines and forfeitures after conviction by final judgment.
law, two conditions must concur: (1) there must be an This provision, however, cannot be interpreted as denying
actual invasion or rebellion and, (2) public safety must the power of courts to control the enforcement of their
require it. These conditions are not required in the case decisions after the finality. In truth, an accused who has
of the power to call out the armed forces. The only been convicted by final judgment still possesses collateral
criterion is that “whenever it becomes necessary,” the rights and these rights can be claimed in the appropriate
President may call the armed forces “to prevent or courts. For instance, a death convict who becomes insane
suppress lawless violence, invasion or rebellion.” The after his final conviction cannot be executed while in a
implication is that the President is given full discretion and state of insanity (See Article 79 of the Revised Penal
wide latitude in the exercise of the power to call as Code). The suspension of such a death sentence is
compared to the two other powers. undisputably an exercise of judicial power. It is not
usurpation of the presidential power of reprieve though its
If the petitioner fails, by way of proof, to support effect is the same – the temporary suspension of the
the assertion that the President acted without factual execution of the death convict. In the same vein, it
basis, then this Court cannot undertake an independent cannot be denied that Congress can at any time amend

44
R.A. No. 7659 by reducing the penalty of death to life Makabayan] v. Executive Secretary Ronaldo
imprisonment. The effect of such an amendment is like Zamora, G.R. No. 138570, Oct. 10, 2000, En Banc
that of commutation of sentence. But by no stretch of the [Buena])
imagination can the exercise by Congress of its plenary
power to amend laws be considered as a violation of the 94. Which provision of the Constitution applies with
President’s power to commute final sentences of regard to the exercise by the Senate of its
conviction. The powers of the Executive, the Legislative constitutional power to concur with the Visiting Forces
and the Judiciary to save the life of a death convict do not Agreement (VFA)?
exclude each other for the simple reason that there is no
higher right than the right to life. (Echegaray v. Held: The 1987 Philippine Constitution contains
Secretary of Justice, 301 SCRA 96, Jan. 19, 1999, two provisions requiring the concurrence of the Senate on
En Banc [Puno]) treaties or international agreements. Section 21, Article
VII x x x reads:
92. Discuss the nature of a conditional pardon. Is its
grant or revocation by the President subject to judicial “No treaty or international agreement
review? shall be valid and effective unless concurred in by
at least two-thirds of all the Members of the
Held: A conditional pardon is in the nature of a Senate.”
contract between the sovereign power or the Chief
Executive and the convicted criminal to the effect that the Section 25, Article XVIII, provides:
former will release the latter subject to the condition that
if he does not comply with the terms of the pardon, he “After the expiration in 1991 of the
will be recommitted to prison to serve the unexpired Agreement between the Republic of the Philippines and
portion of the sentence or an additional one . By the the United States of America concerning Military Bases,
pardonee’s consent to the terms stipulated in this foreign military bases, troops, or facilities shall not be
contract, the pardonee has thereby placed himself under allowed in the Philippines except under a treaty duly
the supervision of the Chief Executive or his delegate who concurred in by the Senate and, when the Congress so
is duty-bound to see to it that the pardonee complies with requires, ratified by a majority of the votes cast by the
the terms and conditions of the pardon. Under Section people in a national referendum held for that purpose,
64(i) of the Revised Administrative Code, the Chief and recognized as a treaty by the other contracting
Executive is authorized to order “the arrest and re- State.”
incarceration of any such person who, in his judgment,
shall fail to comply with the condition, or conditions of his Section 21, Article VII deals with treaties or
pardon, parole, or suspension of sentence.” It is now a international agreements in general, in which case, the
well-entrenched rule in this jurisdiction that this exercise concurrence of at least two-thirds (2/3) of all the
of presidential judgment is beyond judicial scrutiny. The Members of the Senate is required to make the subject
determination of the violation of the conditional pardon treaty, or international agreement, valid and binding on
rests exclusively in the sound judgment of the Chief the part of the Philippines. This provision lays down the
Executive, and the pardonee, having consented to place general rule on treaties or international agreements and
his liberty on conditional pardon upon the judgment of the applies to any form of treaty with a wide variety of subject
power that has granted it, cannot invoke the aid of the matter, such as, but not limited to, extradition or tax
courts, however erroneous the findings may be upon treaties or those economic in nature. All treaties or
which his recommitment was ordered. international agreements entered into by the Philippines,
regardless of subject matter, coverage, or particular
Xxx designation or appellation, requires the concurrence of the
Ultimately, solely vested in the Chief Executive, Senate to be valid and effective.
who in the first place was the exclusive author of the
conditional pardon and of its revocation, is the corollary In contrast, Section 25, Article XVIII is a special
prerogative to reinstate the pardon if in his own provision that applies to treaties which involve the
judgment, the acquittal of the pardonee from the presence of foreign military bases, troops or facilities in
subsequent charges filed against him, warrants the same. the Philippines. Under this provision, the concurrence of
Courts have no authority to interfere with the grant by the the Senate is only one of the requisites to render
President of a pardon to a convicted criminal. It has been compliance with the constitutional requirements and to
our fortified ruling that a final judicial pronouncement as consider the agreement binding on the Philippines.
to the guilt of a pardonee is not a requirement for the Section 25, Article XVIII further requires that “foreign
President to determine whether or not there has been a military bases, troops, or facilities” may be allowed in the
breach of the terms of a conditional pardon. There is Philippines only by virtue of a treaty duly concurred in by
likewise nil a basis for the courts to effectuate the the Senate, ratified by a majority of the votes cast in a
reinstatement of a conditional pardon revoked by the national referendum held for that purpose if so required
President in the exercise of powers undisputably solely by Congress, and recognized as such by the other
and absolutely in his office. (In Re: Wilfredo contracting State.
Sumulong Torres, 251 SCRA 709, Dec. 29, 1995
[Hermosisima]) It is our considered view that both constitutional
provisions, far from contradicting each other, actually
93. Who has the power to ratify a treaty? share some common ground. These constitutional
provisions both embody phrases in the negative and thus,
Held: In our jurisdiction, the power to ratify is are deemed prohibitory in mandate and character. In
vested in the President and not, as commonly believed, in particular, Section 21 opens with the clause “No treaty x x
the legislature. The role of the Senate is limited only to x,” and Section 25 contains the phrase “shall not be
giving or withholding its consent, or concurrence, to the allowed.” Additionally, in both instances, the concurrence
ratification. (BAYAN [Bagong Alyansang

45
of the Senate is indispensable to render the treaty or of said constitutional provision reveals that the
international agreement valid and effective. proscription covers “foreign military bases, troops, or
facilities.” Stated differently, this prohibition is not limited
To our mind, the fact that the President referred to the entry of troops and facilities without any foreign
the VFA to the Senate under Section 21, Article VII, and bases being established. The clause does not refer to
that the Senate extended its concurrence under the same “foreign military bases, troops, or facilities” collectively
provision, is immaterial. For in either case, whether under but treats them as separate and independent subjects.
Section 21, Article VII or Section 25, Article XVIII, the The use of comma and disjunctive word "or" clearly
fundamental law is crystalline that the concurrence of the signifies disassociation and independence of one thing
Senate is mandatory to comply with the strict from the others included in the enumeration, such that,
constitutional requirements. the provision contemplates three different situations - a
military treaty the subject of which could be either (a)
On the whole, the VFA is an agreement which foreign bases (b) foreign troops, or (c) foreign facilities -
defines the treatment of United States troops and any of the three standing alone places it under the
personnel visiting the Philippines. It provides for the coverage of Section 25, Article XVIII.
guidelines to govern such visits of military personnel, and
further defines the rights of the United States and the To this end, the intention of the framers of the
Philippine government in the matter of criminal Charter x x x is consistent with this interpretation x x x.
jurisdiction, movement of vessels and aircraft, importation
and exportation of equipment, materials and supplies. Moreover, military bases established within the
territory of another state is no longer viable because of
Undoubtedly, Section 25, Article XVIII, which the alternatives offered by new means and weapons of
specifically deals with treaties involving foreign military warfare such as nuclear weapons, guided missiles as well
bases, troops, or facilities, should apply in the instant as huge sea vessels that can stay afloat in the sea even
case. To a certain extent and in a limited sense, however, for months and years without returning to their home
the provisions of Section 21, Article VII will find country. These military warships are actually used as
applicability with regard to the issue and for the sole substitutes for a land-home base not only of military
purpose of determining the number of votes required to aircraft but also of military personnel and facilities.
obtain the valid concurrence of the Senate x x x. Besides, vessels are mobile as compared to a land-based
military headquarters. (BAYAN [Bagong Alyansang
It is a finely-imbedded principle in statutory Makabayan] v. Executive Secretary Ronaldo
construction that a special provision or law prevails over a Zamora, G.R. No. 138570 and Companion Cases,
general one. Lex specialis derogat generali. (BAYAN Oct. 10, 2000, 342 SCRA 449, 481-492, En Banc
[Bagong Alyansang Makabayan] v. Executive [Buena])
Secretary Ronaldo Zamora, G.R. No. 138570 and
Companion Cases, Oct. 10, 2000, 342 SCRA 449, 97. Were the requirements of Sec. 25, Art. XVIII of the
481-492, En Banc [Buena]) 1987 Constitution complied with when the Senate
gave its concurrence to the VFA?
95. Should the contention that since the VFA merely
involved the temporary visits of United States Held: Section 25, Article XVIII disallows foreign
personnel engaged in joint military exercises and not military bases, troops, or facilities in the country, unless
a basing agreement, therefore, Sec. 25, Art. XVIII of the following conditions are sufficiently met, viz: (a) it
the Constitution is inapplicable to the VFA, be upheld? must be under a treaty; (b) the treaty must be duly
concurred in by the Senate and, when so required by
Held: It is specious to argue that Section 25, Congress, ratified by a majority of the votes cast by the
Article XVIII is inapplicable to mere transient agreements people in a national referendum; and (c) recognized as a
for the reason that there is no permanent placing of treaty by the other contracting state.
structure for the establishment of a military base. On this
score, the Constitution makes no distinction between There is no dispute as to the presence of the first
“transient” and “permanent”. Certainly, we find nothing in two requisites in the case of the VFA. The concurrence
Section 25, Article XVIII that requires foreign troops or handed by the Senate through Resolution No. 18 is in
facilities to be stationed or placed permanently in the accordance with the provisions of the Constitution,
Philippines. whether under the general requirement in Section 21,
Article VII, or the specific mandate mentioned in Section
It is a rudiment in legal hermeneutics that when 25, Article XVIII, the provision in the latter article
no distinction is made by law, the Court should not requiring ratification by a majority of the votes cast in a
distinguish - Ubi lex non distinguit nec nos distinguire national referendum being unnecessary since Congress
debemos. (BAYAN [Bagong Alyansang Makabayan] has not required it.
v. Executive Secretary Ronaldo Zamora, G.R. No.
138570 and Companion Cases, Oct. 10, 2000, 342 As to the matter of voting, Section 21, Article VII
SCRA 449, 481-492, En Banc [Buena]) particularly requires that a treaty or international
agreement, to be valid and effective, must be concurred
96. Will it be correct to argue that since no foreign in by at least two-thirds of all the members of the Senate.
military bases, but merely troops and facilities, are On the other hand, Section 25, Article XVIII simply
involved in the VFA, therefore, Section 25, Article provides that the treaty be “duly concurred in by the
XVIII of the Constitution is not controlling? Senate.”

Held: In like manner, we do not subscribe to the Applying the foregoing constitutional provisions, a
argument that Section 25, Article XVIII is not controlling two-thirds vote of all the members of the Senate is clearly
since no foreign military bases, but merely foreign troops required so that the concurrence contemplated by law
and facilities, are involved in the VFA. Notably, a perusal may be validly obtained and deemed present. While it is

46
true that Section 25, Article XVIII requires, among other
things, that the treaty - the VFA, in the instant case - be xxx
“duly concurred in by the Senate,” it is very true however
that said provision must be related and viewed in light of With the ratification of the VFA, which is
the clear mandate embodied in Section 21, Article VII, equivalent to final acceptance, and with the exchange of
which in more specific terms, requires that the notes between the Philippines and the United States of
concurrence of a treaty, or international agreement, be America, it now becomes obligatory and incumbent on our
made by a two-thirds vote of all the members of the part, under the principles of international law, to be bound
Senate. Indeed, Section 25, Article XVIII must not be by the terms of the agreement. (BAYAN [Bagong
treated in isolation to Section 21, Article VII. Alyansang Makabayan] v. Executive Secretary
Ronaldo Zamora, G.R. No. 138570 and Companion
As noted, the “concurrence requirement” under Cases, Oct. 10, 2000, 342 SCRA 449, 481-492, En
Section 25, Article XVIII must be construed in relation to Banc [Buena])
the provisions of Section 21, Article VII. In a more
particular language, the concurrence of the Senate 98. Are the “Balikatan” exercises covered by the Visiting
contemplated under Section 25, Article XVIII means that Forces Agreement?
at least two-thirds of all the members of the Senate
favorably vote to concur with the treaty - the VFA in the Held: The holding of “Balikatan 02-1” must be
instant case. studied in the framework of the treaty antecedents to
which the Philippines bound itself. The first of these is the
xxx Mutual Defense Treaty (MDT, for brevity). The MDT has
been described as the “core” of the defense relationship
Having resolved that the first two requisites between the Philippines and its traditional ally, the United
prescribed in Section 25, Article XVIII are present, we States. Its aim is to enhance the strategic and
shall now pass upon and delve on the requirement that technological capabilities of our armed forces through
the VFA should be recognized as a treaty by the United joint training with its American counterparts; the
States of America. “Balikatan” is the largest such training exercise directly
supporting the MDT’s objectives. It is this treaty to which
xxx the VFA adverts and the obligations thereunder which it
seeks to reaffirm.
This Court is of the firm view that the phrase
“recognized as a treaty” means that the other contracting The lapse of the US-Philippine Bases Agreement
party accepts or acknowledges the agreement as a treaty. in 1992 and the decision not to renew it created a vacuum
To require the other contracting state, The United States in US-Philippine defense relations, that is, until it was
of America in this case, to submit the VFA to the United replaced by the Visiting Forces Agreement. It should be
States Senate for concurrence pursuant to its Constitution, recalled that on October 10, 2000, by a vote of eleven to
is to accord strict meaning to the phrase. three, this Court upheld the validity of the VFA (BAYAN,
et. al. v. Zamora, et. al., 342 SCRA 449 [2000]). The VFA
Well-entrenched is the principle that the words provides the “regulatory mechanism” by which “United
used in the Constitution are to be given their ordinary States military and civilian personnel [may visit]
meaning except where technical terms are employed, in temporarily in the Philippines in connection with activities
which case the significance thus attached to them approved by the Philippine Government.” It contains
prevails. Its language should be understood in the sense provisions relative to entry and departure of American
they have in common use. personnel, driving and vehicle registration, criminal
jurisdiction, claims, importation and exportation,
Moreover, it is inconsequential whether the United movement of vessels and aircraft, as well as the duration
States treats the VFA only as an executive agreement of the agreement and its termination. It is the VFA which
because, under international law, an executive agreement gives continued relevance to the MDT despite the passage
is as binding as a treaty. To be sure, as long as the VFA of years. Its primary goal is to facilitate the promotion of
possesses the elements of an agreement under optimal cooperation between American and Philippine
international law, the said agreement is to be taken military forces in the event of an attack by a common foe.
equally as a treaty.
The first question that should be addressed is
xxx whether “Balikatan 02-1” is covered by the Visiting Forces
Agreement. To resolve this, it is necessary to refer to the
The records reveal that the United States VFA itself. Not much help can be had therefrom,
Government, through Ambassador Thomas C. Hubbard, unfortunately, since the terminology employed is itself the
has stated that the United States government has fully source of the problem. The VFA permits United States
committed to living up to the terms of the VFA. For as personnel to engage, on an impermanent basis, in
long as the United States of America accepts or “activities,” the exact meaning of which was left
acknowledges the VFA as a treaty, and binds itself further undefined. The expression is ambiguous, permitting a
to comply with its obligations under the treaty, there is wide scope of undertakings subject only to the approval of
indeed marked compliance with the mandate of the the Philippine government. The sole encumbrance placed
Constitution. on its definition is couched in the negative, in that United
States personnel must “abstain from any activity
Worth stressing too, is that the ratification, by the inconsistent with the spirit of this agreement, and in
President, of the VFA and the concurrence of the Senate particular, from any political activity.” All other activities,
should be taken as a clear and unequivocal expression of in other words, are fair game.
our nation's consent to be bound by said treaty, with the
concomitant duty to uphold the obligations and We are not completely unaided, however. The
responsibilities embodied thereunder. Vienna Convention on the Law of Treaties, which contains

47
provisos governing interpretations of international
agreements, state x x x. 100. What are the requisites before the Court can
exercise the power of judicial review?
It is clear from the foregoing that the cardinal rule
of interpretation must involve an examination of the text, Held: 1. The time-tested standards for the
which is presumed to verbalize the parties’ intentions. exercise of judicial review are: (1) the existence of an
The Convention likewise dictates what may be used as appropriate case; (2) an interest personal and substantial
aids to deduce the meaning of terms, which it refers to as by the party raising the constitutional question; (3) the
the context of the treaty, as well as other elements may plea that the function be exercised at the earliest
be taken into account alongside the aforesaid context. X opportunity; and (4) the necessity that the constitutional
xx question be passed upon in order to decide the case
(Separate Opinion, Kapunan, J., in Isagani Cruz v.
The Terms of Reference rightly fall within the Secretary of Environment and Natural Resources,
context of the VFA. et al., G.R. No. 135385, Dec. 6, 2000, En Banc)

After studied reflection, it appeared farfetched 2. When questions of constitutional significance


that the ambiguity surrounding the meaning of the word are raised, the Court can exercise its power of judicial
“activities” arose from accident. In our view, it was review only if the following requisites are complied with,
deliberately made that way to give both parties a certain namely: (1) the existence of an actual and appropriate
leeway in negotiation. In this manner, visiting US forces case; (2) a personal and substantial interest of the party
may sojourn in Philippine territory for purposes other than raising the constitutional question; (3) the exercise of
military. As conceived, the joint exercises may include judicial review is pleaded at the earliest opportunity; and
training on new techniques of patrol and surveillance to (4) the constitutional question is the lis mota of the case.
protect the nation’s marine resources, sea search-and- (Integrated Bar of the Philippines v. Hon. Ronaldo
destroy operations to assist vessels in distress, disaster B. Zamora, G.R. No. 141284, Aug. 15, 2000, En
relief operations, civic action projects such as the building Banc [Kapunan])
of school houses, medical and humanitarian missions, and
the like. 101. What are the requisites for the proper exercise of
the power of judicial review? Illustrative case.
Under these auspices, the VFA gives legitimacy to
the current Balikatan exercises. It is only logical to Held: Respondents assert that the petition fails
assume that “Balikatan 02-1,” a “mutual anti-terrorism to satisfy all the four requisites before this Court may
advising, assisting and training exercise,” falls under the exercise its power of judicial review in constitutional
umbrella of sanctioned or allowable activities in the cases. Out of respect for the acts of the Executive
context of the agreement. Both the history and intent of department, which is co-equal with this Court,
the Mutual Defense Treaty and the VFA support the respondents urge this Court to refrain from reviewing the
conclusion that combat-related activities – as opposed to constitutionality of the ad interim appointments issued by
combat itself – such as the one subject of the instant the President to Benipayo, Borra and Tuason unless all the
petition, are indeed authorized. (Arthur D. Lim and four requisites are present. X x x
Paulino R. Ersando v. Honorable Executive
Secretary, G.R. No. 151445, April 11, 2002, En Banc Respondents argue that the second, third and
[De Leon]) fourth requisites are absent in this case. Respondents
maintain that petitioner does not have a personal and
99. What is the power of impoundment of the President? substantial interest in the case because she has not
What are its principal sources? sustained a direct injury as a result of the ad interim
appointments of Benipayo, Borra and Tuason and their
Held: Impoundment refers to the refusal of the assumption of office. Respondents point out that
President, for whatever reason, to spend funds made petitioner does not claim to be lawfully entitled to any of
available by Congress. It is the failure to spend or the positions assumed by Benipayo, Borra or Tuason.
obligate budget authority of any type. Neither does petitioner claim to be directly injured by the
appointments of these three respondents.
Proponents of impoundment have invoked at least
three principal sources of the authority of the President. Respondents also contend that petitioner failed to
Foremost is the authority to impound given to him either question the constitutionality of the ad interim
expressly or impliedly by Congress. Second is the appointments at the earliest opportunity. Petitioner filed
executive power drawn from the President’s role as the petition only on August 3, 2001 despite the fact that
Commander-in-Chief. Third is the Faithful Execution the ad interim appointments of Benipayo, Borra and
Clause. Tuason were issued as early as March 22, 2001.
Moreover, the petition was filed after the third time that
The proponents insist that a faithful execution of these three respondents were issued ad interim
the laws requires that the President desist from appointments.
implementing the law if doing so would prejudice public
interest. An example given is when through efficient and Respondents insist that the real issue in this case
prudent management of a project, substantial savings are is the legality of petitioner’s reassignment from the EID to
made. In such a case, it is sheer folly to expect the the Law Department. Consequently, the constitutionality
President to spend the entire amount budgeted in the law. of the ad interim appointments is not the lis mota of this
(PHILCONSA v. Enriquez, 235 SCRA 506, Aug. 9, case.
1994 [Quiason])
We are not persuaded.

The Judicial Department

48
Benipayo reassigned petitioner from the EID, issue raised by petitioner. (Matibag v. Benipayo, 380
where she was Acting Director, to the Law Department, SCRA 49, April 2, 2002, En Banc [Carpio])
where she was placed on detail. Respondents claim that
the reassignment was “pursuant to x x x Benipayo’s 102. What is an “actual case or controversy”?
authority as Chairman of the Commission on Elections,
and as the Commission’s Chief Executive Officer.” Held: An “actual case or controversy” means an
Evidently, respondents’ anchor the legality of petitioner’s existing case or controversy which is both ripe for
reassignment on Benipayo’s authority as Chairman of the resolution and susceptible of judicial determination, and
COMELEC. The real issue then turns on whether or not that which is not conjectural or anticipatory, or that which
Benipayo is the lawful Chairman of the COMELEC. Even if seeks to resolve hypothetical or feigned constitutional
petitioner is only an Acting director of the EID, her problems. A petition raising a constitutional question does
reassignment is without legal basis if Benipayo is not the not present an “actual controversy,” unless it alleges a
lawful COMELEC Chairman, an office created by the legal right or power. Moreover, it must show that a
Constitution. conflict of rights exists, for inherent in the term
“controversy” is the presence of opposing views or
On the other hand, if Benipayo is the lawful contentions. Otherwise, the Court will be forced to
COMELEC Chairman because he assumed office in resolve issues which remain unfocused because they lack
accordance with the Constitution, then petitioner’s such concreteness provided when a question emerges
reassignment is legal and she has no cause to complain precisely framed from a clash of adversary arguments
provided the reassignment is in accordance with the Civil exploring every aspect of a multi-faceted situation
Service Law. Clearly, petitioner has a personal and embracing conflicting and demanding interests. The
material stake in the resolution of the constitutionality of controversy must also be justiciable; that is, it must be
Benipayo’s assumption of office. Petitioner’s personal and susceptible of judicial determination. (Integrated Bar of
substantial injury, if Benipayo is not the lawful COMELEC the Philippines v. Hon. Ronaldo B. Zamora, G.R. No.
Chairman, clothes her with the requisite locus standi to 141284, Aug. 15, 2000, En Banc [Kapunan])
raise the constitutional issue in this petition.
103. Petitioners Isagani Cruz and Cesar Europa
Respondents harp on petitioner’s belated act of brought a suit for prohibition and mandamus as
questioning the constitutionality of the ad interim citizens and taxpayers, assailing the constitutionality
appointments of Benipayo, Borra and Tuason. Petitioner of certain provisions of Republic Act No. 8371,
filed the instant petition only on August 3, 2001, when the otherwise known as the Indigenous Peoples Rights
first ad interim appointments were issued as early as Act of 1997 (IPRA), and its Implementing Rules and
March 22, 2001. However, it is not the date of filing of Regulations. A preliminary issue resolved by the SC
the petition that determines whether the constitutional was whether the petition presents an actual
issue was raised at the earliest opportunity. The earliest controversy.
opportunity to raise a constitutional issue is to raise it in
the pleadings before a competent court that can resolve Held: Courts can only decide actual
the same, such that, “if it is not raised in the pleadings, it controversies, not hypothetical questions or cases . The
cannot be considered on appeal.” Petitioner questioned threshold issue, therefore, is whether an “appropriate
the constitutionality of the ad interim appointments of case” exists for the exercise of judicial review in the
Benipayo, Borra and Tuason when she filed her petition present case.
before this Court, which is the earliest opportunity for
pleading the constitutional issue before a competent body. Xxx
Furthermore, this Court may determine, in the exercise of
sound discretion, the time when a constitutional issue may In the case at bar, there exists a live controversy
be passed upon. There is no doubt petitioner raised the involving a clash of legal rights. A law has been enacted,
constitutional issue on time. and the Implementing Rules and Regulations approved.
Money has been appropriated and the government
Moreover, the legality of petitioner’s reassignment agencies concerned have been directed to implement the
hinges on the constitutionality of Benipayo’s ad interim statute. It cannot be successfully maintained that we
appointment and assumption of office. Unless the should await the adverse consequences of the law in
constitutionality of Benipayo’s ad interim appointment and order to consider the controversy actual and ripe for
assumption of office is resolved, the legality of petitioner’s judicial resolution. It is precisely the contention of the
reassignment from the EID to the Law Department cannot petitioners that the law, on its face, constitutes an
be determined. Clearly, the lis mota of this case is the unconstitutional abdication of State ownership over lands
very constitutional issue raised by petitioner. of the public domain and other natural resources.
Moreover, when the State machinery is set into motion to
In any event, the issue raised by petitioner is of implement an alleged unconstitutional statute, this Court
paramount importance to the public. The legality of the possesses sufficient authority to resolve and prevent
directives and decisions made by the COMELEC in the imminent injury and violation of the constitutional process.
conduct of the May 14, 2001 national elections may be (Separate Opinion, Kapunan, J., in Isagani Cruz v.
put in doubt if the constitutional issue raised by petitioner Secretary of Environment and Natural Resources,
is left unresolved. In keeping with this Court’s duty to et al., G.R. No. 135385, Dec. 6, 2000, En Banc)
determine whether other agencies of government have
remained within the limits of the Constitution and have 104. When is an action considered “moot”? May the
not abused the discretion given them, this Court may court still resolve the case once it has become moot
even brush aside technicalities of procedure and resolve and academic?
any constitutional issue raised. Here the petitioner has
complied with all the requisite technicalities. Moreover, Held: 1. It is alleged by respondent that, with
public interest requires the resolution of the constitutional respect to the PCCR [Preparatory Commission on

49
Constitutional Reform], this case has become moot and proclaimed and its officials appointed, the case
academic. We agree. before Us cannot truly be viewed as already moot
and academic. Continuation of the existence of
An action is considered “moot” when it no longer this newly proclaimed province which petitioners
presents a justiciable controversy because the issues strongly profess to have been illegally born,
involved have become academic or dead. Under E.O. No. deserves to be inquired into by this Tribunal so
43, the PCCR was instructed to complete its task on or that, if indeed, illegality attaches to its creation,
before June 30, 1999. However, on February 19, 1999, the commission of that error should not provide
the President issued Executive Order No. 70 (E.O. No. the very excuse for perpetration of such wrong.
70), which extended the time frame for the completion of For this Court to yield to the respondents’ urging
the commission’s work x x x. The PCCR submitted its that, as there has been fait accompli, then this
recommendations to the President on December 20, 1999 Court should passively accept and accede to the
and was dissolved by the President on the same day. It prevailing situation is an unacceptable suggestion.
had likewise spent the funds allocated to it. Thus, the Dismissal of the instant petition, as respondents
PCCR has ceased to exist, having lost its raison d’ être. so propose is a proposition fraught with mischief.
Subsequent events have overtaken the petition and the Respondents’ submission will create a dangerous
Court has nothing left to resolve. precedent. Should this Court decline now to
perform its duty of interpreting and indicating
The staleness of the issue before us is made more what the law is and should be, this might tempt
manifest by the impossibility of granting the relief prayed again those who strut about in the corridors of
for by petitioner. Basically, petitioner asks this Court to power to recklessly and with ulterior motives,
enjoin the PCCR from acting as such. Clearly, prohibition create, merge, divide and/or alter the boundaries
is an inappropriate remedy since the body sought to be of political subdivisions, either brazenly or
enjoined no longer exists. It is well-established that stealthily, confident that this Court will abstain
prohibition is a preventive remedy and does not lie to from entertaining future challenges to their acts if
restrain an act that is already fait accompli. At this point, they manage to bring about a fait accompli.”
any ruling regarding the PCCR would simply be in the (City of Pasig v. COMELEC, 314 SCRA 179, Sept. 10,
nature of an advisory opinion, which is definitely beyond 1999, En Banc [Ynares-Santiago])
the permissible scope of judicial power. (Gonzales v.
Narvasa, 337 SCRA 733, Aug. 14, 2000, En Banc 106. On May 1, 2001, President Macapagal-Arroyo,
[Gonzaga-Reyes]) faced by an “angry and violent mob armed with
explosives, firearms, bladed weapons, clubs, stones
2. The petition which was filed by private and other deadly weapons” assaulting and attempting
respondents before the trial court sought the issuance of to break into Malacanang, issued Proclamation No. 38
a writ of mandamus, to command petitioners to admit declaring that there was a state of rebellion in the
them for enrolment. Taking into account the admission of National Capital Region. She likewise issued General
private respondents that they have finished their Nursing Order No. 1 directing the Armed Forces of the
course at the Lanting College of Nursing even before the Philippines and the Philippine National Police to
promulgation of the questioned decision, this case has suppress the rebellion in the National Capital Region.
clearly been overtaken by events and should therefore be Warrantless arrests of several alleged leaders and
dismissed. However, the case of Eastern Broadcasting promoters of the “rebellion” were thereafter effected.
Corporation (DYRE) v. Dans is the authority for the view Hence, several petitions were filed before the SC
that “even if a case were moot and academic, a statement assailing the declaration of State of Rebellion by
of the governing principle is appropriate in the resolution President Gloria Macapagal-Arroyo and the
of dismissal for the guidance not only of the parties but of warrantless arrests allegedly effected by virtue
others similarly situated.” We shall adhere to this view thereof.
and proceed to dwell on the merits of this petition.
(University of San Agustin, Inc. v. Court of Appeals, Held: All the foregoing petitions assail the
230 SCRA 761, 770, March 7, 1994 [Nocon]) declaration of state of rebellion by President Gloria
Macapagal-Arroyo and the warrantless arrests allegedly
105. Should the Court still resolve the case despite that effected by virtue thereof, as having no basis both in fact
the issue has already become moot and academic? and in law. Significantly, on May 6, 2001, President
Exception. Macapagal-Arroyo ordered the lifting of the declaration of
a “state of rebellion” in Metro Manila. Accordingly, the
Held: Neither do we agree that merely because instant petitions have been rendered moot and academic.
a plebiscite had already been held in the case of the As to petitioners’ claim that the proclamation of a “state of
proposed Barangay Napico, the petition of the Municipality rebellion” is being used by the authorities to justify
of Cainta has already been rendered moot and academic. warrantless arrests, the Secretary of Justice denies that it
The issue raised by the Municipality of Cainta in its has issued a particular order to arrest specific persons in
petition before the COMELEC against the holding of the connection with the “rebellion.” He states that what is
plebiscite for the creation of Barangay Napico are still extant are general instructions to law enforcement officers
pending determination before the Antipolo Regional Trial and military agencies to implement Proclamation No. 38.
Court. x x x. With this declaration, petitioners’ apprehensions as
to warrantless arrests should be laid to rest. (Lacson v.
In Tan v. Commission on Elections, we struck Perez, 357 SCRA 756, May 10, 2001, En Banc
down the moot and academic argument as follows – [Melo])

“Considering that the legality of the 107. In connection with the May 11, 1998 elections,
plebiscite itself is challenged for non-compliance the COMELEC issued a resolution prohibiting the
with constitutional requisites, the fact that such conduct of exit polls on the ground, among others,
plebiscite had been held and a new province that it might cause disorder and confusion considering

50
the randomness of selecting interviewees, which uphold the rule of law and the Constitution, the
further makes the exit polls unreliable. The Integrated Bar of the Philippines (IBP) filed a petition
constitutionality of this resolution was challenged by before the SC questioning the validity of the order of
ABS-CBN Broadcasting Corporation as violative of the President commanding the deployment and
freedom of expression. The Solicitor General utilization of the Philippine Marines to assist the
contends that the petition has been rendered moot Philippine National Police (PNP) in law enforcement by
and academic because the May 11, 1998 election has joining the latter in visibility patrols around the
already been held and done with and, therefore, there metropolis. The Solicitor General questioned the legal
is no longer any actual controversy to be resolved. standing of the IBP to file the petition? Resolve.
Resolve.
Held: In the case at bar, the IBP primarily
Held: While the assailed Resolution referred anchors its standing on its alleged responsibility to uphold
specifically to the May 11, 1998 election, its implications the rule of law and the Constitution. Apart from this
on the people’s fundamental freedom of expression declaration, however, the IBP asserts no other basis in
transcend the past election. The holding of periodic support of its locus standi. The mere invocation by the
elections is a basic feature of our democratic government. IBP of its duty to preserve the rule of law and nothing
By its very nature, exit polling is tied up with elections. more, while undoubtedly true, is not sufficient to clothe it
To set aside the resolution of the issue now will only with standing in this case. This is too general an interest
postpone a task that could well crop up again in future which is shared by other groups and the whole citizenry.
elections. Based on the standards above-stated, the IBP has failed
to present a specific and substantial interest in the
In any event, in Salonga v. Cruz Pano, the Court resolution of the case. Its fundamental purpose which,
had occasion to reiterate that it “also has the duty to under Section 2, Rule 139-A of the Rules of Court, is to
formulate guiding and controlling constitutional principles, elevate the standards of the law profession and to
precepts, doctrines, or rules. It has the symbolic function improve the administration of justice is alien to, and
of educating bench and bar on the extent of protection cannot be affected by the deployment of the Marines. x x
given by constitutional guarantees.” Since the x Moreover, the IBP x x x has not shown any specific
fundamental freedoms of speech and of the press are injury which it has suffered or may suffer by virtue of the
being invoked here, we have resolved to settle, for the questioned governmental act. Indeed, none of its
guidance of posterity, whether they likewise protect the members x x x has sustained any form of injury as a
holding of exit polls and the dissemination of data derived result of the operation of the joint visibility patrols.
therefrom. (ABS-CBN Broadcasting Corporation v. Neither is it alleged that any of its members has been
COMELEC, G.R. No. 133486, Jan. 28, 2000, En Banc arrested or that their civil liberties have been violated by
[Panganiban]) the deployment of the Marines. What the IBP projects as
injurious is the supposed “militarization” of law
108. What is the meaning of "legal standing" or locus enforcement which might threaten Philippine democratic
standi? institutions and may cause more harm than good in the
long run. Not only is the presumed “injury” not personal
Held: 1. “Legal standing” or locus standi has in character, it is likewise too vague, highly speculative
been defined as a personal and substantial interest in the and uncertain to satisfy the requirement of standing.
case such that the party has sustained or will sustain Since petitioner has not successfully established a direct
direct injury as a result of the governmental act that is and personal injury as a consequence of the questioned
being challenged. The term “interest” means a material act, it does not possess the personality to assail the
interest, an interest in issue affected by the decree, as validity of the deployment of the Marines. (Integrated
distinguished from mere interest in the question involved, Bar of the Philippines v. Hon. Ronaldo B. Zamora,
or a mere incidental interest. The gist of the question of G.R. No. 141284, Aug. 15, 2000, En Banc
standing is whether a party alleges “such personal stake [Kapunan])
in the outcome of the controversy as to assure that
concrete adverseness which sharpens the presentation of 110. Considering the lack of requisite standing of the
issues upon which the court depends for illumination of IBP to file the petition questioning the validity of the
difficult constitutional questions.” (Integrated Bar of order of the President to deploy and utilize the
the Philippines v. Hon. Ronaldo B. Zamora, G.R. No. Philippine Marines to assist the PNP in law
141284, Aug. 15, 2000) enforcement, may the Court still properly take
cognizance of the case?
2. In addition to the existence of an actual case
or controversy, a person who assails the validity of a Held: Having stated the foregoing, it must be
statute must have a personal and substantial interest in emphasized that this Court has the discretion to take
the case, such that, he has sustained, or will sustain, a cognizance of a suit which does not satisfy the
direct injury as a result of its enforcement . Evidently, the requirement of legal standing when paramount interest is
rights asserted by petitioners as citizens and taxpayers are involved. In not a few cases, the Court has adopted a
held in common by all the citizens, the violation of which liberal attitude on the locus standi of a petitioner where
may result only in a “generalized grievance”. Yet, in a the petitioner is able to craft an issue of transcendental
sense, all citizen's and taxpayer's suits are efforts to air significance to the people. Thus, when the issues raised
generalized grievances about the conduct of government are of paramount importance to the public, the Court may
and the allocation of power. (Separate Opinion, brush aside technicalities of procedure. In this case, a
Kapunan, J., in Isagani Cruz v. Secretary of reading of the petition shows that the IBP has advanced
Environment and Natural Resources, et al., G.R. No. constitutional issues which deserve the attention of this
135385, Dec. 6, 2000, En Banc) Court in view of their seriousness, novelty and weight as
precedents. Moreover, because peace and order are
109. Asserting itself as the official organization of under constant threat and lawless violence occurs in
Filipino lawyers tasked with the bounden duty to increasing tempo, undoubtedly aggravated by the

51
Mindanao insurgency problem, the legal controversy of the law or the Constitution. Thus, a taxpayer’s action is
raised in the petition almost certainly will not go away. It properly brought only when there is an exercise by
will stare us in the face again. It, therefore, behooves the Congress of its taxing or spending power. This was our
Court to relax the rules on standing and to resolve the ruling in a recent case wherein petitioners
issue now, rather than later. (Integrated Bar of the Telecommunications and Broadcast Attorneys of the
Philippines v. Hon. Ronaldo B. Zamora, G.R. No. Philippines (TELEBAP) and GMA Network, Inc. questioned
141284, Aug. 15, 2000) the validity of Section 92 of B.P. Blg. 881 (otherwise
known as the “Omnibus Election Code”) requiring radio
111. Discuss the nature of a taxpayer’s suit. When and television stations to give free air time to the
may it be allowed? Commission on Elections during the campaign period
(Telecommunications and Broadcast Attorneys of the
Held: 1. Petitioner and respondents agree that Philippines, Inc. v. Commission on Elections, 289 SCRA
to constitute a taxpayer's suit, two requisites must be 337 [1998]). The Court held that petitioner TELEBAP did
met, namely, that public funds are disbursed by a political not have any interest as a taxpayer since the assailed law
subdivision or instrumentality and in doing so, a law is did not involve the taxing or spending power of Congress.
violated or some irregularity is committed, and that the
petitioner is directly affected by the alleged ultra vires act . Many other rulings have premised the grant or
The same pronouncement was made in Kilosbayan, Inc. v. denial of standing to taxpayers upon whether or not the
Guingona, Jr., where the Court also reiterated its liberal case involved a disbursement of public funds by the
stance in entertaining so-called taxpayer's suits, especially legislature. In Sanidad v. Commission on Elections, the
when important issues are involved. A closer examination petitioners therein were allowed to bring a taxpayer’s suit
of the facts of this case would readily demonstrate that to question several presidential decrees promulgated by
petitioner's standing should not even be made an issue then President Marcos in his legislative capacity calling for
here, “since standing is a concept in constitutional law and a national referendum, with the Court explaining that –
here no constitutional question is actually involved.”
X x x [i]t is now an ancient rule that the
In the case at bar, disbursement of public funds valid source of a statute – Presidential Decrees
was only made in 1975 when the Province bought the are of such nature – may be contested by one
lands from Ortigas at P110.00 per square meter in line who will sustain a direct injury as a result of its
with the objectives of P.D. 674. Petitioner never referred enforcement. At the instance of taxpayers, laws
to such purchase as an illegal disbursement of public providing for the disbursement of public funds
funds but focused on the alleged fraudulent reconveyance may be enjoined, upon the theory that the
of said property to Ortigas because the price paid was expenditure of public funds by an officer of the
lower than the prevailing market value of neighboring lots. State for the purpose of executing an
The first requirement, therefore, which would make this unconstitutional act constitutes a misapplication of
petition a taxpayer's suit is absent. The only remaining such funds. The breadth of Presidential Decree
justification for petitioner to be allowed to pursue this No. 991 carries an appropriation of Five Million
action is whether it is, or would be, directly affected by Pesos for the effective implementation of its
the act complained of. As we stated in Kilosbayan, Inc. v. purposes. Presidential Decree No. 1031
Morato, appropriates the sum of Eight Million Pesos to
carry out its provisions. The interest of the
“Standing is a special concern in aforenamed petitioners as taxpayers in the lawful
constitutional law because in some cases suits are expenditure of these amounts of public money
brought not by parties who have been personally sufficiently clothes them with that personality to
injured by the operation of a law or by official litigate the validity of the Decrees appropriating
action taken, but by concerned citizens, taxpayers said funds x x x.
or voters who actually sue in the public interest.
Hence the question in standing is whether such In still another case, the Court held that petitioners – the
parties have 'alleged such a personal stake in the Philippine Constitution Association, Inc., a non-profit civic
outcome of the controversy as to assure that organization – had standing as taxpayers to question the
concrete adverseness which sharpens the constitutionality of Republic Act No. 3836 insofar as it
presentation of issues upon which the court so provides for retirement gratuity and commutation of
largely depends for illumination of difficult vacation and sick leaves to Senators and Representatives
constitutional questions.' (Citing Baker v. Carr, and to the elective officials of both houses of Congress
369 U.S. 186, 7 L. Ed. 2d 633 [1962])” (Philippine Constitution Association, Inc. v. Gimenez).
And in Pascual v. Secretary of Public Works, the Court
Undeniably, as a taxpayer, petitioner would somehow be allowed petitioner to maintain a taxpayer’s suit assailing
adversely affected by an illegal use of public money. the constitutional soundness of Republic Act No. 920
When, however, no such unlawful spending has been appropriating P85,000 for the construction, repair and
shown x x x, petitioner, even as a taxpayer, cannot improvement of feeder roads within private property. All
question the transaction validly executed by and between these cases involved the disbursement of public funds by
the Province and Ortigas for the simple reason that it is means of a law.
not privy to said contract. In other words, petitioner has
absolutely no cause of action, and consequently no locus Meanwhile, in Bugnay Construction and
standi, in the instant case. (The Anti-Graft League of Development Corporation v. Laron, the Court declared
the Philippines, Inc. v. San Juan, 260 SCRA 250, that the trial court was wrong in allowing respondent
253-255, Aug. 1, 1996, En Banc [Romero]) Ravanzo to bring an action for injunction in his capacity as
a taxpayer in order to question the legality of the contract
2. A taxpayer is deemed to have the standing to of lease covering the public market entered into between
raise a constitutional issue when it is established that the City of Dagupan and petitioner. The Court declared
public funds have been disbursed in alleged contravention that Ravanzo did not possess the requisite standing to

52
bring such taxpayer’s suit since “[o]n its face, and there is reason of the implementation of the questioned
no evidence to the contrary, the lease contract entered administrative issuance.
into between petitioner and the City shows that no public
funds have been or will be used in the construction of the A justiciable controversy has been defined as, “a
market building.” definite and concrete dispute touching on the legal
relations of parties having adverse legal interests” which
Coming now to the instant case, it is readily may be resolved by a court of law through the application
apparent that there is no exercise by Congress of its of a law. Courts have no judicial power to review cases
taxing or spending power. The PCCR was created by the involving political questions and as a rule, will desist from
President by virtue of E.O. No. 43, as amended by E.O. taking cognizance of speculative or hypothetical cases,
No. 70. Under Section 7 of E.O. No. 43, the amount of P3 advisory opinions and in cases that has become moot .
million is “appropriated” for its operational expenses “to Subject to certain well-defined exceptions courts will not
be sourced from the funds of the Office of the President.” touch an issue involving the validity of a law unless there
x x x The appropriations for the PCCR were authorized by has been a governmental act accomplished or performed
the President, not by Congress. In fact, there was no that has a direct adverse effect on the legal right of the
appropriation at all. “In a strict sense, appropriation has person contesting its validity. In the case of PACU v.
been defied ‘as nothing more than the legislative Secretary of Education the petition contesting the validity
authorization prescribed by the Constitution that money of a regulation issued by the Secretary of Education
may be paid out of the Treasury,’ while appropriation requiring private schools to secure a permit to operate
made by law refers to ‘the act of the legislature setting was dismissed on the ground that all the petitioners have
apart or assigning to a particular use a certain sum to be permits and are actually operating under the same. The
used in the payment of debt or dues from the State to its petitioners questioned the regulation because of the
creditors.’” The funds used for the PCCR were taken from possibility that the permit might be denied them in the
funds intended for the Office of the President, in the future. This Court held that there was no justiciable
exercise of the Chief Executive’s power to transfer funds controversy because the petitioners suffered no wrong by
pursuant to Section 25 (5) of Article VI of the the implementation of the questioned regulation and
Constitution. therefore, they are not entitled to relief. A mere
apprehension that the Secretary of Education will
In the final analysis, it must be stressed that the withdraw the permit does not amount to justiciable
Court retains the power to decide whether or not it will controversy. The questioned regulation in the PACU case
entertain a taxpayer’s suit . In the case at bar, there being may be questioned by a private school whose permit to
no exercise by Congress of its taxing or spending power, operate has been revoked or one whose application
petitioner cannot be allowed to question the creation of therefore has been denied.
the PCCR in his capacity as a taxpayer, but rather, he
must establish that he has a “personal and substantial This Court cannot rule on the basis of petitioners’
interest in the case and that he has sustained or will speculation that the DENR will approve the application of
sustain direct injury as a result of its enforcement.” In the heirs of Carantes. There must be an actual
other words, petitioner must show that he is a real party governmental act which directly causes or will imminently
in interest – that he will stand to be benefited or injured cause injury to the alleged legal right of the petitioner to
by the judgment or that he will be entitled to the avails of possess the land before the jurisdiction of this Court may
the suit. Nowhere in his pleadings does petitioner be invoked. There is no showing that the petitioners were
presume to make such a representation. (Gonzales v. being evicted from the land by the heirs of Carantes under
Narvasa, 337 SCRA 733, Aug. 14, 2000, En Banc orders from the DENR. The petitioners’ allegation that
[Gonzaga-Reyes]) certain documents from the DENR were shown to them by
the heirs of Carantes to justify eviction is vague, and it
112. What is the meaning of “justiciable controversy” would appear that the petitioners did not verify if indeed
as requisite for the proper exercise of the power of the respondent DENR or its officers authorized the
judicial review? Illustrative case. attempted eviction. Suffice it to say that by the
petitioners’ own admission that the respondents are still
Held: From a reading of the records it appears to processing and have not approved the application of the
us that the petition was prematurely filed. Under the heirs of Carantes, the petitioners alleged right to possess
undisputed facts there is as yet no justiciable controversy the land is not violated nor is in imminent danger of being
for the court to resolve and the petition should have been violated, as the DENR may or may not approve Carantes’
dismissed by the appellate court on this ground. application. Until such time, the petitioners are simply
speculating that they might be evicted from the premises
We gather from the allegations of the petition and at some future time. Borrowing from the
that of the petitioner’s memorandum that the alleged pronouncements of this Court in the PACU case, “They
application for certificate of ancestral land claim (CALC) (the petitioners) have suffered no wrong under the terms
filed by the heirs of Carantes under the assailed DENR of the law – and, naturally need no relief in the form they
special orders has not been granted nor the CALC applied now seek to obtain.” If indeed the heirs of Carantes are
for, issued. The DENR is still processing the application of trying to enter the land and disturbing the petitioners’
the heirs of Carantes for a certificate of ancestral land possession thereof even without prior approval by the
claim, which the DENR may or may not grant. It is DENR of the claim of the heirs of Carantes, the case is
evident that the adverse legal interests involved in this simply one of forcible entry. (Cutaran v. DENR, 350
case are the competing claims of the petitioners and that SCRA 697, Jan. 31, 2001, 3rd Div. [Gonzaga-Reyes])
of the heirs of Carantes to possess a common portion of a
piece of land. As the undisputed facts stand there is no 113. What is a justiciable controversy? What are
justiciable controversy between the petitioners and the political questions?
respondents as there is no actual or imminent violation of
the petitioners’ asserted right to possess the land by Held: As a general proposition, a controversy is
justiciable if it refers to a matter which is appropriate for

53
court review. It pertains to issues which are inherently at all in contemplation of law, as where the power is
susceptible of being decided on grounds recognized by exercised in an arbitrary and despotic manner by reason
law. Nevertheless, the Court does not automatically of passion or hostility. Under this definition, a court is
assume jurisdiction over actual constitutional cases without power to directly decide matters over which full
brought before it even in instances that are ripe for discretionary authority has been delegated. But while this
resolution. One class of cases wherein the Court hesitates Court has no power to substitute its judgment for that of
to rule on are “political questions.” The reason is that Congress or of the President, it may look into the question
political questions are concerned with issues dependent of whether such exercise has been made in grave abuse
upon the wisdom, not the legality, of a particular act or of discretion. A showing that plenary power is granted
measure being assailed. Moreover, the political question either department of government may not be an obstacle
being a function of the separation of powers, the courts to judicial inquiry, for the improvident exercise or abuse
will not normally interfere with the workings of another thereof may give rise to justiciable controversy.
co-equal branch unless the case shows a clear need for (Integrated Bar of the Philippines v. Hon. Ronaldo
the courts to step in to uphold the law and the B. Zamora, G.R. No. 141284, Aug. 15, 2000, En
Constitution. Banc [Kapunan])

As Tanada v. Angara puts it, political questions 114. Is the legitimacy of the assumption to the
refer “to those questions which, under the Constitution, Presidency of President Gloria Macapagal Arroyo a
are to be decided by the people in their sovereign political question and, therefore, not subject to judicial
capacity, or in regard to which full discretionary authority review? Distinguish EDSA People Power I from EDSA
has been delegated to the legislative or executive branch People Power II.
of government.” Thus, if an issue is clearly identified by
the text of the Constitution as matters for discretionary Held: Respondents rely on the case of Lawyers
action by a particular branch of government or to the League for a Better Philippines and/or Oliver A. Lozano v.
people themselves then it is held to be a political President Corazon C. Aquino, et al. and related cases to
question. In the classic formulation of Justice Brennan in support their thesis that since the cases at bar involve the
Baker v. Carr, “[p]rominent on the surface of any case legitimacy of the government of respondent Arroyo, ergo,
held to involve a political question is found a textually they present a political question. A more cerebral reading
demonstrable constitutional commitment of the issue to a of the cited cases will show that they are inapplicable. In
coordinate political department; or a lack of judicially the cited cases, we held that the government of former
discoverable and manageable standards for resolving it; or President Aquino was the result of a successful revolution
the impossibility of deciding without an initial policy by the sovereign people, albeit a peaceful one. No less
determination of a kind clearly for nonjudicial discretion; than the Freedom Constitution declared that the Aquino
or the impossibility of a court's undertaking independent government was installed through a direct exercise of the
resolution without expressing lack of the respect due power of the Filipino people “in defiance of the provisions
coordinate branches of government; or an unusual need of the 1973 Constitution, as amended.” It is familiar
for unquestioning adherence to a political decision already learning that the legitimacy of a government sired by a
made; or the potentiality of embarrassment from successful revolution by people power is beyond judicial
multifarious pronouncements by various departments on scrutiny for that government automatically orbits out of
the one question.” the constitutional loop. In checkered contrast, the
government of respondent Arroyo is not revolutionary in
The 1987 Constitution expands the concept of character. The oath that she took at the EDSA Shrine is
judicial review by providing that “[T]he Judicial power the oath under the 1987 Constitution. In her oath, she
shall be vested in one Supreme Court and in such lower categorically swore to preserve and defend the 1987
courts as may be established by law. Judicial power Constitution. Indeed, she has stressed that she is
includes the duty of the courts of justice to settle actual discharging the powers of the presidency under the
controversies involving rights which are legally authority of the 1987 Constitution.
demandable and enforceable, and to determine whether
or not there has been a grave abuse of discretion In fine, the legal distinction between EDSA People
amounting to lack or excess of jurisdiction on the part of Power I and EDSA People Power II is clear. EDSA I
any branch or instrumentality of the Government.” (Article involves the exercise of the people power of revolution
VIII, Sec. 1 of the 1987 Constitution) Under this which overthrows the whole government. EDSA II is an
definition, the Court cannot agree x x x that the issue exercise of people power of freedom of speech and
involved is a political question beyond the jurisdiction of freedom of assembly to petition the government for
this Court to review. When the grant of power is redress of grievances which only affected the office of the
qualified, conditional or subject to limitations, the issue of President. EDSA I is extra constitutional and the
whether the prescribed qualifications or conditions have legitimacy of the new government that resulted from it
been met or the limitations respected, is justiciable - the cannot be the subject of judicial review, but EDSA II is
problem being one of legality or validity, not its wisdom . intra constitutional and the resignation of the sitting
Moreover, the jurisdiction to delimit constitutional President that it caused and the succession of the Vice
boundaries has been given to this Court. When political President as President are subject to judicial review.
questions are involved, the Constitution limits the EDSA I presented a political question; EDSA II involves
determination as to whether or not there has been a legal questions. X x x
grave abuse of discretion amounting to lack or excess of
jurisdiction on the part of the official whose action is being Needless to state, the cases at bar pose legal and
questioned. not political questions. The principal issues for resolution
require the proper interpretation of certain provisions in
By grave abuse of discretion is meant simply the 1987 Constitution, notably Section 1 of Article II, and
capricious or whimsical exercise of judgment that is patent Section 8 of Article VII, and the allocation of
and gross as to amount to an evasion of positive duty or a governmental powers under Section 11 of Article VII. The
virtual refusal to perform a duty enjoined by law, or to act issues likewise call for a ruling on the scope of presidential

54
immunity from suit. They also involve the correct
calibration of the right of petitioner against prejudicial In the exercise of this jurisdiction, lower courts
publicity. As early as the 1803 case of Marbury v. are advised to act with the utmost circumspection,
Madison, the doctrine has been laid down that “it is bearing in mind the consequences of a declaration of
emphatically the province and duty of the judicial unconstitutionality upon the stability of laws, no less than
department to say what the law is x x x.” Thus, on the doctrine of separation of powers. As the
respondent’s invocation of the doctrine of political questioned act is usually the handiwork of the legislative
question is but a foray in the dark. (Joseph E. Estrada or the executive departments, or both, it will be prudent
v. Aniano Desierto, G.R. Nos. 146710-15, March 2, for such courts, if only out of a becoming modesty, to
2001, En Banc [Puno]) defer to the higher judgment of this Court in the
consideration of its validity, which is better determined
115. Is the President’s power to call out the armed after a thorough deliberation by a collegiate body and
forces as their Commander-in-Chief in order to with the concurrence of the majority of those who
prevent or suppress lawless violence, invasion or participated in its discussion. (Drilon v. Lim, 235 SCRA
rebellion subject to judicial review, or is it a political 135, 139-140, Aug. 4, 1994, En Banc [Cruz])
question? Clarify.
117. What cases are to be heard by the Supreme Court
Held: When the President calls the armed forces en banc?
to prevent or suppress lawless violence, invasion or
rebellion, he necessarily exercises a discretionary power Held: Under Supreme Court Circular No. 2-89,
solely vested in his wisdom. This is clear from the intent dated February 7, 1989, as amended by the Resolution of
of the framers and from the text of the Constitution itself. November 18, 1993:
The Court, thus, cannot be called upon to overrule the
President's wisdom or substitute its own. However, this X x x [t]he following are considered en banc
does not prevent an examination of whether such power cases:
was exercised within permissible constitutional limits or
whether it was exercised in a manner constituting grave 1) Cases in which the constitutionality or validity
abuse of discretion. In view of the constitutional intent to of any treaty, international or executive
give the President full discretionary power to determine agreement, law, executive order, or
the necessity of calling out the armed forces, it is presidential decree, proclamation, order,
incumbent upon the petitioner to show that the instruction, ordinance, or regulation is in
President's decision is totally bereft of factual basis. The question;
present petition fails to discharge such heavy burden as 2) Criminal cases in which the appealed decision
there is no evidence to support the assertion that there imposes the death penalty;
exists no justification for calling out the armed forces. 3) Cases raising novel questions of law;
There is, likewise, no evidence to support the proposition 4) Cases affecting ambassadors, other public
that grave abuse was committed because the power to ministers and consuls;
call was exercised in such a manner as to violate the 5) Cases involving decisions, resolutions or
constitutional provision on civilian supremacy over the orders of the Civil Service Commission,
military. In the performance of this Court's duty of Commission on Elections, and Commission on
“purposeful hesitation” before declaring an act of another Audit;
branch as unconstitutional, only where such grave abuse 6) Cases where the penalty to be imposed is the
of discretion is clearly shown shall the Court interfere with dismissal of a judge, officer or employee of
the President's judgment. To doubt is to sustain. the judiciary, disbarment of a lawyer, or
(Integrated Bar of the Philippines v. Hon. Ronaldo either the suspension of any of them for a
B. Zamora, G.R. No. 141284, Aug. 15, 2000, En period of more than one (1) year or a fine
Banc [Kapunan]) exceeding P10,000.00 or both;
7) Cases where a doctrine or principle laid down
116. Do lower courts have jurisdiction to consider the by the court en banc or in division may be
constitutionality of a law? If so, how should they act modified or reversed;
in the exercise of this jurisdiction? 8) Cases assigned to a division which in the
opinion of at least three (3) members thereof
Held: We stress at the outset that the lower merit the attention of the court en banc and
court had jurisdiction to consider the constitutionality of are acceptable to a majority of the actual
Section 187, this authority being embraced in the general membership of the court en banc; and
definition of the judicial power to determine what are the 9) All other cases as the court en banc by a
valid and binding laws by the criterion of their conformity majority of its actual membership may deem
to the fundamental law. Specifically, BP 129 vests in the of sufficient importance to merit its attention.
regional trial courts jurisdiction over all civil cases in which (Firestone Ceramics, Inc. v. Court of Appeals,
the subject of the litigation is incapable of pecuniary 334 SCRA 465, 471-472, June 28, 2000, En Banc
estimation, even as the accused in a criminal action has [Purisima])
the right to question in his defense the constitutionality of
a law he is charged with violating and of the proceedings 118. What is fiscal autonomy? The fiscal autonomy
taken against him, particularly as they contravene the Bill clause?
of Rights. Moreover, Article VIII, Section 5(2), of the
Constitution vests in the Supreme Court appellate Held: As envisioned in the Constitution, the fiscal
jurisdiction over final judgments and orders of lower autonomy enjoyed by the Judiciary, the Civil Service
courts in all cases in which the constitutionality or validity Commission, the Commission on Audit, the Commission
of any treaty, international or executive agreement, law, on Elections, and the Office of the Ombudsman
presidential decree, proclamation, order, instruction, contemplates a guarantee of full flexibility to allocate and
ordinance, or regulation is in question. utilize their resources with the wisdom and dispatch that

55
their needs require. It recognizes the power and authority as to lay the basis for a criminal or administrative
to levy, assess and collect fees, fix rates of compensation complaint for rendering an unjust judgment or
not exceeding the highest rates authorized by law for order. That prerogative belongs to the courts
compensation and pay plans of the government and alone.
allocate and disburse such sums as may be provided by
law or prescribed by them in the course of the discharge This having been said, we find that the
of their functions. Ombudsman acted in accordance with law and
jurisprudence when he referred the cases against Judge
Fiscal autonomy means freedom from outside Pelayo to the Supreme Court for appropriate action. (De
control. The Judiciary, the Constitutional Commissions, Vera v. Pelayo, 335 SCRA 281, July 6, 2000, 1st Div.
and the Ombudsman must have the independence and [Pardo])
flexibility needed in the discharge of their constitutional
duties. The imposition of restrictions and constraints on 120. What is a Memorandum Decision?
the manner the independent constitutional offices allocate
and utilize the funds appropriated for their operations is Held: A Memorandum Decision is a “specie of
anathema to fiscal autonomy and violative not only of the succinctly written decisions by appellate courts in
express mandate of the Constitution but especially as accordance with the provisions of Section 40, B.P. Blg.
regards the Supreme Court, of the independence and 129 on the grounds of expediency, practicality,
separation of powers upon which the entire fabric of our convenience and docket status of our courts.”
constitutional system is based. (Bengzon v. Drilon, (Francisco v. Permskul, 173 SCRA 324, 333 [1989])
208 SCRA 133, April 15, 1992, En Banc [Gutierrez])
121. Discuss the validity of “Memorandum Decisions.”
119. May the Ombudsman validly entertain criminal
charges against a judge of the regional trial court in Held: 1. The constitutional mandate that no
connection with his handling of cases before the decision shall be rendered by any court without
court. expressing therein clearly and distinctly the facts and the
law on which it is based does not preclude the validity of
Held: Petitioner criticizes the jurisprudence “memorandum decisions” which adopt by reference the
(Maceda v. Vasquez, 221 SCRA 464 [1993] and Dolalas v. findings of fact and conclusions of law contained in the
Office of the Ombudsman-Mindanao, 265 SCRA 818 decisions of inferior tribunals. X x x
[1996]) cited by the Office of the Ombudsman as
erroneous and not applicable to his complaint. He insists Hence, even in this jurisdiction, incorporation by
that since his complaint involved a criminal charge against reference is allowed if only to avoid the cumbersome
a judge, it was within the authority of the Ombudsman reproduction of the decision of the lower courts, or
not the Supreme Court to resolve whether a crime was portions thereof, in the decisions of the higher court. This
committed and the judge prosecuted therefor. is particularly true when the decision sought to be
incorporated is a lengthy and thorough discussion of the
The petition cannot succeed. facts and conclusions arrived at x x x. (Oil and Natural
Gas Commission v. Court of Appeals, 293 SCRA 26,
Xxx July 23, 1998 [Martinez])

We agree with the Solicitor General that the 2. We have sustained decisions of lower courts as
Ombudsman committed no grave abuse of discretion having substantially or sufficiently complied with the
warranting the writs prayed for. The issues have been constitutional injunction notwithstanding the laconic and
settled in the case of In Re: Joaquin Borromeo. There, terse manner in which they were written and even if
we laid down the rule that before a civil or criminal action “there [was left] much to be desired in terms of [their]
against a judge for a violation of Arts. 204 and 205 clarity, coherence and comprehensibility” provided that
(knowingly rendering an unjust judgment or order) can be they eventually set out the facts and the law on which
entertained, there must first be “a final and authoritative they were based, as when they stated the legal
judicial declaration” that the decision or order in question qualifications of the offense constituted by the facts
is indeed “unjust.” The pronouncement may result from proved, the modifying circumstances, the participation of
either: the accused, the penalty imposed and the civil liability; or
discussed the facts comprising the elements of the offense
(a) an action of certiorari or prohibition in a that was charged in the information, and accordingly
higher court impugning the validity of the rendered a verdict and imposed the corresponding
judgment; or penalty; or quoted the facts narrated in the prosecution’s
(b) an administrative proceeding in the Supreme memorandum but made their own findings and
Court against the judge precisely for assessment of evidence, before finally agreeing with the
promulgating an unjust judgment or order. prosecution’s evaluation of the case.

Likewise, the determination of whether a judge We have also sanctioned the use of memorandum
has maliciously delayed the disposition of the case is also decisions x x x. We have also declared that memorandum
an exclusive judicial function (In Re: Borromeo, supra, at decisions comply with the constitutional mandate.
461).
In Francisco v. Permskul, however, we laid the
“To repeat, no other entity or official of conditions for the validity of memorandum decisions,
the government, not the prosecution or thus:
investigation service of any other branch, not any
functionary thereof, has competence to review a The memorandum decision, to be valid,
judicial order or decision – whether final and cannot incorporate the findings of fact and the
executory or not – and pronounce it erroneous so conclusions of law of the lower court only by

56
remote reference, which is to say that the We therefore reiterate our admonition in Nicos
challenged decision is not easily and immediately Industrial Corporation v. Court of Appeals, in that while
available to the person reading the memorandum we conceded that brevity in the writing of decisions is an
decision. For the incorporation by reference to be admirable trait, it should not and cannot be substituted for
allowed, it must provide for direct access to the substance; and again in Francisco v. Permskul, where we
facts and the law being adopted, which must be cautioned that expediency alone, no matter how
contained in a statement attached to the said compelling, cannot excuse non-compliance with the
decision. In other words, the memorandum constitutional requirements.
decision authorized under Section 40 of B.P. Blg.
129 should actually embody the findings of fact This is not to discourage the lower courts to write
and conclusions of law of the lower court in an abbreviated and concise decisions, but never at the
annex attached to and made an indispensable expense of scholarly analysis, and more significantly, of
part of the decision. justice and fair play, lest the fears expressed by Justice
Feria as the ponente in Romero v. Court of Appeals come
It is expected that this requirement will true, i.e., if an appellate court failed to provide the appeal
allay the suspicion that no study was made of the the attention it rightfully deserved, said court deprived the
decision of the lower court and that its decision appellant of due process since he was accorded a fair
was merely affirmed without a prior examination opportunity to be heard by a fair and responsible
of the facts and the law on which it is based. The magistrate. This situation becomes more ominous in
proximity at least of the annexed statement criminal cases, as in this case, where not only property
should suggest that such examination has been rights are at stake but also the liberty if not the life of a
undertaken. It is, of course, also understood that human being.
the decision being adopted should, to begin with,
comply with Article VIII, Section 14 as no amount Faithful adherence to the requirements of Section
of incorporation or adoption will rectify its 14, Article VIII of the Constitution is indisputably a
violation. paramount component of due process and fair play. It is
likewise demanded by the due process clause of the
The Court finds necessary to emphasize Constitution. The parties to a litigation should be
that the memorandum decision should be informed of how it was decided, with an explanation of
sparingly used lest it become an additive excuse the factual and legal reasons that led to the conclusions of
for judicial sloth. It is an additional condition for the court. The court cannot simply say that judgment is
the validity of this kind of decision may be rendered in favor of X and against Y and just leave it at
resorted to only in cases where the facts are in that without any justification whatsoever for its action.
the main accepted by both parties and easily The losing party is entitled to know why he lost, so he
determinable by the judge and there are no may appeal to the higher court, if permitted, should he
doctrinal complications involved that will require believe that the decision should be reversed. A decision
an extended discussion of the laws involved. The that does not clearly and distinctly state the facts and the
memorandum decision may be employed in law on which it is based leaves the parties in the dark as
simple litigations only, such as ordinary collection to how it was reached and is precisely prejudicial to the
cases, where the appeal is obviously groundless losing party, who is unable to pinpoint the possible errors
and deserves no more than the time needed to of the court for review by a higher tribunal. More than
dismiss it. that, the requirement is an assurance to the parties that,
in reaching judgment, the judge did so through the
Xxx processes of legal reasoning. It is, thus, a safeguard
against the impetuosity of the judge, preventing him from
Henceforth, all memorandum decisions deciding ipse dixit. Vouchsafed neither the sword nor the
shall comply with the requirements herein set purse by the Constitution but nonetheless vested with the
forth as to the form prescribed and the occasions sovereign prerogative of passing judgment on the life,
when they may be rendered. Any deviation will liberty or property of his fellowmen, the judge must
summon the strict enforcement of Article VIII, ultimately depend on the power of reason for sustained
Section 14 of the Constitution and strike down the public confidence in the justness of his decision.
flawed judgment as a lawless disobedience.
Thus the Court has struck down as void, decisions
Tested against these standards, we find that the of lower courts and even of the Court of Appeals whose
RTC decision at bar miserably failed to meet them and, careless disregard of the constitutional behest exposed
therefore, fell short of the constitutional injunction. The their sometimes cavalier attitude not only to their
RTC decision is brief indeed, but it is starkly hallow, magisterial responsibilities but likewise to their avowed
otiosely written, vacuous in its content and trite in its fealty to the Constitution.
form. It achieved nothing and attempted at nothing, not
even at a simple summation of facts which could easily be Thus, we nullified or deemed to have failed to
done. Its inadequacy speaks for itself. comply with Section 14, Article VIII of the Constitution, a
decision, resolution or order which: contained no analysis
We cannot even consider or affirm said RTC of the evidence of the parties nor reference to any legal
decision as a memorandum decision because it failed to basis in reaching its conclusions; contained nothing more
comply with the measures of validity laid down in than a summary of the testimonies of the witnesses of
Francisco v. Permskul. It merely affirmed in toto the both parties; convicted the accused of libel but failed to
MeTC decision without saying more. A decision or cite any legal authority or principle to support conclusions
resolution, especially one resolving an appeal, should that the letter in question was libelous; consisted merely
directly meet the issues for resolution; otherwise, the of one (1) paragraph with mostly sweeping
appeal would be pointless generalizations and failed to support its conclusion of
parricide; consisted of five (5) pages, three (3) pages of

57
which were quotations from the labor arbiter’s decision Haydee Yorac as Acting Chairperson of the COMELEC.
including the dispositive portion and barely a page (two This Court ruled that:
[2] short paragraphs of two [2] sentences each) of its
own discussion or reasonings; was merely based on the “A designation as Acting Chairman is by its very
findings of another court sans transcript of stenographic terms essentially temporary and therefore
notes, or failed to explain the factual and legal bases for revocable at will. No cause need be established
the award of moral damages. to justify its revocation. Assuming its validity, the
designation of the respondent as Acting Chairman
In the same vein do we strike down as a nullity of the Commission on Elections may be withdrawn
the RTC decision in question. (Yao v. Court of Appeals, by the President of the Philippines at any time and
344 SCRA 202, Oct. 24, 2000, 1st Div. [Davide]) for whatever reason she sees fit. It is doubtful if
the respondent, having accepted such
122. What are the distinctive features and purpose of a designation, will not be estopped from challenging
memorandum decision? its withdrawal.

Held: In Francisco v. Permskul (173 SCRA 324, The Constitution provides for many safeguards to
333 [1989], the Court described “[t]he distinctive features the independence of the Commission on Elections,
of a memorandum decision are, first, it is rendered by an foremost among which is the security of tenure of
appellate court, second, it incorporates by reference the its members. That guarantee is not available to
findings of fact or the conclusions of law contained in the the respondent as Acting Chairman of the
decision, order, or ruling under review. Most likely, the Commission on Elections by designation of the
purpose is to affirm the decision, although it is not President of the Philippines.”
impossible that the approval of the findings of facts by the
lower court may lead to a different conclusion of law by Earlier, in Nacionalista Party v. Bautista, a case
the higher court. At any rate, the reason for allowing the decided under the 1935 Constitution, which did not have a
incorporation by reference is evidently to avoid the provision prohibiting temporary or acting appointments to
cumbersome reproduction of the decision of the lower the COMELEC, this Court nevertheless declared
court, or portions thereof, in the decision of the higher unconstitutional the designation of the Solicitor General as
court. The idea is to avoid having to repeat in the body of acting member of the COMELEC. This Court ruled that the
the latter decision the findings or conclusions of the lower designation of an acting Commissioner would undermine
court since they are being approved or adopted anyway. the independence of the COMELEC and hence violate the
(Yao v. Court of Appeals, 344 SCRA 202, Oct. 24, Constitution. We declared then: “It would be more in
2000, 1st Div. [Davide]) keeping with the intent, purpose and aim of the framers
of the Constitution to appoint a permanent Commissioner
123. Does the period for decision making under Section than to designate one to act temporarily.” (Matibag v.
15, Article VIII, 1987 Constitution, apply to the Benipayo, 380 SCRA 49, April 2, 2002, En Banc
Sandiganbayan? Explain. [Carpio])

Held: The above provision does not apply to the 125. Is the constitutional power of the COA to examine
Sandiganbayan. The provision refers to regular courts of and audit government banks and agencies exclusive?
lower collegiate level that in the present hierarchy applies Does it preclude a concurrent audit by a private
only to the Court of Appeals. external auditor?

The Sandiganbayan is a special court of the same Held: The resolution of the primordial issue of
level as the Court of Appeals and possessing all the whether or not the COA has the sole and exclusive power
inherent powers of a court of justice, with functions of a to examine and audit government banks involves an
trial court. interpretation of Section 2, Article IX-D of the 1987
Constitution. This Section provides as follows:
Thus, the Sandiganbayan is not a regular court
but a special one. (Re: Problem of Delays in Cases “Sec. 2. (1) The Commission on Audit
Before the Sandiganbayan, A.M. No. 00-8-05-SC, shall have the power, authority, and duty to
Nov. 28, 2001, En Banc [Pardo]) examine, audit, and settle all accounts pertaining
to the revenue and receipts of, and expenditures
or uses of funds and property, owned and held in
The Constitutional Commissions trust by, or pertaining to, the Government, or any
of its subdivisions, agencies, or instrumentalities,
124. Why does the Constitution prohibit the President including government-owned or controlled
from appointing in an acting or temporary capacity corporations with original charters, x x x.
the Chairman and Commissioners of the Constitutional
Commissions? Explain. “(2) The Commission shall have the
exclusive authority, subject to the limitations in
Held: [A] temporary or acting appointee does this Article, to define the scope of its audit and
not enjoy security of tenure, no matter how briefly. examination, establish the techniques and
methods required therefore, and promulgate
This is the kind of appointment that the accounting and auditing rules and regulations,
Constitution prohibits the President from making to the including those for the prevention and
three independent constitutional commissions, including disallowance of irregular, unnecessary, excessive,
the COMELEC. Thus, in Brillantes v. Yorac, this Court extravagant, or unconscionable expenditures, or
struck down as unconstitutional the designation by then uses of government funds and properties.”
President Corazon Aquino of Associate Commissioner (Emphasis supplied)

58
The COA vigorously asserts that under the first certain government agencies in addition to the COA audit,
paragraph of Section 2, the COA enjoys the sole and as when there is a private investment in a government-
exclusive power to examine and audit all government controlled corporation, or when a government corporation
agencies, including the DBP. The COA contends this is is privatized or publicly listed, or as in the case at bar
similar to its sole and exclusive authority, under the same when the government borrows money from abroad.
paragraph of the same section, to define the scope of its
audit, promulgate auditing rules and regulations, including In these instances the government enters the
rules on the disallowance of unnecessary expenditures of marketplace and competes with the rest of the world in
government agencies. The bare language of Section 2, attracting investments or loans. To succeed, the
however, shows that the COA’s power under the first government must abide with the reasonable business
paragraph is not declared exclusive, while its authority practices of the marketplace. Otherwise no investor or
under the second paragraph is expressly declared creditor will do business with the government, frustrating
“exclusive.” There is a significant reason for this marked government efforts to attract investments or secure loans
difference in language. that may be critical to stimulate moribund industries or
resuscitate a badly shattered national economy as in the
During the deliberations of the Constitutional case at bar. By design the Constitution is flexible enough
Commission, Commissioner Serafin Guingona proposed to meet these exigencies. Any attempt to nullify this
the addition of the word “exclusive” in the first paragraph flexibility in the instances mentioned, or in similar
of Section 2, thereby granting the COA the sole and instances, will be ultra vires, in the absence of a statute
exclusive power to examine and audit all government limiting or removing such flexibility.
agencies. However, the Constitutional Commission
rejected the addition of the word “exclusive” in the first The deliberations of the Constitutional
paragraph of Section 2 and Guingona was forced to Commission reveal eloquently the intent of Section 2,
withdraw his proposal. X x x Article IX-D of the Constitution. As this Court has ruled
repeatedly, the intent of the law is the controlling factor in
Xxx the interpretation of the law . If a law needs
interpretation, the most dominant influence is the intent
In sharp contrast, the Constitutional Commission of the law. The intent of the law is that which is
placed the word “exclusive” to qualify the authority of the expressed in the words of the law, which should be
COA under the second paragraph of the same Section 2. discovered within its four corners aided, if necessary, by
This word “exclusive” did not appear in the counterpart its legislative history. In the case of Section 2, Article IX-
provisions of Section 2 in the 1935 and 1973 D of the Constitution, the intent of the framers of the
Constitutions. There is no dispute that the COA’s Constitution is evident from the bare language of Section
authority under the second paragraph of Section 2 is 2 itself. The deliberations of the Constitutional
exclusive as the language of the Constitution admits of no Commission confirm expressly and even elucidate further
other meaning. Thus, the COA has the exclusive authority this intent beyond any doubt whatsoever.
to decide on disallowances of unnecessary government
expenditures. Other government agencies and their There is another constitutional barrier to the
officials, as well as private auditors engaged by them, COA’s insistence of exclusive power to examine and audit
cannot in any way intrude into this exclusive function of all government agencies. The COA’s claim clashes directly
the COA. with the Central Bank’s constitutional power of
“supervision” over banks under Section 20, Article XII of
The qualifying word “exclusive” in the second the Constitution. X x x
paragraph of Section 2 cannot be applied to the first
paragraph which is another sub-section of Section 2. A Historically, the Central Bank has been conducting
qualifying word is intended to refer only to the phrase to periodic and special examination and audit of banks to
which it is immediately associated, and not to a phrase determine the soundness of their operations and the
distantly located in another paragraph or sub-section. safety of the deposits of the public. Undeniably, the
Thus, the first paragraph of Section 2 must be read the Central Bank’s power of “supervision” includes the power
way it appears, without the word “exclusive,” signifying to examine and audit banks, as the banking laws have
that non-COA auditors can also examine and audit always recognized this power of the Central Bank. Hence,
government agencies. Besides, the framers of the the COA’s power to examine and audit government banks
Constitution intentionally omitted the word “exclusive” in must be reconciled with the Central Bank’s power to
the first paragraph of Section 2 precisely to allow supervise the same banks. The inevitable conclusion is
concurrent audit by private external auditors. that the COA and the Central Bank have concurrent
jurisdiction, under the Constitution, to examine and audit
The clear and unmistakable conclusion from a government banks.
reading of the entire Section 2 is that the COA’s power to
examine and audit is non-exclusive. On the other hand, However, despite the Central Bank’s concurrent
the COA’s authority to define the scope of its audit, jurisdiction over government banks, the COA’s audit still
promulgate auditing rules and regulations, and disallow prevails over that of the Central Bank since the COA is the
unnecessary expenditures is exclusive. constitutionally mandated auditor of government banks.
And in matters falling under the second paragraph of
Xxx Section 2, Article IX-D of the Constitution, the COA’s
jurisdiction is exclusive. Thus, the Central Bank is devoid
Manifestly, the express language of the of authority to allow or disallow expenditures of
Constitution, and the clear intent of its framers, point to government banks since this function belongs exclusively
only one indubitable conclusion – the COA does not have to the COA. (Development Bank of the Philippines
the exclusive power to examine and audit government v. Commission on Audit, 373 SCRA 356, January
agencies. The framers of the Constitution were fully 16, 2002, En Banc [Carpio])
aware of the need to allow independent private audit of

59
126. Between the COA’s findings and conclusions and null and void ab initio. Rudimentary is the precept that
that of private auditors, which should prevail? there can be no valid appointment to a non-vacant
position. Accordingly, Adiong’s appointment on 11 March
Held: Moreover, as the constitutionally- 1998 for a term of two years, pursuant to Section 8 of RA
mandated auditor of all government agencies, the COA’s 8551, is null and void. X x x. Therefore, based on our
findings and conclusions necessarily prevail over those of foregoing disquisition, there should no longer be any
private auditors, at least insofar as government agencies doubt as to the proper execution of our 25 January 2000
and officials are concerned. The superiority or decision – all the Commissioners appointed under RA
preponderance of the COA audit over private audit can be 8551 should be removed from office, in order to give way
gleaned from the records of the Constitutional to the reinstatement of petitioners and respondent
Commission x x x. The findings and conclusions of the Adiong. (Canonizado v. Aguirre, 351 SCRA 659, Feb.
private auditor may guide private investors or creditors 15, 2001, En Banc [Gonzaga-Reyes])
who require such private audit. Government agencies and
officials, however, remain bound by the findings and
conclusions of the COA, whether the matter falls under THE INHERENT POWERS OF THE STATE
the first or second paragraph of Section 2, unless of
course such findings and conclusions are modified or Police Power
reversed by the courts.
129. Define Police Power and clarify its scope.
127. May the power of the COA to examine and audit
government agencies be validly taken away from it? Held: 1. Police power is an inherent attribute of
sovereignty. It has been defined as the power vested by
Held: The power of the COA to examine and the Constitution in the legislature to make, ordain, and
audit government agencies, while non-exclusive, cannot establish all manner of wholesome and reasonable laws,
be taken away from the COA. Section 3, Article IX-C of statutes and ordinances, either with penalties or without,
the Constitution mandates that: not repugnant to the Constitution, as they shall judge to
be for the good and welfare of the commonwealth, and
“Sec. 3. No law shall be passed for the subjects of the same. The power is plenary and its
exempting any entity of the Government or its scope is vast and pervasive, reaching and justifying
subsidiary in any guise whatsoever, or any measures for public health, public safety, public morals,
investment of public funds, from the jurisdiction and the general welfare.
of the Commission on Audit.”
It bears stressing that police power is lodged
The mere fact that private auditors may audit government primarily in the National Legislature. It cannot be
agencies does not divest the COA of its power to examine exercised by any group or body of individuals not
and audit the same government agencies. possessing legislative power. The National Legislature,
(Development Bank of the Philippines v. however, may delegate this power to the President and
Commission on Audit, 373 SCRA 356, January 16, administrative boards as well as the lawmaking bodies of
2002, En Banc [Carpio]) municipal corporations or local government units . Once
delegated, the agents can exercise only such legislative
powers as are conferred on them by the national
B. CONSTITUTIONAL LAW lawmaking body. (Metropolitan Manila Development
Authority v. Bel-Air Village Association, Inc., 328
128. What is the effect of declaration of SCRA 836, 843-844, March 27, 2000, 1st Div.
unconstitutionality of a law? Illustrative case. [Puno])

Held: Respondents are seeking a reconsideration 2. Police power as an inherent attribute of


of the Court’s 25 January 2000 decision, wherein we sovereignty is the power to prescribe regulations to
declared Section 8 of Republic Act No. 8551 (RA 8551) to promote the health, morals, peace, education, good order
be violative of petitioners’ constitutionally mandated right or safety and general welfare of the people (Binay v.
to security of tenure. As a consequence of our ruling, we Domingo, 201 SCRA 508). The State, through the
held that petitioners’ removal as commissioners of the legislature, has delegated the exercise of police power to
National Police Commission (NAPOLCOM) and the local government units, as agencies of the State, in order
appointment of new Commissioners in their stead were to effectively accomplish and carry out the declared
nullities and ordered the reinstatement of petitioners and objects of their creation (Tatel v. Muncipality of Virac, 207
the payment of full backwages to be computed from the SCRA 157). This delegation of police power is embodies
date they were removed from office. in the general welfare clause of the Local Government
Code x x x.
Xxx
The scope of police power has been held to be so
An unconstitutional act is not a law; it confers no comprehensive as to encompass almost all matters
rights, imposes no duties, and affords no protection. affecting the health, safety, peace, order, morals, comfort
Therefore, the unavoidable consequence of the Court’s and convenience of the community. Police power is
declaration that Section 8 of RA 8551 violates the essentially regulatory in nature and the power to issue
fundamental law is that all acts done pursuant to such licenses or grant business permits, if exercised for a
provision shall be null and void, including the removal of regulatory and not revenue-raising purpose, is within the
petitioners and Adiong from their positions in the ambit of this power (Procter and Gamble Phils. v. The
NAPOLCOM and the appointment of new commissioners in Muncicipality of Jagna, 94 SCRA 894). (Acebedo Optical
their stead. When a regular government employee is Company, Inc. v. Court of Appeals, 329 SCRA 314,
illegally dismissed, his position does not become vacant 325-326, March 31, 2000, En Banc [Purisima])
and the new appointment made in order to replace him is

60
130. How should laws that grant the right to exercise a 132. Discuss the nature of the authority of local
part of the police power of the State be construed? government units to issue or grant licenses or
permits.
Held: Lest the idea gets lost in the shoals of our
subconsciousness, let us not forget that PAGCOR is Held: [T]he issuance of business licenses and
engaged in business affected with public interest. The permits by a municipality or city is essentially regulatory in
phrase “affected with public interest” means that an nature. The authority, which devolved upon local
industry is subject to control for the public good; it has government units to issue or grant such licenses or
been considered as the equivalent of “subject to the permits, is essentially in the exercise of the police power
exercise of the police power.” Perforce, a legislative of the State within the contemplation of the general
franchise to operate jai-alai is imbued with public interest welfare clause of the Local Government Code. (Acebedo
and involves an exercise of police power. The familiar Optical Company, Inc. v. Court of Appeals, 329
rule is that laws which grant the right to exercise a part of SCRA 314, 335, March 31, 2000, En Banc
the police power of the state are to be construed strictly [Purisima])
and any doubt must be resolved against the grant. The
legislature is regarded as the guardian of society, and 133. Does Article 263(g) of the Labor Code (vesting
therefore is not presumed to disable itself or abandon the upon the Secretary of Labor the discretion to
discharge of its duty. Thus, courts do not assume that determine what industries are indispensable to the
the legislature intended to part away with its power to national interest and thereafter, assume jurisdiction
regulate public morals. The presumption is influenced by over disputes in said industries) violate the workers’
constitutional considerations. Constitutions are widely constitutional right to strike?
understood to withhold from legislatures any authority to
bargain away their police power for the power to protect Held: Said article does not interfere with the
the public interest is beyond abnegation. workers’ right to strike but merely regulates it, when in
the exercise of such right, national interests will be
It is stressed that the case at bar does not involve affected. The rights granted by the Constitution are not
a franchise to operate a public utility (such as water, absolute. They are still subject to control and limitation to
transportation, communication or electricity) – the ensure that they are not exercised arbitrarily. The
operation of which undoubtedly redounds to the benefit of interests of both the employers and the employees are
the general public. What is claimed is an alleged intended to be protected and not one of them is given
legislative grant of a gambling franchise – a franchise to undue preference.
operate jai-alai. A statute which legalizes a gambling
activity or business should be strictly construed and every The Labor Code vests upon the Secretary of Labor
reasonable doubt must be resolved to limit the powers the discretion to determine what industries are
and rights claimed under its authority. (Del Mar v. indispensable to national interest. Thus, upon the
PAGCOR, 346 SCRA 485, Nov. 29, 2000, En Banc determination of the Secretary of Labor that such industry
[Puno]) is indispensable to the national interest, it will assume
jurisdiction over the labor dispute of said industry. The
131. Discuss why rates to be charged by public utilities assumption of jurisdiction is in the nature of police power
like MERALCO are subject to State regulation. measure. This is done for the promotion of the common
good considering that a prolonged strike or lockout can be
Held: The regulation of rates to be charged by inimical to the national economy. The Secretary of Labor
public utilities is founded upon the police power of the acts to maintain industrial peace. Thus, his certification
State and statutes prescribing rules for the control and for compulsory arbitration is not intended to impede the
regulations of public utilities are a valid exercise thereof. workers’ right to strike but to obtain a speedy settlement
When private property is used for a public purpose and is of the dispute. (Philtread Workers Union [PTWU] v.
affected with public interest, it ceases to be juris privati Confesor, 269 SCRA 393, March 12, 1997)
only and becomes subject to regulation. The regulation is
to promote the common good. Submission to regulation 134. May solicitation for religious purposes be subject
may be withdrawn by the owner by discontinuing use; but to proper regulation by the State in the exercise of
as long as the use of the property is continued, the same police power?
is subject to public regulation.
Held: Whence, even the exercise of religion may
In regulating rates charged by public utilities, the be regulated, at some slight inconvenience, in order that
State protects the public against arbitrary and excessive the State may protect its citizens from injury. Without
rates while maintaining the efficiency and quality of doubt, a State may protect its citizens from fraudulent
services rendered. However, the power to regulate rates solicitation by requiring a stranger in the community,
does not give the State the right to prescribe rates which before permitting him publicly to solicit funds for any
are so low as to deprive the public utility of a reasonable purpose, to establish his identity and his authority to act
return on investment. Thus, the rates prescribed by for the cause which he purports to represent. The State
the State must be one that yields a fair return on is likewise free to regulate the time and manner of
the public utility upon the value of the property solicitation generally, in the interest of public safety,
performing the service and one that is reasonable peace, comfort, or convenience.
to the public for the service rendered. The fixing of
just and reasonable rates involves a balancing of the It does not follow, therefore, from the
investor and the consumer interests. (Republic of the constitutional guarantees of the free exercise of religion
Philippines v. Manila Electric Company, G.R. No. that everything which may be so called can be tolerated .
141314, Nov. 15, 2002, 3rd Div. [Puno]) It has been said that a law advancing a legitimate
governmental interest is not necessarily invalid as one
interfering with the “free exercise” of religion merely
because it also incidentally has a detrimental effect on the

61
adherents of one or more religion. Thus, the general The primary purpose of the statute regulating the
regulation, in the public interest, of solicitation, which practice of optometry is to insure that optometrical
does not involve any religious test and does not services are to be rendered by competent and licensed
unreasonably obstruct or delay the collection of funds, is persons in order to protect the health and physical welfare
not open to any constitutional objection, even though the of the people from the dangers engendered by unlicensed
collection be for a religious purpose. Such regulation practice. Such purpose may be fully accomplished
would not constitute a prohibited previous restraint on the although the person rendering the service is employed by
free exercise of religion or interpose an inadmissible a corporation (Silver v. Lansburgh and Brother, 72 App DC
obstacle to its exercise. 77, 11 F2d 518, 128 ALR 582; 61 Am Jur 2d 289).

Even with numerous regulative laws in existence, Furthermore, it was ruled that the employment of
it is surprising how many operations are carried on by a qualified optometrist by a corporation is not against
persons and associations who, secreting their activities public policy (Georgia State Examiners v. Friedman’s
under the guise of benevolent purposes, succeed in Jewelers, 183 Ga 669, 189 SE 238). Unless prohibited by
cheating and defrauding a generous public. It is in fact statutes, a corporation has all the contractual rights that
amazing how profitable the fraudulent schemes and an individual has (State ex rel. McKittrick v. Gate City
practices are to people who manipulate them. The State Optical Co., 339 Mo 427, 97 SW 2d 89) and it does not
has authority under the exercise of its police power to become the practice of medicine or optometry because of
determine whether or not there shall be restrictions on the presence of a physician or optometrist (Dickson v.
soliciting by unscrupulous persons or for unworthy causes Flynn, 246 App Div 341, 286 NYS 225). The
or for fraudulent purposes. That solicitation of manufacturing, selling, trading and bartering of
contributions under the guise of charitable and benevolent eyeglasses and spectacles as articles of merchandise do
purposes is grossly abused is a matter of common not constitute the practice of optometry (State ex rel.
knowledge. Certainly the solicitation of contributions in Brother v. Beck Jewelry Enterprises, Inc. 220 Ind. 276, 41
good faith for worthy purposes should not be denied, but NE 2d 622, 141 ALR 876 [61 Am Jur 187]; Kindy
somewhere should be lodged the power to determine Opticians, Inc. v. State Board of Examiners in Optometry,
within reasonable limits the worthy from the unworthy . 1939, 291 Mich 152, 289 NW 112, 113; New Jersey State
The objectionable practices of unscrupulous persons are Bd. Of Optometrists v. S.S. Kresge Co., 113 NJL 287, 174
prejudicial to worthy and proper charities which naturally A 353).
suffer when the confidence of the public in campaigns for
the raising of money for charity is lessened or destroyed. Xxx
Some regulation of public solicitation is, therefore, in the
public interest. To accomplish the objective of the regulation, a
state may provide by statute that corporations cannot sell
To conclude, solicitation for religious purposes eyeglasses, spectacles, and lenses unless a duly licensed
may be subject to proper regulation by the State in the physician or a duly qualified optometrist is in charge of,
exercise of police power. (Centeno v. Villalon- and in personal attendance at the place where such
Pornillos, 236 SCRA 197, Sept. 1, 1994 [Regalado]) articles are sold (Roschen v. Ward, 279 US 337, 73 L Ed
722, 49 S Ct 336). In such a case, the patient’s primary
135. Does a corporation or individual not himself and essential safeguard lies in the optometrist’s control of
licensed, have a right to hire and employ licensed the “treatment” by means of prescription and preliminary
optometrists? Will the employment of a qualified and final examination (Small and Maine Board of
optometrist by a corporation go against public policy? Registration and Examination in Optometry, 293 A 2d
786).
Held: From the foregoing, it is thus evident that
Congress has not adopted a unanimous position on the In analogy, it is noteworthy that private hospitals
matter of prohibition of indirect practice of optometry by are maintained by corporations incorporated for the
corporations, specifically on the hiring and employment of purpose of furnishing medical and surgical treatment. In
licensed optometrists by optical corporations. It is clear the course of providing such treatments, these
that Congress left the resolution of such issue for judicial corporations employ physicians, surgeons and medical
determination, and it is therefore proper for this Court to practitioners, in the same way that in the course of
resolve the issue. manufacturing and selling eyeglasses, eye frames and
optical lenses, optical shops hire licensed optometrists to
Even in the United States, jurisprudence varies examine, prescribe and dispense ophthalmic lenses. No
and there is a conflict of opinions among the federal one has ever charged that these corporations are engaged
courts as to the right of a corporation or individual not in the practice of medicine. There is indeed no valid basis
himself licensed, to hire and employ licensed optometrists for treating corporations engaged in the business of
(128 ALR 586). running optical shops differently. (Acebedo Optical
Company, Inc. v. Court of Appeals, 329 SCRA 314,
Courts have distinguished between optometry as 331-333, March 31, 2000, En Banc [Purisima])
a learned profession in the category of law and medicine,
and optometry as a mechanical art. And, insofar as the 136. What powers of the State are involved in the
courts regartd optometry as merely a mechanical art, they implementation of the Comprehensive Agrarian
have tended to find nothing objectionable in the making Reform Law (CARL)? Discuss.
and selling of eyeglasses, spectacles and lenses by
corporations so long as the patient is actually examined Held: The implementation of the CARL is an
and prescribed for by qualified practitioners (House of exercise of the State’s police power and the power of
$8.50 Eyeglasses, Inc. v. State Board of Optometry, 288 eminent domain. To the extent that the CARL prescribes
Ala 349, 261 So 2d 27; State ex. Rel. Board of Optometry retention limits to the landowners, there is an exercise of
v. Sears Roebuck and Co., 102 Ariz 175, 427 Pd 126). police power for the regulation of private property in
accordance with the Constitution. But where, to carry out

62
such regulation, the owners are deprived of lands they constitutions attach to the right of property of the citizens,
own in excess of the maximum area allowed, there is also constrains the strict observance of the substantial
a taking under the power of eminent domain. The taking provisions of the law which are prescribed as modes of
contemplated is not a mere limitation of the use of the the exercise of the power, and to protect it from abuse x x
land. What is required is the surrender of the title to and x.
physical possession of the said excess and all beneficial
rights accruing to the owner in favor of the farmer The power of eminent domain is essentially
beneficiary. The Bill of Rights provides that “[n]o person legislative in nature. It is firmly settled, however, that
shall be deprived of life, liberty or property without due such power may be validly delegated to local government
process of law.” The CARL was not intended to take away units, other public entities and public utilities, although
property without due process of law. The exercise of the the scope of this delegated legislative power is necessarily
power of eminent domain requires that due process be narrower than that of the delegating authority and may
observed in the taking of private property. (Roxas & only be exercised in strict compliance with the terms of
Co., Inc. v. Court of Appeals, 321 SCRA 106, Dec. the delegating law. (Heirs of Alberto Suguitan v. City
17, 1999, En Banc [Puno]) of Mandaluyong, 328 SCRA 137, 144-146, March
14, 2000, 3rd Div. [Gonzaga-Reyes])

The Power of Eminent Domain 2. Eminent domain is a fundamental State power


that is inseparable from sovereignty. It is government’s
137. What is Eminent Domain? right to appropriate, in the nature of a compulsory sale to
the State, private property for public use or purpose.
Held: 1. Eminent domain is the right or power Inherently possessed by the national legislature, the
of a sovereign state to appropriate private property to power of eminent domain may be validly delegated to
particular uses to promote public welfare . It is an local governments, other public entities and public
indispensable attribute of sovereignty; a power grounded utilities. For the taking of private property by the
in the primary duty of government to serve the common government to be valid, the taking must be for public
need and advance the general welfare. Thus, the right of purpose and there must be just compensation. (Moday
eminent domain appertains to every independent v. Court of Appeals, 268 SCRA 586, February 20,
government without the necessity for constitutional 1997)
recognition. The provisions found in modern constitutions
of civilized countries relating to the taking of property for 138. Discuss the nature of the right of eminent domain
the public use do not by implication grant the power to and the limitations thereof.
the government, but limit a power which would otherwise
be without limit. Thus, our own Constitution provides that Held: The right of eminent domain is usually
“[p]rivate property shall not be taken for public use understood to be an ultimate right of the sovereign power
without just compensation.” Furthermore, the due to appropriate any property within its territorial
process and equal protection clauses act as additional sovereignty for a public purpose. Fundamental to the
safeguards against the arbitrary exercise of this independent existence of a State, it requires no
governmental power. recognition by the Constitution, whose provisions are
taken as being merely confirmatory of its presence and as
Since the exercise of the power of eminent being regulatory, at most, in the due exercise of the
domain affects an individual’s right to private property, a power. In the hands of the legislature, the power is
constitutionally-protected right necessary for the inherent, its scope matching that of taxation, even that of
preservation and enhancement of personal dignity and police power itself, in many respects. It reaches to every
intimately connected with the rights to life and liberty , the form of property the State needs for public use and, as an
need for its circumspect operation cannot be old case so puts it, all separate interests of individuals in
overemphasized. In City of Manila v. Chinese Community property are held under a tacit agreement or implied
of Manila we said: reservation vesting upon the sovereign the right to
resume the possession of the property whenever the
The exercise of the right of eminent public interest so requires it.
domain, whether directly by the State, or by its
authorized agents, is necessarily in derogation of The ubiquitous character of eminent domain is
private rights, and the rule in that case is that the manifest in the nature of the expropriation proceedings.
authority must be strictly construed. No species Expropriation proceedings are not adversarial in the
of property is held by individuals with greater conventional sense, for the condemning authority is not
tenacity, and none is guarded by the Constitution required to assert any conflicting interest in the property.
and the laws more sedulously, than the right to Thus, by filing the action, the condemnor in effect merely
the freehold of inhabitants. When the legislature serves notice that it is taking title and possession of the
interferes with that right, and, for greater public property, and the defendant asserts title or interest in the
purposes, appropriates the land of ah individual property, not to prove a right to possession, but to prove
without his consent, the plain meaning of the law a right to compensation for the taking.
should not be enlarged by doubt[ful]
interpretation. (Bensley v. Mountainlake Water Obviously, however, the power is not without its
Co., 13 Cal., 306 and cases cited [73 Am. Dec., limits: first, the taking must be for public use, and second,
576]) that just compensation must be given to the private
owner of the property. These twin proscriptions have
The statutory power of taking property from the their origin in the recognition of the necessity for
owner without his consent is one of the most delicate achieving balance between the State interests, on the one
exercise of governmental authority. It is to be watched hand, and private rights, upon the other hand, by
with jealous scrutiny. Important as the power may be to effectively restraining the former and affording protection
the government, the inviolable sanctity which all free to the latter. In determining “public use,” two approaches

63
are utilized – the first is public employment or the actual This Court has ruled that the
use by the public, and the second is public advantage or taking to be valid must be for public use.
benefit. It is also useful to view the matter as being There was a time when it was felt that a
subject to constant growth, which is to say that as society literal meaning should be attached to
advances, its demands upon the individual so increases, such a requirement. Whatever project is
and each demand is a new use to which the resources of undertaken must be for the public to
the individual may be devoted. (Republic of the enjoy, as in the case of streets or parks.
Philippines v. The Hon. Court of Appeals, G.R. No. Otherwise, expropriation is not allowable.
146587, July 2, 2002, 1st Div. [Vitug]) It is not anymore. As long as the purpose
of the taking is public, then the power of
139. State some limitations on the exercise of the eminent domain comes into play . . . It is
power of Eminent Domain. accurate to state then that at present
whatever may be beneficially employed
Held: The limitations on the power of eminent for the general welfare satisfies the
domain are that the use must be public, compensation requirement of public use. (Heirs of
must be made and due process of law must be observed. Juancho Ardona v. Reyes, 125 SCRA 220
The Supreme Court, taking cognizance of such issues as [1983] at 234-235 quoting E. Fernando,
the adequacy of compensation, necessity of the taking the Constitution of the Philippines 523-4
and the public use character or the purpose of the taking, [2nd Ed. 1977])
has ruled that the necessity of exercising eminent domain
must be genuine and of a public character. Government The term “public use” has acquired a
may not capriciously choose what private property should more comprehensive coverage. To the literal
be taken. (Moday v. Court of Appeals, 268 SCRA import of the term signifying strict use or
586, February 20, 1997) employment by the public has been added the
broader notion of indirect public benefit or
140. Discuss the expanded notion of “public use” in advantage.
eminent domain proceedings.
In Manosca v. Court of Appeals, this Court has
Held: The City of Manila, acting through its also held that what ultimately emerged is a concept of
legislative branch, has the express power to acquire public use which is just as broad as “public welfare.”
private lands in the city and subdivide these lands into
home lots for sale to bona fide tenants or occupants Respondent PEZA expropriated the subject parcel
thereof, and to laborers and low-salaried employees of the of land pursuant to Proclamation No. 1980 x x x issued by
city. former President Ferdinand Marcos. Meanwhile, the
power of eminent domain of respondent is contained in its
That only a few could actually benefit from the original charter, Presidential Decree No. 66 x x x.
expropriation of the property does not diminish its public
character. It is simply not possible to provide all at once Accordingly, subject Lot 1406-B was expropriated
land and shelter for all who need them. “for the construction . . . of terminal facilities, structures
and approaches thereto.” The authority is broad enough
Corollary to the expanded notion of public use, to give the respondent substantial leeway in deciding for
expropriation is not anymore confined to vast tracts of what public use the expropriated property would be
land and landed estates. It is therefore of no moment utilized. Pursuant to this broad authority, respondent
that the land sought to be expropriated in this case is less leased a portion of the lot to commercial banks while the
than half a hectare only. rest was made a transportation terminal. Said public
purposes were even reaffirmed by Republic Act No. 7916,
Through the years, the public use requirement in a law amending respondent PEZA’s original charter x x x.
eminent domain has evolved into a flexible concept,
influenced by changing conditions. Public use now In Manila Railroad Co. v. Mitchel, this Court has
includes the broader notion of indirect public benefit or ruled that in the exercise of eminent domain, only as
advantage, including in particular, urban land reform and much land can be taken as is necessary for the legitimate
housing. (Filstream International Incorporated v. purpose of the condemnation. The term “necessary,” in
CA, 284 SCRA 716, Jan. 23, 1998 [Francisco]) this connection, does not mean absolutely indispensable
but requires only a reasonable necessity of the taking for
141. What is the meaning of “public use” in eminent the stated purpose, growth and future needs of the
domain proceedings? Illustrative case. enterprise. The respondent cannot attain a self-sustaining
and viable ECOZONE if inevitable needs in the expansion
Held: This Court holds that respondent in the surrounding areas are hampered by the mere
(Philippine Export Processing Zone) has the legal authority refusal of the private landowners to part with their
to expropriate the subject Lot 1406-B and that the same properties. The purpose of creating an ECOZONE and
was for a valid public purpose. In Sumulong v. Guerrero, other facilities is better served if respondent directly owns
this Court has ruled that, the areas subject of the expansion program.

The “public use” requirement for a valid X x x The expropriation of Lot 1406-B for the
exercise of the power of eminent domain is a purpose of being leased to banks and for the construction
flexible and evolving concept influenced by of a terminal has the purpose of making banking and
changing conditions. In this jurisdiction, the transportation facilities easily accessible to the persons
statutory and judicial trend has been summarized working at the industries located in PEZA. The
as follows: expropriation of adjacent areas therefore comes as a
matter of necessity to bring life to the purpose of the law.
In such a manner, PEZA’s goal of being a major force in

64
the economic development of the country would be 2. We have ruled that the concept of just
realized. Furthermore, this Court has already ruled that: compensation embraces not only the correct
determination of the amount to be paid to the owners of
X x x [T]he Legislature may directly the land, but also the payment of the land within a
determine the necessity for appropriating private reasonable time from its taking. Without prompt
property for a particular improvement for public payment, compensation cannot be considered “just”
use, and it may select the exact location of the inasmuch as the property owner is made to suffer the
improvement. In such a case, it is well-settled consequences of being immediately deprived of his land
that the utility of the proposed improvement, the while being made to wait for a decade or more before
existence of the public necessity for its actually receiving the amount necessary to cope with his
construction, the expediency of constructing it, loss. Payment of just compensation should follow as a
the suitableness of the location selected, are all matter of right immediately after the order of
questions exclusively for the legislature to expropriation is issued. Any delay in payment must be
determine, and the courts have no power to counted from said order. However, the delay to
interfere or to substitute their own views for those constitute a violation of due process must be
of the representatives of the people. unreasonable and inexcusable; it must be deliberately
done by a party in order to defeat the ends of justice.
In the absence of some constitutional or (Estate of Salud Jimenez v. PEZA, 349 SCRA 240,
statutory provisions to the contrary, the necessity Jan. 16, 2001, 2nd Div. [De Leon])
and expediency of exercising the right of eminent
domain are questions essentially political and not 143. The constitutionality of Sec. 92 of B.P. Blg. 881
judicial in their character. (City of Manila v. (requiring radio and television station owners and
Chinese Community of Manila, 40 Phil. 349 operators to give to the Comelec radio and television
[1919]) time free of charge) was challenged on the ground,
among others, that it violated the due process clause
Inasmuch as both Presidential Decree No. 66 and Republic and the eminent domain provision of the Constitution
Act No. 7916, bestow respondent with authority to by taking airtime from radio and television
develop terminal facilities and banking centers, this Court broadcasting stations without payment of just
will not question the respondent’s lease of certain portions compensation. Petitioners claim that the primary
of the expropriated lot to banks, as well as the source of revenue of radio and television stations is
construction of terminal facilities. the sale of airtime to advertisers and that to require
these stations to provide free airtime is to authorize a
Petitioner contends that respondent is bound by taking which is not “a de minimis temporary limitation
the representations of its Chief Civil Engineer when the or restraint upon the use of private property.” Will
latter testified before the trial court that the lot was to be you sustain the challenge?
devoted for the construction of government offices.
Anent this issue, suffice it to say that PEZA can vary the Held: All broadcasting, whether by radio or by
purpose for which a condemned lot will be devoted to, television stations, is licensed by the government.
provided that the same is for public use. Petitioner Airwave frequencies have to be allocated as there are
cannot impose or dictate on the respondent what facilities more individuals who want to broadcast than there are
to establish for as long as the same are for public frequencies to assign. A franchise is thus a privilege
purpose. (Estate of Salud Jimenez v. PEZA, 349 subject, among other things, to amendment by Congress
SCRA 240, Jan. 16, 2001, 2nd Div. [De Leon]) in accordance with the constitutional provision that “any
such franchise or right granted x x x shall be subject to
142. Discuss the meaning of “just compensation” in amendment, alteration or repeal by the Congress when
eminent domain proceedings. Does it include the the common good so requires.” (Art. XII, Sec. 11)
payment of “interest” and, if so, how is it to be
computed? Indeed, provisions for Comelec Time have been
made by amendment of the franchises of radio and
Held: 1. The constitutional limitation of “just television broadcast stations and such provisions have not
compensation” is considered to be the sum equivalent to been thought of as taking property without just
the market value of the property, broadly described to be compensation. Art. XII, Sec. 11 of the Constitution
the price fixed by the seller in open market in the usual authorizes the amendment of franchises for “the common
and ordinary course of legal action and competition or the good.” What better measure can be conceived for the
fair value of the property as between one who receives, common good than one for free airtime for the benefit not
and one who desires to sell it, fixed at the time of the only of candidates but even more of the public,
actual taking by the government. Thus, if property is particularly the voters, so that they will be fully informed
taken for public use before compensation is deposited of the issues in an election? “[I]t is the right of the
with the court having jurisdiction over the case, the final viewers and listeners, not the right of the broadcasters,
compensation must include interests on its just value to which is paramount.”
be computed from the time the property is taken to the
time when compensation is actually paid or deposited with Nor indeed can there be any constitutional
the court. In fine, between the taking of the property and objection to the requirement that broadcast stations give
the actual payment, legal interests accrue in order to free airtime. Even in the United States, there are
place the owner in a position as good as (but not better responsible scholars who believe that government controls
than) the position he was in before the taking occurred . on broadcast media can constitutionally be instituted to
(Republic of the Philippines v. The Hon. Court of ensure diversity of views and attention to public affairs to
Appeals, G.R. No. 146587, July 2, 2002, 1st Div. further the system of free expression. For this purpose,
[Vitug]) broadcast stations may be required to give free airtime to
candidates in an election.

65
In truth, radio and television broadcasting an expropriation suit on the ground that there was no
companies, which are given franchises, do not own the prior offer precludes another suit raising the same issue; it
airwaves and frequencies through which they transmit cannot, however, bar the State or its agent from
broadcast signals and images. They are merely given the thereafter complying with this requirement, as prescribed
temporary privilege of using them. Since a franchise is a by law, and subsequently exercising its power of eminent
mere privilege, the exercise of the privilege may domain over the same property. (Municipality of
reasonably be burdened with the performance by the Paranaque v. V.M. Realty Corporation, 292 SCRA
grantee of some form of public service. 678, July 20, 1998 [Panganiban])

In the granting of the privilege to operate 145. Discuss how expropriation may be initiated, and
broadcast stations and thereafter supervising radio and the two stages in expropriation.
television stations, the State spends considerable public
funds in licensing and supervising such stations. It would Held: Expropriation may be initiated by court
be strange if it cannot even require the licensees to action or by legislation. In both instances, just
render public service by giving free airtime. compensation is determined by the courts.

The claim that petitioner would be losing The expropriation of lands consists of two stages.
P52,380,000.00 in unrealized revenue from advertising is As explained in Municipality of Binan v. Garcia, reiterated
based on the assumption that airtime is “finished product” in National Power Corp. v. Jocson:
which, it is said, become the property of the company, like
oil produced from refining or similar natural resources The first is concerned with the
after undergoing a process for their production. As held determination of the authority of the plaintiff to
in Red Lion Broadcasting Co. v. F.C.C., which upheld the exercise the power of eminent domain and the
right of a party personally attacked to reply, “licenses to propriety of its exercise in the context of the facts
broadcast do not confer ownership of designated involved in the suit. It ends with an order, if not
frequencies, but only the temporary privilege of using dismissal of the action, "of condemnation
them.” Consequently, “a license permits broadcasting, declaring that the plaintiff has a lawful right to
but the licensee has no constitutional right to be the one take the property sought to be condemned, for
who holds the license or to monopolize a radio frequency the public use or purpose declared in the
to the exclusion of his fellow citizens. There is nothing in complaint, upon the payment of just
the First Amendment which prevents the government compensation to be determined as of the date of
from requiring a licensee to share his frequency with the filing of the complaint" x x x.
others and to conduct himself as a proxy or fiduciary with
obligations to present those views and voices which are The second phase of the eminent domain
representative of his community and which would action is concerned with the determination by the
otherwise, by necessity, be barred from the airwaves.” As court of “the just compensation for the property
radio and television broadcast stations do not own the sought to be taken.” This is done by the court
airwaves, no private property is taken by the requirement with the assistance of not more than three (3)
that they provide airtime to the Comelec. (TELEBAP, commissioners x x x.
Inc. v. COMELEC, 289 SCRA 337, April 21, 1998
[Mendoza]) It is only upon the completion of these two stages
that expropriation is said to have been completed.
144. May eminent domain be barred by "res judicata" (Republic v. Salem Investment Corporation, et. al.,
or "law of the case"? G.R. No. 137569, June 23, 2000, 2nd Div.
[Mendoza])
Held: The principle of res judicata, which finds
application in generally all cases and proceedings, cannot 146. May the owner of the property expropriated still
bar the right of the State or its agents to expropriate dispose of that property before the payment of just
private property. The very nature of eminent domain, as compensation? When does title over the property
an inherent power of the State, dictates that the right to expropriated pass to the expropriator?
exercise the power be absolute and unfettered even by a
prior judgment or res judicata. The scope of eminent Held: 1. [I]t is only upon payment of just
domain is plenary and, like police power, can “reach every compensation that title over the property passes to the
form of property which the State might need for public government. Therefore, until the action for expropriation
use.” All separate interests of individuals in property are has been completed and terminated, ownership over the
held of the government under this tacit agreement or property being expropriated remains with the registered
implied reservation. Notwithstanding the grant to owner. Consequently, the latter can exercise all rights
individuals, the eminent domain, the highest and most pertaining to an owner, including the right to dispose of
exact idea of property, remains in the government, or in his property, subject to the power of the State ultimately
the aggregate body of the people in their sovereign to acquire it through expropriation. (Republic v. Salem
capacity; and they have the right to resume the Investment Corporation, et. al., G.R. No. 137569,
possession of the property whenever the public interest June 23, 2000, 2nd Div. [Mendoza])
requires it.” Thus, the State or its authorized agent
cannot be forever barred from exercising said right by 2. The De la Ramas make much of the fact that
reason alone of previous non-compliance with any legal ownership of the land was transferred to the government
requirement. because the equitable and the beneficial title was already
acquired by it in 1983, leaving them with only the naked
While the principle of res judicata does not title. However, as this Court held in Association of Small
denigrate the right of the State to exercise eminent Landowners in the Phil., Inc. v. Secretary of Agrarian
domain, it does apply to specific issues decided in a Reform:
previous case. For example, a final judgment dismissing

66
The recognized rule, indeed, is that title legislative branch of government. For local governments
to the property expropriated shall pass from the to be able to wield the power, it must, by enabling law, be
owner to the expropriator only upon full payment delegated to it by the national legislature, but even then,
of the just compensation. Jurisprudence on this this delegated power of eminent domain is not, strictly
settled principle is consistent both here and in speaking, a power of eminent, but only of inferior, domain
other democratic jurisdictions. X x x or only as broad or confined as the real authority would
(Republic v. Salem Investment Corporation, et. al., want it to be.
G.R. No. 137569, June 23, 2000, 2nd Div.
[Mendoza]) Thus, in Valdehueza v. Republic where the private
landowners had remained unpaid ten years after the
147. Do the two (2) stages in expropriation apply only termination of the expropriation proceedings, this Court
to judicial, and not to legislative, expropriation? ruled –

Held: We see no point in distinguishing between “The points in dispute are whether such
judicial and legislative expropriation as far as the two payment can still be made and, if so, in what
stages mentioned above are concerned. Both involve amount. Said lots have been the subject of
these stages and in both the process is not completed expropriation proceedings. By final and executory
until payment of just compensation is made. (Republic judgment in said proceedings, they were
v. Salem Investment Corporation, et. al., G.R. No. condemned for public use, as part of an airport,
137569, June 23, 2000, 2nd Div. [Mendoza]) and ordered sold to the government. x x x It
follows that both by virtue of the judgment, long
148. Is prior unsuccessful negotiation a condition final, in the expropriation suit, as well as the
precedent for the exercise of eminent domain? annotations upon their title certificates, plaintiffs
are not entitled to recover possession of their
Held: Citing Iron and Steel Authority v. Court of expropriated lots – which are still devoted to the
Appeals, petitioner insists that before eminent domain public use for which they were expropriated – but
may be exercised by the state, there must be a showing only to demand the fair market value of the same.
of prior unsuccessful negotiation with the owner of the
property to be expropriated. Said relief may be granted under
plaintiffs’ prayer for: ‘such other remedies, which
This contention is not correct. As pointed out by may be deemed just and equitable under the
the Solicitor General the current effective law on premises’.”
delegated authority to exercise the power of eminent
domain is found in Section 12, Book III of the Revised The Court proceeded to reiterate its pronouncement in
Administrative Code, which provides: Alfonso v. Pasay City where the recovery of possession of
property taken for public use prayed for by the unpaid
“SEC. 12. Power of Eminent Domain – landowner was denied even while no requisite
The President shall determine when it is expropriation proceedings were first instituted. The
necessary or advantageous to exercise the power landowner was merely given the relief of recovering
of eminent domain in behalf of the National compensation for his property computed at its market
Government, and direct the Solicitor General, value at the time it was taken and appropriated by the
whenever he deems the action advisable, to State.
institute expropriation proceedings in the proper
court.” The judgment rendered by the Bulacan RTC in
1979 on the expropriation proceedings provides not only
The foregoing provision does not require prior for the payment of just compensation to herein
unsuccessful negotiation as a condition precedent for the respondents but likewise adjudges the property
exercise of eminent domain. In Iron and Steel Authority condemned in favor of petitioner over which parties, as
v. Court of Appeals, the President chose to prescribe this well as their privies, are bound. Petitioner has occupied,
condition as an additional requirement instead. In the utilized and, for all intents and purposes, exercised
instant case, however, no such voluntary restriction was dominion over the property pursuant to the judgment.
imposed. (SMI Development Corporation v. The exercise of such rights vested to it as the condemnee
Republic, 323 SCRA 862, Jan. 28, 2000, 3rd Div. indeed has amounted to at least a partial compliance or
[Panganiban]) satisfaction of the 1979 judgment, thereby preempting
any claim of bar by prescription on grounds of non-
149. When may the property owner be entitled to the execution. In arguing for the return of their property on
return of the expropriated property in eminent domain the basis of non-payment, respondents ignore the fact
cases? that the right of the expropriatory authority is far from
that of an unpaid seller in ordinary sales, to which the
Held: 1. In insisting on the return of the remedy of rescission might perhaps apply. An in rem
expropriated property, respondents would exhort on the proceeding, condemnation acts upon the property. After
pronouncement in Provincial Government of Sorsogon v. condemnation, the paramount title is in the public under a
Vda. De Villaroya where the unpaid landowners were new and independent title; thus, by giving notice to all
allowed the alternative remedy of recovery of the property claimants to a disputed title, condemnation proceedings
there in question. It might be borne in mind that the case provide a judicial process for securing better title against
involved the municipal government of Sorsogon, to which all the world than may be obtained by voluntary
the power of eminent domain is not inherent, but merely conveyance. (Republic of the Philippines v. The Hon.
delegated and of limited application. The grant of the Court of Appeals, G.R. No. 146587, July 2, 2002,
power of eminent domain to local governments under 1st Div. [Vitug])
Republic Act No. 7160 cannot be understood as being the
pervasive and all-encompassing power vested in the

67
2. Though the respondent has committed a those exempted from real estate taxes are lands, buildings
misdeed to petitioner, we cannot, however, grant the and improvements actually, directly and exclusively used
petitioner’s prayer for the return of the expropriated Lot for religious, charitable or educational purposes. Father
No. 1406-B. The Order of expropriation dated July 11, Joaquin G. Bernas, an eminent authority on the
1991, has long become final and executory. Petitioner Constitution and also a member of the Concom, adhered
cited Provincial Government of Sorsogon v. Rosa E. Vda. to the same view that the exemption created by said
De Villaroya to support its contention that it is entitled to provision pertained only to property taxes.
a return of the lot where this Court ruled that “under
ordinary circumstances, immediate return to the owners In his treatise on taxation, Mr. Justice Jose C.
of the unpaid property is the obvious remedy.” However, Vitug concurs, stating that "[t]he tax exemption covers
the said statement was not the ruling in that case. As in property taxes only." (Commissioner of Internal
other cases where there was no prompt payment by the Revenue v. CA, 298 SCRA 83, Oct. 14, 1998
government, this Court declared in Sorsogon that “the [Panganiban])
Provincial Government of Sorsogon is expected to
immediately pay as directed. Should any further delay be 152. Under Article XIV, Section 4, paragraph 3 of the
encountered, the trial court is directed to seize any 1987 Constitution, "[A]ll revenues and assets of non-
patrimonial property or cash savings of the province in the stock, non-profit educational institutions used actually,
amount necessary to implement this decision.” However, directly, and exclusively for educational purposes shall
this Court also stressed and declared in that case that “in be exempt from taxes and duties." YMCA alleged that
cases where land is taken for public use, public interest, it "is a non-profit educational institution whose
however, must be considered.” (Estate of Salud revenues and assets are used actually, directly and
Jimenez v. PEZA, 349 SCRA 240, Jan. 16, 2001, 2nd exclusively for educational purposes so it is exempt
Div. [De Leon]) from taxes on its properties and income."

Held: We reiterate that private respondent is


The Power of Taxation exempt from the payment of property tax, but not income
tax on the rentals from its property. The bare allegation
150. Can taxes be subject to off-setting or alone that it is a non-stock, non-profit educational
compensation? institution is insufficient to justify its exemption from the
payment of income tax.
Held: Taxes cannot be subject to compensation
for the simple reason that the government and the [L]aws allowing tax exemption are construed
taxpayer are not creditors and debtors of each other . strictissimi juris. Hence, for the YMCA to be granted the
There is a material distinction between a tax and debt. exemption it claims under the abovecited provision, it
Debts are due to the Government in its corporate must prove with substantial evidence that (1) it falls under
capacity, while taxes are due to the Government in its the classification non-stock, non-profit educational
sovereign capacity. It must be noted that a distinguishing institution; and (2) the income it seeks to be exempted
feature of a tax is that it is compulsory rather than a from taxation is used actually, directly, and exclusively for
matter of bargain. Hence, a tax does not depend upon educational purposes. However, the Court notes that not
the consent of the taxpayer. If any taxpayer can defer a scintilla of evidence was submitted by private
the payment of taxes by raising the defense that it still respondent to prove that it met the said requisites.
has a pending claim for refund or credit, this would (Commissioner of Internal Revenue v. CA, 298
adversely affect the government revenue system. A SCRA 83, Oct. 14, 1998 [Panganiban])
taxpayer cannot refuse to pay his taxes when they fall due
simply because he has a claim against the government or 153. Is the YMCA an educational institution within the
that the collection of a tax is contingent on the result of purview of Article XIV, Section 4, par. 3 of the
the lawsuit it filed against the government. (Philex Constitution?
Mining Corporation v. Commissioner of Internal
Revenue, 294 SCRA 687, Aug. 28, 1998 [Romero]) Held: We rule that it is not. The term
“educational institution” or “institution of learning” has
151. Under Article VI, Section 28, paragraph 3 of the acquired a well-known technical meaning, of which the
1987 Constitution, "[C]haritable institutions, churches members of the Constitutional Commission are deemed
and parsonages or convents appurtenant thereto, cognizant. Under the Education Act of 1982, such term
mosques, non-profit cemeteries, and all lands, refers to schools. The school system is synonymous with
buildings, and improvements, actually, directly and formal education, which “refers to the hierarchically
exclusively used for religious, charitable or educational structured and chronologically graded learnings organized
purposes shall be exempt from taxation." YMCA and provided by the formal school system and for which
claims that the income earned by its building leased to certification is required in order for the learner to progress
private entities and that of its parking space is through the grades or move to the higher levels.” The
likewise covered by said exemption. Resolve. Court has examined the “Amended Articles of
Incorporation” and “By-Laws” of the YMCA, but found
Held: The debates, interpellations and nothing in them that even hints that it is a school or an
expressions of opinion of the framers of the Constitution educational institution.
reveal their intent that which, in turn, may have guided
the people in ratifying the Charter. Such intent must be Furthermore, under the Education Act of 1982,
effectuated. even non-formal education is understood to be school-
based and “private auspices such as foundations and
Accordingly, Justice Hilario G. Davide, Jr., a civic-spirited organizations” are ruled out. It is settled
former constitutional commissioner, who is now a member that the term “educational institution,” when used in laws
of this Court, stressed during the Concom debates that "x granting tax exemptions, refers to a “x x x school
x x what is exempted is not the institution itself x x x; seminary, college or educational establishment x x x.” (84

68
CJS 566) Therefore, the private respondent cannot be Double taxation usually takes place when a
deemed one of the educational institutions covered by the person is resident of a contracting state and derives
constitutional provision under consideration. income from, or owns capital in, the other contracting
(Commissioner of Internal Revenue v. CA, 298 state and both states impose tax on that income or
SCRA 83, Oct. 14, 1998 [Panganiban]) capital. In order to eliminate double taxation, a tax treaty
resorts to several methods. First, it sets out the
154. May the PCGG validly commit to exempt from all respective rights to tax of the state of source or situs and
forms of taxes the properties to be retained by the of the state of residence with regard to certain classes of
Marcos heirs in a Compromise Agreement between income or capital. In some cases, an exclusive right to
the former and the latter? tax is conferred on one of the contracting states;
however, for other items of income or capital, both states
Held: The power to tax and to grant exemptions are given the right to tax, although the amount of tax that
is vested in the Congress and, to a certain extent, in the may be imposed by the state of source is limited.
local legislative bodies. Section 28(4), Article VI of the
Constitution, specifically provides: “No law granting any The second method for the elimination of double
tax exemption shall be passed without the concurrence of taxation applies whenever the state of source is given a
a majority of all the members of the Congress.” The full or limited right to tax together with the state of
PCGG has absolutely no power to grant tax exemptions, residence. In this case, the treaties make it incumbent
even under the cover of its authority to compromise ill- upon the state of residence to allow relief in order to
gotten wealth cases. avoid double taxation. There are two methods of relief -
the exemption method and the credit method. In the
Even granting that Congress enacts a law exemption method, the income or capital which is taxable
exempting the Marcoses from paying taxes on their in the state of source or situs is exempted in the state of
properties, such law will definitely not pass the test of the residence, although in some instances it may be taken
equal protection clause under the Bill of Rights. Any into account in determining the rate of tax applicable to
special grant of tax exemption in favor only of the Marcos the taxpayer's remaining income or capital. On the other
heirs will constitute class legislation. It will also violate hand, in the credit method, although the income or capital
the constitutional rule that “taxation shall be uniform and which is taxed in the state of source is still taxable in the
equitable.” (Chavez v. PCGG, 299 SCRA 744, Dec. 9, state of residence, the tax paid in the former is credited
1998 [Panganiban]) against the tax levied in the latter. The basic difference
between the two methods is that in the exemption
155. Discuss the purpose of tax treaties? method, the focus is on the income or capital itself,
whereas the credit method focuses upon the tax.
Held: The RP-US Tax Treaty is just one of a (Commissioner of Internal Revenue v. S.C. Johnson
number of bilateral treaties which the Philippines has and Son, Inc., 309 SCRA 87, 102-103, June 25,
entered into for the avoidance of double taxation. The 1999)
purpose of these international agreements is to reconcile
the national fiscal legislations of the contracting parties in 158. What is the rationale for reducing the tax rate in
order to help the taxpayer avoid simultaneous taxation in negotiating tax treaties?
two different jurisdictions. More precisely, the tax
conventions are drafted with a view towards the Held: In negotiating tax treaties, the underlying
elimination of international juridical double taxation x x x. rationale for reducing the tax rate is that the Philippines
(Commissioner of Internal Revenue v. S.C. Johnson will give up a part of the tax in the expectation that the
and Son, Inc., 309 SCRA 87, 101-102, June 25, tax given up for this particular investment is not taxed by
1999, 3rd Div. [Gonzaga-Reyes]) the other country. (Commissioner of Internal
Revenue v. S.C. Johnson and Son, Inc., 309 SCRA
156. What is “international juridical double taxation”? 87, 103, June 25, 1999)

Held: It is defined as the imposition of


comparable taxes in two or more states on the same THE BILL OF RIGHTS
taxpayer in respect of the same subject matter and for
identical periods. (Commissioner of Internal The Due Process Clause
Revenue v. S.C. Johnson and Son, Inc., 309 SCRA
87, 102, June 25, 1999) 159. Discuss the Due Process Clause. Distinguish
substantive due process from procedural due process.
157. What is the rationale for doing away with
international juridical double taxation? What are the Held: Section 1 of the Bill of Rights lays down
methods resorted to by tax treaties to eliminate what is known as the “due process clause” of the
double taxation? Constitution.

Held: The apparent rationale for doing away In order to fall within the aegis of this provision,
with double taxation is to encourage the free flow of two conditions must concur, namely, that there is a
goods and services and the movement of capital, deprivation and that such deprivation is done without
technology and persons between countries, conditions proper observance of due process. When one speaks of
deemed vital in creating robust and dynamic economies. due process of law, however, a distinction must be made
Foreign investments will only thrive in a fairly predictable between matters of procedure and matters of substance.
and reasonable international investment climate and the In essence, procedural due process “refers to the method
protection against double taxation is crucial in creating or manner by which the law is enforced,” while
such a climate. substantive due process “requires that the law itself, not
merely the procedures by which the law would be
enforced, is fair, reasonable, and just.” (Corona v.

69
United Harbor Pilots Association of the Phils., 283
SCRA 31, Dec. 12, 1997 [Romero]) Before harbor pilots can earn a license to practice
their profession, they literally have to pass through the
160. Respondents United Harbor Pilots Association of proverbial eye of a needle by taking, not one but five
the Philippines argue that due process was not examinations, each followed by actual training and
observed in the adoption of PPA-AO No. 04-92 which practice. X x x
provides that: “(a)ll existing regular appointments
which have been previously issued by the Bureau of Their license is granted in the form of an
Customs or the PPA shall remain valid up to 31 appointment which allows them to engage in pilotage until
December 1992 only,” and “(a)ll appointments to they retire at the age of 70 years. This is a vested right.
harbor pilot positions in all pilotage districts shall, Under the terms of PPA-AO No. 04-92, “[a]ll existing
henceforth, be only for a term of one (1) year from regular appointments which have been previously issued
date of effectivity subject to renewal or cancellation by the Bureau of Customs or the PPA shall remain valid up
by the Philippine Ports Authority after conduct of a to 31 December 1992 only,” and “(a)ll appointments to
rigid evaluation of performance,” allegedly because no harbor pilot positions in all pilotage districts shall,
hearing was conducted whereby “relevant henceforth, be only for a term of one (1) year from date
government agencies” and the harbor pilots of effectivity subject to renewal or cancellation by the
themselves could ventilate their views. They also Authority after conduct of a rigid evaluation of
contended that the sole and exclusive right to the performance.”
exercise of harbor pilotage by pilots has become
vested and can only be “withdrawn or shortened” by It is readily apparent that PPA-AO No. 04-92
observing the constitutional mandate of due process unduly restricts the right of harbor pilots to enjoy their
of law. profession before their compulsory retirement. In the
past, they enjoyed a measure of security knowing that
Held: They are obviously referring to the after passing five examinations and undergoing years of
procedural aspect of the enactment. Fortunately, the on-the-job training, they would have a license which they
Court has maintained a clear position in this regard, a could use until their retirement, unless sooner revoked by
stance it has stressed in the recent case of Lumiqued v. the PPA for mental or physical unfitness. Under the new
Hon. Exevea, where it declared that “(a)s long as a party issuance, they have to contend with an annual
was given the opportunity to defend his interests in due cancellation of their license which can be temporary or
course, he cannot be said to have been denied due permanent depending on the outcome of their
process of law, for this opportunity to be heard is the very performance evaluation. Veteran pilots and neophytes
essence of due process. Moreover, this constitutional alike are suddenly confronted with one-year terms which
mandate is deemed satisfied if a person is granted an ipso facto expire at the end of that period. Renewal of
opportunity to seek reconsideration of the action or ruling their license is now dependent on a “rigid evaluation of
complained of.” performance” which is conducted only after the license
has already been cancelled. Hence, the use of the term
In the case at bar, respondents questioned PPA- “renewal.” It is this pre-evaluation cancellation which
AO No. 04-92 no less than four times before the matter primarily makes PPA-AO No. 04-92 unreasonable and
was finally elevated to this Tribunal. Their arguments on constitutionally infirm. In a real sense, it is a deprivation
this score, however, failed to persuade. X x x of property without due process of law. (Corona v.
United Harbor Pilots Association of the Phils., 283
Neither does the fact that the pilots themselves SCRA 31, December 12, 1997 [Romero])
were not consulted in any way taint the validity of the
administrative order. As a general rule, notice and 161. Does the due process clause encompass the right
hearing, as the fundamental requirements of procedural to be assisted by counsel during an administrative
due process, are essential only when an administrative inquiry?
body exercises its quasi-judicial function. In the
performance of its executive or legislative functions, such Held: The right to counsel, which cannot be
as issuing rules and regulations, an administrative body waived unless the waiver is in writing and in the presence
need not comply with the requirements of notice and of counsel, is a right afforded a suspect or an accused
hearing. during custodial investigation. It is not an absolute right
and may, thus, be invoked or rejected in a criminal
Upon the other hand, it is also contended that the proceeding and, with more reason, in an administrative
sole and exclusive right to the exercise of harbor pilotage inquiry. In the case at bar, petitioners invoke the right of
by pilots is a settled issue. Respondents aver that said an accused in criminal proceedings to have competent
right has become vested and can only be “withdrawn or and independent counsel of his own choice. Lumiqued,
shortened” by observing the constitutional mandate of however, was not accused of any crime in the proceedings
due process of law. Their argument has thus shifted from below. The investigation conducted by the committee x x
the procedural to one of substance. It is here where PPA- x was for the sole purpose of determining if he could be
AO No. 04-92 fails to meet the condition set by the held administratively liable under the law for the
organic law. complaints filed against him. x x x As such, the hearing
conducted by the investigating committee was not part of
Pilotage, just like other professions, may be a criminal prosecution. X x x
practiced only by duly licensed individuals. Licensure is
“the granting of license especially to practice a While investigations conducted by an
profession.” It is also “the system of granting licenses (as administrative body may at times be akin to a criminal
for professional practice) in accordance with established proceeding, the fact remains that under existing laws, a
standards.” A license is a right or permission granted by party in an administrative inquiry may or may not be
some competent authority to carry on a business or do an assisted by counsel, irrespective of the nature of the
act which, without such license, would be illegal. charges and of the respondent's capacity to represent

70
himself, and no duty rests on such a body to furnish the x x (People v. Dela Piedra, 350 SCRA 163,
person being investigated with counsel. In an Jan. 24, 2001, 1st Div. [Kapunan])
administrative proceeding x x x a respondent x x x has the
option of engaging the services of counsel or not. x x x 2. The doctrine has been formulated in various
Thus, the right to counsel is not imperative in ways, but is commonly stated to the effect that a statute
administrative investigations because such inquiries are establishing a criminal offense must define the offense
conducted merely to determine whether there are facts with sufficient definiteness that persons of ordinary
that merit disciplinary measures against erring public intelligence can understand what conduct is prohibited by
officers and employees, with the purpose of maintaining the statute. It can only be invoked against that specie of
the dignity of government service. legislation that is utterly vague on its face, i.e., that which
cannot be clarified either by a saving clause or by
The right to counsel is not indispensable to due construction.
process unless required by the Constitution or the law.
Lumiqued v. Exevea, 282 SCRA 125, Nov. 18, 1997 A statute or act may be said to be vague when it
[Romero]) lacks comprehensible standards that men of common
intelligence must necessarily guess at its meaning and
162. Discuss the “Void for Vagueness” Doctrine, and differ in its application. In such instance, the statute is
why is it repugnant to the Constitution. Distinguish a repugnant to the Constitution in two (2) respects – it
“perfectly vague act” from “legislation couched in violated due process for failure to accord persons,
imprecise language.” especially the parties targeted by it, fair notice of what
conduct to avoid; and, it leaves law enforcers unbridled
Held: 1. Due process requires that the terms of discretion in carrying out its provisions and becomes an
a penal statute must be sufficiently explicit to inform arbitrary flexing of the Government muscle. But the
those who are subject to it what conduct on their part will doctrine does not apply as against legislations that are
render them liable to its penalties. A criminal statute that merely couched in imprecise language but which
“fails to give a person of ordinary intelligence fair notice nonetheless specify a standard though defectively
that his contemplated conduct is forbidden by the phrased; or to those that are apparently ambiguous yet
statute,” or is so indefinite that “it encourages arbitrary fairly applicable to certain types of activities. The first
and erratic arrests and convictions,” is void for vagueness . may be “saved” by proper construction, while no
The constitutional vice in a vague or indefinite statute is challenge may be mounted as against the second
the injustice to the accused in placing him on trial for an whenever directed against such activities. With more
offense, the nature of which he is given no fair warning. reason, the doctrine cannot be invoked where the assailed
statute is clear and free from ambiguity, as in this case.
We reiterated these principles in People v.
Nazario: The test in determining whether a criminal statute
is void for uncertainty is whether the language conveys a
As a rule, a statute or act may be said to sufficiently definite warning as to the proscribed conduct
be vague when it lacks comprehensible standards when measured by common understanding and practice .
that men “of common intelligence must It must be stressed, however, that the “vagueness”
necessarily guess at its meaning and differ as to doctrine merely requires a reasonable degree of certainty
its application.” It is repugnant to the for the statute to be upheld – not absolute precision or
Constitution in two respects: (1) it violates due mathematical exactitude, as petitioner seems to suggest.
process for failure to accord persons, especially Flexibility, rather than meticulous specificity, is permissible
the parties targeted by it, fair notice of the as long as the metes and bounds of the statute are clearly
conduct to avoid; and (2) it leaves law enforcers delineated. An act will not be held invalid merely because
unbridled discretion in carrying out its provisions it might have been more explicit in its wordings or
and become an arbitrary flexing of the detailed in its provisions, especially where, because of the
Government muscle. nature of the act, it would be impossible to provide all the
details in advance as in all other statutes. (Joseph
We added, however, that: Ejercito Estrada v. Sandiganbayan [Third Division],
G.R. No. 148560, Nov. 19, 2001, En Banc
X x x the act must be utterly vague on its [Bellosillo])
face, that is to say, it cannot be clarified by either
a saving clause or by construction. Thus, in 163. Does Article 13 (b) of the Labor Code defining
Coates v. City of Cincinnati, the U.S. Supreme “recruitment and placement” violate the due process
Court struck down an ordinance that had made it clause?
illegal for “three or more persons to assemble on
any sidewalk and there conduct themselves in a Held: In support of her submission that Article
manner annoying to persons passing by.” Clearly, 13 (b) is void for vagueness, appellant invokes People v.
the ordinance imposed no standard at all Panis, where this Court x x x “criticized” the definition of
“because one may never know in advance what “recruitment and placement” x x x.
annoys some people but does not annoy others.”
Appellant further argues that the acts that
Coates highlights what has been referred constitute “recruitment and placement” suffer from
to as a “perfectly vague” act whose obscurity is overbreadth since by merely “referring” a person for
evident on its face. It is to be distinguished, employment, a person may be convicted of illegal
however, from legislation coached in imprecise recruitment.
language – but which nonetheless specifies a
standard though defectively phrased – in which These contentions cannot be sustained.
case, it may be “saved” by proper construction. X

71
Appellant’s reliance on People v. Panis is judge in its application; the counsel, in defending one
misplaced. The issue in Panis was whether, under the charged with its violation; and more importantly, the
proviso of Article 13(b), the crime of illegal recruitment accused, in identifying the realm of the proscribed
could be committed only “whenever two or more persons conduct. Indeed, it can be understood with little difficulty
are in any manner promised or offered any employment that what the assailed statute punishes is the act of a
for a fee.” The Court held in the negative x x x. public officer in amassing or accumulating ill-gotten
wealth of at least P50,000,000.00 through a series or
X x x The Court, in Panis, merely bemoaned the combination of acts enumerated in Sec. 1, par. (d), of the
lack of records that would help shed light on the meaning Plunder Law.
of the proviso. The absence of such records
notwithstanding, the Court was able to arrive at a In fact, the amended Information itself closely
reasonable interpretation of the proviso by applying tracks the language of the law, indicating with reasonable
principles in criminal law and drawing from the language certainty the various elements of the offense which
and intent of the law itself. Section 13(b), therefore, is petitioner is alleged to have committed x x x.
not a “perfectly vague act” whose obscurity is evident on
its face. If at all, the proviso therein is merely couched in We discern nothing in the foregoing that is vague
imprecise language that was salvaged by proper or ambiguous – as there is obviously none – that will
construction. It is not void for vagueness. confuse petitioner in his defense. Although subject to
proof, these factual assertions clearly show that the
Xxx elements of the crime are easily understood and provide
adequate contrast between the innocent and the
That Section 13(b) encompasses what appellant prohibited acts. Upon such unequivocal assertions,
apparently considers as customary and harmless acts such petitioner is completely informed of the accusations
as “labor or employment referral” (“referring” an against him as to enable him to prepare for an intelligent
applicant, for employment to a prospective employer) defense.
does not render the law overbroad. Evidently, appellant
misapprehends concept of overbreadth. Petitioner, however, bewails the failure of the law
to provide for the statutory definition of the terms
A statute may be said to be overbroad where it “combination” and “series” in the key phrase “a
operates to inhibit the exercise of individual freedoms combination or series of overt or criminal acts” found in
affirmatively guaranteed by the Constitution, such as the Sec. 1, par. (d), and Sec. 2, and the word “pattern” in
freedom of speech or religion. A generally worded Sec. 4. These omissions, according to petitioner, render
statute, when construed to punish conduct which cannot the Plunder Law unconstitutional for being impermissibly
be constitutionally punished is unconstitutionally vague to vague and overbroad and deny him the right to be
the extent that it fails to give adequate warning of the informed of the nature and cause of the accusation
boundary between the constitutionally permissible and the against him, hence, violative of his fundamental right to
constitutionally impermissible applications of the statute . due process.

In Blo Umpar Adiong v. Commission on Elections, The rationalization seems to us to be pure


for instance, we struck down as void for overbreadth sophistry. A statute is not rendered uncertain and void
provisions prohibiting the posting of election propaganda merely because general terms are used therein, or
in any place – including private vehicles – other than in because of the employment of terms without defining
the common poster areas sanctioned by the COMELEC. them; much less do we have to define every word we use.
We held that the challenged provisions not only deprived Besides, there is no positive constitutional or statutory
the owner of the vehicle the use of his property but also command requiring the legislature to define each and
deprived the citizen of his right to free speech and every word in an enactment. Congress is not restricted in
information. The prohibition in Adiong, therefore, was so the form of expression of its will, and its inability to so
broad that it covered even constitutionally guaranteed define the words employed in a statute will not necessarily
rights and, hence, void for overbreadth. In the present result in the vagueness or ambiguity of the law so long as
case, however, appellant did not even specify what the legislative will is clear, or at least, can be gathered
constitutionally protected freedoms are embraced by the from the whole act, which is distinctly expressed in the
definition of “recruitment and placement” that would Plunder Law.
render the same constitutionally overbroad. (People v.
Dela Piedra, 350 SCRA 163, Jan. 24, 2001, 1st Div. Moreover, it is a well-settled principle of legal
[Kapunan]) hermeneutics that words of a statute will be interpreted in
their natural, plain and ordinary acceptation and
164. Is the Plunder Law unconstitutional for being signification, unless it is evident that the legislature
vague? intended a technical or special legal meaning to those
words. The intention of the lawmakers – who are,
Held: As it is written, the Plunder Law contains ordinarily, untrained philologists and lexicographers – to
ascertainable standards and well-defined parameters use statutory phraseology in such a manner is always
which would enable the accused to determine the nature presumed. Thus, Webster’s New Collegiate Dictionary
of his violation. Section 2 is sufficiently explicit in its contains the following commonly accepted definition of
description of the acts, conduct and conditions required or the words “combination” and “series.”
forbidden, and prescribes the elements of the crime with
reasonable certainty and particularity. X x x Combination – the result or product of
combining; the act or process of combining. To
As long as the law affords some comprehensible combine is to bring into such close relationship as
guide or rule that would inform those who are subject to it to obscure individual characters.
what conduct would render them liable to its penalties, its
validity would be sustained. It must sufficiently guide the

72
Series – a number of things or events of
the same class coming one after another in spatial The void-for-vagueness doctrine states
and temporal succession. that “a statute which either forbids or requires the
doing of an act in terms so vague that men of
That Congress intended the words “combination” common intelligence must necessarily guess at its
and “series” to be understood in their popular meanings is meaning and differ as to its application violates
pristinely evident from the legislative deliberations on the the first essential of due process of law.”
bill which eventually became RA 7080 or the Plunder Law (Connally v. General Constr. Co., 269 U.S. 385,
x x x. 391, 70 L. Ed. 328 [1926] cited in Ermita-Malate
Hotel and Motel Operators Ass’n. v. City Mayor, 20
Xxx SCRA 849, 867 [1967]) The overbreadth
doctrine, on the other hand, decrees that “a
Thus when the Plunder Law speaks of governmental purpose may not be achieved by
“combination,” it is referring to at least two (2) acts falling means which sweep unnecessarily broadly and
under different categories or enumeration provided in Sec. thereby invade the area of protected freedoms.”
1, par. (d), e.g., raids on the public treasury in Sec. 1, (NAACP v. Alabama, 377 U.S. 288, 307, 12, 2 L.
par. (d), subpar. (1), and fraudulent conveyance of assets Ed 325, 338 [1958]; Shelton v. Tucker, 364 U.S.
belonging to the National Government under Sec. 1, par. 479, 5 L. Ed. 2d 231 [1960])
(d), subpar. (3).
A facial challenge is allowed to be made
On the other hand, to constitute a “series” there to a vague statute and to one which is overbroad
must be two (2) or more overt or criminal acts falling because of possible “chilling effect” upon
under the same category of enumeration found in Sec. 1, protected speech. The theory is that “[w]hen
par. (d), say, misappropriation, malversation and raids on statutes regulate or proscribe speech and no
the public treasury, all of which fall under Sec. 1, par. (d), readily apparent construction suggests itself as a
subpar. (1). Verily, had the legislature intended a vehicle for rehabilitating the statutes in a single
technical or distinctive meaning for “combination” and prosecution, the transcendent value to all society
“series,” it would have taken greater pains in specifically of constitutionally protected expression is deemed
providing for it in the law. to justify allowing attacks on overly broad statutes
with no requirement that the person making the
As for “pattern,” we agree with the observations attack demonstrate that his own conduct could
of the Sandiganbayan that this term is sufficiently defined not be regulated by a statute drawn with narrow
in Sec. 4, in relation to Sec. 1, par. (d), and Sec. 2 – specificity.” (Gooding v. Wilson, 405 U.S. 518,
521, 31 L. Ed. 2d 408, 413 [1972] [internal
x x x under Sec. 1 (d) of the law, a quotation marks omitted]) The possible harm to
‘pattern’ consists of at least a combination or society in permitting some unprotected speed to
series of overt or criminal acts enumerated go unpunished is outweighed by the possibility
in subsections (1) to (6) of Sec. 1 (d). that the protected speech of others may be
Secondly, pursuant to Sec. 2 of the law, the deterred and perceived grievances left to fester
pattern of overt or criminal acts is directed because of possible inhibitory effects of overly
towards a common purpose or goal which is broad statutes.
to enable the public officer to amass,
accumulate or acquire ill-gotten wealth. This rationale does not apply to penal
And thirdly, there must either be an ‘overall statutes. Criminal statutes have general in
unlawful scheme’ or ‘conspiracy’ to achieve terrorem effect resulting from their very
said common goal. As commonly understood, the existence, and, if facial challenge is allowed for
term ‘overall unlawful scheme’ indicates a ‘general this reason alone, the State may well be
plan of action or method’ which the principal prevented from enacting laws against socially
accused and public officer and others conniving harmful conduct. In the area of criminal law, the
with him follow to achieve the aforesaid common law cannot take chances as in the area of free
goal. In the alternative, if there is no such overall speech.
scheme or where the schemes or methods used
by multiple accused vary, the overt or criminal The overbreadth and vagueness doctrine
acts must form part of a conspiracy to attain a then have special application only to free speech
common goal. cases. They are inapt for testing the validity of
penal statutes. As the U.S. Supreme Court put it,
Xxx in an opinion by Chief Justice Rehnquist, “we have
not recognized an ‘overbreadth’ doctrine outside
Hence, it cannot plausibly be contended that the the limited context of the First Amendment.” In
law does not give a fair warning and sufficient notice of Broadwick v. Oklahoma (413 U.S. 601, 612-613,
what it seeks to penalize. Under the circumstances, 37 L Ed. 2d 830, 840-841 [1973]), the Court ruled
petitioner’s reliance on the “void-for-vagueness” doctrine that “claims of facial overbreadth have been
is manifestly misplaced. entertained in cases involving statutes which, by
their terms, seek to regulate only spoken words”
Xxx and, again, that “overbreadth claims, if
entertained at all, have been curtailed when
Moreover, we agree with, hence we adopt, the invoked against ordinary criminal laws that are
observations of Mr. Justice Vicente V. Mendoza during the sought to be applied to protected conduct.” For
deliberations of the Court that the allegations that the this reason, it has been held that “a facial
Plunder Law is vague and overbroad do not justify a facial challenge to a legislative act is the most difficult
review of its validity – challenge to mount successfully, since the

73
challenger must establish that no set of constitutional questions, whichever way
circumstances exists under which the Act would they might be decided.
be valid.” (United States v. Salerno, supra.) As
for the vagueness doctrine, it is said that a litigant For these reasons, “on its face” invalidation of
may challenge a statute on its face only if it is statutes has been described as “manifestly strong
vague in all its possible applications. “A plaintiff medicine,” to be employed “sparingly and only as
who engages in some conduct that is clearly a last resort,” (Broadwick v. Oklahoma, 413 U.S.
proscribed cannot complain of the vagueness of at 613, 37 L.Ed.2d at 841; National Endowment
the law as applied to the conduct of others.” for the Arts v. Finley, 524 U.S. 569, 580 [1998])
(Village of Hoffman Estates v. Flipside, Hoffman and is generally disfavored (FW/PBS, Inc. v. City
Estates, Inc., 455 U.S. 489, 494-95, 71 L Ed. 2d of Dallas, 493 U.S. 223, 107 L.Ed.2d 603 [1990];
362, 369 [1982]) Cruz v. Secretary of Environment and Natural
Resources, G.R. No. 135385, 6 December 2000
In sum, the doctrines of strict scrutiny, [Mendoza, J., Separate Opinion]). In determining
overbreadth, and vagueness are analytical tools the constitutionality of a statute, therefore, its
developed for testing “on their faces” statutes in provisions which are alleged to have been violated
free speech cases or, as they are called in in a case must be examined in the light of the
American law, First Amendment cases. They conduct with which the defendant is charged
cannot be made to do service when what is (United States v. National Dairy Prod. Corp., 372
involved is a criminal statute. With respect to U.S. 29, 32-33, 9 L.Ed.2d 561, 565-6 [1963])
such statute, the established rule is that “one to
whom application of a statute is constitutional will In light of the foregoing disquisition, it is evident
not be heard to attack the statute on the ground that the purported ambiguity of the Plunder Law, so
that impliedly it might also be taken as applying tenaciously claimed and argued at length by petitioner, is
to other persons or other situations in which its more imagined than real. Ambiguity, where none exists,
application might be unconstitutional.” (United cannot be created by dissecting parts and words in the
States v. Raines, 362 U.S. 17, 21, 4 L. Ed. 2d 524, statute to furnish support to critics who cavil at the want
529 [1960]. The paradigmatic case is Yazoo & of scientific precision in the law. Every provision of the
Mississippi Valley RR. v. Jackson Vinegar Co., 226 law should be construed in relation and with reference to
U.S. 217, 57 l. Ed. 193 [1912]) As has been every other part. To be sure, it will take more than
pointed out, “vagueness challenges in the First nitpicking to overturn the well-entrenched presumption of
Amendment context, like overbreadth challenges constitutionality and validity of the Plunder Law. A
typically produce facial invalidation, while statutes fortiori, petitioner cannot feign ignorance of what the
found to be vague as a matter of due process Plunder Law is all about. Being one of the Senators who
typically are invalidated [only] ‘as applied’ to a voted for its passage, petitioner must be aware that the
particular defendant.” (G. Gunther & K. Sullivan, law was extensively deliberated upon by the Senate and
Constitutional Law 1299 [2001]) Consequently, its appropriate committees by reason of which he even
there is no basis for petitioner’s claim that this registered his affirmative vote with full knowledge of its
Court review the Anti-Plunder Law on its face and legal implications and sound constitutional anchorage.
in its entirety. (Joseph Ejercito Estrada v. Sandiganbayan [Third
Division], G.R. No. 148560, Nov. 19, 2001, En Banc
Indeed, “on its face” invalidation of [Bellosillo])
statutes results in striking them down entirely on
the ground that they might be applied to parties 165. Does an extraditee have the right to notice and
not before the Court whose activities are hearing during the evaluation stage of an extradition
constitutionally protected (Id. at 1328). It proceeding?
constitutes a departure from the case and
controversy requirement of the Constitution and Held: Considering that in the case at bar, the
permits decisions to be made without concrete extradition proceeding is only at its evaluation stage, the
factual settings and in sterile abstract contexts nature of the right being claimed by the private
(Constitution, Art. VIII, Sections 1 and 5. respondent is nebulous and the degree of prejudice he
Compare Angara v. Electoral Commission, 63 Phil. will allegedly suffer is weak, we accord greater weight to
139, 158 [1936]). But, as the U.S. Supreme the interests espoused by the government thru the
Court pointed out in Younger v. Harris (401 U.S. petitioner Secretary of Justice. X x x
37, 52-53, 27 L. Ed. 2d 669, 680 [1971]; others
omitted.) In tilting the balance in favor of the interests of
the State, the Court stresses that it is not ruling that the
[T]he task of analyzing a private respondent has no right to due process at all
proposed statute, pinpointing its throughout the length and breadth of the extradition
deficiencies, and requiring correction of proceedings. Procedural due process requires a
these deficiencies before the statute is determination of what process is due, when it is due, and
put into effect, is rarely if ever an the degree of what is due. Stated otherwise, a prior
appropriate task for the judiciary. The determination should be made as to whether procedural
combination of the relative remoteness of protections are at all due and when they are due, which in
the controversy, the impact on the turn depends on the extent to which an individual will be
legislative process of the relief sought, "condemned to suffer grievous loss." We have explained
and above all the speculative and why an extraditee has no right to notice and hearing
amorphous nature of the required line-by- during the evaluation stage of the extradition process. As
line analysis of detailed statutes, x x x aforesaid, P.D. No. 1069 which implements the RP-US
ordinarily results in a kind of case that is Extradition Treaty affords an extraditee sufficient
wholly unsatisfactory for deciding opportunity to meet the evidence against him once the

74
petition is filed in court. The time for the extraditee to In this light, would it be proper and just for the
know the basis of the request for his extradition is merely government to increase the risk of violating its treaty
moved to the filing in court of the formal petition for obligations in order to accord Respondent Jimenez his
extradition. The extraditee's right to know is momentarily personal liberty in the span of time that it takes to resolve
withheld during the evaluation stage of the extradition the Petition for Extradition? His supposed immediate
process to accommodate the more compelling interest of deprivation of liberty without due process that he had
the State to prevent escape of potential extraditees which previously shunned pales against the government’s
can be precipitated by premature information of the basis interest in fulfilling its Extradition Treaty obligations and in
of the request for his extradition. No less compelling at cooperating with the world community in the suppression
that stage of the extradition proceedings is the need to be of crime. Indeed, “[c]onstitutional liberties do not exist in
more deferential to the judgment of a co-equal branch of a vacuum; the due process rights accorded to individuals
the government, the Executive, which has been endowed must be carefully balanced against exigent and palpable
by our Constitution with greater power over matters government interest.”
involving our foreign relations. Needless to state, this
balance of interests is not a static but a moving balance Too, we cannot allow our country to be a haven
which can be adjusted as the extradition process moves for fugitives, cowards and weaklings who, instead of
from the administrative stage to the judicial stage and to facing the consequences of their actions, choose to run
the execution stage depending on factors that will come and hide. Hence, it would not be good policy to increase
into play. In sum, we rule that the temporary hold on the risk of violating our treaty obligations if, through
private respondent's privilege of notice and hearing is a overprotection or excessively liberal treatment, persons
soft restraint on his right to due process which will not sought to be extradited are able to evade arrest or escape
deprive him of fundamental fairness should he decide to from our custody. In the absence of any provision – in
resist the request for his extradition to the United States. the Constitution, the law or the treaty – expressly
There is no denial of due process as long as fundamental guaranteeing the right to bail in extradition proceedings,
fairness is assured a party. (Secretary of Justice v. adopting the practice of not granting them bail, as a
Hon. Ralph C. Lantion, G.R. No. 139465, Oct. 17, general rule, would be a step towards deterring fugitives
2000, En Banc [Puno]) from coming to the Philippines to hide from or evade their
prosecutors.
166. Will Mark Jimenez’s detention prior to the
conclusion of the extradition proceedings not amount The denial of bail as a matter of course in
to a violation of his right to due process? extradition cases falls into place with and gives life to
Article 14 (It states: “If the person sought consents in
Held: Contrary to his contention, his detention writing to surrender to the Requesting State, the
prior to the conclusion of the extradition proceedings does Requested State may surrender the person as
not amount to a violation of his right to due process. We expeditiously as possible without further proceedings.”) of
iterate the familiar doctrine that the essence of due the Treaty, since this practice would encourage the
process is the opportunity to be heard but, at the same accused to voluntarily surrender to the requesting state to
time, point out that the doctrine does not always call for a cut short their detention here. Likewise, their detention
prior opportunity to be heard. Where the circumstances – pending the resolution of extradition proceedings would
such as those present in an extradition case – call for it, a fall into place with the emphasis of the Extradition Law on
subsequent opportunity to be heard is enough. In the the summary nature of extradition cases and the need for
present case, respondent will be given full opportunity to their speedy disposition. (Government of the United
be heard subsequently, when the extradition court hears States of America v. Hon. Guillermo Purganan, G.R.
the Petition for Extradition. Hence, there is no violation of No. 148571, Sept. 24, 2002, En Banc [Panganiban])
his right to due process and fundamental fairness.

Contrary to the contention of Jimenez, we find no The Equal Protection Clause


arbitrariness, either, in the immediate deprivation of his
liberty prior to his being heard. That his arrest and 167. Explain and discuss the equal protection of the
detention will not be arbitrary is sufficiently ensured by law clause.
(1) the DOJ’s filing in court the Petition with its supporting
documents after a determination that the extradition Held: 1. The equal protection of the law is
request meets the requirements of the law and the embraced in the concept of due process, as every unfair
relevant treaty; (2) the extradition judge’s independent discrimination offends the requirements of justice and fair
prima facie determination that his arrest will best serve play. It has nonetheless been embodied in a separate
the ends of justice before the issuance of a warrant for his clause in Article III, Sec. 1, of the Constitution to provide
arrest; and (3) his opportunity, once he is under the for a more specific guaranty against any form of undue
court’s custody, to apply for bail as an exception to the favoritism or hostility from the government. Arbitrariness
no-initial-bail rule. in general may be challenged on the basis of the due
process clause. But if the particular act assailed partakes
It is also worth noting that before the US of an unwarranted partiality or prejudice, the sharper
government requested the extradition of respondent, weapon to cut it down is the equal protection clause.
proceedings had already been conducted in that country.
But because he left the jurisdiction of the requesting state According to a long line of decisions, equal
before those proceedings could be completed, it was protection simply requires that all persons or things
hindered from continuing with the due processes similarly situated should be treated alike, both as to rights
prescribed under its laws. His invocation of due process conferred and responsibilities imposed. Similar subjects,
now had thus become hollow. He already had that in other words, should not be treated differently, so as to
opportunity in the requesting state; yet, instead of taking give undue favor to some and unjustly discriminate
it, he ran away. against others.

75
The equal protection clause does not require the After a careful study, the ineluctable conclusion is
universal application of the laws on all persons or things that the classification under Section 44 of RA 8189
without distinction. This might in fact sometimes result in satisfies the aforestated requirements.
unequal protection, as where, for example, a law
prohibiting mature books to all persons, regardless of age, The singling out of election officers in order to
would benefit the morals of the youth but violate the "ensure the impartiality of election officials by preventing
liberty of adults. What the clause requires is equality them from developing familiarity with the people of their
among equals as determined according to a valid place of assignment" does not violate the equal protection
classification. By classification is meant the grouping of clause of the Constitution.
persons or things similar to each other in certain
particulars and different from all others in these same In Lutz v. Araneta, it was held that "the
particulars. (Philippine Judges Association v. Prado, legislature is not required by the Constitution to adhere to
227 SCRA 703, 711-712, Nov. 11, 1993, En Banc a policy of 'all or none'". This is so for underinclusiveness
[Cruz]) is not an argument against a valid classification. It may
be true that all other officers of COMELEC referred to by
2. The equal protection clause exists to prevent petitioners are exposed to the same evils sought to be
undue favor or privilege. It is intended to eliminate addressed by the statute. However, in this case, it can be
discrimination and oppression based on inequality. discerned that the legislature thought the noble purpose
Recognizing the existence of real difference among men, of the law would be sufficiently served by breaking an
the equal protection clause does not demand absolute important link in the chain of corruption than by breaking
equality. It merely requires that all persons shall be up each and every link thereof. Verily, under Section 3(n)
treated alike, under like circumstances and conditions of RA 8189, election officers are the highest officials or
both as to the privileges conferred and liabilities enforced . authorized representatives of the COMELEC in a city or
Thus, the equal protection clause does not absolutely municipality. It is safe to say that without the complicity
forbid classifications x x x. If the classification is based on of such officials, large-scale anomalies in the registration
real and substantial differences; is germane to the of voters can hardly be carried out. (Agripino A. De
purpose of the law; applies to all members of the same Guzman, Jr., et al. v. COMELEC (G.R. No. 129118,
class; and applies to current as well as future conditions, July 19, 2000, en Banc [Purisima])
the classification may not be impugned as violating the
Constitution's equal protection guarantee. A distinction 169. Are there substantial distinctions between print
based on real and reasonable considerations related to a media and broadcast media to justify the requirement
proper legislative purpose x x x is neither unreasonable, for the latter to give free airtime to be used by the
capricious nor unfounded. (Himagan v. People, 237 Comelec to inform the public of qualifications and
SCRA 538, Oct. 7, 1994, En Banc [Kapunan]) program of government of candidates and political
parties during the campaign period? Discuss.
168. Congress enacted R.A. No. 8189 which provides,
in Section 44 thereof, that "No Election Officer shall Held: There are important differences in the
hold office in a particular city or municipality for more characteristics of the two media which justify their
than four (4) years. Any election officer who, either differential treatment for free speech purposes. Because
at the time of the approval of this Act or subsequent of the physical limitations of the broadcast spectrum, the
thereto, has served for at least four (4) years in a government must, of necessity, allocate broadcast
particular city or municipality shall automatically be frequencies to those wishing to use them. There is no
reassigned by the Commission to a new station similar justification for government allocation and
outside the original congressional district." regulation of the print media.
Petitioners, who are City and Municipal Election
Officers, theorize that Section 44 of RA 8189 is In the allocation of limited resources, relevant
violative of the "equal protection clause" of the 1987 conditions may validly be imposed on the grantees or
Constitution because it singles out the City and licensees. The reason for this is that the government
Municipal Election Officers of the COMELEC as spends public funds for the allocation and regulation of
prohibited from holding office in the same city or the broadcast industry, which it does not do in the case of
municipality for more than four (4) years. They print media. To require radio and television broadcast
maintain that there is no substantial distinction industry to provide free airtime for the Comelec Time is a
between them and other COMELEC officials, and fair exchange for what the industry gets.
therefore, there is no valid classification to justify the
objective of the provision of law under attack. From another point of view, the SC has also held
Resolve. that because of the unique and pervasive influence of the
broadcast media, “[n]ecessarily x x x the freedom of
Held: The Court is not persuaded by petitioners' television and radio broadcasting is somewhat lesser in
arguments. The "equal protection clause" of the 1987 scope than the freedom accorded to newspaper and print
Constitution permits a valid classification under the media.” (TELEBAP, Inc. v. COMELEC, 289 SCRA 337,
following conditions: April 21, 1998 [Mendoza])

1) The classification must rest on substantial 170. Does the death penalty law (R.A. No. 7659)
distinction; violate the equal protection clause considering that, in
2) The classification must be germane to the effect, it punishes only people who are poor,
purpose of the law; uneducated, and jobless?
3) The classification must not be limited to
existing conditions only; and Held: R.A. No. 7659 specifically provides that
4) The classification must apply equally to all “[T]he death penalty shall be imposed if the crime of rape
members of the same class. is committed x x x when the victim is a religious or a child
below seven (7) years old.” Apparently, the death penalty

76
law makes no distinction. It applies to all persons and to (International School, Inc.), its "international character"
all classes of persons – rich or poor, educated or notwithstanding.
uneducated, religious or non-religious. No particular
person or classes of persons are identified by the law The School contends that petitioner has not
against whom the death penalty shall be exclusively adduced evidence that local-hires perform work equal to
imposed. The law punishes with death a person who shall that of foreign-hires. The Court finds this argument a
commit rape against a child below seven years of age. little cavalier. If an employer accords employees the
Thus, the perpetration of rape against a 5-year old girl same position and rank, the presumption is that these
does not absolve or exempt an accused from the employees perform equal work. This presumption is
imposition of the death penalty by the fact that he is poor, borne by logic and human experience. If the employer
uneducated, jobless, and lacks catechetical instruction. pays one employee less than the rest, it is not for that
To hold otherwise will not eliminate but promote employee to explain why he receives less or why the
inequalities. others receive more. That would be adding insult to
injury. The employer has discriminated against that
In Cecilleville Realty and Service Corporation v. employee; it is for the employer to explain why the
CA, the SC clarified that compassion for the poor is an employee is treated unfairly.
imperative of every humane society but only when the
recipient is not a rascal claiming an undeserved privilege. The employer in this case failed to discharge this
(People v. Jimmy Mijano y Tamora, G.R. No. burden. There is no evidence here that foreign-hires
129112, July 23, 1999, En Banc [Per Curiam]) perform 25% more efficiently or effectively than the local-
hires. Both groups have similar functions and
171. The International School Alliance of Educators responsibilities, which they perform under similar working
(ISAE) questioned the point-of-hire classification conditions.
employed by International School, Inc. to justify
distinction in salary rates between foreign-hires and The School cannot invoke the need to entice
local-hires, i.e., salary rates of foreign-hires are higher foreign-hires to leave their domicile to rationalize the
by 25% than their local counterparts, as distinction in salary rates without violating the principle of
discriminatory and, therefore, violates the equal equal work for equal pay.
protection clause. The International School
contended that this is necessary in order to entice Xxx
foreign-hires to leave their domicile and work here.
Resolve. While we recognize the need of the School to
attract foreign-hires, salaries should not be used as an
Held: That public policy abhors inequality and enticement to the prejudice of local-hires. The local-hires
discrimination is beyond contention. Our Constitution and perform the same services as foreign-hires and they ought
laws reflect the policy against these evils. X x x to be paid the same salaries as the latter. For the same
reason, the "dislocation factor" and the foreign-hires'
International law, which springs from general limited tenure also cannot serve as valid bases for the
principles of law, likewise proscribes discrimination x x x. distinction in salary rates. The dislocation factor and
The Universal Declaration of Human Rights, the limited tenure affecting foreign-hires are adequately
International Covenant on Economic, Social and Cultural compensated by certain benefits accorded them which are
Rights, the International Convention on the Elimination of not enjoyed by local-hires, such as housing,
All Forms of Racial Discrimination, the Convention against transportation, shipping costs, taxes and home leave
Discrimination in Education, the Convention (No. 111) travel allowances.
Concerning Discrimination in Respect of Employment and
Occupation - all embody the general principle against The Constitution enjoins the State to “protect the
discrimination, the very antithesis of fairness and justice. rights of workers and promote their welfare”, “to afford
The Philippines, through its Constitution, has incorporated labor full protection.” The State, therefore, has the right
this principle as part of its national laws. and duty to regulate the relations between labor and
capital. These relations are not merely contractual but
[I]t would be an affront to both the spirit and are so impressed with public interest that labor contracts,
letter of these provisions if the State, in spite of its collective bargaining agreements included, must yield to
primordial obligation to promote and ensure equal the common good. Should such contracts contain
employment opportunities, closes its eyes to unequal and stipulations that are contrary to public policy, courts will
discriminatory terms and conditions of employment x x x. not hesitate to strike down these stipulations.

Discrimination, particularly in terms of wages, is In this case, we find the point-of-hire classification
frowned upon by the Labor Code. Article 135, for employed by respondent School to justify the distinction in
example, prohibits and penalizes the payment of lesser the salary rates of foreign-hires and local-hires to be an
compensation to a female employee as against a male invalid classification. There is no reasonable distinction
employee for work of equal value. Article 248 declares it between the services rendered by foreign-hires and local-
an unfair labor practice for an employer to discriminate in hires. The practice of the School of according higher
regards to wages in order to encourage or discourage salaries to foreign-hires contravenes public policy and,
membership in any labor organization. X x x certainly, does not deserve the sympathy of this Court.
(International School Alliance of Educators (ISAE)
The foregoing provisions impregnably v. Hon. Leonardo A. Quisumbing, G.R. No. 128845,
institutionalize in this jurisdiction the long honored legal June 1, 2000, 1st Div. [Kapunan])
truism of “Equal pay for equal work.” Persons who work
with substantially equal qualifications, skill, effort and 172. Accused-appellant Romeo G. Jalosjos filed a
responsibility, under similar conditions, should be paid motion before the Court asking that he be allowed to
similar salaries. This rule applies to the School fully discharge the duties of a Congressman, including

77
attendance at legislative sessions and committee public office gives priority to any other right or interest,
meetings despite his having been convicted in the first including the police power of the State. (People v.
instance of a non-bailable offense. Does being an Jalosjos, 324 SCRA 689, Feb. 3, 2000, En Banc
elective official result in a substantial distinction that [Ynares-Santiago])
allows different treatment? Is being a Congressman a
substantial differentiation which removes the accused- 173. Appellant, who was charged with Illegal
appellant as a prisoner from the same class as all Recruitment in the RTC of Zamboanga City, invokes
persons validly confined under law? the equal protection clause in her defense. She points
out that although the evidence purportedly shows that
Held: In the ultimate analysis, the issue before Jasmine Alejandro handed out application forms and
us boils down to a question of constitutional equal even received Lourdes Modesto’s payment, appellant
protection. was the only one criminally charged. Alejandro, on
the other hand, remained scot-free. From this,
Xxx appellant concludes that the prosecution discriminated
against her on grounds of regional origins. Appellant
The performance of legitimate and even essential is a Cebuana while Alejandro is a Zamboanguena, and
duties by public officers has never been an excuse to free the alleged crime took place in Zamboanga City.
a person validly in prison. The duties imposed by the
“mandate of the people” are multifarious. The accused- Held: The argument has no merit.
appellant asserts that the duty to legislate ranks highest in
the hierarchy of government. The accused-appellant is The prosecution of one guilty while others equally
only one of 250 members of the House of guilty are not prosecuted, however, is not, by itself, a
Representatives, not to mention the 24 members of the denial of the equal protection of the laws. Where the
Senate, charged with the duties of legislation. Congress official action purports to be in conformity to the statutory
continues to function well in the physical absence of one classification, an erroneous or mistaken performance of
or a few of its members. Depending on the exigency of the statutory duty, although a violation of the statute, is
Government that has to be addressed, the President or not without more a denial of the equal protection of the
the Supreme Court can also be deemed the highest for laws. The unlawful administration by officers of a statute
that particular duty. The importance of a function fair on its face, resulting in its unequal application to those
depends on the need for its exercise. The duty of a who are entitled to be treated alike, is not a denial of
mother to nurse her infant is most compelling under the equal protection, unless there is shown to be present in it
law of nature. A doctor with unique skills has the duty to an element of intentional or purposeful discrimination.
save the lives of those with a particular affliction. An This may appear on the face of the action taken with
elective governor has to serve provincial constituents. A respect to a particular class or person, or it may only be
police officer must maintain peace and order. Never had shown by extrinsic evidence showing a discriminatory
the call of a particular duty lifted a prisoner into a design over another not to be inferred from the action
different classification from those others who are validly itself. But a discriminatory purpose is not presumed,
restrained by law. there must be a showing of “clear and intentional
discrimination.” Appellant has failed to show that, in
A strict scrutiny of classifications is essential lest charging appellant in court, that there was a “clear and
wittingly or otherwise, insidious discriminations are made intentional discrimination” on the part of the prosecuting
in favor of or against groups or types of individuals. officials.

The Court cannot validate badges of inequality. The discretion of who to prosecute depends on
The necessities imposed by public welfare may justify the prosecution’s sound assessment whether the evidence
exercise of government authority to regulate even if before it can justify a reasonable belief that a person has
thereby certain groups may plausibly assert that their committed an offense. The presumption is that the
interests are disregarded. prosecuting officers regularly performed their duties, and
this presumption can be overcome only by proof to the
We, therefore, find that election to the position of contrary, not by mere speculation. Indeed, appellant has
Congressman is not a reasonable classification in criminal not presented any evidence to overcome this
law enforcement. The functions and duties of the office presumption. The mere allegation that appellant, a
are not substantial distinctions which lift him from the Cebuana, was charged with the commission of a crime,
class of prisoners interrupted in their freedom and while a Zamboanguena, the guilty party in appellant’s
restricted in liberty of movement. Lawful arrest and eyes, was not, is insufficient to support a conclusion that
confinement are germane to the purposes of the law and the prosecution officers denied appellant equal protection
apply to all those belonging to the same class. of the laws.

Xxx There is also common sense practicality in


sustaining appellant’s prosecution.
It can be seen from the foregoing that
incarceration, by its nature, changes an individual’s status While all persons accused of crime are to
in society. Prison officials have the difficult and often be treated on a basis of equality before the law, it
thankless job of preserving the security in a potentially does not follow that they are to be protected in
explosive setting, as well as of attempting to provide the commission of crime. It would be
rehabilitation that prepare inmates for re-entry into the unconscionable, for instance, to excuse a
social mainstream. Necessarily, both these demands defendant guilty of murder because others have
require the curtailment and elimination of certain rights . murdered with impunity. The remedy for unequal
enforcement of the law in such instances does not
Premises considered, we are constrained to rule lie in the exoneration of the guilty at the expense
against the accused-appellant’s claim that re-election to of society x x x. Protection of the law will be

78
extended to all persons equally in the pursuit of determines whether a warrant of arrest should be
their lawful occupations, but no person has the issued against the accused, i.e., whether there is
right to demand protection of the law in the a necessity for placing him under immediate
commission of a crime (People v. Montgomery, custody in order not to frustrate the ends of
117 P.2d 437 [1941]). justice. Thus, even if both should base their
findings on one and the same proceeding or
Likewise, evidence, there should be no confusion as to their
distinct objectives.
[i]f the failure of prosecutors to enforce
the criminal laws as to some persons should be Second, since their objectives are
converted into a defense for others charged with different, the judge cannot rely solely on the
crime, the result would be that the trial of the report of the prosecutor in finding probable cause
district attorney for nonfeasance would become to justify the issuance of a warrant of arrest.
an issue in the trial of many persons charged with Obviously and understandably, the contents of the
heinous crimes and the enforcement of law would prosecutor’s report will support his own
suffer a complete breakdown (State v. Hicks, 325 conclusion that there is reason to charge the
P.2d 794 [1958]). accused for an offense and hold him for trial.
(People v. Dela Piedra, 350 SCRA 163, Jan. 24, However, the judge must decide independently.
2001, 1st Div. [Kapunan]) Hence, he must have supporting evidence, other
than the prosecutor’s bare report, upon which to
legally sustain his own findings on the existence
The Right against Unreasonable Searches and (or nonexistence) of probable cause to issue an
Seizures arrest order. This responsibility of determining
personally and independently the existence or
174. Discuss the constitutional requirement that a nonexistence of probable cause is lodged in him
judge, in issuing a warrant of arrest, must determine by no less than the most basic law of the land.
probable cause “personally.” Distinguish Parenthetically, the prosecutor could ease the
determination of probable cause by the prosecutor burden of the judge and speed up the litigation
and determination of probable cause by the judge. process by forwarding to the latter not only the
information and his bare resolution finding
Held: It must be stressed that the 1987 probable cause, but also so much of the records
Constitution requires the judge to determine probable and the evidence on hand as to enable the His
cause “personally,” a requirement which does not appear Honor to make his personal and separate judicial
in the corresponding provisions of our previous finding on whether to issue a warrant of arrest.
constitutions. This emphasis evinces the intent of the
framers to place a greater degree of responsibility upon Lastly, it is not required that the complete
trial judges than that imposed under previous or entire records of the case during the
Constitutions. preliminary investigation be submitted to and
examined by the judge. We do not intend to
In Soliven v. Makasiar, this Court pronounced: unduly burden trial courts by obliging them to
examine the complete records of every case all
“What the Constitution underscores is the the time simply for the purpose of ordering the
exclusive and personal responsibility of the issuing arrest of an accused. What is required, rather, is
judge to satisfy himself of the existence of that the judge must have sufficient supporting
probable cause. In satisfying himself of the documents (such as the complaint, affidavits,
existence of probable cause for the issuance of a counter-affidavits, sworn statements of witnesses
warrant of arrest, the judge is not required to or transcript of stenographic notes, if any) upon
personally examine the complainant and his which to make his independent judgment or, at
witnesses. Following established doctrine and the very least, upon which to verify the findings of
procedure, he shall: (1) personally evaluate the the prosecutor as to the existence of probable
report and the supporting documents submitted cause. The point is: he cannot rely solely and
by the fiscal regarding the existence of probable entirely on the prosecutor’s recommendation, as
cause and, on the basis thereof, issue a warrant Respondent Court did in this case. Although the
of arrest; or (2) if in the basis thereof he finds no prosecutor enjoys the legal presumption of
probable cause, he may disregard the fiscal’s regularity in the performance of his official duties
report and require the submission of supporting and functions, which in turn gives his report the
affidavits of witnesses to aid him in arriving at a presumption of accuracy, the Constitution, we
conclusion as to the existence of probable cause.” repeat, commands the judge to personally
determine probable cause in the issuance of
Ho v. People (Ibid.) summarizes existing warrants of arrest. This Court has consistently
jurisprudence on the matter as follows: held that a judge fails in his bounden duty if he
relies merely on the certification or the report of
“Lest we be too repetitive, we only wish the investigating officer.” (Citations omitted)
to emphasize three vital matters once more: First,
as held in Inting, the determination of probable In the case at bench, respondent admits that he
cause by the prosecutor is for a purpose different issued the questioned warrant as there was “no reason for
from that which is to be made by the judge. (him) to doubt the validity of the certification made by the
Whether there is reasonable ground to believe Assistant Prosecutor that a preliminary investigation was
that the accused is guilty of the offense charged conducted and that probable cause was found to exist as
and should be held for trial is what the prosecutor against those charged in the information filed.” The
passes upon. The judge, on the other hand, statement is an admission that respondent relied solely

79
and completely on the certification made by the fiscal that It is neither fair nor licit to allow police officers to
probable cause exists as against those charged in the search a place different from that stated in the warrant on
information and issued the challenged warrant of arrest the claim that the place actually searched – although not
on the sole basis of the prosecutor’s findings and that specified in the warrant – is exactly what they had in
recommendations. He adopted the judgment of the view when they applied for the warrant and had
prosecutor regarding the existence of probable cause as demarcated in their supporting evidence. What is
his own. (Abdula v. Guiani, 326 SCRA 1, Feb. 18, material in determining the validity of a search is the place
2000, 3rd Div. [Gonzaga-Reyes]) stated in the warrant itself, not what applicants had in
their thoughts, or had represented in the proofs they
175. In an application for search warrant, the submitted to the court issuing the warrant. (People v.
application was accompanied by a sketch of the Court of Appeals, 291 SCRA 400, June 26, 1998
compound at 516 San Jose de la Montana St., Mabolo, [Narvasa])
Cebu City, indicating the 2-storey residential house of
private respondent with a large “X” enclosed in a 177. Is a respondent in an Extradition Proceeding
square. Within the same compound are residences of entitled to notice and hearing before the issuance of a
other people, workshops, offices, factories and warrant of arrest?
warehouse. The search warrant issued, however,
merely indicated the address of the compound which Held: Both parties cite Section 6 of PD 1069 in
is 516 San Jose de la Montana St., Mabolo, Cebu City. support of their arguments. X x x
Did this satisfy the constitutional requirement under
Section 2, Article III that the place to be searched Does this provision sanction RTC Judge
must be particularly described? Purganan’s act of immediately setting for hearing the
issuance of a warrant of arrest? We rule in the negative.
Held: This Court has held that the applicant
should particularly describe the place to be searched and 1. On the Basis of the Extradition Law
the person or things to be seized, wherever and whenever
it is feasible. In the present case, it must be noted that It is significant to note that Section 6 of PD 1069,
the application for a search warrant was accompanied by our Extradition Law, uses the word “immediate” to qualify
a sketch of the compound at 516 San Jose de la Montana the arrest of the accused. This qualification would be
St., Mabolo, Cebu City. The sketch indicated the 2-storey rendered nugatory by setting for hearing the issuance of
residential house of private respondent with a large "X" the arrest warrant. Hearing entails sending notices to the
enclosed in a square. Within the same compound are opposing parties, receiving facts and arguments from
residences of other people, workshops, offices, factories them, and giving them time to prepare and present such
and warehouse. With this sketch as the guide, it could facts and arguments. Arrest subsequent to a hearing can
have been very easy to describe the residential house of no longer be considered “immediate.” The law could not
private respondent with sufficient particularity so as to have intended the word as a mere superfluity but, on the
segregate it from the other buildings or structures inside whole, as a means of impairing a sense of urgency and
the same compound. But the search warrant merely swiftness in the determination of whether a warrant of
indicated the address of the compound which is 516 San arrest should be issued.
Jose de la Montana St., Mabolo, Cebu City. This
description of the place to be searched is too general and By using the phrase “if it appears,” the law further
does not pinpoint the specific house of private conveys that accuracy is not as important as speed at
respondent. Thus, the inadequacy of the description of such early stage. The trial court is not expected to make
the residence of private respondent sought to be searched an exhaustive determination to ferret out the true and
has characterized the questioned search warrant as a actual situation, immediately upon the filing of the
general warrant, which is violative of the constitutional petition. From the knowledge and the material then
requirement. (People v. Estrada, 296 SCRA 383, available to it, the court is expected merely to get a good
400, [Martinez]) first impression – a prima facie finding – sufficient to
make a speedy initial determination as regards the arrest
176. Can the place to be searched, as set out in the and detention of the accused.
warrant, be amplified or modified by the officers’ own
personal knowledge of the premises, or the evidence Xxx
they adduce in support of their application for the
warrant? We stress that the prima facie existence of
probable cause for hearing the petition and, a priori, for
Held: Such a change is proscribed by the issuing an arrest warrant was already evident from the
Constitution which requires inter alia the search warrant Petition itself and its supporting documents. Hence, after
to particularly describe the place to be searched as well as having already determined therefrom that a prima facie
the persons or things to be seized. It would concede to finding did exist, respondent judge gravely abused his
police officers the power of choosing the place to be discretion when he set the matter for hearing upon
searched, even if it not be that delineated in the warrant. motion of Jimenez.
It would open wide the door to abuse of the search
process, and grant to officers executing a search warrant Moreover, the law specifies that the court sets a
that discretion which the Constitution has precisely hearing upon receipt of the answer or upon failure of the
removed from them. The particularization of the accused to answer after receiving the summons. In
description of the place to be searched may properly be connection with the matter of immediate arrest, however,
done only by the Judge, and only in the warrant itself; it the word “hearing” is notably absent from the provision.
cannot be left to the discretion of the police officers Evidently, had the holding of a hearing at that stage been
conducting the search. intended, the law could have easily so provided. It also
bears emphasizing at this point that extradition
proceedings are summary (See Sec. 9, PD 1069) in

80
nature. Hence, the silence of the Law and the Treaty possession of a weapon or evidence he can destroy, a
leans to the more reasonable interpretation that there is valid arrest must precede the search. The process cannot
no intention to punctuate with a hearing every little step be reversed.
in the entire proceedings.
In a search incidental to a lawful arrest,
Xxx as the precedent arrest determines the validity of
the incidental search, the legality of the arrest is
Verily x x x sending to persons sought to be questioned in a large majority of these cases,
extradited a notice of the request for their arrest and e.g., whether an arrest was merely used as a
setting it for hearing at some future date would give them pretext for conducting a search. In this instance,
ample opportunity to prepare and execute an escape. the law requires that there be first a lawful arrest
Neither the Treaty nor the Law could have intended that before a search can be made – the process
consequence, for the very purpose of both would have cannot be reversed. (Malacat v. Court of Appeals,
been defeated by the escape of the accused from the 283 SCRA 159, 175 [1997])
requested state. (People v. Chua Ho San, 308 SCRA 432, June 17,
1999, En Banc [Davide, Jr., C.J.])
2. On the Basis of the Constitution
179. What is the “plain view” doctrine? What are its
Even Section 2 of Article III of our Constitution x requisites? Discuss.
x x does not require a notice or a hearing before the
issuance of a warrant of arrest. X x x Held: 1. Objects falling in plain view of an
officer who has a right to be in the position to have that
To determine probable cause for the issuance of view are subject to seizure even without a search warrant
arrest warrants, the Constitution itself requires only the and may be introduced in evidence. The “plain view”
examination – under oath or affirmation – of complainants doctrine applies when the following requisites concur: (a)
and the witnesses they may produce. There is no the law enforcement officer in search of the evidence has
requirement to notify and hear the accused before the a prior justification for an intrusion or is in a position from
issuance of warrants of arrest. which he can view a particular area; (b) the discovery of
the evidence in plain view is inadvertent; (c) it is
In Ho v. People and in all the cases cited therein, immediately apparent to the officer that the item he
never was a judge required to go to the extent of observes may be evidence of a crime, contraband or
conducting a hearing just for the purpose of personally otherwise subject to seizure. The law enforcement officer
determining probable cause for the issuance of a warrant must lawfully make an initial intrusion or properly be in a
of arrest. All we required was that the “judge must have position from which he can particularly view the area . In
sufficient supporting documents upon which to make his the course of such lawful intrusion, he came inadvertently
independent judgment, or at the very least, upon which to across a piece of evidence incriminating the accused . The
verify the findings of the prosecutor as to the existence of object must be open to eye and hand and its discovery
probable cause.” inadvertent.

In Webb v. De Leon, the Court categorically It is clear that an object is in plain view if the
stated that a judge was not supposed to conduct a object itself is plainly exposed to sight. The difficulty
hearing before issuing a warrant of arrest x x x. arises when the object is inside a closed container.
Where the object seized was inside a closed package, the
At most, in cases of clear insufficiency of evidence object itself is not in plain view and therefore cannot be
on record, judges merely further examine complainants seized without a warrant. However, if the package
and their witnesses. In the present case, validating the proclaims its contents, whether by its distinctive
act of respondent judge and instituting the practice of configuration, its transparency, or if its contents are
hearing the accused and his witnesses at this early stage obvious to an observer, then the contents are in plain
would be discordant with the rationale for the entire view and may be seized. In other words, if the package is
system. If the accused were allowed to be heard and such that an experienced observer could infer from its
necessarily to present evidence during the prima facie appearance that it contains the prohibited article, then the
determination for the issuance of a warrant of arrest, article is deemed in plain view. It must be immediately
what would stop him from presenting his entire plethora apparent to the police that the items that they observe
of defenses at this stage – if he so desires – in his effort may be evidence of a crime, contraband or otherwise
to negate a prima facie finding? Such a procedure could subject to seizure. (People v. Doria, 301 SCRA 668,
convert the determination of a prima facie case into a full- Jan. 22, 1999, En Banc [Puno, J.])
blown trial of the entire proceedings and possibly make
trial of the main case superfluous. This scenario is also 2. For the doctrine to apply, the following
anathema to the summary nature of extraditions. elements must be present:
(Government of the United States of America v.
Hon. Guillermo Purganan, G.R. No. 148571, Sept. a) a prior valid intrusion based on the valid
24, 2002, En Banc [Panganiban]) warrantless arrest in which the police are
legally present in the pursuit of their official
178. What is “search incidental to a lawful arrest”? duties;
Discuss. b) the evidence was inadvertently discovered by
the police who have the right to be where
Held: While a contemporaneous search of a they are; and
person arrested may be effected to discover dangerous c) the evidence must be immediately apparent;
weapons or proofs or implements used in the commission and
of the crime and which search may extend to the area d) plain view justified mere seizure of evidence
within his immediate control where he might gain without further search.

81
In the instant case, recall that PO2 Balut testified 180. What is a “stop-and-frisk” search?
that they first located the marijuana plants before
appellant was arrested without a warrant. Hence, there Held: 1. In the landmark case of Terry v. Ohio,
was no valid warrantless arrest which preceded the search a stop-and-frisk was defined as the vernacular designation
of appellant’s premises. Note further that the police team of the right of a police officer to stop a citizen on the
was dispatched to appellant’s kaingin precisely to search street, interrogate him, and pat him for weapon(s):
for and uproot the prohibited flora. The seizure of
evidence in “plain view” applies only where the police “x x x (W)here a police officer observes
officer is not searching for evidence against the accused, an unusual conduct which leads him reasonably to
but inadvertently comes across an incriminating object . conclude in light of his experience that criminal
Clearly, their discovery of the cannabis plants was not activity may be afoot and that the persons with
inadvertent. We also note the testimony of SPO2 Tipay whom he is dealing may be armed and presently
that upon arriving at the area, they first had to “look dangerous, where in the course of investigating
around the area” before they could spot the illegal plants. this behavior he identified himself as a policeman
Patently, the seized marijuana plants were not and make reasonable inquiries, and where
“immediately apparent” and “further search” was needed. nothing in the initial stages of the encounter
In sum, the marijuana plants in question were not in serves to dispel his reasonable fear for his own or
“plain view” or “open to eye and hand.” The “plain view” others’ safety, he is entitled for the protection of
doctrine, thus, cannot be made to apply. himself or others in the area to conduct a
carefully limited search of the outer clothing of
Nor can we sustain the trial court’s conclusion such persons in an attempt to discover weapons
that just because the marijuana plants were found in an which might be used to assault him. Such a
unfenced lot, appellant could not invoke the protection search is a reasonable search under the Fourth
afforded by the Charter against unreasonable searches by Amendment, and any weapon seized may
agents of the State. The right against unreasonable properly be introduced in evidence against the
searches and seizures is the immunity of one’s person, person from whom they were taken.” (Herrera, A
which includes his residence, his papers, and other Handbook on Arrest, Search and Seizure and
possessions. The guarantee refers to “the right of Custodial Investigation, 1995 ed., p. 185; and
personal security” of the individual. X x x, what is sought Terry v. Ohio, supra, p. 911)
to be protected against the State’s unlawful intrusion are
persons, not places. To conclude otherwise would not In allowing such a search, the United States
only mean swimming against the stream, it would also Supreme Court held that the interest of effective crime
lead to the absurd logic that for a person to be immune prevention and detection allows a police officer to
against unreasonable searches and seizures, he must be approach a person, in appropriate circumstances and
in his home or office, within a fenced yard or a private manner, for purposes of investigating possible criminal
place. The Bill of Rights belongs as much to the person in behavior even though there is insufficient probable cause
the street as to the individual in the sanctuary of his to make an actual arrest.
bedroom. (People v. Abe Valdez, G.R. No. 129296,
Sept. 25, 2000, En Banc [Quisumbing]) In admitting in evidence two guns seized during
the stop-and-frisk, the US Supreme Court held that what
3. Considering its factual milieu, this case falls justified the limited search was the more immediate
squarely under the plain view doctrine. interest of the police officer in taking steps to assure
himself that the person with whom he was dealing was
When Spencer wrenched himself free from the not armed with a weapon that could unexpectedly and
grasp of PO2 Gaviola, he instinctively ran towards the fatally be used against him.
house of appellant. The members of the buy-bust team
were justified in running after him and entering the house It did not, however, abandon the rule that the
without a search warrant for they were hot in the heels of police must, whenever practicable, obtain advance judicial
a fleeing criminal. Once inside the house, the police approval of searches and seizures through the warrant
officers cornered Spencer and recovered the buy-bust procedure, excused only by exigent circumstances.
money from him. They also caught appellant in flagrante (Manalili v. CA, 280 SCRA 400, Oct. 9, 1997
delicto repacking the marijuana bricks which were in full [Panganiban])
view on top of a table. x x x.
2. We now proceed to the justification for and
Hence, appellant’s subsequent arrest was likewise allowable scope of a “stop-and-frisk” as a “limited
lawful, coming as it is within the purview of Section 5(a) protective search of outer clothing for weapons,” as laid
of Rule 113 of the 1985 Rules on Criminal Procedure x x down in Terry, thus:
x.
We merely hold today that where a police
Section 5(a) is commonly referred to as the rule officer observes unusual conduct which leads him
on in flagrante delicto arrests. Here two elements must reasonably to conclude in light of his experience
concur: (1) the person to be arrested must execute an that criminal activity may be afoot and that the
overt act indicating that he has just committed, is actually persons with whom he is dealing may be armed
committing, or is attempting to commit a crime; and (2) and presently dangerous, where in the course of
such overt act is done in the presence or within the view investigating this behavior he identifies himself as
of the arresting officer. Thus, when appellant was seen a policeman and makes reasonable inquiries, and
repacking the marijuana, the police officers were not only where nothing in the initial stages of the
authorized but also duty-bound to arrest him even without encounter serves to dispel his reasonable fear for
a warrant. (People v. Elamparo, 329 SCRA 404, 414- his own or others’ safety, he is entitled for the
415, March 31, 2000, 2nd Div. [Quisumbing]) protection of himself and others in the area to

82
conduct a carefully limited search of the outer We take judicial notice of the existence of the
clothing of such persons in an attempt to discover COMELEC resolution imposing a gun ban during the
weapons which might be used to assault him. election period issued pursuant to Section 52(c) in relation
Such a search is a reasonable search under the to Section 26(q) of the Omnibus Election Code (Batas
Fourth Amendment (Terry, at 911. In fact, the Pambansa Blg. 881). The national and local elections in
Court noted that the ‘sole justification’ for a stop- 1995 were held on 8 May, the second Monday of the
and-frisk was the ‘protection of the police officer month. The incident, which happened on 5 April 1995,
and others nearby’; while the scope of the search was well within the election period.
conducted in the case was limited to patting down
the outer clothing of petitioner and his Xxx
companions, the police officer did not place his
hands in their pockets nor under the outer surface The checkpoint herein conducted was in
of their garments until he had felt weapons, and pursuance of the gun ban enforced by the COMELEC. The
then he merely reached for and removed the COMELEC would be hard put to implement the ban if its
guns. This did not constitute a general deputized agents were limited to a visual search of
exploratory search, Id.) pedestrians. It would also defeat the purpose for which
such ban was instituted. Those who intend to bring a gun
Other notable points of Terry are that while during said period would know that they only need a car
probable cause is not required to conduct a “stop-and- to be able to easily perpetrate their malicious designs.
frisk,” it nevertheless holds that mere suspicion or a
hunch will not validate a “stop-and-frisk.” A genuine The facts adduced do not constitute a ground for
reason must exist, in light of the police officer’s a violation of the constitutional rights of the accused
experience and surrounding conditions, to warrant the against illegal search and seizure. PO3 Suba admitted
belief that the person detained has weapons concealed that they were merely stopping cars they deemed
about him. Finally, a “stop-and-frisk” serves a two-fold suspicious, such as those whose windows are heavily
interest: (1) the general interest of effective crime tinted just to see if the passengers thereof were carrying
prevention and detection, which underlies the recognition guns. At best they would merely direct their flashlights
that a police officer may, under appropriate circumstances inside the cars they would stop, without opening the car’s
and in an appropriate manner, approach a person for doors or subjecting its passengers to a body search.
purposes of investigating possible criminal behavior even There is nothing discriminatory in this as this is what the
without probable cause; and (2) the more pressing situation demands.
interest of safety and self-preservation which permit the
police officer to take steps to assure himself that the We see no need for checkpoints to be announced
person with whom he deals is not armed with a deadly x x x. Not only would it be impractical, it would also
weapon that could unexpectedly and fatally be used forewarn those who intend to violate the ban. Even so,
against the police officer. (Malacat v. Court of badges of legitimacy of checkpoints may still be inferred
Appeals, 283 SCRA 159, Dec. 12, 1997 [Davide]) from their fixed location and the regularized manner in
which they are operated. (People v. Usana, 323 SCRA
181. Are searches at checkpoints valid? Discuss. 754, Jan. 28, 2000, 1st Div. [Davide, CJ])

Held: This Court has ruled that not all 183. Do the ordinary rights against unreasonable
checkpoints are illegal. Those which are warranted by the searches and seizures apply to searches conducted at
exigencies of public order and are conducted in a way the airport pursuant to routine airport security
least intrusive to motorists are allowed. For, admittedly, procedures?
routine checkpoints do intrude, to a certain extent, on
motorists’ right to “free passage without interruption,” but Held: Persons may lose the protection of the
it cannot be denied that, as a rule, it involves only a brief search and seizure clause by exposure of their persons or
detention of travelers during which the vehicle’s occupants property to the public in a manner reflecting a lack of
are required to answer a brief question or two. For as subjective expectation of privacy, which expectation
long as the vehicle is neither searched nor its occupants society is prepared to recognize as reasonable. Such
subjected to a body search, and the inspection of the recognition is implicit in airport security procedures. With
vehicle is limited to a visual search, said routine checks increased concern over airplane hijacking and terrorism
cannot be regarded as violative of an individual’s right has come increased security at the nation’s airports.
against unreasonable search. In fact, these routine Passengers attempting to board an aircraft routinely pass
checks, when conducted in a fixed area, are even less through metal detectors; their carry-on baggage as well
intrusive. (People v. Usana, 323 SCRA 754, Jan. 28, as checked luggage are routinely subjected to x-ray scans.
2000, 1st Div. [Davide, CJ]) Should these procedures suggest the presence of
suspicious objects, physical searches are conducted to
182. Illustrative case of valid search at checkpoint determine what the objects are. There is little question
conducted to implement the gun ban law during that such searches are reasonable, given their minimal
election. Is it still necessary that checkpoints be pre- intrusiveness, the gravity of the safety interests involved,
announced? and the reduced privacy expectations associated with
airline travel. Indeed, travelers are often notified through
Held: Accused-appellants assail the manner by airport public address systems, signs, and notices in their
which the checkpoint in question was conducted. They airline tickets that they are subject to search and, if any
contend that the checkpoint manned by elements of the prohibited materials or substances are found, such would
Makati Police should have been announced. They also be subject to seizure. These announcements place
complain of its having been conducted in an arbitrary and passengers on notice that ordinary constitutional
discriminatory manner. protections against warrantless searches and seizures do
not apply to routine airport procedures. (People v. Leila

83
Johnson, G.R. No. 138881, Dec. 18, 2000, 2nd Div. sanctions can medicines and drugs circulate the
[Mendoza]) market. We cannot afford to take any risk, for
the life and health of the citizenry are as precious
184. May the constitutional protection against as the existence of the State.”
unreasonable searches and seizures be extended to
acts committed by private individuals? 186. Do Regional Trial Courts have competence to pass
upon the validity or regularity of seizure and forfeiture
Held: As held in People v. Marti, the proceedings conducted by the Bureau of Customs and
constitutional protection against unreasonable searches to enjoin or otherwise interfere with these
and seizures refers to the immunity of one's person from proceedings?
interference by government and it cannot be extended to
acts committed by private individuals so as to bring it Held: In Jao v. Court of Appeals, this Court,
within the ambit of alleged unlawful intrusion. (People v. reiterating its rulings x x x said:
Mendoza, 301 SCRA 66, Jan. 18, 1999, 1st Div.
[Melo]) There is no question that Regional Trial
Courts are devoid of any competence to pass
185. Should the seized drugs (which are upon the validity or regularity of seizure and
pharmaceutically correct but not properly forfeiture proceedings conducted by the Bureau of
documented) subject of an illegal search (because the Customs and to enjoin or otherwise interfere with
applicant “failed to allege in the application for search these proceedings. The Collector of Customs
warrant that the subject drugs for which she was sitting in seizure and forfeiture proceedings has
applying for search warrant were either fake, exclusive jurisdiction to hear and determine all
misbranded, adulterated, or unregistered,”) be questions touching on the seizure and forfeiture
returned to the owner? of dutiable goods. The Regional Trial Courts are
precluded from assuming cognizance over such
Ans.: No. Instead, they should be promptly matters even through petitions of certiorari,
disposed of in the manner provided by law in order to prohibition or mandamus.
insure that the same do not fall into the wrong hands who
might use the drugs underground. As clarified in People It is likewise well-settled that the
v. Judge Estrella T. Estrada (G.R No. 124461, June provisions of the Tariff and Customs Code and
26, 2000, Spcl. 2nd Div. [Ynares-Santiago]): that of Republic Act No. 1125, as amended,
otherwise known as “An Act Creating the Court of
“With the State's obligation to protect and Tax Appeals,” specify the proper fora and
promote the right to health of the people and procedure for the ventilation of any legal
instill health consciousness among them (Article objections or issues raised concerning these
II, Section 15, 1987 Constitution), in order to proceedings. Thus, actions of the Collector of
develop a healthy and alert citizenry (Article XIV, Customs are appealable to the Commissioner of
Section 19[1]), it became mandatory for the Customs, whose decision, in turn, is subject to the
government to supervise and control the exclusive appellate jurisdiction of the Court of Tax
proliferation of drugs in the market. The Appeals and from there to the Court of Appeals.
constitutional mandate that "the State shall adopt
an integrated and comprehensive approach to The rule that Regional Trial Courts have
health development which shall endeavor to make no review powers over such proceedings is
essential goods, health and other social services anchored upon the policy of placing no
available to all people at affordable cost" (Article unnecessary hindrance on the government’s drive,
XIII, Section 11) cannot be neglected. This is not only to prevent smuggling and other frauds
why "the State shall establish and maintain an upon Customs, but more importantly, to render
effective food and drug regulatory system." effective and efficient the collection of import and
(Article XIII, Section 12) The BFAD is the export duties due the State, which enables the
government agency vested by law to make a government to carry out the functions it has been
mandatory and authoritative determination of the instituted to perform.
true therapeutic effect of drugs because it
involves technical skill which is within its special Even if the seizure by the Collector of
competence. The health of the citizenry should Customs were illegal, x x x we have said that such
never be compromised. To the layman, medicine act does not deprive the Bureau of Customs of
is a cure that may lead to better health. jurisdiction thereon.
(Bureau of Customs v. Ogario, 329 SCRA 289, 296-
If the seized 52 boxes of drugs are 298, March 30, 2000, 2nd Div. [Mendoza])
pharmaceutically correct but not properly
documented, they should be promptly disposed of 187. Accused-appellant assails the validity of his arrest
in the manner provided by law in order to ensure and his subsequent convictions for the two crimes.
that the same do not fall into the wrong hands Both the trial court and the Court of Appeals found
who might use the drugs underground. X x x. that the arrest and subsequent seizure were legal.
The policy of the law enunciated in R.A. No. 8203
is to protect the consumers as well as the licensed Held: In the cases at bar, the police saw the gun
businessmen. Foremost among these consumers tucked in appellant’s waist when he stood up. The gun
is the government itself which procures medicines was plainly visible. No search was conducted as none was
and distributes them to the local communities necessary. Accused-appellant could not show any license
through direct assistance to the local health for the firearm, whether at the time of his arrest or
centers or through outreach and charity thereafter. Thus, he was in effect committing a crime in
programs. Only with the proper government the presence of the police officers. No warrant of arrest

84
was necessary in such a situation, it being one of the disembarking from the vessel. It was only when the
recognized exceptions under the Rules. informer pointed to him as the carrier of the marijuana
that he suddenly became suspect and so subject to
As a consequence of appellant’s valid warrantless apprehension.”
arrest, he may be lawfully searched for dangerous
weapons or anything which may be used as proof of the Likewise, in People v. Mengote, the Court did not
commission of an offense, without a search warrant, as consider “eyes x x x darting from side to side x x x [while]
provided in Rule 126, Section 12. This is a valid search holding x x x [one’s] abdomen,” in a crowded street at
incidental to a lawful arrest. The subsequent discovery in 11:30 in the morning, as overt acts and circumstances
his car of drug paraphernalia and the crystalline sufficient to arouse suspicion and indicative of probable
substance, which, was later identified as shabu, though in cause. According to the Court, “[b]y no stretch of the
a distant place from where the illegal possession of imagination could it have been inferred from these acts
firearm was committed, cannot be said to have been that an offense had just been committed, or was actually
made during an illegal search. As such, the seized items being committed, or was at least being attempted in [the
do not fall within the exclusionary clause x x x. Hence, arresting officers’] presence.” So also, in People v.
not being fruits of the poisonous tree x x x the objects Encinada, the Court ruled that no probable cause is
found at the scene of the crime, such as the firearm, the gleanable from the act of riding a motorela while holding
shabu and the drug paraphernalia, can be used as two plastic baby chairs.
evidence against appellant. Besides, it has been held that
drugs discovered as a result of a consented search is Then, too, in Malacat v. Court of Appeals, the trial
admissible in evidence. (Citations omitted.) (People v. court concluded that petitioner was attempting to commit
Go, 354 SCRA 338, Mar. 14, 2001, 1st Div. [Ynares- a crime as he was “’standing at the corner of Plaza
Santiago]) Miranda and Quezon Boulevard’ with his eyes ‘moving
very fast’ and ‘looking at every person that come (sic)
188. Discuss the nature of an “in flagrante delicto” nearer (sic) to them.’” In declaring the warrantless arrest
warrantless arrest. Illustrative case. therein illegal, the Court said:

Held: In the case at bar, the court a quo Here, there could have been no valid in
anchored its judgment of conviction on a finding that the flagrante delicto … arrest preceding the search in
warrantless arrest of accused-appellants, and the light of the lack of personal knowledge on the
subsequent search conducted by the peace officers, are part of Yu, the arresting officer, or an overt
valid because accused-appellants were caught in flagrante physical act, on the part of petitioner, indicating
delicto in possession of prohibited drugs. This brings us that a crime had just been committed, was being
to the issue of whether or not the warrantless arrest, committed or was going to be committed.
search and seizure in the present case fall within the
recognized exceptions to the warrant requirement. It went on to state that –

In People v. Chua Ho San, the Court held that in Second, there was nothing in petitioner’s
cases of in flagrante delicto arrests, a peace officer or a behavior or conduct which could have reasonably
private person may, without a warrant, arrest a person elicited even mere suspicion other than that his
when, in his presence, the person to be arrested has eyes were “moving very fast” – an observation
committed, is actually committing, or is attempting to which leaves us incredulous since Yu and his
commit an offense. The arresting office, therefore, must teammates were nowhere near petitioner and it
have personal knowledge of such fact or, as a recent case was already 6:60 p.m., thus presumably dusk.
law adverts to, personal knowledge of facts or Petitioner and his companions were merely
circumstances convincingly indicative or constitutive of standing at the corner and were not creating any
probable cause. As discussed in People v. Doria, probable commotion or trouble x x x.
cause means an actual belief or reasonable grounds of
suspicion. The grounds of suspicion are reasonable when, Third, there was at all no ground,
in the absence of actual belief of the arresting officers, the probable or otherwise, to believe that petitioner
suspicion that the person to be arrested is probably guilty was armed with a deadly weapon. None was
of committing the offense, is based on actual facts, i.e., visible to Yu, for as he admitted, the alleged
supported by circumstances sufficiently strong in grenade was “discovered” “inside the front
themselves to create the probable cause of guilt of the waistline” of petitioner, and from all indications as
person to be arrested. A reasonable suspicion therefore to the distance between Yu and petitioner, any
must be founded on probable cause, coupled with good telltale bulge, assuming that petitioner was indeed
faith on the part of the peace officers making the arrest. hiding a grenade, could not have been visible to
Yu.
As applied to in flagrante delicto arrests, it is
settled that “reliable information” alone, absent any overt Clearly, to constitute a valid in flagrante delicto
act indicative of a felonious enterprise in the presence and arrest, two requisites must concur: (1) the person to be
within the view of the arresting officers, are not sufficient arrested must execute an overt act indicating that he has
to constitute probable cause that would justify an in just committed, is actually committing, or is attempting to
flagrante delicto arrest. Thus, in People v. Aminnudin, it commit a crime; and (2) such overt act is done in the
was held that “the accused-appellant was not, at the presence or within the view of the arresting officer.
moment of his arrest, committing a crime nor was it
shown that he was about to do so or that he had just In the case at bar, accused-appellants manifested
done so. What he was doing was descending the no outward indication that would justify their arrest. In
gangplank of the M/V Wilcon 9 and there was no outward holding a bag on board a trisikad, accused-appellants
indication that called for his arrest. To all appearances, could not be said to be committing, attempting to commit
he was like any of the other passengers innocently or have committed a crime. It matters not that accused-

85
appellant Molina responded “Boss, if possible we will settle was likewise illegal. Consequently, the marijuana seized
this” to the request of SPO1 Pamplona to open the bag. by the peace officers could not be admitted as evidence
Such response which allegedly reinforced the “suspicion” against accused-appellants, and the Court is thus, left
of the arresting officers that accused-appellants were with no choice but to find in favor of accused-appellants.
committing a crime, is an equivocal statement which (People v. Molina, 352 SCRA 174, Feb. 19, 2001, En
standing alone will not constitute probable cause to effect Banc [Ynares-Santiago])
an in flagrante delicto arrest. Note that were it not for
SPO1 Marino Paguidopon (who did not participate in the
arrest but merely pointed accused-appellants to the The Right to Privacy of Communications and
arresting officers), accused-appellants could not be Correspondence
subject of any suspicion, reasonable or otherwise.
189. What is the Anti-Wire Tapping Act (R.A. 4200)?
While SPO1 Paguidopon claimed that he and his
informer conducted a surveillance of accused-appellant Ans.: R.A. 4200 is a special law prohibiting and
Mula, SPO1 Paguidopon, however, admitted that he only penalizing wire-tapping. Section 1 thereof provides:
learned Mula’s name and address after the arrest. What
is more, it is doubtful if SPO1 Paguidopon indeed Sec. 1. It shall be unlawful for any
recognized accused-appellant Mula. It is worthy to note person, not being authorized by all the parties to
that, before the arrest, he was able to see Mula in person any private communication or spoken word, to tap
only once, pinpointed to him by his informer while they any wire or cable, or by using any other device or
were on the side of the road. These circumstances could arrangement, to secretly overhear, intercept, or
not have afforded SPO1 Paguidopon a closer look at record such communication or spoken word by
accused-appellant Mula, considering that the latter was using a device commonly known as a dictaphone
then driving a motorcycle when SPO1 Paguidopon caught or dictagraph or detectaphone or walkie-talkie or
a glimpse of him. With respect to accused-appellant tape-recorder, or however otherwise described:
Molina, SPO1 Paguidopon admitted that he had never
seen him before the arrest. It shall also be unlawful for any person,
be he a participant or not in the act or acts
This belies the claim of SPO1 Pamplona that he penalized in the next preceding sentence, to
knew the name of accused-appellants even before the knowingly possess any tape record, wire record,
arrest x x x. disc record, or any other such record, or copies
thereof, of any communication or spoken word
The aforesaid testimony of SPO1 Pamplona, secured either before or after the effective date of
therefore, is entirely baseless. SPO1 Pamplona could not this Act in the manner prohibited by this law; or
have learned the name of accused-appellants from SPO1 to replay the same for any other person or
Paguidopon because Paguidopon himself, who allegedly persons; or to communicate the contents thereof,
conducted the surveillance, was not even aware of either verbally or in writing, or to furnish
accused-appellants’ name and address prior to the arrest. transcriptions thereof, whether complete or
partial, to any other person: Provided, That the
Evidently, SPO1 Paguidopon, who acted as use of such record or any copies thereof as
informer of the arresting officers, more so the arresting evidence in any civil, criminal investigation or trial
officers themselves, could not have been certain of of offenses mentioned in section 3 hereof, shall
accused-appellants’ identity, and were, from all not be covered by this prohibition.
indications, merely fishing for evidence at the time of the
arrest. Section 4 thereof also provides:

Compared to People v. Encinada, the arresting Sec. 4. Any communication or spoken


officer in the said case knew appellant Encinada even word, or the existence, contents, substance,
before the arrest because of the latter’s illegal gambling purport, effect, or meaning of the same or any
activities, thus, lending at least a semblance of validity on part thereof, or any information therein contained
the arrest effected by the peace officers. Nevertheless, obtained or secured by any person in violation of
the Court declared in said case that the warrantless arrest the preceding sections of this Act shall not be
and the consequent search were illegal, holding that admissible in evidence in any judicial, quasi-
“[t]he prosecution’s evidence did not show any suspicious judicial, legislative or administrative hearing or
behavior when the appellant disembarked from the ship or investigation.
while he rode the motorela. No act or fact demonstrating
a felonious enterprise could be ascribed to appellant 190. Private respondent Rafael S. Ortanez filed with
under such bare circumstances.” (People v. Encinada, the Regional Trial Court of Quezon City a complaint
supra.) for annulment of marriage with damages against his
wife, herein petitioner, Teresita Salcedo-Ortanez, on
Moreover, it could not be said that accused- grounds of lack of marriage license and/or
appellants waived their right against unreasonable psychological incapacity of the petitioner. Among the
searches and seizure. Implied acquiescence to the exhibits offered by private respondent were three (3)
search, if there was any, could not have been more than cassette tapes of alleged telephone conversations
mere passive conformity given under intimidating or between petitioner and unidentified persons. The trial
coercive circumstances and is thus considered no consent court issued the assailed order admitting all of the
at all within the purview of the constitutional guarantee. evidence offered by private respondent, including tape
recordings of telephone conversations of petitioner
Withal, the Court holds that the arrest of accused- with unidentified persons. These tape recordings
appellants does not fall under the exceptions allowed by were made and obtained when private respondent
the rules. Hence, the search conducted on their person allowed his friends from the military to wire tap his

86
home telephone. Did the trial court act properly when Indeed, if we extend our judicial gaze we will find
it admitted in evidence said tape recordings? that the right of privacy is recognized and enshrined in
several provisions of our Constitution. (Morfe v. Mutuc,
Held: Republic Act No. 4200 entitled “An Act to 22 SCRA 424, 444 [1968]; Cortes, The Constitutional
Prohibit and Penalize Wire Tapping and Other Related Foundations of Privacy, p. 18 [1970]). It is expressly
Violations of the Privacy of Communication, and For Other recognized in Section 3(1) of the Bill of Rights x x x.
Purposes” expressly makes such tape recordings Other facets of the right to privacy are protected in
inadmissible in evidence. x x x. various provisions of the Bill of Rights (viz: Secs. 1, 2, 6,
8, and 17. (Ople v. Torres, G.R. No. 127685, July 23,
Clearly, respondent trial court and Court of 1998 [Puno])
Appeals failed to consider the afore-quoted provisions of
the law in admitting in evidence the cassette tapes in 193. What are the zones of privacy recognized and
question. Absent a clear showing that both parties to the protected in our laws?
telephone conversations allowed the recording of the
same, the inadmissibility of the subject tapes is mandatory Held: The Civil Code provides that “[e]very
under Rep. Act No. 4200. person shall respect the dignity, personality, privacy and
peace of mind of his neighbors and other persons” and
Additionally, it should be mentioned that the punishes as actionable torts several acts by a person of
above-mentioned Republic Act in Section 2 thereof meddling and prying into the privacy of another. It also
imposes a penalty of imprisonment of not less than six (6) holds a public officer or employee or any private individual
months and up to six (6) years for violation of said Act. liable for damages for any violation of the rights and
(Salcedo-Ortanez v. Court of Appeals, 235 SCRA liberties of another person, and recognizes the privacy of
111, Aug. 4, 1994 [Padilla]) letters and other private communications. The Revised
Penal Code makes a crime the violation of secrets by an
191. Two local media men in Lucena City went to the officer, the revelation of trade and industrial secrets, and
police station to report alleged indecent show in one trespass to dwelling. Invasion of privacy is an offense in
night establishment in the City. At the station, there special laws like the Anti-Wiretapping Law (R.A. 4200),
was a heated argument between police officer the Secrecy of Bank Deposits (R.A. 1405) and the
Navarro and Lingan, one of the two media men, Intellectual Property Code (R.A. 8293). The Rules of
which led to fisticuffs. Lingan fell and his head hit the Court on privileged communication likewise recognize the
pavement which caused his death. During the trial, privacy of certain information (Section 24, Rule 130[c],
Jalbuena, the other media man, testified. Presented Revised Rules on Evidence). (Ople v. Torres, G.R. No.
in evidence to confirm his testimony was a voice 127685, July 23, 1998 [Puno])
recording he had made of the heated discussion at
the police station between accused police officer 194. Discuss why Administrative Order No. 308 (issued
Navarro and the deceased, Lingan, which was taken by the President prescribing for a National ID system
without the knowledge of the two. Is the voice for all citizens to facilitate business transactions with
recording admissible in evidence in view of R.A. 4200, government agencies engaged in the delivery of basic
which prohibits wire-tapping? services and social security provisions) should be
declared unconstitutional.
Held: [J]albuena’s testimony is confirmed by the
voice recording he had made. It may be asked whether Held: We prescind from the premise that the
the tape is admissible in view of R.A. No. 4200, which right to privacy is a fundamental right guaranteed by the
prohibits wire tapping. The answer is in the affirmative. x Constitution, hence, it is the burden of government to
x x. show that A.O. No. 308 is justified by some compelling
state interest and that it is narrowly drawn. A.O. No. 308
[T]he law prohibits the overhearing, intercepting, is predicated on two considerations: (1) the need to
or recording of private communications (Ramirez v. Court provide our citizens and foreigners with the facility to
of Appeals, 248 SCRA 590 [1995]). Since the exchange conveniently transact business with basic service and
between petitioner Navarro and Lingan was not private, social security providers and other government
its tape recording is not prohibited. (Felipe Navarro v. instrumentalities and (2) the need to reduce, if not totally
Court of Appeals, G.R. No. 121087, Aug. 26, 1999, eradicate, fraudulent transactions and misrepresentations
2nd Div. [Mendoza]) by persons seeking basic services. It is debatable
whether these interests are compelling enough to warrant
the issuance of A.O. No. 308. But what is not arguable is
The Right to Privacy the broadness, the vagueness, the overbreadth of A.O.
No. 308 which if implemented will put our people’s right to
192. Is there a constitutional right to privacy? privacy in clear and present danger.

Held: The essence of privacy is the “right to be The heart of A.O. No. 308 lies in its Section 4
let alone.” In the 1965 case of Griswold v. Connecticut which provides for a Population Reference Number (PRN)
(381 U.S. 479, 14 L. ed. 2D 510 [1965]), the United as a “common reference number to establish a linkage
States Supreme Court gave more substance to the right of among concerned agencies” through the use of
privacy when it ruled that the right has a constitutional “Biometrics Technology” and “computer application
foundation. It held that there is a right of privacy which designs.”
can be found within the penumbras of the First, Third,
Fourth, Fifth and Ninth Amendments x x x. In the 1968 It is noteworthy that A.O. No. 308 does not state
case of Morfe v. Mutuc (22 SCRA 424, 444-445), we what specific biological characteristics and what particular
adopted the Griswold ruling that there is a constitutional biometrics technology shall be used to identify people who
right to privacy x x x. will seek its coverage. Considering the banquet of options
available to the implementors of A.O. No. 308, the fear

87
that it threatens the right to privacy of our people is not interest and that the law, rule, or regulation is narrowly
groundless. drawn to preclude abuses. This approach is demanded by
the 1987 Constitution whose entire matrix is designed to
A.O. No. 308 should also raise our antennas for a protect human rights and to prevent authoritarianism. In
further look will show that it does not state whether case of doubt, the least we can do is to lean towards the
encoding of data is limited to biological information alone stance that will not put in danger the rights protected by
for identification purposes. x x x. Clearly, the the Constitution.
indefiniteness of A.O. No. 308 can give the government
the roving authority to store and retrieve information for a The right to privacy is one of the most threatened
purpose other than the identification of the individual rights of man living in a mass society. The threats
through his PRN. emanate from various sources – governments, journalists,
employers, social scientists, etc. In the case at bar, the
The potential for misuse of the data to be threat comes from the executive branch of government
gathered under A.O. No. 308 cannot be underplayed x x which by issuing A.O. No. 308 pressures the people to
x. The more frequent the use of the PRN, the better the surrender their privacy by giving information about
chance of building a huge and formidable information themselves on the pretext that it will facilitate delivery of
base through the electronic linkage of the files. The data basic services. Given the record-keeping power of the
may be gathered for gainful and useful government computer, only the indifferent will fail to perceive the
purposes; but the existence of this vast reservoir of danger that A.O. No. 308 gives the government the power
personal information constitutes a covert invitation to to compile a devastating dossier against unsuspecting
misuse, a temptation that may be too great for some of citizens. x x x [W]e close with the statement that the
our authorities to resist. right to privacy was not engraved in our Constitution for
flattery. (Ople v. Torres, G.R. No. 127685, July 23,
It is plain and we hold that A.O. No. 308 falls 1998 [Puno])
short of assuring that personal information which will be
gathered about our people will only be processed for 195. Should in camera inspection of bank accounts be
unequivocally specified purposes. The lack of proper allowed? If in the affirmative, under what
safeguards in this regard of A.O. No. 308 may interfere circumstances should it be allowed?
with the individual’s liberty of abode and travel by
enabling authorities to track down his movement; it may Held: The issue is whether petitioner may be
also enable unscrupulous persons to access confidential cited for indirect contempt for her failure to produce the
information and circumvent the right against self- documents requested by the Ombudsman. And whether
incrimination; it may pave the way for “fishing the order of the Ombudsman to have an in camera
expeditions” by government authorities and evade the inspection of the questioned account is allowed as an
right against unreasonable searches and seizures. The exception to the law on secrecy of bank deposits (R.A. No.
possibilities of abuse and misuse of the PRN, biometrics 1405).
and computer technology are accentuated when we
consider that the individual lacks control over what can be An examination of the secrecy of bank deposits
read or placed on his ID, much less verify the correctness law (R.A. No. 1405) would reveal the following
of the data encoded. They threaten the very abuses that exceptions:
the Bill of Rights seeks to prevent.
1) Where the depositor consents in writing;
The ability of a sophisticated data center to 2) Impeachment cases;
generate a comprehensive cradle-to-grave dossier on an 3) By court order in bribery or dereliction of duty
individual and transmit it over a national network is one of cases against public officials;
the most graphic threats of the computer revolution. The 4) Deposit is subject of litigation;
computer is capable of producing a comprehensive dossier 5) Sec. 8, R.A. No. 3019, in cases of unexplained
on individuals out of information given at different times wealth as held in the case of PNB v. Gancayco
and for varied purposes. x x x. Retrieval of stored data is (122 Phil. 503, 508 [1965]).
simple. When information of a privileged character finds
its way into the computer, it can be extracted together The order of the Ombudsman to produce for in
with other data on the subject. Once extracted, the camera inspection the subject accounts with the Union
information is putty in the hands of any person. The end Bank of the Philippines, Julia Vargas Branch, is based on a
of privacy begins. pending investigation at the Office of the Ombudsman
against Amado Lagdameo, et. al. for violation of R.A. No.
[T]he Court will not be true to its role as the 3019, Sec. 3 (e) and (g) relative to the Joint Venture
ultimate guardian of the people’s liberty if it would not Agreement between the Public Estates Authority and
immediately smother the sparks that endanger their rights AMARI.
but would rather wait for the fire that could consume
them. We rule that before an in camera inspection may
be allowed, there must be a pending case before a court
[A]nd we now hold that when the integrity of a of competent jurisdiction. Further, the account must be
fundamental right is at stake, this Court will give the clearly identified, the inspection limited to the subject
challenged law, administrative order, rule or regulation a matter of the pending case before the court of competent
stricter scrutiny. It will not do for the authorities to invoke jurisdiction. The bank personnel and the account holder
the presumption of regularity in the performance of must be notified to be present during the inspection, and
official duties. Nor is it enough for the authorities to such inspection may cover only the account identified in
prove that their act is not irrational for a basic right can the pending case.
be diminished, if not defeated, even when the
government does not act irrationally. They must In Union Bank of the Philippines v. Court of
satisfactorily show the presence of compelling state Appeals, we held that “Section 2 of the Law on Secrecy of

88
Bank Deposits, as amended, declares bank deposits to be danger test to such regulatory measures would be like
‘absolutely confidential’ except: using a sledgehammer to drive a nail when a regular
hammer is all that is needed.
1) In an examination made in the course of a
special or general examination of a bank that The test for this difference in the level of
is specifically authorized by the Monetary justification for the restriction of speech is that content-
Board after being satisfied that there is based restrictions distort public debate, have improper
reasonable ground to believe that a bank motivation, and are usually imposed because of fear of
fraud or serious irregularity has been or is how people will react to a particular speech. No such
being committed and that it is necessary to reasons underlie content-neutral regulations, like
look into the deposit to establish such fraud regulation of time, place and manner of holding public
or irregularity, assemblies under B.P. Blg. 880, the Public Assembly Act of
2) In an examination made by an independent 1985. (Osmena v. COMELEC, 288 SCRA 447, March
auditor hired by the bank to conduct its 31, 1998 [Mendoza])
regular audit provided that the examination is
for audit purposes only and the results 197. What is the most influential test for distinguishing
thereof shall be for the exclusive use of the content-based from content-neutral regulations?
bank,
3) Upon written permission of the depositor, Held: The United States Supreme Court x x x
4) In cases of impeachment, held in United States v. O’ Brien:
5) Upon order of a competent court in cases of
bribery or dereliction of duty of public [A] a governmental regulation is sufficiently
officials, or justified (1) if it is within the constitutional power
6) In cases where the money deposited or of the government; (2) if it furthers an important
invested is the subject matter of the or substantial governmental interest; (3) if the
litigation”. governmental interest is unrelated to the
suppression of free expression; and (4) if the
In the case at bar, there is yet no pending incidental restriction on alleged First Amendment
litigation before any court of competent authority. What freedoms (of speech, expression and press) is no
is existing is an investigation by the Office of the greater than is essential to the furtherance of that
Ombudsman. In short, what the Office of the intesrst (391 U.S. 367, 20 L. Ed. 2df 692, 680
Ombudsman would wish to do is to fish for additional [1968] [bracketed numbers added])
evidence to formally charge Amado Lagdameo, et. al.,
with the Sandiganbayan. Clearly, there was no pending This is so far the most influential test for
case in court which would warrant the opening of the distinguishing content-based from content-neutral
bank account for inspection. (Lourdes T. Marquez v. regulations and is said to have “become canonical in the
Hon. Aniano A. Desierto, G.R. No. 135882, June 27, review of such laws.” (G. Gunther & K. Sullivan,
2001, En Banc [Pardo]) Constitutional Law 1217 [13th ed. 1997]). It is noteworthy
that the O’ Brien test has been applied by this Court in at
least two cases (Adiong v. Comelec, 207 SCRA 712
Freedom of Expression [1992]; Osmena v. Comelec, supra.).

196. Distinguish “content-based restrictions” on free Under this test, even if a law furthers an
speech from “content-neutral restrictions,” and give important or substantial governmental interest, it should
example of each. be invalidated if such governmental interest is “not
unrelated to the suppression of free expression.”
Held: Content-based restrictions are imposed Moreover, even if the purpose is unrelated to the
because of the content of the speech and are, therefore, suppression of free speech, the law should nevertheless
subject to the clear-and-present danger test. For be invalidated if the restriction on freedom of expression
example, a rule such as that involved in Sanidad v. is greater than is necessary to achieve the governmental
Comelec, prohibiting columnists, commentators, and purpose in question. (Social Weather Stations, Inc. v.
announcers from campaigning either for or against an Comelec, G.R. No. 147571, May 5, 2001, En Banc
issue in a plebiscite must have compelling reason to [Mendoza])
support it, or it will not pass muster under strict scrutiny.
These restrictions are censorial and therefore they bear a 198. Does the conduct of exit poll by ABS CBN present
heavy presumption of constitutional invalidity. In addition, a clear and present danger of destroying the
they will be tested for possible overbreadth and credibility and integrity of the electoral process as it
vagueness. has the tendency to sow confusion considering the
randomness of selecting interviewees, which further
Content-neutral restrictions, on the other hand, makes the exit poll highly unreliable, to justify the
like Sec. 11(b) of R.A. No. 6646, which prohibits the sale promulgation of a Comelec resolution prohibiting the
or donation of print space and air time to political same?
candidates during the campaign period, are not concerned
with the content of the speech. These regulations need Held: Such arguments are purely speculative and
only a substantial governmental interest to support them. clearly untenable. First, by the very nature of a survey,
A deferential standard of review will suffice to test their the interviewees or participants are selected at random,
validity. The clear-and-present danger rule is so that the results will as much as possible be
inappropriate as a test for determining the constitutional representative or reflective of the general sentiment or
validity of laws, like Sec. 11(b) of R.A. No. 6646, which view of the community or group polled. Second, the
are not concerned with the content of political ads but survey result is not meant to replace or be at par with the
only with their incidents. To apply the clear-and-present official Comelec count. It consists merely of the opinion

89
of the polling group as to who the electorate in general Held: For reason hereunder given, we hold that
has probably voted for, based on the limited data Section 5.4 of R.A. No. 9006 constitutes an
gathered from polled individuals. Finally, not at stake are unconstitutional abridgment of freedom of speech,
the credibility and the integrity of the elections, which are expression, and the press.
exercises that are separate and independent from the exit
polls. The holding and the reporting of the results of exit To be sure, Section 5.4 lays a prior restraint on
polls cannot undermine those of the elections, since the freedom of speech, expression, and the press by
former is only part of the latter. If at all, the outcome of prohibiting the publication of election survey results
one can only be indicative of the other. affecting candidates within the prescribed periods of
fifteen (15) days immediately preceding a national
The COMELEC’s concern with the possible election and seven (7) days before a local election.
noncommunicative effect of exit polls – disorder and Because of the preferred status of the constitutional rights
confusion in the voting centers – does not justify a total of speech, expression, and the press, such a measure is
ban on them. Undoubtedly, the assailed Comelec vitiated by a weighty presumption of invalidity. Indeed,
Resolution is too broad, since its application is without “any system of prior restraints of expression comes to this
qualification as to whether the polling is disruptive or not. Court bearing a heavy presumption against its
There is no showing, however, that exit polls or the constitutional validity x x x. The Government ‘thus carries
means to interview voters cause chaos in voting centers. a heavy burden of showing justification for the
Neither has any evidence been presented proving that the enforcement of such restraint.’” There is thus a reversal
presence of exit poll reporters near an election precinct of the normal presumption of validity that inheres in every
tends to create disorder or confuse the voters. legislation.

Moreover, the prohibition incidentally prevents the Nor may it be argued that because of Art. IX-C,
collection of exit poll data and their use for any purpose. Sec. 4 of the Constitution, which gives the Comelec
The valuable information and ideas that could be derived supervisory power to regulate the enjoyment or utilization
from them, based on the voters’ answers to the survey of franchise for the operation of media of communication,
questions will forever remain unknown and unexplored. no presumption of invalidity attaches to a measure like
Unless the ban is restrained, candidates, researchers, Sec. 5.4. For as we have pointed out in sustaining the
social scientists and the electorate in general would be ban on media political advertisements, the grant of power
deprived of studies on the impact of current events and of to the Comelec under Art. IX-C, Sec. 4 is limited to
election-day and other factors on voters’ choices. ensuring “equal opportunity, time, space, and the right to
reply” as well as uniform and reasonable rates of charges
The absolute ban imposed by the Comelec for the use of such media facilities for “public information
cannot, therefore, be justified. It does not leave open any campaigns and forums among candidates.”
alternative channel of communication to gather the type
of information obtained through exit polling. On the other Xxx
hand, there are other valid and reasonable ways and
means to achieve the Comelec end of avoiding or Nor can the ban on election surveys be justified
minimizing disorder and confusion that may be brought on the ground that there are other countries x x x which
about by exit surveys. similarly impose restrictions on the publication of election
surveys. At best this survey is inconclusive. It is
With foregoing premises, it is concluded that the noteworthy that in the United States no restriction on the
interest of the state in reducing disruption is outweighed publication of election survey results exists. It cannot be
by the drastic abridgment of the constitutionally argued that this is because the United States is a mature
guaranteed rights of the media and the electorate. Quite democracy. Neither are there laws imposing an embargo
the contrary, instead of disrupting elections, exit polls – on survey results, even for a limited period, in other
properly conducted and publicized – can be vital tools for countries. x x x.
the holding of honest, orderly, peaceful and credible
elections; and for the elimination of election-fixing, fraud What test should then be employed to determine
and other electoral ills. (ABS-CBN Broadcasting the constitutional validity of Section 5.4? The United
Corporation v. COMELEC, G.R. No. 133486, Jan. 28, States Supreme Court x x x held in United States v. O’
2000, En Banc [Panganiban]) Brien:

199. Section 5.4 of R.A. No. 9006 (Fair Election Act) [A] government regulation is sufficiently justified
which provides: “Surveys affecting national candidates (1) if it is within the constitutional power of the
shall not be published fifteen (15) days before an government; (2) if it furthers an important or
election and surveys affecting local candidates shall substantial governmental interest; (3) if the
not be published seven (7) days before an election.” governmental interest is unrelated to the
The Social Weather Stations, Inc. (SWS), a private suppression of free expression; and (4) if the
non-stock, non-profit social research institution incidental restriction on alleged First Amendment
conducting surveys in various fields; and Kamahalan freedoms (of speech, expression and press) is no
Publishing Corporation, publisher of the Manila greater than is essential to the furtherance of that
Standard, a newspaper of general circulation, which interest (391 U.S. 367, 20 L. Ed. 2d 692, 680
features newsworthy items of information including [1968] [bracketed numbers added]).
election surveys, challenged the constitutionality of
aforesaid provision as it constitutes a prior restraint This is so far the most influential test for
on the exercise of freedom of speech without any distinguishing content-based from content-neutral
clear and present danger to justify such restraint. regulations and is said to have “become canonical in the
Should the challenge be sustained? review of such laws.” It is noteworthy that the O’ Brien
test has been applied by this Court in at least two cases
(Adiong v. Comelec; Osmena v. Comelec).

90
be for a limited time, but the curtailment of the right of
Under this test, even if a law furthers an expression is direct, absolute, and substantial. It
important or substantial governmental interest, it should constitutes a total suppression of a category of speech
be invalidated if such governmental interest is “not and is not made less so because it is only for a period of
unrelated to the suppression of free expression.” fifteen (15) days immediately before a national election
Moreover, even if the purpose is unrelated to the and seven (7) days immediately before a local election.
suppression of free speech, the law should nevertheless
be invalidated if the restriction on freedom of expression This sufficiently distinguishes Sec. 5.4 from R.A.
is greater than is necessary to achieve the governmental No. 6646, Sec. 11(b), which this Court found to be valid in
purpose in question. National Press Club v. Comelec, and Osmena v. Comelec.
For the ban imposed by R.A. No. 6646, Sec. 11(b) is not
Our inquiry should accordingly focus on these two only authorized by a specific constitutional provision (Art.
considerations as applied to Sec. 5.4. IX-C, Sec. 4), but it also provided an alternative so that,
as this Court pointed out in Osmena, there was actually
First. Sec. 5.4 fails to meet criterion (3) of the O’ no ban but only a substitution of media advertisements by
Brien test because the causal connection of expression to the Comelec space, and Comelec hour.
the asserted governmental interest makes such interest
“not unrelated to the suppression of free expression.” By Second. Even if the governmental interest sought
prohibiting the publication of election survey results to be promoted is unrelated to the suppression of speech
because of the possibility that such publication might and the resulting restriction of free expression is only
undermine the integrity of the election, Sec. 5.4 actually incidental, Sec. 5.4 nonetheless fails to meet criterion (4)
suppresses a whole class of expression, while allowing the of the O’ Brien test, namely, that the restriction be not
expression of opinion concerning the same subject matter greater than is necessary to further the governmental
by newspaper columnists, radio and TV commentators, interest. As already stated, Sec. 5.4. aims at the
armchair theorists, and other opinion makers. In effect, prevention of last-minute pressure on voters, the creation
Sec. 5.4 shows a bias for a particular subject matter, if of bandwagon effect, “junking” of weak or “losing”
not viewpoint, by preferring personal opinion to statistical candidates, and resort to the form of election cheating
results. The constitutional guarantee of freedom of called “dagdag-bawas.” Praiseworthy as these aims of
expression means that “the government has no power to the regulation might be, they cannot be attained at the
restrict expression because of its message, its ideas, its sacrifice of the fundamental right of expression, when
subject matter, or its contents.” The inhibition of speech such aim can be more narrowly pursued by punishing
should be upheld only if the expression falls within one of unlawful acts, rather than speech because of
the few unprotected categories dealt with in Chaplinsky v. apprehension that such speech creates the danger of such
New Hampshire, thus: evils. Thus, under the Administrative Code of 1987 (Bk.
V, Tit. I, Subtit. C, Ch 1, Sec. 3[1]), the Comelec is given
There are certain well-defined and the power:
narrowly limited classes of speech, the prevention
and punishment of which have never been To stop any illegal activity, or confiscate,
thought to raise any Constitutional problem. tear down, and stop any unlawful, libelous,
These include the lewd and obscene, the profane, misleading or false election propaganda, after due
the libelous, and the insulting or ‘fighting’ words – notice and hearing.
those which by their very utterance inflict injury
or tend to incite an immediate breach of the This is surely a less restrictive means than the
peace. [S]uch utterances are no essential part of prohibition contained in Sec. 5.4. Pursuant to this power
any exposition of ideas, and are of such slight of the Comelec, it can confiscate bogus survey results
social value as a step to truth that any benefit calculated to mislead voters. Candidates can have their
that may be derived from them is clearly own surveys conducted. No right of reply can be invoked
outweighed by the social interest in order and by others. No principle of equality is involved. It is a free
morality. market to which each candidate brings his ideas. As for
the purpose of the law to prevent bandwagon effects, it is
Nor is there justification for the prior restraint doubtful whether the Government can deal with this
which Sec. 5.4 lays on protected speech. In Near v. natural-enough tendency of some voters. Some voters
Minnesota, it was held: want to be identified with the “winners.” Some are
susceptible to the herd mentality. Can these be
[T]he protection even as to previous restraint is legitimately prohibited by suppressing the publication of
not absolutely unlimited. But the limitation has survey results which are a form of expression? It has
been recognized only in exceptional cases x x x. been held that “[mere] legislative preferences or beliefs
No one would question but that a government respecting matters of public convenience may well
might prevent actual obstruction to its recruiting support regulation directed at other personal activities,
service or the publication of the sailing dates of but be insufficient to justify such as diminishes the
transports or the number and location of troops. exercise of rights so vital to the maintenance of
On similar grounds, the primary requirements of democratic institutions.”
decency may be enforced against obscene
publications. The security of the community life To summarize then, we hold that Sec. 5.4. is
may be protected against incitements to acts of invalid because (1) it imposes a prior restraint on the
violence and the overthrow by force of orderly freedom of expression, (2) it is a direct and total
government x x x. suppression of a category of expression even though such
suppression is only for a limited period, and (3) the
Thus, x x x the prohibition imposed by Sec. 5.4 governmental interest sought to be promoted can be
cannot be justified on the ground that it is only for a achieved by means other than the suppression of freedom
limited period and is only incidental. The prohibition may of expression. (Social Weather Stations, Inc., v.

91
COMELEC, G.R. No. 147571, May 5, 2001, En Banc based on established facts, then it is immaterial that the
[Mendoza]) opinion happens to be mistaken, as long as it might
reasonably be inferred from the facts. (Borjal v. CA,
200. The question for determination in this case is the 301 SCRA 1, Jan. 14, 1999, 2nd Div. [Bellosillo])
liability for libel of a citizen who denounces a
barangay official for misconduct in office. The 202. What is the “raison d’etre” for the New York
Regional Trial Court of Manila x x x found petitioner Times v. Sullivan (376 US 254) holding that honest
guilty x x x on the ground that petitioner failed to criticisms on the conduct of public officials and public
prove the truth of the charges and that he was figures are insulated from libel judgments?
“motivated by vengeance in uttering the defamatory
statement.” Held: The guarantees of freedom of speech and
press prohibit a public official or public figure from
Held: The decision appealed from should be recovering damages for a defamatory falsehood relating
reversed. to his official conduct unless he proves that the statement
was made with actual malice, i.e., with knowledge that it
In denouncing the barangay chairman in this was false or with reckless disregard of whether it was
case, petitioner and the other residents of the Tondo false or not.
Foreshore Area were not only acting in their self-interest
but engaging in the performance of a civic duty to see to The raison d’etre for the New York Times doctrine
it that public duty is discharged faithfully and well by was that to require critics of official conduct to guarantee
those on whom such duty is incumbent. The recognition the truth of all their factual assertions on pain of libel
of this right and duty of every citizen in a democracy is judgments would lead to self-censorship, since would-be
inconsistent with any requirement placing on him the critics would be deterred from voicing out their criticisms
burden of proving that he acted with good motives and even if such were believed to be true, or were in fact true,
for justifiable ends. because of doubt whether it could be proved or because
of fear of the expense of having to prove it . (Borjal v.
For that matter, even if the defamatory statement CA, 301 SCRA 1, Jan. 14, 1999, 2nd Div. [Bellosillo])
is false, no liability can attach if it relates to official
conduct, unless the public official concerned proves that 203. Columnist Art Borjal of The Philippine Star wrote
the statement was made with actual malice – that is, with in his column that petitioner (private respondent), the
knowledge that it was false or with reckless disregard of Executive Director and Spokesman of the FNCLT (First
whether it was false or not. This is the gist of the ruling National Conference on Land Transportation), a joint
in the landmark case of New York Times v. Sullivan, which project of the government and the private sector to
this Court has cited with approval in several of its own raise funds to improve the state of land transportation
decisions. This is the rule of “actual malice.” In this case, in the country, engaged in shady and anomalous
the prosecution failed to prove not only that the charges deals. He was sued for damages arising from libel by
made by petitioner were false but also that petitioner petitioner (private respondent) and subsequently
made them with knowledge of their falsity or with reckless found liable by the trial court and made to pay
disregard of whether they were false or not. damages. On appeal, the SC reversed applying the
doctrine of fair comment.
A rule placing on the accused the burden of
showing the truth of allegations of official misconduct Held: [W]e deem private respondent a public
and/or good motives and justifiable ends for making such figure within the purview of the New York Times ruling. X
allegations would not only be contrary to Art. 361 of the xx
Revised Penal Code. It would, above all, infringe on the
constitutionally guaranteed freedom of expression. Such The FNCLT (First National Conference on Land
a rule would deter citizens from performing their duties as Transportation) was an undertaking infused with public
members of a self-governing community. Without free interest. It was promoted as a joint project of the
speech and assembly, discussions of our most abiding government and the private sector, and organized by top
concerns as a nation would be stifled. As Justice Brandies government officials and prominent businessmen. For this
has said, “public discussion is a political duty” and the reason, it attracted media mileage and drew public
“greatest menace to freedom is an inert people.” (Whitney attention not only to the conference itself but to the
v. California) (Vasquez v. Court of Appeals, 314 personalities behind as well. As its Executive Director and
SCRA 460, Sept. 15, 1999, En Banc [Mendoza]) spokesman, private respondent consequently assumed the
status of a public figure.
201. Discuss the "doctrine of fair comment" as a valid
defense in an action for libel or slander. But even assuming ex-gratia argumenti that
private respondent, despite the position he occupied in
Held: Fair commentaries on matters of public the FNCLT, would not qualify as a public figure, it does
interest are privileged and constitute a valid defense in an not necessarily follow that he could not validly be the
action for libel or slander. The doctrine of fair comment subject of a public comment even if he was not a public
means that while in general every discreditable imputation official or at least a public figure, for he could be, as long
publicly made is deemed false, because every man is as he was involved in a public issue. If a matter is a
presumed innocent until his guilt is judicially proved, and subject of public or general interest, it cannot suddenly
every false imputation is deemed malicious, nevertheless, become less so merely because a private individual is
when the discreditable imputation is directed against a involved or because in some sense the individual did not
public person in his public capacity, it is not necessarily voluntarily choose to become involved. The public’s
actionable. In order that such discreditable imputation to primary interest is in the event; the public focus is on the
a public official may be actionable, it must either be a conduct of the participant and the content, effect and
false allegation of fact or a comment based on a false significance of the conduct, not the participant’s prior
supposition. If the comment is an expression of opinion,

92
anonymity or notoriety. (Borjal v. CA, 301 SCRA 1, Canete (38 Phil. 253, 265 [1918]), this Court ruled that
Jan. 14, 1999, 2nd Div. [Bellosillo]) publications which are privileged for reasons of public
policy are protected by the constitutional guaranty of
204. Who is a “public figure,” and therefore subject to freedom of speech. This constitutional right cannot be
public comment? abolished by the mere failure of the legislature to give it
express recognition in the statute punishing libel. (Borjal
Held: At any rate, we have also defined “public v. CA, 301 SCRA 1, Jan. 14, 1999, 2nd Div.
figure” in Ayers Production Pty., Ltd. v. Capulong as – [Bellosillo])

X x x a person who, by his accomplishments, 208. The Office of the Mayor of Las Pinas refused to
fame, mode of living, or by adopting a profession issue permit to petitioners to hold rally a rally in front
or calling which gives the public a legitimate of the Justice Hall of Las Pinas on the ground that it
interest in his doings, his affairs and his character, was prohibited under Supreme Court En Banc
has become a ‘public personage.’ He is, in other Resolution dated July 7,1998 in A.M. No. 98-7-02-SC,
words, a celebrity. Obviously, to be included in entitled, "Re: Guidelines on the Conduct of
this category are those who have achieved some Demonstrations, Pickets, Rallies and Other Similar
degree of reputation by appearing before the Gatherings in the Vicinity of the Supreme Court and
public, as in the case of an actor, a professional All Other Courts." Petitioners thus initiated the instant
baseball player, a pugilist, or any other proceedings. They submit that the Supreme Court
entertainer. The list is, however, broader than gravely abused its discretion and/or acted without or
this. It includes public officers, famous inventors in excess of jurisdiction in promulgating those
and explorers, war heroes and even ordinary guidelines.
soldiers, infant prodigy, and no less a personage
than the Great Exalted Ruler of the lodge. It Held: We shall first dwell on the critical
includes, in short, anyone who has arrived at a argument made by petitioners that the rules constitute an
position where the public attention is focused abridgment of the people's aggregate rights of free
upon him as a person. (Borjal v. CA, 301 SCRA speech, free expression, peaceful assembly and
1, Jan. 14, 1999, 2nd Div. [Bellosillo]) petitioning government for redress of grievances citing
Sec. 4, Article III of the 1987 Constitution that “no law
205. What are the types of privileged communications? shall be passed abridging” them.

Held: A privileged communication may be either It is true that the safeguarding of the people's
absolutely privileged or qualifiedly privileged. Absolutely freedom of expression to the end that individuals may
privileged communications are those which are not speak as they think on matters vital to them and that
actionable even if the author acted in bad faith. An falsehoods may be exposed through the processes of
example is found in Sec. 11, Art. VI, of the 1987 education and discussion, is essential to free government .
Constitution which exempts a member of Congress from But freedom of speech and expression despite its
liability for any speech or debate in the Congress or in any indispensability has its limitations. It has never been
Committee thereof. Upon the other hand, qualifiedly understood as the absolute right to speak whenever,
privileged communications containing defamatory however, and wherever one pleases, for the manner,
imputations are not actionable unless found to have been place, and time of public discussion can be constitutionally
made without good intention or justifiable motive. To this controlled. [T]he better policy is not liberty untamed but
genre belong “private communications” and “fair and true liberty regulated by law where every freedom is exercised
report without any comments or remarks.” (Borjal v. in accordance with law and with due regard for the rights
CA, 301 SCRA 1, Jan. 14, 1999, 2nd Div. [Bellosillo]) of others.

206. Is the enumeration of qualifiedly privileged Conventional wisdom tells us that the realities of
communications under Article 354 of the Revised life in a complex society preclude an absolutist
Penal Code exclusive? interpretation of freedom of expression where it does not
involve pure speech but speech plus physical actions like
Held: Indisputably, petitioner Borjal’s questioned picketing. There are other significant societal values that
writings are not within the exceptions of Art. 354 of The must be accommodated and when they clash, they must
Revised Penal Code for x x x they are neither private all be weighed with the promotion of the general welfare
communications nor fair and true report without any of the people as the ultimate objective. In balancing
comments or remarks. However, this does not necessarily these values, this Court has accorded freedom of
mean that they are not privileged. To be sure, the expression a preferred position in light of its more
enumeration under Art. 354 is not an exclusive list of comparative importance. Hence, our rulings now musty in
qualifiedly privileged communications since fair years hold that only the narrowest time, place and
commentaries on matters of public interest are likewise manner regulations that are specifically tailored to serve
privileged. (Borjal v. CA, 301 SCRA 1, Jan. 14, 1999, an important governmental interest may justify the
2nd Div. [Bellosillo]) application of the balancing of interests test in derogation
of the people's right of free speech and expression.
207. Discuss the origin of the rule on privileged Where said regulations do not aim particularly at the evils
communication. Did it originate in the nation’s penal within the allowable areas of state control but, on the
laws, or in the Bill of Rights guaranteeing freedom of contrary, sweep within their ambit other activities as to
speech and of the press? operate as an overhanging threat to free discussion, or
where upon their face they are so vague, indefinite, or
Held: The rule on privileged communications had inexact as to permit punishment of the fair use of the
its genesis not in the nation’s penal code but in the Bill of right of free speech, such regulations are void.
Rights of the Constitution guaranteeing freedom of speech
and of the press. As early as 1918, in United States v.

93
Prescinding from this premise, the Court reiterates public forum place into a “no rally” zone. Thus, they
that judicial independence and the fair and orderly accuse this Court of x x x violating the principle of
administration of justice constitute paramount separation of powers.
governmental interests that can justify the regulation of
the public's right of free speech and peaceful assembly in We reject these low watts arguments. Public
the vicinity of courthouses. In the case of In Re: Emil P. places historically associated with the free exercise of
Jurado, the Court pronounced in no uncertain terms that: expressive activities, such as streets, sidewalks, and
parks, are considered, without more, to be public fora. In
“x x x freedom of expression needs on other words, it is not any law that can imbue such places
occasion to be adjusted to and accommodated with the public nature inherent in them. But even in such
with the requirements of equally important public public fora, it is settled jurisprudence that the government
interests. One of these fundamental public may restrict speech plus activities and enforce reasonable
interests is the maintenance of the integrity and time, place, and manner regulations as long as the
orderly functioning of the administration of restrictions are content-neutral, are narrowly tailored to
justice. There is no antinomy between free serve a significant governmental interest, and leave open
expression and the integrity of the system of ample alternative channels of communication.
administering justice. For the protection and
maintenance of freedom of expression itself can Contrary therefore to petitioners’ impression, B.P.
be secured only within the context of a Blg. 880 did not establish streets and sidewalks, among
functioning and orderly system of dispensing other places, as public fora. A close look at the law will
justice, within the context, in other words, of reveal that it in fact prescribes reasonable time, place,
viable independent institutions for delivery of and manner regulations. Thus, it requires a written
justice which are accepted by the general permit for the holding of public assemblies in public places
community. x x x” (In Re: Emil P. Jurado, 243 subject, even, to the right of the mayor to modify the
SCRA 299, 323-324 [1995]) place and time of the public assembly, to impose a
rerouting of the parade or street march, to limit the
It is sadly observed that judicial independence volume of loud speakers or sound system and to prescribe
and the orderly administration of justice have been other appropriate restrictions on the conduct of the public
threatened not only by contemptuous acts inside, but also assembly.
by irascible demonstrations outside, the courthouses.
They wittingly or unwittingly, spoil the ideal of sober, non- The existence of B.P. Blg. 880, however, does not
partisan proceedings before a cold and neutral judge. preclude this Court from promulgating rules regulating
Even in the United States, a prohibition against picketing conduct of demonstrations in the vicinity of courts to
and demonstrating in or near courthouses, has been ruled assure our people of an impartial and orderly
as valid and constitutional notwithstanding its limiting administration of justice as mandated by the Constitution.
effect on the exercise by the public of their liberties. X x x To insulate the judiciary from mob pressure, friendly or
otherwise, and isolate it from public hysteria, this Court
The administration of justice must not only be fair merely moved away the situs of mass actions within a
but must also appear to be fair and it is the duty of this 200-meter radius from every courthouse. In fine, B.P.
Court to eliminate everything that will diminish if not Blg. 880 imposes general restrictions to the time, place
destroy this judicial desideratum. To be sure, there will and manner of conducting concerted actions. On the
be grievances against our justice system for there can be other hand, the resolution of this Court regulating
no perfect system of justice but these grievances must be demonstrations adds specific restrictions as they involve
ventilated through appropriate petitions, motions or other judicial independence and the orderly administration of
pleadings. Such a mode is in keeping with the respect justice. There is thus no discrepancy between the two
due to the courts as vessels of justice and is necessary if sets of regulatory measures. Simply put, B.P. Blg. 880
judges are to dispose their business in a fair fashion. It is and the assailed resolution complement each other. We
the traditional conviction of every civilized society that so hold following the rule in legal hermeneutics that an
courts must be insulated from every extraneous influence apparent conflict between a court rule and a statutory
in their decisions. The facts of a case should be provision should be harmonized and both should be given
determined upon evidence produced in court, and should effect if possible. (In Re: Petition to Annul En Banc
be uninfluenced by bias, prejudice or sympathies. (In Re: Resolution A.M. 98-7-02-SC - Ricardo C. Valmonte
Petition to Annul En Banc Resolution A.M. 98-7-02- and Union of Lawyers and Advocates for
SC - Ricardo C. Valmonte and Union of Lawyers and Transparency in Government [ULAT], G.R. No.
Advocates for Transparency in Government [ULAT], 134621, Sept. 29, 1998)
G.R. No. 134621, Sept. 29, 1998)
210. Should live media coverage of court proceedings
209. Did the Supreme Court commit an act of judicial be allowed?
legislation in promulgating En Banc Resolution A.M.
98-7-02-SC, entitled, “Re: Guidelines on the Conduct Held: The propriety of granting or denying
of Demonstrations, Pickets, Rallies and Other Similar permission to the media to broadcast, record, or
Gatherings in the Vicinity of the Supreme Court and photograph court proceedings involves weighing the
All Other Courts?” constitutional guarantees of freedom of the press, the
right of the public to information and the right to public
Held: Petitioners also claim that this Court trial, on the one hand, and on the other hand, the due
committed an act of judicial legislation in promulgating process rights of the defendant and the inherent and
the assailed resolution. They charge that this Court constitutional power of the courts to control their
amended provisions of Batas Pambansa (B.P.) Blg. 880, proceedings in order to permit the fair and impartial
otherwise known as “the Public Assembly Act,” by administration of justice. Collaterally, it also raises issues
converting the sidewalks and streets within a radius of on the nature of the media, particularly television and its
two hundred (200) meters from every courthouse from a

94
role in society, and of the impact of new technologies on the Hearing of President Corazon C. Aquino's Libel
law. Case, dated Oct. 22, 1991)

Xxx 211. Should the Court allow live media coverage of the
anticipated trial of the plunder and other criminal
Courts do not discriminate against radio and cases filed against former President Joseph E. Estrada
television media by forbidding the broadcasting or before the Sandiganbayan in order “to assure the
televising of a trial while permitting the newspaper public of full transparency in the proceedings of an
reporter access to the courtroom, since a television or unprecedented case in our history” as requested by
news reporter has the same privilege, as the news the Kapisanan ng mga Brodkaster ng Pilipinas?
reporter is not permitted to bring his typewriter or printing
press into the courtroom. Held: The propriety of granting or denying the
instant petition involve the weighing out of the
In Estes v. Texas, the United States Supreme constitutional guarantees of freedom of the press and the
Court held that television coverage of judicial proceedings right to public information, on the one hand, and the
involves an inherent denial of due process rights of a fundamental rights of the accused, on the other hand,
criminal defendant. Voting 5-4, the Court through Mr. along with the constitutional power of a court to control
Justice Clark, identified four (4) areas of potential its proceedings in ensuring a fair and impartial trial.
prejudice which might arise from the impact of the
cameras on the jury, witnesses, the trial judge and the When these rights race against one another,
defendant. The decision in part pertinently stated: jurisprudence tells us that the right of the accused must
be preferred to win.
“Experience likewise has established the
prejudicial effect of telecasting on witnesses. With the possibility of losing not only the precious
Witnesses might be frightened, play to the liberty but also the very life of an accused, it behooves all
camera, or become nervous. They are subject to to make absolutely certain that an accused receives a
extraordinary out-of-court influences which might verdict solely on the basis of a just and dispassionate
affect their testimony. Also, telecasting not only judgment, a verdict that would come only after the
increases the trial judge's responsibility to avoid presentation of credible evidence testified to by unbiased
actual prejudice to the defendant; it may as well witnesses unswayed by any kind of pressure, whether
affect his own performance. Judges are human open or subtle, in proceedings that are devoid of
beings also and are subject to the same histrionics that might detract from its basic aim to ferret
psychological reactions as laymen. For the veritable facts free from improper influence, and decreed
defendant, telecasting is a form of mental by a judge with an unprejudiced mind, unbridled by
harassment and subjects him to excessive public running emotions or passions.
exposure and distracts him from the effective
presentation of his defense. Due process guarantees the accused a
presumption of innocence until the contrary is proved in a
“The television camera is a powerful trial that is not lifted above its individual settings nor
weapon which intentionally or inadvertently can made an object of public’s attention and where the
destroy an accused and his case in the eyes of the conclusions reached are induced not by any outside force
public.” or influence but only by evidence and argument given in
open court, where fitting dignity and calm ambiance is
Representatives of the press have no special demanded.
standing to apply for a writ of mandate to compel a court
to permit them to attend a trial, since within the Witnesses and judges may very well be men and
courtroom a reporter's constitutional rights are no greater women of fortitude, able to thrive in hardy climate, with
than those of any other member of the public. Massive every reason to presume firmness of mind and resolute
intrusion of representatives of the news media into the endurance, but it must also be conceded that “television
trial itself can so alter or destroy the constitutionally can work profound changes in the behavior of the people
necessary judicial atmosphere and decorum that the it focuses on.” Even while it may be difficult to quantify
requirements of impartiality imposed by due process of the influence, or pressure that media can bring to bear on
law are denied the defendant and a defendant in a them directly and through the shaping of public opinion, it
criminal proceeding should not be forced to run a gauntlet is a fact, nonetheless, that, indeed, it does so in so many
of reporters and photographers each time he enters or ways and in varying degrees. The conscious or
leaves the courtroom. unconscious effect that such a coverage may have on the
testimony of witnesses and the decision of judges cannot
Considering the prejudice it poses to the be evaluated but, it can likewise be said, it is not at all
defendant's right to due process as well as to the fair and unlikely for a vote of guilt or innocence to yield to it . It
orderly administration of justice, and considering further might be farcical to build around them an impregnable
that the freedom of the press and the right of the people armor against the influence of the most powerful media of
to information may be served and satisfied by less public opinion.
distracting, degrading and prejudicial means, live radio
and television coverage of court proceedings shall not be To say that actual prejudice should first be
allowed. Video footages of court hearings for news present would leave to near nirvana the subtle threats to
purposes shall be restricted and limited to shots of the justice that a disturbance of the mind so indispensable to
courtroom, the judicial officers, the parties and their the calm and deliberate dispensation of justice can create.
counsel taken prior to the commencement of official The effect of television may escape the ordinary means of
proceedings. No video shots or photographs shall be proof, but it is not far-fetched for it to gradually erode our
permitted during the trial proper. (Supreme Court En basal conception of a trial such as we know it now.
Banc Resolution Re: Live TV and Radio Coverage of

95
An accused has a right to a public trial but it is a Held: 1. The right to religious profession and
right that belongs to him, more than anyone else, where worship has a two-fold aspect, viz., freedom to believe
his life or liberty can be held critically in balance. A public and freedom to act on one's belief. The first is absolute
trial aims to ensure that he is fairly dealt with and would as long as the belief is confined within the realm of
not be unjustly condemned and that his rights are not thought. The second is subject to regulation where the
compromised in secret conclaves of long ago. A public belief is translated into external acts that affect the public
trial is not synonymous with publicized trial; it only implies welfare. (Iglesia Ni Cristo v. CA, 259 SCRA 529, July
that the court doors must be open to those who wish to 26, 1996 [Puno])
come, sit in the available seats, conduct themselves with
decorum and observe the trial process. In the 2. The constitutional inhibition of legislation on
constitutional sense, a courtroom should have enough the subject of religion has a double aspect. On the one
facilities for a reasonable number of the public to observe hand, it forestalls compulsion by law of the acceptance of
the proceedings, not too small as to render the openness any creed or the practice of any form of worship.
negligible and not too large as to distract the trial Freedom of conscience and freedom to adhere to such
participants from their proper functions, who shall then be religious organization or form of worship as the individual
totally free to report what they have observed during the may choose cannot be restricted by law. On the other
proceedings. hand, it safeguards the free exercise of the chosen form
of religion. Thus, the Constitution embraces two
The courts recognize the constitutionally concepts, that is, freedom to believe and freedom to act.
embodied freedom of the press and the right to public The first is absolute but, in the nature of things, the
information. It also approves of media’s exalted power to second cannot be. Conduct remains subject to regulation
provide the most accurate and comprehensive means of for the protection of society. The freedom to act must
conveying the proceedings to the public and in have appropriate definitions to preserve the enforcement
acquainting the public with the judicial process in action; of that protection. In every case, the power to regulate
nevertheless, within the courthouse, the overriding must be so exercised, in attaining a permissible end, as
consideration is still the paramount right of the accused to not to unduly infringe on the protected freedom.
due process which must never be allowed to suffer
diminution in its constitutional proportions. Justice Clark Whence, even the exercise of religion may be
thusly pronounced, “while a maximum freedom must be regulated, at some slight inconvenience, in order that the
allowed the press in carrying out the important function of State may protect its citizens from injury. X x x
informing the public in a democratic society, its exercise
must necessarily be subject to the maintenance of It does not follow, therefore, from the
absolute fairness in the judicial process.” constitutional guarantees of the free exercise of religion
that everything which may be so called can be tolerated .
Xxx It has been said that a law advancing a legitimate
governmental interest is not necessarily invalid as one
The Integrated Bar of the Philippines x x x interfering with the “free exercise” of religion merely
expressed its own concern on the live television and radio because it also incidentally has a detrimental effect on the
coverage of the criminal trials of Mr. Estrada; to adherents of one or more religion . (Centeno v.
paraphrase: Live television and radio coverage can negate Villalon-Pornillos, 236 SCRA 197, Sept. 1, 1994
the rule on exclusion of witnesses during the hearings [Regalado])
intended to assure a fair trial; at stake in the criminal trial
is not only the life and liberty of the accused but the very 213. Discuss why the Gerona ruling (justifying the
credibility of the Philippine criminal justice system, and expulsion from public schools of children of Jehovah’s
live television and radio coverage of the trial could allow Witnesses who refuse to salute the flag and sing the
the “hooting throng” to arrogate unto themselves the task national anthem during flag ceremony as prescribed
of judging the guilt of the accused, such that the verdict by the Flag Salute Law) should be abandoned.
of the court will be acceptable only if popular; and live
television and radio coverage of the trial will not subserve Held: Our task here is extremely difficult, for the
the ends of justice but will only pander to the desire for 30-year old decision of this court in Gerona upholding the
publicity of a few grandstanding lawyers. flag salute law and approving the expulsion of students
who refuse to obey it, is not lightly to be trifled with.
Xxx
It is somewhat ironic however, that after the
Unlike other government offices, courts do not Gerona ruling had received legislative cachet by its
express the popular will of the people in any sense which, incorporation in the Administrative Code of 1987, the
instead, are tasked to only adjudicate controversies on the present Court believes that the time has come to
basis of what alone is submitted before them. A trial is reexamine it. The idea that one may be compelled to
not a free trade of ideas. Nor is a competing market of salute the flag, sing the national anthem, and recite the
thoughts the known test of truth in a courtroom. (Re: patriotic pledge, during a flag ceremony on pain of being
Request Radio-TV coverage of the Trial in the dismissed from one’s job or of being expelled from school,
Sandiganbayan of the Plunder Cases against the is alien to the conscience of the present generation of
former President Joseph E. Estrada, A.M. No. 01-4- Filipinos who cut their teeth on the Bill of Rights which
03-SC, June 29, 2001, En Banc [Vitug]) guarantees their rights to free speech (The flag salute,
singing the national anthem and reciting the patriotic
pledge are all forms of utterances.) and the free exercise
Freedom of Religion of religious profession and worship.

212. Discuss the two aspects of freedom of religion. Religious freedom is a fundamental right which is
entitled to the highest priority and the amplest protection

96
among human rights, for it involves the relationship of Moreover, the expulsion of members of Jehovah’s
man to his Creator. Witnesses from the schools where they are enrolled will
violate their right as Philippine citizens, under the 1987
Xxx Constitution, to receive free education, for it is the duty of
the State to “protect and promote the right of all citizens
Petitioners stress x x x that while they do not take to quality education x x x and to make such education
part in the compulsory flag ceremony, they do not engage accessible to all” (Sec. 1, Art. XIV).
in “external acts” or behavior that would offend their
countrymen who believe in expressing their love of In Victoriano v. Elizalde Rope Workers’ Union, we
country through the observance of the flag ceremony. upheld the exemption of members of the Iglesia Ni Cristo,
They quietly stand at attention during the flag ceremony from the coverage of a closed shop agreement between
to show their respect for the rights of those who choose their employer and a union because it would violate the
to participate in the solemn proceedings. Since they do teaching of their church not to join any labor group x x x.
not engage in disruptive behavior, there is no warrant for
their expulsion. Xxx

“The sole justification for a prior restraint We hold that a similar exemption may be
or limitation on the exercise of religious freedom accorded to the Jehovah’s Witnesses with regard to the
(according to the late Chief Justice Claudio observance of the flag ceremony out of respect for their
Teehankee in his dissenting opinion in German v. religious beliefs, however “bizarre” those beliefs may
Barangan, 135 SCRA 514, 517) is the existence of seem to others. Nevertheless, their right not to
a grave and present danger of a character both participate in the flag ceremony does not give them a
grave and imminent, of a serious evil to public right to disrupt such patriotic exercises. Paraphrasing the
safety, public morals, public health or any other warning cited by this Court in Non v. Dames II, while the
legitimate public interest, that the State has a highest regard must be afforded their right to the free
right (and duty) to prevent.” Absent such a exercise of their religion, “this should not be taken to
threat to public safety, the expulsion of the mean that school authorities are powerless to discipline
petitioners from the schools is not justified. them” if they should commit breaches of the peace by
actions that offend the sensibilities, both religious and
The situation that the Court directly predicted in patriotic, of other persons. If they quietly stand at
Gerona that: attention during the flag ceremony while their classmates
and teachers salute the flag, sing the national anthem and
“[T]he flag ceremony will become a thing recite the patriotic pledge, we do not see how such
of the past or perhaps conducted with very few conduct may possibly disturb the peace, or pose “a grave
participants, and the time will come when we and present danger of a serious evil to public safety,
would have citizens untaught and uninculcated in public morals, public health or any other legitimate public
and not imbued with reverence for the flag and interest that the State has a right (and duty) to prevent .”
love of country, admiration for national heroes, (Ebralinag v. The Division Superintendent of
and patriotism – a pathetic, even tragic situation, Schools of Cebu, 219 SCRA 256, 269-273, March 1,
and all because a small portion of the school 1993, En Banc [Grino-Aquino])
population imposed its will, demanded and was
granted an exemption.” 214. A pre-taped TV program of the Iglesia Ni Cristo
(INC) was submitted to the MTRCB for review. The
has not come to pass. We are not persuaded that by latter classified it as “rated X” because it was shown
exempting the Jehovah’s Witnesses from saluting the flag, to be attacking another religion. The INC protested
singing the national anthem and reciting the patriotic by claiming that its religious freedom is per se beyond
pledge, this religious group which admittedly comprises a review by the MTRCB. Should this contention be
“small portion of the school population” will shake up our upheld?
part of the globe and suddenly produce a nation
“untaught and uninculcated in and unimbued with Held: The Iglesia Ni Cristo's postulate that its
reverence for the flag, patriotism, love of country and religious freedom is per se beyond review by the MTRCB
admiration for national heroes. After all, what the should be rejected. Its public broadcast on TV of its
petitioners seek only is exemption from the flag religious programs brings it out of the bosom of internal
ceremony, not exclusion from the public schools where belief. Television is a medium that reaches even the eyes
they may study the Constitution, the democratic way of and ears of children. The exercise of religious freedom
life and form of government, and learn not only the arts, can be regulated by the State when it will bring about the
sciences, Philippine history and culture but also receive clear and present danger of a substantive evil which the
training for a vocation or profession and be taught the State is duty-bound to prevent, i.e., serious detriment to
virtues of “patriotism, respect for human rights, the more overriding interest of public health, public
appreciation for national heroes, the rights and duties of morals, or public welfare. A laissez faire policy on the
citizenship, and moral and spiritual values (Sec. 3[2], Art. exercise of religion can be seductive to the liberal mind
XIV, 1987 Constitution) as part of the curricula. Expelling but history counsels the Court against its blind adoption as
or banning the petitioners from Philippine schools will religion is and continues to be a volatile area of concern in
bring about the very situation that this Court had feared in our society today. “For sure, we shall continue to subject
Gerona. Forcing a small religious group, through the iron any act pinching the space for the free exercise of religion
hand of the law, to participate in a ceremony that violates to a heightened scrutiny but we shall not leave its rational
their religious beliefs, will hardly be conducive to love of exercise to the irrationality of man. For when religion
country or respect for duly constituted authorities. divides and its exercise destroys, the State should not
stand still.” (Iglesia Ni Cristo v. CA, 259 SCRA 529,
Xxx July 26, 1996 [Puno])

97
215. Did the MTRCB act correctly when it rated “X” the would impose a condition on the exercise of a
Iglesia Ni Cristo's pre-taped TV program simply constitutional right. It is for the same reason that
because it was found to be “attacking” another religious rallies are exempted from the requirement of
religion? prior permit for public assemblies and other uses of public
parks and streets (B.P. Blg. 880, Sec. 3[a]). To read the
Held: The MTRCB may disagree with the Decree, therefore, as including within its reach
criticisms of other religions by the Iglesia Ni Cristo but solicitations for religious purposes would be to construe it
that gives it no excuse to interdict such criticisms, in a manner that it violates the Free Exercise of Religion
however unclean they may be. Under our constitutional Clause of the Constitution x x x. (Concurring Opinion,
scheme, it is not the task of the State to favor any religion Mendoza, V.V., J., in Centeno v. Villalon-Pornillos,
by protecting it against an attack by another religion. 236 SCRA 197, Sept. 1, 1994)
Religious dogma and beliefs are often at war and to
preserve peace among their followers, especially the 217. What is a purely ecclesiastical affair to which the
fanatics, the establishment clause of freedom of religion State can not meddle?
prohibits the State from leaning towards any religion. Vis-
à-vis religious differences, the State enjoys no banquet of Held: An ecclesiastical affair is “one that
options. Neutrality alone is its fixed and immovable concerns doctrine, creed, or form of worship of the
stance. In fine, the MTRCB cannot squelch the speech of church, or the adoption and enforcement within a
the INC simply because it attacks another religion. In a religious association of needful laws and regulations for
State where there ought to be no difference between the the government of the membership, and the power of
appearance and the reality of freedom of religion, the excluding from such associations those deemed not
remedy against bad theology is better theology. The worthy of membership.” Based on this definition, an
bedrock of freedom of religion is freedom of thought and ecclesiastical affair involves the relationship between the
it is best served by encouraging the marketplace of church and its members and relate to matters of faith,
dueling ideas. When the luxury of time permits, the religious doctrines, worship and governance of the
marketplace of ideas demands that speech should be met congregation. To be concrete, examples of this so-called
by more speech for it is the spark of opposite speech, the ecclesiastical affairs to which the State cannot meddle are
heat of colliding ideas, that can fan the embers of truth. proceedings for excommunication, ordinations of religious
(Iglesia Ni Cristo v. CA, 259 SCRA 529, July 26, ministers, administration of sacraments and other
1996 [Puno]) activities with attached religious significance. (Pastor
Dionisio V. Austria v. NLRC, G.R. No. 124382, Aug.
216. Is solicitation for the construction of a church 16, 1999, 1st Div. [Kapunan])
covered by P.D. No. 1564 and, therefore, punishable
if done without the necessary permit for solicitation 218. Petitioner is a religious minister of the Seventh
from the DSWD? Day Adventist (SDA). He was dismissed because of
alleged misappropriation of denominational funds,
Held: First. Solicitation of contributions for the willful breach of trust, serious misconduct, gross and
construction of a church is not solicitation for “charitable habitual neglect of duties and commission of an
or public welfare purpose” but for a religious purpose, and offense against the person of his employer’s duly
a religious purpose is not necessarily a charitable or public authorized representative. He filed an illegal
welfare purpose. A fund campaign for the construction or termination case against the SDA before the labor
repair of a church is not like fund drives for needy families arbiter. The SDA filed a motion to dismiss invoking
or victims of calamity or for the construction of a civic the doctrine of separation of Church and State.
center and the like. Like solicitation of subscription to Should the motion be granted?
religious magazines, it is part of the propagation of
religious faith or evangelization. Such solicitation calls Held: Where what is involved is the relationship
upon the virtue of faith, not of charity, save as those of the church as an employer and the minister as an
solicited for money or aid may not belong to the same employee and has no relation whatsoever with the
religion as the solicitor. Such solicitation does not engage practice of faith, worship or doctrines of the church, i.e.,
the philanthropic as much as the religious fervor of the the minister was not excommunicated or expelled from
person who is solicited for contribution. the membership of the congregation but was terminated
from employment, it is a purely secular affair.
Second. The purpose of the Decree is to protect Consequently, the suit may not be dismissed invoking the
the public against fraud in view of the proliferation of fund doctrine of separation of church and the state. (Pastor
campaigns for charity and other civic projects. On the Dionisio V. Austria v. NLRC, G.R. No. 124382, Aug.
other hand, since religious fund drives are usually 16, 1999, 1st Div. [Kapunan])
conducted among those belonging to the same religion,
the need for public protection against fraudulent
solicitations does not exist in as great a degree as does The Right of the People to Information on Matters
the need for protection with respect to solicitations for of Public Concern
charity or civic projects as to justify state regulation.
219. Discuss the scope of the right to information on
Third. To require a government permit before matters of public concern.
solicitation for religious purpose may be allowed is to lay a
prior restraint on the free exercise of religion. Such Held: In Valmonte v. Belmonte, Jr., the Court
restraint, if allowed, may well justify requiring a permit emphasized that the information sought must be “matters
before a church can make Sunday collections or enforce of public concern,” access to which may be limited by law.
tithing. But in American Bible Society v. City of Manila, we Similarly, the state policy of full public disclosure extends
precisely held that an ordinance requiring payment of a only to “transactions involving public interest” and may
license fee before one may engage in business could not also be “subject to reasonable conditions prescribed by
be applied to the appellant's sale of bibles because that law.” As to the meanings of the terms “public interest”

98
and “public concern,” the Court, in Legaspi v. Civil Service
Commission, elucidated: Held: In Chavez v. PCGG (299 SCRA 744,
Dec. 9, 1998 [Panganiban]), the SC enumerated the
“In determining whether or not a recognized restrictions to the right of the people to
particular information is of public concern there is information on matters of public concern, as follows:
no rigid test which can be applied. ‘Public
concern’ like ‘public interest’ is a term that eludes 1) National security matters and intelligence
exact definition. Both terms embrace a broad information. This jurisdiction recognizes the
spectrum of subjects which the public may want common law holding that there is a
to know, either because these directly affect their governmental privilege against public
lives, or simply because such matters naturally disclosure with respect to state secrets
arouse the interest of an ordinary citizen. In the regarding military, diplomatic and other
final analysis, it is for the courts to determine on a national security matters. Likewise,
case by case basis whether the matter at issue is information on inter-government exchanges
of interest or importance, as it relates to or affects prior to the conclusion of treaties and
the public.” executive agreements may be subject to
reasonable safeguards for the sake of national
Considered a public concern in the above- interest;
mentioned case was the “legitimate concern of citizens to 2) Trade or industrial secrets (pursuant to the
ensure that government positions requiring civil service Intellectual Property Code [R.A. No. 8293,
eligibility are occupied only by persons who are eligibles.” approved on June 6, 1997] and other related
So was the need to give the general public adequate laws) and banking transactions (pursuant to
notification of various laws that regulate and affect the the Secrecy of Bank Deposits Act [R.A. No.
actions and conduct of citizens, as held in Tanada. 1405, as amended]);
Likewise did the “public nature of the loanable funds of 3) Criminal matters, such as those relating to the
the GSIS and the public office held by the alleged apprehension, the prosecution and the
borrowers (members of the defunct Batasang Pambansa)” detention of criminals, which courts may not
qualify the information sought in Valmonte as matters of inquire into prior to such arrest, detention and
public interest and concern. In Aquino-Sarmiento v. prosecution;
Morato, the Court also held that official acts of public 4) Other confidential information. The Ethical
officers done in pursuit of their official functions are public Standards Act (R.A. No. 6713, enacted on
in character; hence, the records pertaining to such official February 20, 1989) further prohibits public
acts and decisions are within the ambit of the officials and employees from using or
constitutional right of access to public records. divulging “confidential or classified
information officially known to them by
Under Republic Act No. 6713, public officials and reason of their office and not made available
employees are mandated to “provide information on their to the public.” (Sec. 7[c], ibid.) Other
policies and procedures in clear and understandable acknowledged limitations to information
language, [and] ensure openness of information, public access include diplomatic correspondence,
consultations and hearing whenever appropriate x x x,” closed door Cabinet meetings and executive
except when “otherwise provided by law or when required sessions of either house of Congress, as well
by the public interest.” In particular, the law mandates as the internal deliberations of the Supreme
free public access, at reasonable hours, to the annual Court.
performance reports of offices and agencies of
government and government-owned or controlled 221. Is the recovery of the alleged ill-gotten wealth of
corporations; and the statements of assets, liabilities and the Marcoses a matter of public concern subject to
financial disclosures of all public officials and employees. this right?

In general, writings coming into the hands of Held: With such pronouncements of our
public officers in connection with their official functions government x x x there is no doubt that the recovery of
must be accessible to the public, consistent with the policy the Marcoses’ alleged ill-gotten wealth is a matter of
of transparency of governmental affairs. This principle is public concern and imbued with public interest. We may
aimed at affording the people an opportunity to determine also add that “ill-gotten wealth” refers to assets and
whether those to whom they have entrusted the affairs of properties purportedly acquired, directly or indirectly, by
the government are honestly, faithfully and competently former President Marcos, his immediate family, relatives
performing their functions as public servants. Undeniably, and close associates through or as a result of their
the essence of democracy lies in the free-flow of thought; improper or illegal use of government funds or properties;
but thoughts and ideas must be well-informed so that the or their having taken undue advantage of their public
public would gain a better perspective of vital issues office; or their use of powers, influences or relationships,
confronting them and, thus, be able to criticize as well as “resulting in their unjust enrichment and causing grave
participate in the affairs of the government in a damage and prejudice to the Filipino people and the
responsible, reasonable and effective manner. Certainly, Republic of the Philippines.” Clearly, the assets and
it is by ensuring an unfettered and uninhibited exchange properties referred to supposedly originated from the
of ideas among a well-informed public that a government government itself. To all intents and purposes, therefore,
remains responsive to the changes desired by the people . they belong to the people. As such, upon reconveyance
(Chavez v. PCGG, 299 SCRA 744, Dec. 9, 1998, they will be returned to the public treasury, subject only
[Panganiban]) to the satisfaction of positive claims of certain persons as
may be adjudged by competent courts. Another declared
220. What are some of the recognized restrictions to overriding consideration for the expeditious recovery of ill-
the right of the people to information on matters of gotten wealth is that it may be used for national economic
public concern? recovery.

99
circulars, rules, and regulations, not through
We believe the foregoing disquisition settles the collective bargaining agreements.”
question of whether petitioner has a right to respondents'
disclosure of any agreement that may be arrived at After delving into the intent of the framers of the
concerning the Marcoses’ purported ill-gotten wealth. Constitution, the Court affirmed the above rule in Social
(Chavez v. PCGG, 299 SCRA 744, Dec. 9, 1998 Security System Employees Association (SSSEA) v. Court
[Panganiban]) of Appeals and explained:

“Government employees may, therefore,


Freedom of Association through their unions or associations, either
petition the Congress for the betterment of the
222. Does the right of civil servants to organize include terms and conditions of employment which are
their right to strike? Clarify. within the ambit of legislation or negotiate with
the appropriate government agencies for the
Held: Specifically, the right of civil servants to improvement of those which are not fixed by law.
organize themselves was positively recognized in If there be any unresolved grievances, the dispute
Association of Court of Appeals Employees (ACAE) v. may be referred to the Public Sector Labor-
Ferrer-Calleja. But, as in the exercise of the rights of free Management Council for appropriate action. But
expression and of assembly, there are standards for employees in the civil service may not resort to
allowable limitations such as the legitimacy of the strikes, walkouts and other temporary work
purposes of the association, the overriding considerations stoppages, like workers in the private sector, to
of national security and the preservation of democratic pressure the Government to accede to their
institutions. demands. As now provided under Sec. 4, Rule III
of the Rules and Regulations to Govern the
As regards the right to strike, the Constitution Exercise of the Right of Government Employees to
itself qualifies its exercise with the proviso “in accordance Self-Organization, which took effect after the
with law.” This is a clear manifestation that the state instant dispute arose, ‘[t]he terms and conditions
may, by law, regulate the use of this right, or even deny of employment in the government, including any
certain sectors such right. Executive Order No. 180 which political subdivision or instrumentality thereof and
provides guidelines for the exercise of the right of government-owned and controlled corporations
government workers to organize, for instance, implicitly with original charters are governed by law and
endorsed an earlier CSC circular which “enjoins under pain employees therein shall not strike for the purpose
of administrative sanctions, all government officers and of securing changes [thereto].’”
employees from staging strikes, demonstrations, mass (Jacinto v. Court of Appeals, 281 SCRA 657,
leaves, walkouts and other forms of mass action which Nov. 14, 1997, En Banc [Panganiban])
will result in temporary stoppage or disruption of public
service” (CSC Memorandum Circular No. 6, s. 1987, dated 223. Petitioners public school teachers walked out of
April 21, 1987) by stating that the Civil Service law and their classes and engaged in mass actions during
rules governing concerted activities and strikes in the certain dates in September 1990 protesting the
government service shall be observed. alleged unlawful withholding of their salaries and
other economic benefits. They also raised national
It is also settled in jurisprudence that, in general, issues, such as the removal of US bases and the
workers in the public sector do not enjoy the right to repudiation of foreign debts, in their mass actions.
strike. Alliance of Concerned Government Workers v. They refused to return to work despite orders to do so
Minister of Labor and Employment rationalized the and subsequently were found guilty of conduct
proscription thus: prejudicial to the best interests of the service for
having absented themselves without proper authority,
“The general rule in the past and up to from their schools during regular school days, and
the present is that the ‘terms and conditions of penalized. They denied that they engaged in “strike”
employment in the Government, including any but claimed that they merely exercised a
political subdivision or instrumentality thereof are constitutionally guaranteed right – the right to
governed by law.’ x x x. Since the terms and peaceably assemble and petition the government for
conditions of government employment are fixed redress of grievances - and, therefore, should not
by law, government workers cannot use the same have been penalized. Should their contention be
weapons employed by the workers in the private upheld?
sector to secure concessions from their
employers. The principle behind labor unionism in Held: Petitioners, who are public schoolteachers
private industry is that industrial peace cannot be and thus government employees, do not seek to establish
secured through compulsion by law. Relations that they have a right to strike. Rather, they tenaciously
between private employers and their employees insist that their absences during certain dates in
rest on an essentially voluntary basis. Subject to September 1990 were a valid exercise of their
the minimum requirements of wage laws and constitutional right to engage in peaceful assembly to
other labor and welfare legislation, the terms and petition the government for a redress of grievances. They
conditions of employment in the unionized private claim that their gathering was not a strike, therefore, their
sector are settled through the process of participation therein did not constitute any offense.
collective bargaining. In government MPSTA v. Laguio and ACT v. Carino, in which this Court
employment, however, it is the legislature and, declared that “these ‘mass actions’ were to all intents and
where properly given delegated power, the purposes a strike; they constituted a concerted and
administrative heads of government which fix the unauthorized stoppage of, or absence from, work which it
terms and conditions of employment. And this is was the teachers’ duty to perform, undertaken for
effected through statutes or administrative essentially economic reasons,” should not principally

100
resolve the present case, as the underlying facts are of the 1973 Constitution, which have been greatly
allegedly not identical. enhanced and expanded in the 1987 Constitution by
placing them under a separate Article (Article XIII). The
Strike, as defined by law, means any temporary Article on Social Justice was aptly described as the “heart
stoppage of work done by the concerted action of of the new Charter” by the President of the 1986
employees as a result of an industrial or labor dispute . A Constitutional Commission, retired Justice Cecilia Munoz
labor dispute includes any controversy or matter Palma. Social justice is identified with the broad scope of
concerning terms and conditions of employment; or the the police power of the state and requires the extensive
association or representation of persons in negotiating, use of such power. x x x.
fixing, maintaining, changing or arranging the terms and
conditions of employment, regardless of whether the The constitutional prohibition against impairing
disputants stand in the proximate relation of employers contractual obligations is not absolute and is not to be
and employees. With these premises, we now evaluate read with literal exactness. It is restricted to contracts
the circumstances of the instant petition. with respect to property or some object of value and
which confer rights that may be asserted in a court of
It cannot be denied that the mass action or justice; it has no application to statutes relating to public
assembly staged by the petitioners resulted in the non- subjects within the domain of the general legislative
holding of classes in several public schools during the powers of the State and involving the public rights and
corresponding period. Petitioners do not dispute that the public welfare of the entire community affected by it. It
grievances for which they sought redress concerned the does not prevent a proper exercise by the State of its
alleged failure of public authorities - essentially, their police power by enacting regulations reasonably necessary
“employers” - to fully and justly implement certain laws to secure the health, safety, morals, comfort, or general
and measures intended to benefit them materially x x x. welfare of the community, even though contracts may
And probably to clothe their action with permissible thereby be affected, for such matters cannot be placed by
character, they also raised national issues such as the contract beyond the power of the State to regulate and
removal of the U.S. bases and the repudiation of foreign control them.
debt. In Balingasan v. Court of Appeals, however, this
Court said that the fact that the conventional term “strike” Verily, the freedom to contract is not absolute; all
was not used by the participants to describe their contracts and all rights are subject to the police power of
common course of action was insignificant, since the the State and not only may regulations which affect them
substance of the situation, and not its appearance, was be established by the State, but all such regulations must
deemed controlling. be subject to change from time to time, as the general
well-being of the community may require, or as the
Moreover, the petitioners here x x x were not circumstances may change, or as experience may
penalized for the exercise of their right to assemble demonstrate the necessity. And under the Civil Code,
peacefully and to petition the government for a redress of contracts of labor are explicitly subject to the police power
grievances. Rather, the Civil Service Commission found of the State because they are not ordinary contracts but
them guilty of conduct prejudicial to the best interest of are impressed with public interest. Article 1700 thereof
the service for having absented themselves without expressly provides:
proper authority, from their schools during regular school
days, in order to participate in the mass protest, their Art. 1700. The relations between capital
absence ineluctably resulting in the non-holding of classes and labor are not merely contractual. They are so
and in the deprivation of students of education, for which impressed with public interest that labor contracts
they were responsible. Had petitioners availed must yield to the common good. Therefore, such
themselves of their free time - recess, after classes, contracts are subject to the special laws on labor
weekends or holidays - to dramatize their grievances and unions, collective bargaining, strikes and lockouts,
to dialogue with the proper authorities within the bounds closed shop, wages, working conditions, hours of
of law, no one - not the DECS, the CSC or even this Court labor and similar subjects.
- could have held them liable for the valid exercise of their
constitutionally guaranteed rights. As it was, the The challenged resolution and memorandum
temporary stoppage of classes resulting from their activity circular being valid implementations of E.O. No. 797
necessarily disrupted public services, the very evil sought (Creating the POEA), which was enacted under the police
to be forestalled by the prohibition against strikes by power of the State, they cannot be struck down on the
government workers. Their act by their nature was ground that they violate the contract clause. To hold
enjoined by the Civil Service law, rules and regulations, otherwise is to alter long-established constitutional
for which they must, therefore, be made answerable. doctrine and to subordinate the police power to the
(Jacinto v. CA, 281 SCRA 657, Nov. 14, 1997, En contract clause. (The Conference of Maritime
Banc [Panganiban]) Manning Agencies, Inc. v. POEA, 243 SCRA 666,
April 21, 1995 [Davide, Jr.])

The Non-Impairment Clause 2. Petitioners pray that the present action should
be barred, because private respondents have voluntarily
224. Is the constitutional prohibition against impairing executed quitclaims and releases and received their
contractual obligations absolute? separation pay. Petitioners claim that the present suit is a
“grave derogation of the fundamental principle that
Held: 1. Nor is there merit in the claim that the obligations arising from a valid contract have the force of
resolution and memorandum circular violate the contract law between the parties and must be complied with in
clause of the Bill of Rights. good faith.”

The executive order creating the POEA was The Court disagrees. Jurisprudence holds that
enacted to further implement the social justice provisions the constitutional guarantee of non-impairment of

101
contract is subject to the police power of the state and to to enjoin the respondent Secretary from receiving,
reasonable legislative regulations promoting health, accepting, processing, renewing or approving new timber
morals, safety and welfare. Not all quitclaims are per se license for, save in cases of renewal, no contract would
invalid or against public policy, except (1) where there is have as yet existed in the other instances. Moreover, with
clear proof that the waiver was wangled from an respect to renewal, the holder is not entitled to it as a
unsuspecting or gullible person, or (2) where the terms of matter of right. (Oposa v. Factoran, Jr., 224 SCRA
settlement are unconscionable on their face. In these 792 [1993])
cases, the law will step in to annul the questionable
transactions. Such quitclaim and release agreements are 5. Anent petitioners’ contention that the forcible
regarded as ineffective to bar the workers from claiming refund of incentive benefits is an unconstitutional
the full measure of their legal rights. impairment of a contractual obligation, suffice it to state
that “[n]ot all contracts entered into by the government
In the case at bar, the private respondents agreed will operate as a waiver of its non-suability; distinction
to the quitclaim and release in consideration of their must be made between its sovereign and proprietary acts.
separation pay. Since they were dismissed allegedly for The acts involved in this case are governmental. Besides,
business losses, they are entitled to separation pay under the Court is in agreement with the Solicitor General that
Article 283 of the Labor Code. And since there was thus the incentive pay or benefit is in the nature of a bonus
no extra consideration for the private respondents to give which is not a demandable or enforceable obligation.
up their employment, such undertakings cannot be (Blaquera v. Alcala, 295 SCRA 366, 446, Sept. 11,
allowed to bar the action for illegal dismissal. (Bogo- 1998, En Banc [Purisima])
Medellin Sugarcane Planters Association, Inc. v.
NLRC, 296 SCRA 108, 124, [Panganiban])
The In-Custodial Investigation Rights of an
3. Only slightly less abstract but nonetheless Accused Person
hypothetical is the contention of CREBA that the
imposition of the VAT on the sales and leases of real 225. State the procedure, guidelines and duties which
estate by virtue of contracts entered prior to the the arresting, detaining, inviting, or investigating
effectivity of the law would violate the constitutional officer or his companions must do and observe at the
provision that “No law impairing the obligation of time of making an arrest and again at and during the
contracts shall be passed.” It is enough to say that the time of the custodial interrogation.
parties to a contract cannot, through the exercise of
prophetic discernment, fetter the exercise of the taxing Held: Lastly, considering the heavy penalty of
power of the State. For not only are existing laws read death and in order to ensure that the evidence against
into contracts in order to fix obligations as between an accused were obtained through lawful means, the
parties, but the reservation of essential attributes of Court, as guardian of the rights of the people lays down
sovereign power is also read into contracts as a basic the procedure, guidelines and duties which the arresting,
postulate of the legal order. The policy of protecting detaining, inviting, or investigating officer or his
contracts against impairment presupposes the companions must do and observe at the time of making
maintenance of a government which retains adequate an arrest and again at and during the time of the custodial
authority to secure the peace and good order of society. interrogation in accordance with the Constitution,
jurisprudence and Republic Act No. 7438 (An Act Defining
In truth, the Contract Clause has never been Certain Rights of Person Arrested, Detained or Under
thought as a limitation on the exercise of the State's Custodial Investigation as well as the Duties of the
power of taxation save only where a tax exemption has Arresting, Detaining, and Investigating Officers and
been granted for a valid consideration. x x x. Providing Penalties for Violations Thereof). It is high-time
(Tolentino v. Secretary of Finance, 235 SCRA 630, to educate our law-enforcement agencies who neglect
685-686, Aug. 25, 1994, En Banc [Mendoza]) either by ignorance or indifference the so-called Miranda
rights which had become insufficient and which the Court
4. Since timber licenses are not contracts, the must update in the light of new legal developments:
non-impairment clause x x x cannot be invoked.
1) The person arrested, detained, invited or
X x x, even if it is to be assumed that the same under custodial investigation must be
are contracts, the instant case does not involve a law or informed in a language known to and
even an executive issuance declaring the cancellation or understood by him of the reason for the
modification of existing timber licenses. Hence, the non- arrest and he must be shown the warrant of
impairment clause cannot as yet be invoked. arrest, if any. Every other warnings,
Nevertheless, granting further that a law has actually information or communication must be in a
been passed mandating cancellations or modifications, the language known to and understood by said
same cannot still be stigmatized as a violation of the non- person;
impairment clause. This is because by its very nature and 2) He must be warned that he has a right to
purpose, such a law could have only been passed in the remain silent and that any statement he
exercise of the police power of the state for the purpose makes may be used as evidence against him;
of advancing the right of the people to a balanced and 3) He must be informed that he has the right to
healthful ecology, promoting their health and enhancing be assisted at all times and have the
their general welfare. x x x. presence of an independent and competent
lawyer, preferably of his own choice;
In short, the non-impairment clause must yield to 4) He must be informed that if he has no lawyer
the police power of the state. or cannot afford the services of a lawyer, one
will be provided for him; and that a lawyer
Finally, it is difficult to imagine x x x how the non- may also be engaged by any person in his
impairment clause could apply with respect to the prayer behalf, or may be appointed by the court

102
upon petition of the person arrested or one contemplates effective communication which results in the
acting on his behalf; subject’s understanding of what is conveyed. Since it is
5) That whether or not the person arrested has comprehension that is sought to be attained, the degree
a lawyer, he must be informed that no of explanation required will necessarily vary and depend
custodial investigation in any form shall be on the education, intelligence, and other relevant personal
conducted except in the presence of his circumstances of the person undergoing investigation. In
counsel of after a valid waiver has been further ensuring the right to counsel, it is not enough that
made; the subject is informed of such right; he should also be
6) The person arrested must be informed that, asked if he wants to avail of the same and should be told
at any time, he has the right to communicate that he could ask for counsel if he so desired or that one
or confer by the most expedient means - could be provided him at his request. If he decides not to
telephone, radio, letter or messenger - with retain a counsel of his choice or avail of one to be
his lawyer (either retained or appointed), any provided for him and, therefore, chooses to waive his
member of his immediate family, or any right to counsel, such waiver, to be valid and effective,
medical doctor, priest or minister chosen by must still be made with the assistance of counsel, who,
him or by any one from his immediate family under prevailing jurisprudence, must be a lawyer.
or by his counsel, or be visited by/confer with (People v. Canoy, 328 SCRA 385, March 17, 2000,
duly accredited national or international non- 1st Div. [Davide, CJ])
government organization. It shall be the
responsibility of the officer to ensure that this 227. What is the meaning of “competent counsel”
is accomplished; under Section 12 of the Bill of Rights?
7) He must be informed that he has the right to
waive any of said rights provided it is made Held: The meaning of “competent counsel” was
voluntarily, knowingly and intelligently and explained in People v. Deniega as follows:
ensure that he understood the same;
8) In addition, if the person arrested waives his “x x x [T]he lawyer called to be present
right to a lawyer, he must be informed that it during such investigation should be as far as
must be done in writing and in the presence reasonably possible, the choice of the individual
of counsel, otherwise, he must be warned undergoing questioning. If the lawyer were one
that the waiver is void even if he insist on his furnished in the accused’s behalf, it is important
waiver and chooses to speak; that he should be competent and independent,
9) That the person arrested must be informed i.e., that he is willing to fully safeguard the
that he may indicate in any manner at any constitutional rights of the accused, as
time or stage of the process that he does not distinguished from one who would merely be
wish to be questioned with warning that once giving a routine, peremptory and meaningless
he makes such indication, the police may not recital of the individual’s rights. In People v.
interrogate him if the same had not yet Basay (219 SCRA 404, 418), this Court stressed
commenced, or the interrogation must cease that an accused’s right to be informed of the right
if it has already begun; to remain silent and to counsel ‘contemplates the
10) The person arrested must be informed that transmission of meaningful information rather
his initial waiver of his right to remain silent, than just the ceremonial and perfunctory
the right to counsel or any of his rights does recitation of an abstract constitutional principle.’
not bar him from invoking it at any time
during the process, regardless of whether he “Ideally therefore, a lawyer engaged for
may have answered some questions or an individual facing custodial investigation (if the
volunteered some statements; latter could not afford one) ‘should be engaged by
11) He must also be informed that any statement the accused (himself), or by the latter’s relative or
or evidence, as the case may be, obtained in person authorized by him to engage an attorney
violation of any of the foregoing, whether or by the court, upon proper petition of the
inculpatory or exculpatory, in whole or in accused or person authorized by the accused to
part, shall be admissible in evidence. file such petition.’ Lawyers engaged by the
(People v. Mahinay, 302 SCRA 455, Feb. 1, 1999, police, whatever testimonials are given as proof of
En Banc [Per Curiam]) their probity and supposed independence, are
generally suspect, as in many areas, the
226. Explain the kind of information that is required to relationship between lawyers and law
be given by law enforcement officers to suspect enforcement authorities can be symbiotic.
during custodial investigation.
“x x x The competent or independent
Held: [I]t is settled that one’s right to be lawyer so engaged should be present from the
informed of the right to remain silent and to counsel beginning to end, i.e., at all stages of the
contemplates the transmission of meaningful information interview, counseling or advising caution
rather just the ceremonial and perfunctory recitation of an reasonably at every turn of the investigation, and
abstract constitutional principle. It is not enough for the stopping the interrogation once in a while either
interrogator to merely repeat to the person under to give advice to the accused that he may either
investigation the provisions of Section 12, Article III of the continue, choose to remain silent or terminate the
1987 Constitution; the former must also explain the interview.”
effects of such provision in practical terms – e.g., what (People v. Espiritu, 302 SCRA 533, Feb. 2, 1999,
the person under investigation may or may not do – and 3rd Div. [Panganiban])
in a language the subject fairly understands. The right to
be informed carries with it a correlative obligation on the
part of the police investigator to explain, and

103
228. Can a PAO lawyer be considered an independent permission before interviewing him. The Supreme Court
counsel within the contemplation of Section 12, Article further ruled that appellant’s verbal confessions to the
III, 1987 Constitution? newsmen are not covered by Section 12(1) and (3) of
Article III of the Constitution and, therefore, admissible in
Held: In People v. Oracoy and People v. evidence.
Bandula, the SC has held that a PAO lawyer can be
considered an independent counsel within the 231. Discuss why lower court’s should act with extreme
contemplation of the Constitution considering that he is caution in admitting in evidence accused’s videotaped
not a special counsel, public or private prosecutor, counsel media confessions.
of the police, or a municipal attorney whose interest is
admittedly adverse to that of the accused-appellant. Held: Apropos the court a quo’s admission of
Thus, the assistance of a PAO lawyer satisfies the accused-appellant’s videotaped confession, we find such
constitutional requirement of a competent and admission proper. The interview was recorded on video
independent counsel for the accused. (People v. Bacor, and it showed accused-appellant unburdening his guilt
306 SCRA 522, April 30, 1999, 2nd Div. [Mendoza]) willingly, openly and publicly in the presence of newsmen.
Such confession does not form part of custodial
229. Is the confession of an accused given investigation as it was not given to police officers but to
spontaneously, freely and voluntarily to the Mayor media men in an attempt to elicit sympathy and
admissible in evidence, considering that the Mayor forgiveness from the public. Besides, if he had indeed
has “operational supervision and control” over the been forced into confessing, he could have easily sought
local police and may arguably be deemed a law succor from the newsmen who, in all likelihood, would
enforcement officer? have been sympathetic with him. X x x

Held: While it is true that a municipal mayor has X x x However, because of the inherent danger in
“operational supervision and control” over the local police the use of television as a medium for admitting one’s
and may arguably be deemed a law enforcement officer guilt, and the recurrence of this phenomenon in several
for purposes of applying Section 12(1) and (3) of Article cases, it is prudent that trial courts are reminded that
III of the Constitution, however, appellant’s confession to extreme caution must be taken in further admitting similar
the mayor was not made in response to any interrogation confessions. For in all probability, the police, with the
by the latter. In fact, the mayor did not question the connivance of unscrupulous media practitioners, may
appellant at all. No police authority ordered appellant to attempt to legitimize coerced extrajudicial confessions and
talk to the mayor. It was appellant himself who place them beyond the exclusionary rule by having an
spontaneously, freely and voluntarily sought the mayor for accused admit an offense on television. Such a situation
a private meeting. The mayor did not know that appellant would be detrimental to the guaranteed rights of the
was going to confess his guilt to him. When appellant accused and thus imperil our criminal justice system.
talked with the mayor as a confidant and not as a law
enforcement officer, his uncounselled confession to him We do not suggest that videotaped confessions
did not violate his constitutional rights. Thus, it has been given before media men by an accused with the
held that the constitutional procedures on custodial knowledge of and in the presence of police officers are
investigation do not apply to a spontaneous statement, impermissible. Indeed, the line between proper and
not elicited through questioning by the authorities, but invalid police techniques and conduct is a difficult one to
given in an ordinary manner whereby appellant orally draw, particularly in cases such as this where it is
admitted having committed the crime. What the essential to make sharp judgments in determining
Constitution bars is the compulsory disclosure of whether a confession was given under coercive physical or
incriminating facts or confessions. The rights under psychological atmosphere.
Section 12 are guaranteed to preclude the slightest use of
coercion by the State as would lead the accused to admit A word of caution then to lower courts: we should
something false, not to prevent him from freely and never presume that all media confessions described as
voluntarily telling the truth. (People v. Andan, 269 voluntary have been freely given. This type of confession
SCRA 95, March 3, 1997) always remains suspect and therefore should be
thoroughly examined and scrutinized. Detection of
230. Are confessions made in response to questions by coerced confessions is admittedly a difficult and arduous
news reporters admissible in evidence? task for the courts to make. It requires persistence and
determination in separating polluted confessions from
Answer: Yes. Confessions made in response to untainted ones. We have a sworn duty to be vigilant and
questions by news reporters, not by the police or any protective of the rights guaranteed by the Constitution.
other investigating officer, are admissible. In People v. (People v. Endino, 353 SCRA 307, Feb. 20, 2001,
Vizcarra, where the accused, under custody, gave 2nd Div. [Bellosillo])
spontaneous answers to a televised interview by several
press reporters in the office of the chief of the CIS, it was 232. Discuss the two kinds of involuntary or coerced
held that statements spontaneously made by a suspect to confessions under Section 12, Article III of the 1987
news reporters on a televised interview are deemed Constitution. Illustrate how the Court should
voluntary and are admissible in evidence. In People v. appreciate said involuntary or coerced confessions.
Andan, 269 SCRA 95, March 3, 1997, it was held that
appellant’s confessions to the news reporters were given Held: There are two kinds of involuntary or
free from any undue influence from the police authorities. coerced confessions treated in this constitutional
The news reporters acted as news reporters when they provision: (1) those which are the product of third degree
interviewed appellant. They were not acting under the methods such as torture, force, violence, threat,
direction and control of the police. They did not force intimidation, which are dealt with in paragraph 2 of
appellant to grant them an interview and reenact the Section 12, and (2) those which are given without the
commission of the crime. In fact, they asked his

104
benefit of Miranda warnings, which are the subject of found to be merely ceremonial and inadequate to transmit
paragraph 1 of the same Section 12. meaningful information to the suspect. Especially in this
case, care should have been scrupulously observed by the
Accused-appellant claims that his confession was police investigator that accused-appellant was specifically
obtained by force and threat. Aside from this bare asked these questions considering that he only finished
assertion, he has shown no proof of the use of force and the fourth grade of the elementary school. x x x
violence on him. He did not seek medical treatment nor
even a physical examination. His allegation that the fact Moreover, Article III, Section 12(1) requires that
that he was made to sign the confession five times is counsel assisting suspects in custodial interrogations be
proof that he refused to sign it. competent and independent. Here, accused-appellant
was assisted by Atty. De los Reyes, who, though
Xxx presumably competent, cannot be considered an
“independent counsel” as contemplated by the law for the
We discern no sign that the confession was reason that he was station commander of the WPD at the
involuntarily executed from the fact that it was signed by time he assisted accused-appellant. x x x.
accused-appellant five times.
This is error. As observed in People v. Bandula,
Xxx the independent counsel required by Article III, Section
12(1) cannot be special counsel, public or private
Extrajudicial confessions are presumed voluntary, prosecutor, municipal attorney, or counsel of the police
and, in the absence of conclusive evidence showing the whose interest is admittedly adverse to the accused. In
declarant’s consent in executing the same has been this case, Atty. De los Reyes, as PC Captain and Station
vitiated, such confession will be sustained. Commander of the WPD, was part of the police force who
could not be expected to have effectively and scrupulously
Moreover, the confession contains details that assisted accused-appellant in the investigation. To allow
only the perpetrator of the crime could have given. x x x. such a happenstance would render illusory the protection
It has been held that voluntariness of a confession may be given to the suspect during custodial investigation.
inferred from its being replete with details which could (People v. Obrero, 332 SCRA 190, 220 – 208, May
possibly be supplied only by the accused, reflecting 17, 2000, 2nd Div. [Mendoza])
spontaneity and coherence which cannot be said of a
mind on which violence and torture have been applied . 233. What are the requirements for an extra-judicial
When the details narrated in an extrajudicial confession confession of an accused to be admissible in
are such that they could not have been concocted by one evidence?
who did not take part in the acts narrated, where the
claim of maltreatment in the extraction of the confession Held: 1. In jurisprudence, no confession can be
is unsubstantiated and where abundant evidence exists admitted in evidence unless it is given:
showing that the statement was voluntarily executed, the
confession is admissible against the declarant. There is 1) Freely and voluntarily, without compulsion,
greater reason for finding a confession to be voluntary inducement or trickery;
where it is corroborated by evidence aliunde which 2) Knowingly based on an effective
dovetails with the essential facts contained in such communication to the individual under
confession. custodial investigation of his constitutional
rights; and
But what renders the confession of accused- 3) Intelligently with full appreciation of its
appellant inadmissible is the fact that accused-appellant importance and comprehension of its
was not given the Miranda warnings effectively. Under consequences.
the Constitution, an uncounseled statement, such as it is
called in the United States from which Article III, Section Once admitted, the confession must inspire
12(1) was derived, is presumed to be psychologically credibility or be one which the normal experience of
coerced. Swept into an unfamiliar environment and mankind can accept as being within the realm of
surrounded by intimidating figures typical of the probability.
atmosphere of police interrogation, the suspect really
needs the guiding hand of counsel. A confession meeting all the foregoing requisites
constitutes evidence of a high order since it is supported
Now, under the first paragraph of this provision, it by the strong presumption that no person of normal mind
is required that the suspect in custodial interrogation must will knowingly, freely and deliberately confess that he is
be given the following warnings: (1) he must be informed the perpetrator of a crime unless prompted by truth and
of his right to remain silent; (2) he must be warned that conscience. When all these requirements are met and the
anything he says can and will be used against him; and confession is admitted in evidence, the burden of proof
(3) he must be told that he has a right to counsel, and that it was obtained by undue pressure, threat or
that if he is indigent, a lawyer will be appointed to intimidation rests upon the accused. (People v. Fabro,
represent him. 277 SCRA 19, Aug. 11, 1997 [Panganiban])

Xxx 2. Numerous decisions of this Court rule that for


an extrajudicial confession to be admissible, it must be: 1)
There was thus only a perfunctory reading of the voluntary; 2) made with the assistance of competent and
Miranda rights to accused-appellant without any effort to independent counsel; 3) express; and 4) in writing.
find out from him whether he wanted to have counsel
and, if so, whether he had his own counsel or he wanted The mantle of protection afforded by the above-
the police to appoint one for him. This kind of giving of quoted constitutional provision covers the period from the
warnings, in several decisions of this Court, has been time a person is taken into custody for the investigation of

105
his possible participation in the commission of a crime or him but, rather, it was adopted in our Constitution to
from the time he is singled out as a suspect in the preclude the slightest coercion as would lead the accused
commission of the offense although not yet in custody. to admit something false. The counsel, however, should
The exclusionary rule is premised on the presumption that never prevent an accused from freely and voluntarily
the defendant is thrust into an unfamiliar atmosphere telling the truth.” (People v. Base, 329 SCRA 158,
running through menacing police interrogation procedures 169-171, March 30, 2000, 1st Div. [Ynares-
where the potentiality for compulsion, physical or Santiago])
psychological is forcefully apparent.
235. Should courts be allowed to distinguish between
However, the rule is not intended as a deterrent preliminary questioning and custodial investigation
to the accused from confessing guilt if he voluntarily and proper when applying the exclusionary rule?
intelligently so desires but to protect the accused from
admitting what he is coerced to admit although untrue . Held: The exclusionary rule sprang from a
(People v. Base, 329 SCRA 158, 169-171, March 30, recognition that police interrogatory procedures lay fertile
2000, 1st Div. [Ynares-Santiago]) grounds for coercion, physical and psychological, of the
suspect to admit responsibility for the crime under
234. Is the choice of a lawyer by a person under investigation. It was not intended as a deterrent to the
custodial investigation who cannot afford the services accused from confessing guilt, if he voluntarily and
of a counsel exclusive as to preclude other equally intelligently so desires but to protect the accused from
competent and independent attorneys from handling admitting what he is coerced to admit although untrue .
his defense? Law enforcement agencies are required to effectively
communicate the rights of a person under investigation
Held: It must be remembered in this regard that and to insure that it is fully understood. Any measure
while the right to counsel is immutable, the option to short of this requirement is considered a denial of such
secure the services of counsel de parte is not absolute. right. Courts are not allowed to distinguish between
Indeed – preliminary questioning and custodial investigation proper
when applying the exclusionary rule. Any information or
The phrase “competent and independent” admission given by a person while in custody which may
and “preferably of his own choice” were explicit appear harmless or innocuous at the time without the
details which were added upon the persistence of competent assistance of an independent counsel should
human rights lawyers in the 1986 Constitutional be struck down as inadmissible. It has been held,
Commission who pointed out cases where, during however, that an admission made to news reporters or to
the martial law period, the lawyers made available a confidant of the accused is not covered by the
to the detainee would be one appointed by the exclusionary rule.
military and therefore beholden to the military.
(Citing I Record of the Constitutional Commission The admission allegedly made by the appellant is
731-734; I Bernas, The Constitution of the not in the form of a written extra-judicial confession; the
Republic of the Philippines, 1987 1st ed., p. 347) admission was allegedly made to the arresting officer
during an “informal talk” at the police station after his
Xxx xxx xxx arrest as a prime suspect in the rape and killing of x x x.
The arresting policeman testified that the appellant
Withal, the word “preferably” under admitted that he was with the victim on the evening of
Section 12(1), Article 3 of the 1987 Constitution January 12, 1994, the probable time of the commission of
does not convey the message that the choice of a the crime and that he carried her on his shoulder but that
lawyer by a person under investigation is he was too drunk to remember what subsequently
exclusive as to preclude other equally competent happened. The arresting policeman admitted that he did
and independent attorneys from handling his not inform the appellant of his constitutional rights to
defense. If the rule were otherwise, then, the remain silent and to counsel. We note that the alleged
tempo of a custodial investigation will be solely in admission is incriminating because it places the accused in
the hands of the accused who can impede, nay, the company of the victim at the time the crime was
obstruct the progress of the interrogation by probably committed.
simply selecting a lawyer who for one reason or
another, is not available to protect his interest. The exclusionary rule applies.
This absurd scenario could not have been
contemplated by the framers of the charter. The accused was under arrest for the rape and
killing of x x x and any statement allegedly made by him
While the initial choice in cases where a person pertaining to his possible complicity in the crime without
under custodial investigation cannot afford the services of prior notification of his constitutional rights is inadmissible
a lawyer is naturally lodged in the police investigators, the in evidence. The policeman’s apparent attempt to
accused really has the final choice as he may reject the circumvent the rule by insisting that the admission was
counsel chosen for him and ask for another one. A lawyer made during an “informal talk” prior to custodial
provided by the investigators is deemed engaged by the investigation prior is not tenable. The appellant was not
accused where he never raised any objection against the invited to the police station as part of a general inquiry for
former’s appointment during the course of the any possible lead to the perpetrators of the crime under
investigation and the accused thereafter subscribes to the investigation. At the time the alleged admission was
veracity of his statement before the swearing officer. made the appellant was in custody and had been arrested
as the prime suspect in the rape and killing of x x x. The
Verily, to be an effective counsel “[a] lawyer need exclusionary rule presumes that the alleged admission
not challenge all the questions being propounded to his was coerced, the very evil the rule stands to avoid.
client. The presence of a lawyer is not intended to stop Supportive of such presumption is the absence of a
an accused from saying anything which might incriminate written extra-judicial confession to that effect and the

106
appellant’s denial in court of the alleged oral admission. right to counsel because it involves a general inquiry into
The alleged admission should be struck down as an unsolved crime and is purely investigatory in nature. It
inadmissible. (People v. Bravo, 318 SCRA 812, Nov. has also been held that an uncounseled identification at
22, 1999, En Banc [Gonzaga-Reyes]) the police line-up does not preclude the admissibility of an
in-court identification. The identification made by the
236. Explain the procedure for out-of-court private complainant in the police line-up pointing to
identification of suspects and the test to determine Pavillare as one of his abductors is admissible in evidence
the admissibility of such identification. although the accused-appellant was not assisted by
counsel. x x x (People v. Pavillare, 329 SCRA 684,
Held: 1. In People v. Teehankee, Jr., the Court 694-695, April 5, 2000, En Banc [Per Curiam])
x x x explained the procedure for out-of-court
identification and the test to determine the admissibility of 238. Petitioner in a case “x x x posits the theory that
such identification. It listed the following ways of since he had no counsel during the custodial
identifying the suspects during custodial investigation: investigation when his urine sample was taken and
show-up, mug shots and line-ups. The Court there ruled: chemically examined, Exhibits “L” and “M,” x x x are
also inadmissible in evidence since his urine sample
“x x x. Out-of-court identification is was derived in effect from an uncounselled extra-
conducted by the police in various ways. It is judicial confession. Petitioner claims that the taking
done thru show-ups where the suspect alone is of his urine sample allegedly violates Article III,
brought face to face with the witness for Section 2 of the Constitution x x x.” Should his
identification. It is done thru mug shots where contentions be upheld?
photographs are shown to the witness to identify
the suspect. It is also done thru line ups where a Held: We are not persuaded. The right to
witness identifies the suspect from a group of counsel begins from the time a person is taken into
persons lined up for the purpose. Since custody and placed under investigation for the
corruption of out-of-court identification commission of a crime, i.e., when the investigating officer
contaminates the integrity of in court starts to ask questions to elicit information and/or
identification during the trial of the case, courts confession or admissions from the accused. Such right is
have fashioned out rules to assure its fairness and guaranteed by the Constitution and cannot be waived
its compliance with the requirements of except in writing and in the presence of counsel.
constitutional due process. In resolving the However, what the Constitution prohibits is the use of
admissibility of and relying on out-of- court physical or moral compulsion to extort communication
identification of suspects, courts have adopted the from the accused, but not an inclusion of his body in
totality of circumstances test where they consider evidence, when it may be material. In fact, an accused
the following factors, viz: (1) the witness’ may validly be compelled to be photographed or
opportunity to view the criminal at the time of the measured, or his garments or shoes removed or replaced,
crime; (2) the witness’ degree of attention at that or to move his body to enable the foregoing things to be
time; (3) the accuracy of any prior description done, without running afoul of the proscription against
given by the witness; (4) the level of certainty testimonial compulsion. The situation in the case at bar
demonstrated by the witness at the identification; falls within the exemption under the freedom from
(5) the length of time between the crime and the testimonial compulsion since what was sought to be
identification; and (6) the suggestiveness of the examined came from the body of the accused. This was a
identification procedure.” (People v. Timon, mechanical act the accused was made to undergo which
281 SCRA 577, Nov. 12, 1997 [Panganiban]) was not meant to unearth undisclosed facts but to
ascertain physical attributes determinable by simple
2. x x x. The totality test has been fashioned observation. In fact, the record shows that petitioner and
precisely to assure fairness as well as compliance with his co-accused were not compelled to give samples of
constitutional requirements of due process in regard to their urine but they in fact voluntarily gave the same
out-of-court identification. These cited factors must be when they were requested to undergo a drug test.
considered to prevent contamination of the integrity of in- (Gutang v. People, 335 SCRA 479, July 11, 2000,
court identifications better. (People v. Gamer, 326 2nd Div. [De Leon])
SCRA 660, Feb. 29, 2000, 2nd Div. [Quisumbing])

237. Does the prohibition for custodial investigation The Right to Bail
conducted without the assistance of counsel extend to
a person in a police line-up? Consequently, is the 239. In bail application where the accused is charged
identification by private complainant of accused who with a capital offense, will it be proper for the judge
was not assisted by counsel during police line-up to grant bail without conducting hearing if the
admissible in evidence? prosecutor interposes no objection to such
application? Why?
Held: The prohibition x x x does not extend to a
person in a police line-up because that stage of an Held: Jurisprudence is replete with decisions
investigation is not yet a part of custodial investigation . It compelling judges to conduct the required hearings in bail
has been repeatedly held that custodial investigation applications, in which the accused stands charged with a
commences when a person is taken into custody and is capital offense. The absence of objection from the
singled out as a suspect in the commission of the crime prosecution is never a basis for the grant of bail in such
under investigation and the police officers begin to ask cases, for the judge has no right to presume that the
questions on the suspect’s participation therein and which prosecutor knows what he is doing on account of
tend to elicit an admission. The stage of an investigation familiarity with the case. “Said reasoning is tantamount to
wherein a person is asked to stand in a police line-up has ceding to the prosecutor the duty of exercising judicial
been held to be outside the mantle of protection of the discretion to determine whether the guilt of the accused is

107
strong. Judicial discretion is the domain of the judge
before whom the petition for provisional liberty will be The trial court correctly denied petitioners’ motion
decided. The mandated duty to exercise discretion has that they be allowed provisional liberty after their
never been reposed upon the prosecutor.” conviction, under their respective bail bonds. Apart from
the fact that they were at large, Section 5, Rule 114 of the
Imposed in Baylon v. Sison was this mandatory Rules of Court, as amended by Supreme Court
duty to conduct a hearing despite the prosecution's Administrative Circular 12-94, provides that:
refusal to adduce evidence in opposition to the application
to grant and fix bail. (Joselito V. Narciso v. Flor Marie Xxx
Sta. Romana-Cruz, G.R. No. 134504, March 17,
2000, 3rd Div. [Panganiban]) The Court, in its discretion, may allow the accused
to continue on provisional liberty under the same bail
240. What are the duties of the judge in cases of bail bond during the period to appeal subject to the consent of
applications where the accused is charged with capital the bondsman.
offense?
The bail bond that the accused previously posted
Held: Basco v. Rapatalo enunciated the following can only be used during the 15-day period to appeal (Rule
duties of the trial judge in such petition for bail: 122) and not during the entire period of appeal. This is
consistent with Section 2(a) of Rule 114 which provides
1) Notify the prosecutor of the hearing of the that the bail “shall be effective upon approval and remain
application for bail or require him to submit in force at all stages of the case, unless sooner cancelled,
his recommendation; until the promulgation of the judgment of the Regional
2) Conduct a hearing of the application for bail Trial Court, irrespective of whether the case was originally
regardless of whether or not the prosecution filed in or appealed to it.” This amendment, introduced by
refuses to present evidence to show that the SC Administrative Circular 12-94 is a departure from the
guilt of the accused is strong for the purpose old rules which then provided that bail shall be effective
of enabling the court to exercise its sound and remain in force at all stages of the case until its full
discretion; determination, and thus even during the period of appeal.
3) Decide whether the evidence of guilt of the Moreover, under the present rule, for the accused to
accused is strong based on the summary of continue his provisional liberty on the same bail bond
evidence of the prosecution; during the period to appeal, consent of the bondsman is
4) If the guilt of the accused is not strong, necessary. From the record, it appears that the
discharge the accused upon the approval of bondsman x x x filed a motion in the trial court x x x for
the bailbond. Otherwise, petition should be the cancellation of petitioners’ bail bond for the latter’s
denied. failure to renew the same upon its expiration. Obtaining
the consent of the bondsman was, thus, foreclosed.
The Court added: “The above-enumerated (Maguddatu v. Court of Appeals, 326 SCRA 362,
procedure should now leave no room for doubt as to the Feb. 23, 2000, 1st Div. [Kapunan])
duties of the trial judge in cases of bail applications. So
basic and fundamental is it to conduct a hearing in 242. Is a condition in an application for bail that
connection with the grant of bail in the proper cases that accused be first arraigned before he could be granted
it would amount to judicial apostasy for any member of bail valid?
the judiciary to disclaim knowledge or awareness thereof.”
Held: In requiring that petitioner be first
Additionally, the court's grant or refusal of bail arraigned before he could be granted bail, the trial court
must contain a summary of the evidence for the apprehended that if petitioner were released on bail he
prosecution, on the basis of which should be formulated could, by being absent, prevent his early arraignment and
the judge's own conclusion on whether such evidence is thereby delay his trial until the complainants got tired and
strong enough to indicate the guilt of the accused. The lost interest in their cases. Hence, to ensure his presence
summary thereof is considered an aspect of procedural at the arraignment, approval of petitioner’s bail bonds
due process for both the prosecution and the defense; its should be deferred until he could be arraigned. After that,
absence will invalidate the grant or the denial of the even if petitioner does not appear, trial can proceed as
application for bail. (Joselito V. Narciso v. Flor Marie long as he is notified of the date of the hearing and his
Sta. Romana-Cruz, G.R. No. 134504, March 17, failure to appear is unjustified, since under Art. III, Sec.
2000, 3rd Div. [Panganiban]) 14(2) of the Constitution, trial in absencia is authorized.
This seems to be the theory of the trial court in its x x x
241. Should the accused who remained at large after order conditioning the grant of bail to petitioner on his
their conviction be allowed provisional liberty? Can arraignment.
the bail bond that the accused previously posted be
used during the entire period of appeal? This theory is mistaken. In the first place x x x in
cases where it is authorized, bail should be granted before
Held: Despite an order of arrest from the trial arraignment, otherwise the accused may be precluded
court and two warnings from the Court of Appeals, from filing a motion to quash. For if the information is
petitioners had remained at large. It is axiomatic that for quashed and the case is dismissed, there would then be
one to be entitled to bail, he should be in the custody of no need for the arraignment of the accused. In the
the law, or otherwise, deprived of liberty. The purpose of second place, the trial court could ensure the presence of
bail is to secure one’s release and it would be incongruous petitioner at the arraignment precisely by granting bail
to grant bail to one who is free. Petitioners’ Compliance and ordering his presence at any stage of the
and Motion x x x came short of an unconditional proceedings, such as arraignment. Under Rule 114, Sec.
submission to respondent court’s lawful order and to its 2(b) of the Rules on Criminal Procedure, one of the
jurisdiction. conditions of bail is that “the accused shall appear before

108
the proper court whenever so required by the court or tyranny, as well as the power to promulgate rules to
these Rules,” while under Rule 116, Sec. 1(b) the protect and enforce constitutional rights. Furthermore,
presence of the accused at the arraignment is required. we believe that the right to due process is broad enough
to include the grant of basic fairness to extraditees.
On the other hand, to condition the grant of bail Indeed, the right to due process extends to the “life,
to an accused on his arraignment would be to place him in liberty or property” of every person. It is “dynamic and
a position where he has to choose between (1) filing a resilient, adaptable to every situation calling for its
motion to quash and thus delay his release on bail application.”
because until his motion to quash can be resolved, his
arraignment cannot be held, and (2) foregoing the filing Accordingly and to best serve the ends of justice,
of a motion to quash so that he can be arraigned at once we believe and so hold that, after a potential extraditee
and thereafter be released on bail. These scenarios has been arrested or placed under the custody of the law,
certainly undermine the accused’s constitutional right not bail may be applied for and granted as an exception, only
to be put on trial except upon valid complaint or upon a clear and convincing showing (1) that, once
information sufficient to charge him with a crime and his granted bail, the applicant will not be a flight risk or a
right to bail. (Lavides v. CA, 324 SCRA 321, Feb. 1, danger to the community; and (2) that there exist special,
2000, 2nd Div. [Mendoza]) humanitarian and compelling circumstances including, as
a matter of reciprocity, those cited by the highest court in
243. Is respondent Mark Jimenez entitled to bail during the requesting state when it grants provisional liberty in
the pendency of the Extradition Proceeding? extradition cases therein.

Held: We agree with petitioner. As suggested by Since this exception has no express or specific
the use of the word “conviction,” the constitutional statutory basis, and since it is derived essentially from
provision on bail x x x, as well as Section 4 of Rule 114 of general principles of justice and fairness, the applicant
the Rules of Court, applies only when a person has been bears the burden of proving the above two-tiered
arrested and detained for violation of Philippine criminal requirement with clarity, precision and emphatic
laws. It does not apply to extradition proceedings, forcefulness. The Court realizes that extradition is
because extradition courts do not render judgments of basically an executive, not a judicial, responsibility arising
conviction or acquittal. from the presidential power to conduct foreign relations.
In its barest concept, it partakes of the nature of police
Moreover, the constitutional right to bail “flows assistance amongst states, which is not normally a judicial
from the presumption of innocence in favor of every prerogative. Hence, any intrusion by the courts into the
accused who should not be subjected to the loss of exercise of this power should be characterized by caution,
freedom as thereafter he would be entitled to acquittal, so that the vital international and bilateral interests of our
unless his guilt be proved beyond reasonable doubt.” It country will not be unreasonably impeded or
follows that the constitutional provision on bail will not compromised. In short, while this Court is ever protective
apply to a case like extradition, where the presumption of of “the sporting idea of fair play,” it also recognizes the
innocence is not an issue. limits of its own prerogatives and the need to fulfill
international obligations. (Government of the United
The provision in the Constitution stating that the States of America v. Hon. Guillermo Purganan, G.R.
right to bail shall not be impaired even when the privilege No. 148571, Sept. 24, 2002, En Banc [Panganiban])
of the writ of habeas corpus is suspended” does not
detract from the rule that the constitutional right to bail is 245. Are there special circumstances compelling
available only in criminal proceedings. It must be noted enough for the Court to grant Mark Jimenez’s request
that the suspension of the privilege of the writ of habeas for provisional release on bail?
corpus finds application “only to persons judicially charged
for rebellion or offenses inherent in or directly connected Held: Along this line, Jimenez contends that
with invasion.” (Sec. 18, Article VII, Constitution) Hence, there are special circumstances that are compelling
the second sentence in the constitutional provision on bail enough for the Court to grant his request for provisional
merely emphasizes the right to bail in criminal release on bail. We have carefully examined these
proceedings for the aforementioned offenses. It cannot circumstances and shall now discuss them.
be taken to mean that the right is available even in
extradition proceedings that are not criminal in nature. 1. Alleged Disenfranchisement

That the offenses for which Jimenez is sought to While his extradition was pending, Respondent
be extradited are bailable in the United States is not an Jimenez was elected as a member of the House of
argument to grant him one in the present case. To stress, Representatives. On that basis, he claims that his
extradition proceedings are separate and distinct from the detention will disenfranchise his Manila district of 600,000
trial for the offenses for which he is charged. He should residents. We are not persuaded. In People v. Jalosjos,
apply for bail before the courts trying the criminal cases the Court has already debunked the disenfranchisement
against him, not before the extradition court. argument x x x.
(Government of the United States of America v.
Hon. Guillermo Purganan, G.R. No. 148571, Sept. It must be noted that even before private
24, 2002, En Banc [Panganiban]) respondent ran for and won a congressional seat in
Manila, it was already of public knowledge that the United
244. What is the exception to the “No Bail” Rule in States was requesting his extradition. Hence, his
Extradition Proceedings? constituents were or should have been prepared for the
consequences of the extradition case against their
Held: The rule x x x is that bail is not a matter of representative, including his detention pending the final
right in extradition cases. However, the judiciary has the resolution of the case. Premises considered and in line
constitutional duty to curb grave abuse of discretion and with Jalosjos, we are constrained to rule against his claim

109
that his election to public office is by itself a compelling against him, a right guaranteed by no less than the
reason to grant him bail. fundamental law of the land (Article III, Section 14[2],
1987 Constitution). Elaborating on the defendant’s right
2. Anticipated Delay to be informed, the Court held in Pecho v. People that the
objectives of this right are:
Respondent Jimenez further contends that
because the extradition proceedings are lengthy, it would 1) To furnish the accused with such a description
be unfair to confine him during the pendency of the case. of the charge against him as will enable him
Again we are not convinced. We must emphasize that to make the defense;
extradition cases are summary in nature. They are 2) To avail himself of his conviction or acquittal
resorted to merely to determine whether the extradition for protection against a further prosecution
petition and its annexes conform to the Extradition Treaty, for the same cause; and
not to determine his guilt or innocence. Neither is it, as a 3) To inform the court of the facts alleged, so
rule, intended to address issues relevant to the that it may decide whether they are sufficient
constitutional rights available to the accused in a criminal in law to support a conviction, if one should
action. be had.

We are not overruling the possibility that It is thus imperative that the Information filed
petitioner may, in bad faith, unduly delay the proceedings. with the trial court be complete – to the end that the
This is quite another matter that is not at issue here. accused may suitably prepare for his defense. Corollary
Thus, any further discussion of this point would be merely to this, an indictment must fully state the elements of the
anticipatory and academic. specific offense alleged to have been committed as it is
the recital of the essentials of a crime which delineates
However, if the delay is due to maneuverings of the nature and cause of accusation against the accused.
respondent, with all the more reason would the grant of
bail not be justified. Giving premium to delay by Xxx
considering it as a special circumstance for the grant of
bail would be tantamount to giving him the power to In the case under scrutiny, the information does
grant bail to himself. It would also encourage him to not allege the minority of the victim x x x although the
stretch out and unreasonably delay the extradition same was proven during the trial x x x. The omission is
proceedings even more. This we cannot allow. not merely formal in nature since doctrinally, an accused
cannot be held liable for more than what he is indicted
3. Not a Flight Risk? for. It matters not how conclusive and convincing the
evidence of guilt may be, but an accused cannot be
Jimenez further claims that he is not a flight risk. convicted of any offense, not charged in the Complaint or
To support this claim, he stresses that he learned of the Information on which he is tried or therein necessarily
extradition request in June 1999; yet, he has not fled the included. He has a right to be informed of the nature of
country. True, he has not actually fled during the the offense with which he is charged before he is put on
preliminary stages of the request for his extradition. Yet, trial. To convict an accused of an offense higher than
this fact cannot be taken to mean that he will not flee as that charged in the Complaint or Information on which he
the process moves forward to its conclusion, as he hears is tried would constitute unauthorized denial of that right.
the footsteps of the requesting government inching closer (People v. Bayya, 327 SCRA 771, March 10, 2000,
and closer. That he has not yet fled from the Philippines En Banc [Purisima])
cannot be taken to mean that he will stand his ground
and still be within reach of our government if and when it
matters; that is, upon the resolution of the Petition for The Right to a Fair Trial
Extradition.
247. What is the purpose of the rule barring trial or
In any event, it is settled that bail may be applied sentence of an insane person? What are the reasons
for and granted by the trial court at anytime after the underlying it?
applicant has been taken into custody and prior to
judgment, even after bail has been previously denied. In Held: The rule barring trial or sentence of an
the present case, the extradition court may continue insane person is for the protection of the accused, rather
hearing evidence on the application for bail, which may be than of the public. It has been held that it is inhuman to
granted in accordance with the guidelines in this Decision. require an accused disabled by God to make a just
(Government of the United States of America v. defense for his life or liberty. To put a legally
Hon. Guillermo Purganan, G.R. No. 148571, Sept. incompetent person on trial or to convict and sentence
24, 2002, En Banc [Panganiban]) him is a violation of the constitutional rights to a fair trial;
and this has several reasons underlying it. For one, the
The Right to be Informed of the Nature and Cause accuracy of the proceedings may not be assured, as an
of Accusation against the Accused incompetent defendant who cannot comprehend the
proceedings may not appreciate what information is
246. What are the objectives of the right to be relevant to the proof of his innocence. Moreover, he is
informed of the nature and cause of accusations not in a position to exercise many of the rights afforded a
against the accused? defendant in a criminal case, e.g., the right to effectively
consult with counsel, the right to testify in his own behalf,
Held: Instructive in this regard is Section 6, Rule and the right to confront opposing witnesses, which rights
110 of the Rules of Court x x x. are safeguards for the accuracy of the trial result.
Second, the fairness of the proceedings may be
The purpose of the above-quoted rule is to inform questioned, as there are certain basic decisions in the
the accused of the nature and cause of the accusation course of a criminal proceeding which a defendant is

110
expected to make for himself, and one of these is his plea. right of an accused to a fair trial is not
Third, the dignity of the proceedings may be disrupted, incompatible to a free press. To be sure,
for an incompetent defendant is likely to conduct himself responsible reporting enhances an accused’s right
in the courtroom in a manner which may destroy the to a fair trial for, as well pointed out, a
decorum of the court. Even if the defendant remains responsible press has always been regarded as
passive, his lack of comprehension fundamentally impairs the handmaiden of effective judicial
the functioning of the trial process. A criminal proceeding administration, especially in the criminal field x x
is essentially an adversarial proceeding. If the defendant x. The press does not simply publish information
is not a conscious and intelligent participant, the about trials but guards against the miscarriage of
adjudication loses its character as a reasoned interaction justice by subjecting the police, prosecutors, and
between an individual and his community and becomes an judicial processes to extensive public scrutiny and
invective against an insensible object. Fourth, it is criticism.
important that the defendant knows why he is being
punished, a comprehension which is greatly dependent Pervasive publicity is not per se prejudicial
upon his understanding of what occurs at trial. An to the right of an accused to fair trial. The mere
incompetent defendant may not realize the moral fact that the trial of appellant was given a day-to-
reprehensibility of his conduct. The societal goal of day, gavel-to-gavel coverage does not by itself
institutionalized retribution may be frustrated when the prove that the publicity so permeated the mind of
force of the state is brought to bear against one who the trial judge and impaired his impartiality. For
cannot comprehend its significance. (People v. Estrada, one, it is impossible to seal the minds of members
333 SCRA 699, 718-719, June 19, 2000, En Banc of the bench from pre-trial and other off-court
[Puno]) publicity of sensational criminal cases. The state
The Right to an Impartial Trial of the art of our communication system brings
news as they happen straight to our breakfast
248. What are the two principal legal and philosophical tables and right to our bedrooms. These news
schools of thought on how to deal with the rain of form part of our everyday menu of the facts and
unrestrained publicity during the investigation and fictions of life. For another, our idea of a fair and
trial of high profile cases? impartial judge is not that of a hermit who is out
of touch with the world. We have not installed
Held: There are two (2) principal legal and the jury system whose members are overly
philosophical schools of thought on how to deal with the protected from publicity lest they lose their
rain of unrestrained publicity during the investigation and impartiality. x x x. Our judges are learned in the
trial of high profile cases. The British approach the law and trained to disregard off-court evidence
problem with the presumption that publicity will prejudice and on-camera performances of parties to a
a jury. Thus, English courts readily stay and stop criminal litigation. Their mere exposure to publications
trials when the right of an accused to fair trial suffers a and publicity stunts does not per se fatally infect
threat. The American approach is different. US courts their impartiality.
assume a skeptical approach about the potential effect of
pervasive publicity on the right of an accused to a fair At best, appellant can only conjure
trial. They have developed different strains of tests to possibility of prejudice on the part of the trial
resolve this issue, i.e., substantial probability of judge due to the barrage of publicity that
irreparable harm, strong likelihood, clear and present characterized the investigation and trial of the
danger, etc. (Estrada v. Desierto, G.R. Nos. 146710- case. In Martelino, et al. v. Alejandro, et al., we
15, March 2, 2001, En Banc [Puno]) rejected this standard of possibility of prejudice
249. Should the Ombudsman be stopped from and adopted the test of actual prejudice as we
conducting the investigation of the cases filed against ruled that to warrant a finding of prejudicial
petitioner (former President) Estrada due to the publicity, there must be allegation and proof that
barrage of prejudicial publicity on his guilt? the judges have been unduly influenced, not
simply that they might be, by the barrage of
Held: Petitioner x x x contends that the publicity. In the case at bar, the records do not
respondent Ombudsman should be stopped from show that the trial judge developed actual bias
conducting the investigation of the cases filed against him against appellant as a consequence of the
due to the barrage of prejudicial publicity on his guilt. He extensive media coverage of the pre-trial and trial
submits that the respondent Ombudsman has developed of his case. The totality of circumstances of the
bias and is all set to file the criminal cases in violation of case does not prove that the trial judge acquired
his right to due process. a fixed opinion as a result of prejudicial publicity
which is incapable of change even by evidence
Xxx presented during the trial. Appellant has the
burden to prove this actual bias and he has not
This is not the first time the issue of trial by discharged the burden.”
publicity has been raised in this Court to stop the trials or
annul convictions in high profile criminal cases. In People We expounded further on this doctrine in the
v. Teehankee, Jr., later reiterated in the case of Larranaga subsequent case of Webb v. Hon. Raul de Leon, etc. and
v. Court of Appeals, et al., we laid down the doctrine that: its companion cases, viz.:

“We cannot sustain appellant’s claim that “Again, petitioners raise the effect of
he was denied the right to impartial trial due to prejudicial publicity on their right to due process
prejudicial publicity. It is true that the print and while undergoing preliminary investigation. We
broadcast media gave the case at bar pervasive find no procedural impediment to its early
publicity, just like all high profile and high stake invocation considering the substantial risk to their
criminal trials. Then and now, we rule that the

111
liberty whole undergoing a preliminary courtroom doors which had long been open to the
investigation. public at the time the First Amendment was
adopted. Moreover, the right of assembly is also
Xxx relevant, having been regarded not only as an
independent right but also as a catalyst to
The democratic settings, media coverage augment the free exercise of the other First
of trials of sensational cases cannot be avoided Amendment rights with which it was deliberately
and oftentimes, its excessiveness has been linked by the draftsmen. A trial courtroom is a
aggravated by kinetic developments in the public place where the people generally – and
telecommunications industry. For sure, few cases representatives of the media – have a right to be
can match the high volume and high velocity of present, and where their presence historically has
publicity that attended the preliminary been thought to enhance the integrity and quality
investigation of the case at bar. Our daily diet of of what takes place.
facts and fiction about the case continues
unabated even today. Commentators still (c) Even though the Constitution
bombard the public with views not too many of contains no provision which by its terms
which are sober and sublime. Indeed, even the guarantees to the public the right to attend
principal actors in the case – the NBI, the criminal trials, various fundamental rights, not
respondents, their lawyers and their sympathizers expressly guaranteed, have been recognized as
– have participated in this media blitz. The indispensable to the enjoyment of enumerated
possibility of media abuses and their threat to a rights. The right to attend criminal trial is implicit
fair trial notwithstanding, criminal trials cannot be in the guarantees of the First Amendment:
completely closed to the press and public. In the without the freedom to attend such trials, which
seminal case of Richmond Newspapers, Inc. v. people have exercised for centuries, important
Virginia, it was wisely held: aspects of freedom of speech and of the press
could be eviscerated.’
‘x x x
Be that as it may, we recognize that
(a) The historical evidence of the pervasive and prejudicial publicity under certain
evolution of the criminal trial in Anglo-American circumstances can deprive an accused of his due
justice demonstrates conclusively that at the process right to fair trial. Thus, in Martelino, et al.
time this Nation’s organic laws were adopted, v. Alejandro, et al., we held that to warrant a
criminal trials both here and in England had long finding of prejudicial publicity there must be
been presumptively open, thus giving assurance allegation and proof that the judges have been
that the proceedings were conducted fairly to all unduly influenced, not simply that they might be,
concerned and discouraging perjury, the by the barrage of publicity. In the case at bar, we
misconduct of participants, or decisions based on find nothing in the records that will prove that the
secret bias or partiality. In addition, the tone and content of the publicity that attended
significant community therapeutic value of public the investigation of petitioners fatally infected the
trials was recognized: when a shocking crime fairness and impartiality of the DOJ Panel.
occurs, a community reaction of outrage and Petitioners cannot just rely on the subliminal
public protest often follows, and thereafter the effects of publicity on the sense of fairness of the
open processes of justice serve an important DOJ Panel, for these are basically unbeknown and
prophylactic purpose, providing an outlet for beyond knowing. To be sure, the DOJ Panel is
community concern, hostility, and emotion. To composed of an Assistant Chief State Prosecutor
work effectively, it is important that society’s and Senior State Prosecutors. Their long
criminal process ‘satisfy the appearance of experience in criminal investigation is a factor to
justice,’ Offutt v. United States, 348 US 11, 14, consider in determining whether they can easily
99 L Ed 11, 75 S Ct 11, which can best be be blinded by the klieg lights of publicity. Indeed,
provided by allowing people to observe such their 26-page Resolution carries no indubitable
process. From this unbroken, uncontradicted indicia of bias for it does not appear that they
history, supported by reasons as valid today as in considered any extra-record evidence except
centuries past, it must be concluded that a evidence properly adduced by the parties. The
presumption of openness inheres in the very length of time the investigation was conducted
nature of a criminal trial under this Nation’s despite it summary nature and the generosity
system of justice, Cf., e.g., Levine v. United with which they accommodated the discovery
States, 362 US 610, 4 L Ed 2d 989, 80 S Ct 1038. motions of petitioners speak well of their fairness.
At no instance, we note, did petitioners seek the
(b) The freedoms of speech, disqualification of any member of the DOJ Panel
press, and assembly, expressly guaranteed by the on the ground of bias resulting from their
First Amendment, share a common core purpose bombardment of prejudicial publicity.”
of assuring freedom of communication on matters
relating to the functioning of government. In Applying the above ruling, we hold that there is not
guaranteeing freedoms such as those of speech enough evidence to warrant this Court to enjoin the
and press, the First Amendment can be read as preliminary investigation of the petitioner by the
protecting the right of everyone to attend trials so respondent Ombudsman. Petitioner needs to offer more
as give meaning to those explicit guarantees; the than hostile headlines to discharge his burden of proof.
First Amendment right to receive information and He needs to show more than weighty social science
ideas means, in the context of trials, that the evidence to successfully prove the impaired capacity of a
guarantees of speech and press, standing alone, judge to render a bias-free decision. Well to note, the
prohibit government from summarily closing cases against the petitioner are still undergoing

112
preliminary investigation by a special panel of prosecutors v. Rondero, 320 SCRA 383, 399-401, Dec. 9, 1999,
in the office of the respondent Ombudsman. No En Banc [Per Curiam])
allegation whatsoever has been made by the petitioner
that the minds of the members of this special panel have 251. Does the right against self-incrimination extend to
already been infected by bias because of the pervasive administrative proceedings?
prejudicial publicity against him. Indeed, the special panel
has yet to come out with its findings and the Court cannot Held: In Pascual v. Board of Medical Examiners
second guess whether its recommendation will be (28 SCRA 344 [1969]), we held that the right against self-
unfavorable to the petitioner. (Estrada v. Desierto, incrimination under Section 17, Article III of the 1987
G.R. Nos. 146710-15, March 2, 2001, En Banc Constitution which is ordinarily available only in criminal
[Puno]) prosecutions, extends to administrative proceedings which
possess a criminal or penal aspect, such as an
administrative investigation of a licensed physician who is
The Right against Self-Incrimination charged with immorality, which could result in his loss of
the privilege to practice medicine if found guilty. The
250. Accused-appellant alleges that while in the Court, citing the earlier case of Cabal v. Kapunan (6 SCRA
custody of police officers, some hair strands were 1059 [1962]), pointed out that the revocation of one’s
taken from him without his consent and submitted to license as a medical practitioner, is an even greater
the NBI for investigation, in violation of his right deprivation than forfeiture of property. (Secretary of
against self-incrimination. Aside from executing a Justice v. Lantion, 322 SCRA 160, 184, Jan. 18,
waiver of the provisions of Article 125 of the Revised 2000, En Banc [Melo])
Penal Code, accused-appellant executed a waiver of
the provisions of Article III, Section 12 of the 252. May the right against self-incrimination be validly
Constitution regarding the rights of an accused during invoked during inquiry in aid of legislation?
custodial investigation. It appears, however, that the
waivers were executed by the accused without the Held: [I]t has been held that “a congressional
assistance of a counsel of his own choice. committee’s right to inquire is ‘subject to all relevant
limitations placed by the Constitution on governmental
Held: The use of evidence against the accused action,’ including ‘the relevant limitations of the Bill of
obtained by virtue of his testimony or admission without Rights’.”
the assistance of counsel while under custodial
investigation is proscribed under Sections 12 and 17, In another case –
Article III of the Constitution x x x.
“x x x the mere semblance of legislative
The aforesaid rules are set forth in the purpose would not justify an inquiry in the face of
Constitution as a recognition of the fact that the the Bill of Rights. The critical element is the
psychological if not physical atmosphere of custodial existence of, and the weight to be ascribed to, the
investigations in the absence of procedural safeguards is interest of the Congress in demanding disclosures
inherently coercive in nature. However, to paraphrase from an unwilling witness. We cannot simply
Justice Sanchez in the case of Chavez v. Court of Appeals assume, however, that every congressional
(24 SCRA 663 [1968]), “Compulsion does not necessarily investigation is justified by a public need that
connote the use of violence; it may be the product of over-balances any private rights affected. To do
unintentional statements. Pressure which operates to so would be to abdicate the responsibility placed
overbear his will, disable him from making a free and by the Constitution upon the judiciary to insure
rational choice or impair his capacity for making rational that the Congress does not unjustifiably encroach
judgment would be sufficient. So is moral coercion upon an individual’s right to privacy nor abridge
tending to force testimony from the unwilling lips of the his liberty of speech, press, religion or assembly.”
defendant.” Needless to say, the above-mentioned (Watkins v. US, 354 USS 178 citing US v. Rumely,
provisions are an affirmation that “coercion can be mental 345 US 41)
as well as physical and that the blood of the accused is
not the only hallmark of an unconstitutional inquisition.” One of the basic rights guaranteed by the
(Blackburn v. Alabama, 361 US 199) Constitution to an individual is the right against self-
incrimination. This right construed as the right to remain
It bears emphasis, however, that under the completely silent may be availed of by the accused in a
above-quoted provisions, what is actually proscribed is the criminal case; but it may be invoked by other witnesses
use of physical or moral compulsion to extort only as questions are asked of them.
communication from the accused-appellant and not the
inclusion of his body in evidence when it may be material. This distinction is enunciated by the Court in
For instance, substance emitted from the body of the Romeo Chavez v. The Honorable Court of Appeals, et al.
accused may be received as evidence in prosecution for thus –
acts of lasciviousness (US v. Tan Teng, 23 Phil. 145
[1912]) and morphine forced out of the mouth of the “Petitioner, as accused, occupies a
accused may also be used as evidence against him (US v. different tier of protection from an ordinary
Ong Siu Hong, 36 Phil. 735 [1917]). Consequently, witness. Whereas an ordinary witness may be
although accused-appellant insists that hair samples were compelled to take the witness stand and claim the
forcibly taken from him and submitted to the NBI for privilege as each question requiring an
forensic examination, the hair samples may be admitted in incriminating answer is shot at him, an accused
evidence against him, for what is proscribed is the use of may altogether refuse to take the witness stand
testimonial compulsion or any evidence communicative in and refuse to answer any and all questions.”
nature acquired from the accused under duress. (People

113
Moreover, this right of the accused is extended to fundamental laws of all civilized countries. Over the
respondents in administrative investigations but only if years, however, came the need to assist government in its
they partake of the nature of a criminal proceeding or task of containing crime for peace and order is a
analogous to a criminal proceeding. In Galman v. necessary matrix of public welfare. To accommodate the
Pamaran, the Court reiterated the doctrine in Cabal v. need, the right against self-incrimination was stripped of
Kapunan to illustrate the right of witnesses to invoke the its absoluteness. Immunity statutes in varying shapes
right against self-incrimination not only in criminal were enacted which would allow government to compel a
proceedings but also in all other types of suit. witness to testify despite his plea of the right against self-
incrimination. To insulate these statutes from the virus of
It was held that: unconstitutionality, a witness is given what has come to
be known as transactional or a use-derivative-use
“We did not therein state that since he is immunity x x x. Quite clearly, these immunity statutes are
not an accused and the case is not a criminal not a bonanza from government. Those given the
case, Cabal cannot refuse to take the witness privilege of immunity paid a high price for it – the
stand and testify, and that he can invoke his right surrender of their precious right to be silent. Our
against self-incrimination only when a question hierarchy of values demands that the right against self-
which tends to elicit an answer that will incrimination and the right to be silent should be accorded
incriminate him is propounded to him. Clearly greater respect and protection. Laws that tend to erode
then, it is not the character of the suit involved the force of these preeminent rights must necessarily be
but the nature of the proceedings that controls. given a liberal interpretation in favor of the individual.
The privilege has consistently been held to extend The government has a right to solve crimes but it must do
to all proceedings sanctioned by law and to all it, rightly. (Mapa, Jr. v. Sandiganbayan, 231 SCRA
cases in which punishment is sought to be visited 783, 805-806, April 26, 1994, En Banc [Puno])
upon a witness, whether a party or not.” The Right against Double Jeopardy
255. Discuss the two kinds of double jeopardy.
We do not here modify these doctrines. If we
presently rule that petitioners may not be compelled by Held: Our Bill of Rights deals with two (2) kinds
the respondent Committee to appear, testify and produce of double jeopardy. The first sentence of Clause 20,
evidence before it, it is only because we hold that the Section 1, Article III of the Constitution ordains that “no
questioned inquiry is not in aid of legislation and, if person shall be twice put in jeopardy of punishment for
pursued, would be violative of the principle of separation the same offense.” The second sentence of said clause
of powers between the legislative and the judicial provides that “if an act is punishable by a law and an
departments of government, ordained by the Constitution. ordinance, conviction or acquittal under either shall
(Bengzon, Jr. v. Senate Blue Ribbon Committee, constitute a bar to another prosecution for the same act.”
203 SCRA 767, Nov. 20, 1991, En Banc [Padilla]) Thus, the first sentence prohibits double jeopardy of
253. What are the two types of immunity statutes? punishment for the same offense whereas, the second
Which has broader scope of protection? contemplates double jeopardy of punishment for the same
act. Under the first sentence, one may be twice put in
Held: Our immunity statutes are of American jeopardy of punishment of the same act, provided that he
origin. In the United States, there are two types of is charged with different offenses, or the offense charged
statutory immunity granted to a witness. They are the in one case is not included in, or does not include, the
transactional immunity and the use-and-derivative-use crime charged in the other case. The second sentence
immunity. Transactional immunity is broader in the scope applies, even if the offense charged are not the same,
of its protection. By its grant, a witness can no longer be owing to the fact that one constitutes a violation of an
prosecuted for any offense whatsoever arising out of the ordinance and the other a violation of statute. If the two
act or transaction. In contrast, by the grant of use-and- charges are based on one and the same act, conviction or
derivative-use immunity, a witness is only assured that his acquittal under either the law or the ordinance shall bar a
or her particular testimony and evidence derived from it prosecution under the other. Incidentally, such conviction
will not be used against him or her in a subsequent or acquittal is not indispensable to sustain the plea of
prosecution. (Mapa, Jr. v. Sandiganbayan, 231 SCRA double jeopardy of punishment or the same offense. So
783, 797-798, April 26, 1994, En Banc [Puno]) long as jeopardy has been attached under one of the
informations charging said offense, the defense may be
254. Is the grant of immunity to an accused willing to availed of in the other case involving the same offense,
testify for the government a special privilege and, even if there has been neither conviction nor acquittal in
therefore, must be strictly construed against the either case.
accused?
Elsewhere stated, where the offense charged are
Held: [W]e reject respondent court’s ruling that penalized either by different sections of the same statute
the grant of section 5 immunity must be strictly construed or by different statutes, the important inquiry relates to
against the petitioners. It simplistically characterized the the identity of offenses charged. The constitutional
grant as a special privilege, as if it was gifted by the protection against double jeopardy is available only where
government, ex gratia. In taking this posture, it misread an identity is shown to exist between the earlier and the
the raison d’ etre and the long pedigree of the right subsequent offenses charged. The question of identity or
against self-incrimination vis-à-vis immunity statutes. lack of identity of offenses is addressed by examining the
essential elements of each of the two offenses charged,
The days of inquisition brought about the most as such elements are set out in the respective legislative
despicable abuses against human rights. Not the least of definitions of the offenses involved. (People v. Quijada,
these abuses is the expert use of coerced confessions to 259 SCRA 191, July 24, 1996)
send to the guillotine even the guiltless. To guard against
the recurrence of this totalitarian method, the right 256. What must be proved to substantiate a claim of
against self-incrimination was ensconced in the double jeopardy? When may legal jeopardy attach?

114
However, this rule admits of two exceptions, namely:
Held: To substantiate a claim of double insufficiency of evidence and denial of the right to speedy
jeopardy, the following must be proven: trial. Double jeopardy may attach when the proceedings
have been prolonged unreasonably, in violation of the
(1) A first jeopardy must have attached prior to accused’s right to speedy trial. (Almario v. Court of
the second; (2) the first jeopardy must have been validly Appeals, 355 SCRA 1, Mar. 22, 2001, 2nd Div.
terminated; (3) the second jeopardy must be for the same [Quisumbing]
offense, or the second offense includes or is necessarily
included in the offense charged in the first information, or 259. If the criminal case was dismissed predicated on
is an attempt to commit the same or is a frustration the right of the accused to speedy trial, but later the
thereof. trial court reconsidered its decision and allowed the
case to be reinstated as it noted that the delay in the
Legal jeopardy attaches only: (1) upon a valid trial was due to circumstances beyond the control of
indictment; (b) before a competent court; (c) after the parties and of the trial court, i.e., the presiding
arraignment; (d) when a valid plea has been entered; and judge was promoted to the Court of Appeals, and his
(e) the case was dismissed or otherwise terminated successor as trial judge was not immediately
without the express consent of the accused. (Cuison v. appointed, nor another judge detailed to his sala, is
CA, 289 SCRA 159, April 15, 1998 [Panganiban]) there violation of the accused’s right against double
jeopardy?
257. In its decision in a criminal case, the Judge
promulgated only the civil aspect of the case, but not Held: Here we must inquire whether there was
the criminal. Will the promulgation of the criminal unreasonable delay in the conduct of the trial so that
aspect later constitute double jeopardy? violation of the right to speedy trial of the accused x x x
resulted. For it must be recalled that in the application of
Held: Petitioner contends that "the promulgation the constitutional guaranty of the right to speedy
by Judge Ramos on April 4, 1995 of the Respondent disposition of cases, particular regard must also be taken
Court's decision of June 30, 1991 by reading its dispositive of the facts and circumstances peculiar to each case.
portion has effectively terminated the criminal cases Both the trial court and the appellate court noted that
against the petitioner x x x." In other words, petitioner after pre-trial of petitioner’s case was terminated x x x
claims that the first jeopardy attached at that point. continuous trial was set x x x. The scheduled hearings,
however, were cancelled when the presiding judge was
The Court is not persuaded. As a rule, a criminal promoted to the Court of Appeals, and his successor as
prosecution includes a civil action for the recovery of trial judge was not immediately appointed, nor another
indemnity. Hence, a decision in such case disposes of judge detailed to his sala.
both the criminal as well as the civil liabilities of an Xxx
accused. Here, trial court promulgated only the civil
aspect of the case, but not the criminal. As observed by respondent appellate court, delay
in the trial was due to circumstances beyond the control
[T]he promulgation of the CA Decision was not of the parties and of the trial court. x x x. Thus, after a
complete. In fact and in truth, the promulgation was not closer analysis of these successive events, the trial court
merely incomplete; it was also void. In excess of its realized that the dates of the hearings were transferred
jurisdiction, the trial judge rendered a substantially for valid grounds. Hence, the trial court set aside its initial
incomplete promulgation on April 4, 1995, and he order and reinstated the cases against petitioner, which
repeated his mistake in his April 12, 1996 Order. We order the appellate court later sustained.
emphasize that grave abuse of discretion rendered the
aforementioned act of the trial court void. Since the That there was no unreasonable delay of the
criminal cases have not yet been terminated, the first proceedings is apparent from the chronology of the
jeopardy has not yet attached. Hence, double jeopardy hearings with the reasons for their postponements or
cannot prosper as a defense. transfers. x x x

We must stress that Respondent Court's There being no oppressive delay in the
questioned Decision did not modify or amend its July 30, proceedings, and no postponements unjustifiably sought,
1991 Decision. It merely ordered the promulgation of the we concur with the conclusion reached by the Court of
judgment of conviction and the full execution of the Appeals that petitioner’s right to speedy trial had not been
penalty it had earlier imposed on petitioner. (Cuison v. infringed. Where the right of the accused to speedy trial
CA, 289 SCRA 159, April 15, 1998 [Panganiban]) had not been violated, there was no reason to support the
initial order of dismissal.
258. What are the exceptions to the rule that the
dismissal of a criminal case resulting in acquittal made It follows that petitioner cannot invoke the
with the express consent of the accused or upon his constitutional right against double jeopardy when that
own motion will not place the accused in double order was reconsidered seasonably. For as petitioner’s
jeopardy? right to speedy trial was not transgressed, this exception
to the fifth element of double jeopardy – that the
Held: In the cases at bar, the order of dismissal defendant was acquitted or convicted, or the case was
based on a violation of the right to speedy trial was made dismissed or otherwise terminated without the express
upon motion by counsel for petitioner before the trial consent of the accused – was not met. The trial court’s
court. It was made at the instance of the accused before initial order of dismissal was upon motion of petitioner’s
the trial court, and with his express consent. Generally, counsel, hence made with the express consent of
the dismissal of a criminal case resulting in acquittal made petitioner. That being the case, despite the
with the express consent of the accused or upon his own reconsideration of said order, double jeopardy did not
motion will not place the accused in double jeopardy. attach. As this Court had occasion to rule in People v.

115
Tampal, reiterated in People v. Leviste, where we clearly show that no second criminal offense was being
overturned an order of dismissal by the trial court imputed to petitioner on appeal. In modifying the lower
predicated on the right to speedy trial – court’s judgment, the appellate court did not modify the
judgment of acquittal. Nor did it order the filing of a
It is true that in an unbroken line of second criminal cases against petitioner for the same
cases, we have held that the dismissal of cases on offense. Obviously, therefore, there was no second
the ground of failure to prosecute is equivalent to jeopardy to speak of. Petitioner’s claim of having been
an acquittal that would bar further prosecution of placed in double jeopardy is incorrect.
the accused for the same offense. It must be
stressed, however, that these dismissals were Our law recognizes two kinds of acquittal, with
predicated on the clear right of the accused to different effects on the civil liability of the accused. First
speedy trial. These cases are not applicable to is an acquittal on the ground that the accused is not the
the petition at bench considering that the right of author of the act or omission complained of. This
the private respondents to speedy trial has not instance closes the door to civil liability, for a person who
been violated by the State. For this reason, has been found to be not the perpetrator of any act or
private respondents cannot invoke their right omission cannot and can never be held liable for such or
against double jeopardy. omission. There being no delict, civil liability ex delicto is
out of the question, and the civil action, if any, which may
Both the trial court and the Court of Appeals were be instituted must be based on grounds other than the
thus not in error when they allowed reinstatement of the delict complained of. This is the situation contemplated in
cases against petitioner. (Almario v. Court of Appeals, Rule 111 of the Rules of Court. The second instance is an
355 SCRA 1, Mar. 22, 2001, 2nd Div. [Quisumbing] acquittal based on reasonable doubt on the guilt of the
accused. In this case, even if the guilt of the accused has
260. Is there double jeopardy when an accused was not been satisfactorily established, he is not exempt from
acquitted in a criminal case for reckless imprudence civil liability which may be proved by preponderance of
but the civil aspect of the case was elevated to the evidence only. This is the situation contemplated in
Court of Appeals and the latter found him liable for Article 29 of the Civil Code, where the civil action for
indemnity and damages? damages is “for the same act or omission.” Although the
two actions have different purposes, the matters
Held: Petitioner opines that the Court of Appeals discussed in the civil case are similar to those discussed in
should not have disturbed the findings of the trial court on the criminal case. However, the judgment in the criminal
the lack of negligence or reckless imprudence under the proceeding cannot be read in evidence in the civil action
guise of determining his civil liability. He argues that the to establish any fact there determined, even though both
trial court’s finding that he was neither imprudent nor actions involve the same act or omission. The reason for
negligent was the basis for his acquittal, and not this rule is that the parties are not the same and
reasonable doubt. He submits that in finding him liable secondarily, different rules of evidence are applicable.
for indemnity and damages, the appellate court not only Hence, notwithstanding herein petitioner’s acquittal, the
placed his acquittal in suspicion, but also put him in Court of Appeals in determining whether Article 29
“double jeopardy.” applied, was not precluded from looking into the question
of petitioner’s negligence or reckless imprudence.
Private respondents contend that while the trial (Manantan v. Court of Appeals, 350 SCRA 387, Jan.
court found that petitioner’s guilt had not been proven 29, 2001, 2nd Div. [Quisumbing])
beyond reasonable doubt, it did not state in clear and
unequivocal terms that petitioner was not recklessly The Right against Ex Post Facto Law and Bill of
imprudent or negligent. Hence, impliedly the trial court Attainder
acquitted him on reasonable doubt. Since civil liability is
not extinguished in criminal cases, if the acquittal is based 261. What is a bill of attainder? Is P.D. 1866 a bill of
on reasonable doubt, the Court of Appeals had to review attainder?
the findings of the trial court to determine if there was a
basis for awarding indemnity and damages. Held: [T]he Court, in People v. Ferrer, defined a
bill of attainder as a legislative act which inflicts
Preliminarily, petitioner’s claim that the decision of punishment on individuals or members of a particular
the appellate court awarding indemnity placed him in group without a judicial trial. Essential to a bill of
double jeopardy is misplaced. x x x. When a person is attainder are a specification of certain individuals or a
charged with an offense and the case is terminated either group of individuals, the imposition of a punishment,
by acquittal or conviction or in any manner without the penal or otherwise, and the lack of judicial trial. This last
consent of the accused, the latter cannot again be element, the total lack of court intervention in the finding
charged with the same or identical offense . This is double of guilt and the determination of the actual penalty to be
jeopardy. For double jeopardy to exist, the following imposed, is the most essential. P.D. No. 1866 does not
elements must be established: (1) a first jeopardy must possess the elements of a bill of attainder. It does not
have attached prior to the second; (2) the first jeopardy seek to inflict punishment without a judicial trial.
must have terminated; and (3) the second jeopardy must Nowhere in the measure is there a finding of guilt and an
be for the same offense as the first. In the instant case, imposition of a corresponding punishment. What the
petitioner had once been placed in jeopardy by the filing decree does is to define the offense and provide for the
of Criminal Case No. 066 and the jeopardy was terminated penalty that may be imposed, specifying the qualifying
by his discharge. The judgment of acquittal became circumstances that would aggravate the offense. There is
immediately final. Note, however, that what was elevated no encroachment on the power of the court to determine
to the Court of Appeals by private respondents was the after due hearing whether the prosecution has proved
civil aspect of Criminal Case No. 066. Petitioner was not beyond reasonable doubt that the offense of illegal
charged anew in CA-G.R. CV No. 19240 with a second possession of firearms has been committed and that the
criminal offense identical to the first offense. The records qualifying circumstances attached to it has been

116
established also beyond reasonable doubt as the LEX REVIEWS AND SEMINARS INC;
Constitution and judicial precedents require. (Misolas v. NATIONAL BAR REVIEW CENTER
Panga, 181 SCRA 648, 659-660, Jan. 30, 1990, En
Banc [Cortes])

262. What is an ex post facto law? Is R.A. No. 8249 an FOUR HUNDRED AND SEVENTY-NINE (479)
ex post facto law? QUESTIONS AND ANSWERS IN POLITICAL
LAW AND PUBLIC INTERNATIONAL LAW
Held: Ex post facto law, generally, prohibits (Culled from Significant Laws and Decisions of
retrospectivity of penal laws. R.A. 8249 is not a penal
the Supreme Court)
law. It is a substantive law on jurisdiction which is not
penal in character. Penal laws are those acts of the
Legislature which prohibit certain acts and establish Attorney EDWIN REY SANDOVAL
penalties for their violations; or those that define crimes, (As of August 25, 2006)
treat of their nature, and provide for their punishment.
R.A. 7975, which amended P.D. 1606 as regards the PART II
Sandiganbayan’s jurisdiction, its mode of appeal and other
procedural matters, has been declared by the Court as not C. ADMINISTRATIVE LAW
a penal law, but clearly a procedural statute, i.e., one
which prescribes rules of procedure by which courts 280. Describe the Administrative Code of 1987.
applying laws of all kinds can properly administer justice.
Not being a penal law, the retroactive application of R.A. Held: The Code is a general law and “incorporates in
8249 cannot be challenged as unconstitutional. a unified document the major structural, functional and
procedural principles of governance (Third Whereas Clause,
Petitioner’s and intervenors’ contention that their Administrative Code of 1987) and “embodies changes in
right to a two-tiered appeal which they acquired under administrative structures and procedures designed to serve the
R.A. 7975 has been diluted by the enactment of R.A. people.” (Fourth Whereas Clause, Administrative Code of 1987)
8249, is incorrect. The same contention has already been The Code is divided into seven (7) books. These books contain
provisions on the organization, powers and general
rejected by the court several times considering that the
administration of departments, bureaus and offices under the
right to appeal is not a natural right but statutory in
executive branch, the organization and functions of the
nature that can be regulated by law. The mode of Constitutional Commissions and other constitutional bodies, the
procedure provided for in the statutory right of appeal is rules on the national government budget, as well as guidelines
not included in the prohibition against ex post facto laws. for the exercise by administrative agencies of quasi-legislative
R.A. 8249 pertains only to matters of procedure, and and quasi-judicial powers. The Code covers both the internal
being merely an amendatory statute it does not partake administration, i.e., internal organization, personnel and
the nature of an ex post facto law. It does not mete out a recruitment, supervision and discipline, and the effects of the
penalty and, therefore, does not come within the functions performed by administrative officials on private
prohibition. Moreover, the law did not alter the rules of individuals or parties outside government. (Ople v. Torres,
evidence or the mode of trial. It has been ruled that G.R. No. 127685, July 23, 1998 [Puno])
adjective statutes may be made applicable to actions
pending and unresolved at the time of their passage. 281. What is Administrative Power?

At any rate, R.A. 8249 has preserved the Held: Administrative power is concerned with the
accused’s right to appeal to the Supreme Court to review work of applying policies and enforcing orders as determined by
questions of law. On the removal of the intermediate proper governmental organs. It enables the President to fix a
review of facts, the Supreme Court still has the power of uniform standard of administrative efficiency and check the
review to determine if the presumption of innocence has official conduct of his agents. To this end, he can issue
been convincingly overcome. (Panfilo M. Lacson v. administrative orders, rules and regulations. (Ople v. Torres,
G.R. No. 127685, July 23, 1998 [Puno])
The Executive Secretary, et. al., G.R. No. 128096,
Jan. 20, 1999 [Martinez])
282. What is an Administrative Order?
GOD BLESS Ǖ Ǖ Ǖ Held: An administrative order is an ordinance issued
by the President which relates to specific aspects in the
administrative operation of government. It must be in harmony
with the law and should be for the sole purpose of implementing
the law and carrying out the legislative policy. (Ople v. Torres,
G.R. No. 127685, July 23, 1998 [Puno])

283. What is the Government of the Republic of the


Philippines?

Ans.: The Government of the Republic of the


Philippines refers to the corporate governmental entity through
which the functions of the government are exercised throughout
the Philippines, including, save as the contrary appears from the
context, the various arms through which political authority is
made effective in the Philippines, whether pertaining to the
autonomous regions, the provincial, city, municipal or barangay
subdivisions or other forms of local government. (Sec. 2[1],
Introductory Provisions, Executive Order No. 292)

284. What is an Agency of the Government?


117
stock; x x x (Sec. 2[13], Introductory Provisions, Executive
Ans.: Agency of the Government refers to any of the Order No. 292)
various units of the Government, including a department,
bureau, office, instrumentality, or government-owned or 292. When is a Government-Owned or Controlled
controlled corporation, or a local government or a distinct unit Corporation deemed to be performing proprietary
therein. (Sec. 2[4], Introductory Provisions, Executive Order function? When is it deemed to be performing
No. 292) governmental function?

285. What is a Department? Held: Government-owned or controlled corporations


may perform governmental or proprietary functions or both,
Ans.: Department refers to an executive department depending on the purpose for which they have been created. If
created by law. For purposes of Book IV, this shall include any the purpose is to obtain special corporate benefits or earn
instrumentality, as herein defined, having or assigned the rank pecuniary profit, the function is proprietary. If it is in the
of a department, regardless of its name or designation. (Sec. interest of health, safety and for the advancement of public good
2[7], Introductory Provisions, Executive Order No. 292) and welfare, affecting the public in general, the function is
governmental. Powers classified as “proprietary” are those
286. What is a Bureau? intended for private advantage and benefit. (Blaquera v.
Alcala, 295 SCRA 366, 425, Sept. 11, 1998, En Banc
Ans.: Bureau refers to any principal subdivision or [Purisima])
unit of any department. For purposes of Book IV, this shall
include any principal subdivision or unit of any instrumentality 293. The Philippine National Red Cross (PNRC) is a
given or assigned the rank of a bureau, regardless of actual government-owned and controlled corporation with
name or designation, as in the case of department-wide regional an original charter under R.A. No. 95, as amended.
offices. (Sec. 2[8], Introductory Provisions, Executive Order Its charter, however, was amended to vest in it the
No. 292) authority to secure loans, be exempted from
payment of all duties, taxes, fees and other charges,
287. What is an Office? etc. With the amendnt of its charter, has it been
Ans.: Office refers, within the framework of “impliedly converted to a private corporation”?
governmental organization, to any major functional unit of a
department or bureau including regional offices. It may also Held: The test to determine whether a corporation is
refer to any position held or occupied by individual persons, government owned or controlled, or private in nature is simple.
whose functions are defined by law or regulation. (Sec. 2[9], Is it created by its own charter for the exercise of a public
Introductory Provisions, Executive Order No. 292) function, or by incorporation under the general corporation law?
Those with special charters are government corporations subject
288. What is a Government Instrumentality? What are to its provisions, and its employees are under the jurisdiction of
included in the term Government Instrumentality? the Civil Service Commission. The PNRC was not “impliedly
converted to a private corporation” simply because its charter
Ans.: A government instrumentality refers to any was amended to vest in it the authority to secure loans, be
agency of the national government, not integrated within the exempted from payment of all duties, taxes, fees and other
department framework, vested with special functions or charges, etc. (Camporedondo v. NLRC, G.R. No. 129049, Aug.
jurisdiction by law, endowed with some if not all corporate 6, 1999, 1st Div. [Pardo])
powers, administering special funds, enjoying operational
autonomy, usually through a charter. The term includes 294. When may the Government not validly invoke the
regulatory agencies, chartered institutions and government- rule that prescription does not run against the
owned or controlled corporations. (Sec. 2[10], Introductory State? Illustrative Case.
Provisions, Executive Order No. 292)
Held: While it is true that prescription does not run
289. What is a Regulatory Agency? against the State, the same may not be invoked by the
government in this case since it is no longer interested in the
Ans.: A regulatory agency refers to any agency subject matter. While Camp Wallace may have belonged to the
expressly vested with jurisdiction to regulate, administer or government at the time Rafael Galvez’s title was ordered
adjudicate matters affecting substantial rights and interest of cancelled in Land Registration Case No. N-361, the same no
private persons, the principal powers of which are exercised by longer holds true today.
a collective body, such as a commission, board or council. (Sec.
2[11], Introductory Provisions, Executive Order No. 292) Republic Act No. 7227, otherwise known as the Base
Conversion and Development Act of 1992, created the Bases
290. What is a Chartered Institution? Conversion and Development Authority. X x x

Ans.: A chartered institution refers to any agency Xxx


organized or operating under a special charter, and vested by
law with functions relating to specific constitutional policies or With the transfer of Camp Wallace to the BCDA, the
objectives. This term includes state universities and colleges government no longer has a right or interest to protect.
and the monetary authority of the State. (Section 2[12], Consequently, the Republic is not a real party in interest and it
Introductory Provisions, Executive Order No. 292) may not institute the instant action. Nor may it raise the defense
of imprescriptibility, the same being applicable only in cases
291. What is a Government-Owned or Controlled where the government is a party in interest. x x x. Being the
Corporation? owner of the areas covered by Camp Wallace, it is the Bases
Conversion and Development Authority, not the Government,
Ans.: Government-owned or controlled corporation which stands to be benefited if the land covered by TCT No. T-
refers to any agency organized as a stock or non-stock 5710 issued in the name of petitioner is cancelled.
corporation, vested with functions relating to public needs
whether governmental or proprietary in nature, and owned by Nonetheless, it has been posited that the transfer of
the Government directly or through its instrumentalities either military reservations and their extensions to the BCDA is
wholly, or, where applicable as in the case of stock corporations, basically for the purpose of accelerating the sound and balanced
to the extent of at least fifty-one (51) per cent of its capital conversion of these military reservations into alternative
118
productive uses and to enhance the benefits to be derived from Philippine Ports Authority against the petitioner therein were the
such property as a measure of promoting the economic and same. To dismiss the complaint in E.B. Marcha would have
social development, particularly of Central Luzon and, in brought needless delay in the settlement of the matter since the
general, the country’s goal for enhancement (Section 2, PPA would have to refile the case on the same claim already
Republic Act No. 7227). It is contended that the transfer of litigated upon. Such is not the case here since to allow the
these military reservations to the Conversion Authority does not government to sue herein enables it to raise the issue of
amount to an abdication on the part of the Republic of its imprescriptibility, a claim which is not available to the BCDA.
interests, but simply a recognition of the need to create a body The rule that prescription does not run against the State does not
corporate which will act as its agent for the realization of its apply to corporations or artificial bodies created by the State for
program. It is consequently asserted that the Republic remains special purposes, it being said that when the title of the Republic
to be the real party in interest and the Conversion Authority has been divested, its grantees, although artificial bodies of its
merely its agent. own creation, are in the same category as ordinary persons. By
raising the claim of imprescriptibility, a claim which cannot be
We, however, must not lose sight of the fact that the raised by the BCDA, the Government not only assists the
BCDA is an entity invested with a personality separate and BCDA, as it did in E.B. Marcha, it even supplants the latter, a
distinct from the government. X x x course of action proscribed by said case.

It may not be amiss to state at this point that the Moreover, to recognize the Government as a proper
functions of government have been classified into governmental party to sue in this case would set a bad precedent as it would
or constituent and proprietary or ministrant. While public allow the Republic to prosecute, on behalf of government-
benefit and public welfare, particularly, the promotion of the owned or controlled corporations, causes of action which have
economic and social development of Central Luzon, may be already prescribed, on the pretext that the Government is the
attributable to the operation of the BCDA, yet it is certain that real party in interest against whom prescription does not run,
the functions performed by the BCDA are basically proprietary said corporations having been created merely as agents for the
in nature. The promotion of economic and social development realization of government programs.
of Central Luzon, in particular, and the country’s goal for
enhancement, in general, do not make the BCDA equivalent to It should also be noted that petitioner is unquestionably
the Government. Other corporations have been created by a buyer in good faith and for value, having acquired the property
government to act as its agents for the realization of its in 1963, or 5 years after the issuance of the original certificate of
programs, the SSS, GSIS, NAWASA and the NIA, to count a title, as a third transferee. If only not to do violence and to give
few, and yet, the Court has ruled that these entities, although some measure of respect to the Torrens System, petitioner must
performing functions aimed at promoting public interest and be afforded some measure of protection. (Shipside
public welfare, are not government-function corporations Incorporated v. Court of Appeals, 352 SCRA 334, Feb. 20,
invested with governmental attributes. It may thus be said that 2001, 3rd Div. [Melo])
the BCDA is not a mere agency of the Government but a
corporate body performing proprietary functions. 295. Discuss the nature and functions of the National
Telecommunications Commission (NTC), and
Xxx analyze its powers and authority as well as the
laws, rules and regulations that govern its
Having the capacity to sue or be sued, it should thus be existence and operations.
the BCDA which may file an action to cancel petitioner’s title,
not the Republic, the former being the real party in interest. Held: The NTC was created pursuant to Executive
One having no right or interest to protect cannot invoke the Order No. 546 x x x. It assumed the functions formerly
jurisdiction of the court as a party plaintiff in an action. A suit assigned to the Board of Communications and the
may be dismissed if the plaintiff or the defendant is not a real Communications Control Bureau, which were both abolished
party in interest. x x x under the said Executive Order. Previously, the NTC’s function
were merely those of the defunct Public Service Commission
However, E.B. Marcha Transport Co., Inc. v. IAC is (PSC), created under Commonwealth Act No. 146, as amended,
cited as authority that the Republic is the proper party to sue for otherwise known as the Public Service Act, considering that the
the recovery of possession of property which at the time of the Board of Communications was the successor-in-interest of the
installation of the suit was no longer held by the national PSC. Under Executive Order No. 125-A, issued in April 1987,
government body but by the Philippine Ports Authrotiy. In E.B. the NTC became an attached agency of the Department of
Marcha, the Court ruled: Transportation and Communications.

It can be said that in suing for the recovery of In the regulatory communications industry, the NTC
the rentals, the Republic of the Philippines, acted as has the sole authority to issue Certificates of Public
principal of the Philippine Ports Authority, directly Convenience and Necessity (CPCN) for the installation,
exercising the commission it had earlier conferred on operation, and maintenance of communications facilities and
the latter as its agent. We may presume that, by doing services, radio communications systems, telephone and
so, the Republic of the Philippines did not intend to telegraph systems. Such power includes the authority to
retain the said rentals for its own use, considering that determine the areas of operations of applicants for
by its voluntary act it had transferred the land in telecommunications services. Specifically, Section 16 of the
question to the Philippine Ports Authority effective Public Service Act authorizes the then PSC, upon notice and
July 11, 1974. The Republic of the Philippines had hearing, to issue Certificates of Public Convenience for the
simply sought to assist, not supplant, the Philippine operation of public services within the Philippines “whenever
Ports Authority, whose title to the disputed property it the Commission finds that the operation of the public service
continues to recognize. We may expect the that the proposed and the authorization to do business will promote the
said rentals, once collected by the Republic of the public interests in a proper and suitable manner.”
Philippines, shall be turned over by it to the Philippine (Commonwealth Act No. 146, Section 16[a]) The procedure
Ports Authority conformably to the purposes of P.D. governing the issuance of such authorizations is set forth in
No. 857. Section 29 of the said Act x x x. (Republic v. Express
Telecommunication Co., Inc., 373 SCRA 316, Jan. 15, 2002,
E.B. Marcha is, however, not on all fours with the case 1st Div. [Ynares-Santiago])
at bar. In the former, the Court considered the Republic a
proper party to sue since the claims of the Republic and the
119
296. Is the filing of the administrative rules and Administrative Order, legally, until it is published, is
regulations with the UP Law Center the operative invalid within the context of Article 2 of Civil Code,
act that gives the rules force and effect? which reads:

Held: In granting Bayantel the provisional authority to “Article 2. Laws shall take effect
operate a CMTS, the NTC applied Rule 15, Section 3 of its after fifteen days following the completion of
1978 Rules of Practice and Procedure, which provides: their publication in the Official Gazette (or in
a newspaper of general circulation in the
Sec. 3. Provisional Relief. – Upon the filing Philippines), unless it is otherwise provided.
of an application, complaint or petition or at any stage X x x”
thereafter, the Board may grant on motion of the
pleader or on its own initiative, the relief prayed for, The fact that the amendments to
based on the pleading, together with the affidavits and Administrative Order No. SOCPEC 89-08-01 were
supporting documents attached thereto, without filed with, and published by the UP Law Center in the
prejudice to a final decision after completion of the National Administrative Register, does not cure the
hearing which shall be called within thirty (30) days defect related to the effectivity of the Administrative
from grant of authority asked for. Order.

Respondent Extelcom, however, contends that the NTC This Court, in Tanada v. Tuvera stated, thus:
should have applied the Revised Rules which were filed with
the Office of the National Administrative Register on February “We hold therefore that all statutes,
3, 1993. These Revised Rules deleted the phrase “on its own including those of local application and private laws, shall be
initiative”; accordingly, a provisional authority may be issued published as a condition for their effectivity, which shall begin
only upon filing of the proper motion before the Commission. fifteen days after publication unless a different effectivity is
fixed by the legislature.
In answer to this argument, the NTC, through the
Secretary of the Commission, issued a certification to the effect Covered by this rule are presidential
that inasmuch as the 1993 Revised Rules have not been decrees and executive orders promulgated by the President in
published in a newspaper of general circulation, the NTC has the exercise of legislative power or, at present, directly
been applying the 1978 Rules. conferred by the Constitution. Administrative Rules and
Regulations must also be published if their purpose is to
The absence of publication, coupled with the enforce or implement existing law pursuant also to a valid
certification by the Commissioner of the NTC stating that the delegation.
NTC was still governed by the 1987 Rules, clearly indicate that
the 1993 Revised Rules have not taken effect at the time of the Interpretative regulations and those
grant of the provisional authority to Bayantel. The fact that the merely internal in nature, that is, regulating only the personnel
1993 Revised Rules were filed with the UP Law Center on of the administrative agency and not the public, need not be
February 3, 1993 is of no moment. There is nothing in the published. Neither is publication required of the so-called
Administrative Code of 1987 which implies that the filing of the letters of instructions issued by administrative superiors
rules with the UP Law Center is the operative act that gives the concerning the rules or guidelines to be followed by their
rules force and effect. Book VII, Chapter 2, Section 3 thereof subordinates in the performance of their duties.
merely states:
Xxx
Filing. – (1) Every agency shall file with the
University of the Philippines Law Center three (3) We agree that the publication must
certified copies of every rule adopted by it. Rules in be in full or it is no publication at all since its purpose is to
force on the date of effectivity of this Code which are inform the public of the contents of the laws.”
not filed within three (3) months from the date shall not
thereafter be the basis of any sanction against any party The Administrative Order under consideration
or persons. is one of those issuances which should be published for
its effectivity, since its purpose is to enforce and
(2) The records officer of the agency, or his implement an existing law pursuant to a valid
equivalent functionary, shall carry out the requirements delegation, i.e., P.D. 1071, in relation to LOI 444 and
of this section under pain of disciplinary action. EO 133.

(3) A permanent register of all rules shall be Thus, publication in the Official Gazette or a
kept by the issuing agency and shall be open to public newspaper of general circulation is a condition sine qua non
inspection. before statutes, rules or regulations can take effect. This is
explicit from Executive Order No. 200, which repealed Article 2
The National Administrative Register is merely a of the Civil Code, and which states that:
bulletin of codified rules and it is furnished only to the Office of
the President, Congress, all appellate courts, the National Laws shall take effect after fifteen days
Library, other public offices or agencies as the Congress may following the completion of their publication either in the
select, and to other persons at a price sufficient to cover Official Gazette or in a newspaper of general circulation in the
publication and mailing or distribution costs (Administrative Philippines, unless it is otherwise provided (E.O. 200, Section
Code of 1987, Book VII, Chapter 2, Section 7). In a similar 1).
case, we held:
The Rules of Practice and Procedure of the NTC,
This does not imply, however, that the subject which implements Section 29 of the Public Service Act, fall
Administrative Order is a valid exercise of such quasi- squarely within the scope of these laws, as explicitly mentioned
legislative power. The original Administrative Order in the case of Tanada v. Tuvera.
issued on August 30, 1989, under which the
respondents filed their applications for importations, Our pronouncement in Tanada v. Tuvera is
was not published in the Official Gazette or in a clear and categorical. Administrative rules and regulations
newspaper of general circulation. The questioned must be published if their purpose is to enforce or implement
120
existing law pursuant to a valid delegation. The only exception National Administrative Register as required by the
are interpretative regulations, those merely internal in nature, Administrative Code of 1987.
or those so-called letters of instructions issued by
administrative superiors concerning the rules and guidelines to POEA Memorandum Circular No. 2, Series of 1983
be followed by their subordinates in the performance of their must likewise be declared ineffective as the same was never
duties (PHILSA International Placement & Services Corp. v. published or filed with the National Administrative Register.
Secretary of Labor, G.R. No. 103144, April 4, 2001, 356 SCRA
174). POEA Memorandum Circular No. 2, Series of 1983
provides for the applicable schedule of placement and
Hence, the 1993 Revised Rules should be published in documentation fees for private employment agencies or
the Official Gazette or in a newspaper of general circulation authority holders. Under the said Order, the maximum amount
before it can take effect. Even the 1993 Revised Rules itself which may be collected from prospective Filipino overseas
mandates that said Rules shall take effect only after their workers is P2,500.00. The said circular was apparently issued
publication in a newspaper of general circulation (Section 20 in compliance with the provisions of Article 32 of the Labor
thereof). In the absence of such publication, therefore, it is the Code x x x.
1978 Rules that govern. (Republic v. Express
Telecommunication Co., Inc., 373 SCRA 316, Jan. 15, 2002, It is thus clear that the administrative circular under
1st Div. [Ynares-Santiago]) consideration is one of those issuances which should be
published for its effectivity, since its purpose is to enforce and
297. May a person be held liable for violation of an implement an existing law pursuant to a valid delegation.
administrative regulation which was not Considering that POEA Administrative Circular No. 2, Series of
published? 1983 has not as yet been published or filed with the National
Administrative Register, the same is ineffective and may not be
Held: Petitioner insists, however, that it cannot be enforced. (Philsa International Placement and Services
held liable for illegal exaction as POEA Memorandum Circular Corporation v. Secretary of Labor and Employment, 356
No. II, Series of 1983, which enumerated the allowable fees SCRA 174, April 4, 2001, 3rd Div., [Gonzaga-Reyes])
which may be collected from applicants, is void for lack of
publication. 298. Does the publication requirement apply as well to
administrative regulations addressed only to a
There is merit in the argument. specific group and not to the general public?

In Tanada v. Tuvera, the Court held, as follows: Held: The Office of the Solicitor General likewise
argues that the questioned administrative circular is not among
“We hold therefore that all statutes, including those requiring publication contemplated by Tanada v. Tuvera
those of local application and private laws, shall be published as as it is addressed only to a specific group of persons and not to
a condition for their effectivity, which shall begin fifteen days the general public.
after publication unless a different effectivity date is fixed by
the legislature. Again, there is no merit in this argument.

Covered by this rule are presidential decrees The fact that the said circular is addressed only to a
and executive orders promulgated by the President in the specified group, namely private employment agencies or
exercise of legislative powers whenever the same are validly authority holders, does not take it away from the ambit of our
delegated by the legislature or, at present, directly conferred by ruling in Tanada v. Tuvera. In the case of Phil. Association of
the Constitution. Administrative rules and regulations must also Service Exporters v. Torres, the administrative circulars
be published if their purpose is to enforce or implement existing questioned therein were addressed to an even smaller group,
law pursuant to a valid delegation. namely Philippine and Hong Kong agencies engaged in the
recruitment of workers for Hong Kong, and still the Court ruled
Interpretative regulations and those merely therein that, for lack of proper publication, the said circulars
internal in nature, that is, regulating only the personnel of the may not be enforced or implemented.
administrative agency and the public, need not be published.
Neither is publication required of the so-called letter of Our pronouncement in Tanada v. Tuvera is clear and
instructions issued by the administrative superiors concerning categorical. Administrative rules and regulations must be
the rules or guidelines to be followed by their subordinates in published if their purpose is to enforce or implement existing
the performance of their duties.” law pursuant to a valid delegation. The only exceptions are
interpretative regulations, those merely internal in nature, or
Applying this doctrine, we have previously declared as those so-called letters of instructions issued by administrative
having no force and effect the following administrative superiors concerning the rules and guidelines to be followed by
issuances: a) Rules and Regulations issued by the Joint Ministry their subordinates in the performance of their duties.
of Health-Ministry of Labor and Employment Accreditation Administrative Circular No. 2, Series of 1983 has not been
Committee regarding the accreditation of hospitals, medical shown to fall under any of these exceptions.
clinics and laboratories; b) Letter of Instruction No. 416
ordering the suspension of payments due and payable by In this regard, the Solicitor General’s reliance on the
distressed copper mining companies to the national government; case of Yaokasin v. Commissioner of Customs is misplaced. In
c) Memorandum Circulars issued by the POEA regulating the the said case, the validity of certain Customs Memorandum
recruitment of domestic helpers to Hong Kong; d) Orders were upheld despite their lack of publication as they
Administrative Order No. SOCPEC 89-08-01 issued by the were addressed to a particular class of persons, the customs
Philippine International Trading Corporation regulating collectors, who were also the subordinates of the Commissioner
applications for importation from the People’s Republic of of the Bureau of Customs. As such, the said Memorandum
China; and e) Corporate Compensation Circular No. 10 issued Orders clearly fall under one of the exceptions to the publication
by the Department of Budget and Management discontinuing requirement, namely those dealing with instructions from an
the payment of other allowances and fringe benefits to administrative superior to a subordinate regarding the
government officials and employees. In all these cited cases, performance of their duties, a circumstance which does not
the administrative issuances questioned therein were uniformly obtain in the case at bench.
struck down as they were not published or filed with the
Xxx
121
certified available funds, the same covers only Phase I of the
To summarize, petitioner should be absolved from the VRIS Project, i.e., the issuance of identification cards for only
three (3) counts of exaction as POEA Administrative Circular 1,000,000 voters in specified areas. In effect, the
No. 2, Series of 1983 could not be the basis of administrative implementation of the VRIS Project will be “segmented” or
sanctions against petitioner for lack of publication. (Philsa “chopped” into several phases. Not only is such arrangement
International Placement and Services Corporation v. Secretary disallowed by our budgetary laws and practices, it is also
of Labor and Employment, 356 SCRA 174, April 4, 2001, 3 rd disadvantageous to the COMELEC because of the uncertainty
Div., [Gonzaga-Reyes]) that will loom over its modernization project for an indefinite
period of time. Should Congress fail to appropriate the amount
299. May a successful bidder compel a government necessary for the completion of the entire project, what good
agency to formalize a contract with it will the accomplished Phase I serve? As expected, the project
notwithstanding that its bid exceeds the amount failed “to sell” with the Department of Budget and
appropriated by Congress for the project? Management. Thus, Secretary Benjamin Diokno, per his letter
of December 1, 2000, declined the COMELEC’s request for the
Held: Enshrined in the 1987 Philippine Constitution is issuance of the Notice of Cash Availability (NCA) and a multi-
the mandate that “no money shall be paid out of the Treasury year obligatory authority to assume payment of the total VRIS
except in pursuance of an appropriation made by law.” (Sec. Project for lack of legal basis. Corollarily, under Section 33 of
29[1], Article VI of the 1987 Constitution) Thus, in the R.A. No. 8760, no agency shall enter into a multi-year contract
execution of government contracts, the precise import of this without a multi-year obligational authority, thus:
constitutional restriction is to require the various agencies to
limit their expenditures within the appropriations made by law “SECTION 33. Contracting Multi-Year
for each fiscal year. Projects. - In the implementation of multi-year projects, no
agency shall enter into a multi-year contract without a multi-
Xxx year Obligational Authority issued by the Department of Budget
and Management for the purpose. Notwithstanding the issuance
It is quite evident from the tenor of the language of the of the multi-year Obligational Authority, the obligation to be
law that the existence of appropriations and the availability of incurred in any given calendar year, shall in no case exceed the
funds are indispensable pre-requisites to or conditions sine qua amount programmed for implementation during said calendar
non for the execution of government contracts. The obvious year.”
intent is to impose such conditions as a priori requisites to the
validity of the proposed contract. Using this as our premise, we Petitioners are justified in refusing to formalize the
cannot accede to PHOTOKINA’s contention that there is contract with PHOTOKINA. Prudence dictated them not to
already a perfected contract. While we held in Metropolitan enter into a contract not backed up by sufficient appropriation
Manila Development Authority v. Jancom Environmental and available funds. Definitely, to act otherwise would be a
Corporation that “the effect of an unqualified acceptance of the futile exercise for the contract would inevitably suffer the vice
offer or proposal of the bidder is to perfect a contract, upon of nullity. x x x
notice of the award to the bidder,” however, such statement
would be inconsequential in a government where the acceptance Xxx
referred to is yet to meet certain conditions. To hold otherwise
is to allow a public officer to execute a binding contract that Verily, the contract, as expressly declared by law, is
would obligate the government in an amount in excess of the inexistent and void ab initio (Article 1409 of the Civil Code of
appropriations for the purpose for which the contract was the Philippines). This is to say that the proposed contract is
attempted to be made. This is a dangerous precedent. without force and effect from the very beginning or from its
incipiency, as if it had never been entered into, and hence,
In the case at bar, there seems to be an oversight of the cannot be validated either by lapse of time or ratification.
legal requirements as early as the bidding stage. The first step
of a Bids and Awards Committee (BAC) is to determine Xxx
whether the bids comply with the requirements. The BAC shall
rate a bid “passed” only if it complies with all the requirements In fine, we rule that PHOTOKINA, though the winning
and the submitted price does not exceed the approved budget for bidder, cannot compel the COMELEC to formalize the contract.
the contract.” (Implementing Rules and Regulations [IRR] for Since PHOTOKINA’s bid is beyond the amount appropriated
Executive Order No. 262, supra.) by Congress for the VRIS Project, the proposed contract is not
binding upon the COMELEC and is considered void x x x.
Extant on the record is the fact that the VRIS Project (Commission on Elections v. Judge Ma. Luisa Quijano-
was awarded to PHOTOKINA on account of its bid in the Padilla, G.R. No. 151992, Sept. 18, 2002, En Banc [Sandoval-
amount of P6.588 Billion Pesos. However, under Republic Act Gutierrez])
No. 8760 (General Appropriations Act, FY 2000, p. 1018,
supra.), the only fund appropriated for the project was P1 300. What is the remedy available to a party who
Billion Pesos and under the Certification of Available Funds contracts with the government contrary to the
(CAF) only P1.2 Billion Pesos was available. Clearly, the requirements of the law and, therefore, void ab
amount appropriated is insufficient to cover the cost of the initio?
entire VRIS Project. There is no way that the COMELEC could
enter into a contract with PHOTOKINA whose accepted bid Held: Of course, we are not saying that the party who
was way beyond the amount appropriated by law for the project. contracts with the government has no other recourse in law.
This being the case, the BAC should have rejected the bid for The law itself affords him the remedy. Section 48 of E.O. No.
being excessive or should have withdrawn the Notice of Award 292 explicitly provides that any contract entered into contrary to
on the ground that in the eyes of the law, the same is null and the above-mentioned requirements shall be void, and “the
void. officers entering into the contract shall be liable to the
Government or other contracting party for any consequent
Xxx damage to the same as if the transaction had been wholly
between private parties.” So when the contracting officer
Even the draft contract submitted by Commissioner transcends his lawful and legitimate powers by acting in excess
Sadain that provides for a contract price in the amount of P1.2 of or beyond the limits of his contracting authority, the
Billion Pesos is unacceptable. x x x While the contract price Government is not bound under the contract. It would be as if
under the draft contract is only P1.2 Billion and, thus, within the the contract in such case were a private one, whereupon, he
122
binds himself, and thus, assumes personal liability thereunder.
Otherwise stated, the proposed contract is unenforceable as to “Evidently, the ‘preventive measures and legal
the Government. aid services’ mentioned in the Constitution refer to
extrajudicial and judicial remedies (including a writ of
While this is not the proceeding to determine where the preliminary injunction) which the CHR may seek from
culpability lies, however, the constitutional mandate cited above the proper courts on behalf of the victims of human
constrains us to remind all public officers that public office is a rights violations. Not being a court of justice, the CHR
public trust and all public officers must at all times be itself has no jurisdiction to issue the writ, for a writ of
accountable to the people. The authority of public officers to preliminary injunction may only be issued ‘by the
enter into government contracts is circumscribed with a heavy judge of any court in which the action is pending
burden of responsibility. In the exercise of their contracting [within his district], or by a Justice of the Court of
prerogative, they should be the first judges of the legality, Appeals, or of the Supreme Court. x x x. A writ of
propriety and wisdom of the contract they entered into. They preliminary injunction is an ancillary remedy. It is
must exercise a high degree of caution so that the Government available only in a pending principal action, for the
may not be the victim of ill-advised or improvident action. preservation or protection of the rights and interest of a
(Commission on Elections v. Judge Ma. Luisa Quijano- party thereto, and for no other purpose.”
Padilla, G.R. No. 151992, Sept. 18, 2002, En Banc [Sandoval-
Gutierrez]) The Commission does have legal standing to indorse, for
appropriate action, its findings and recommendations to any
301. Does the Commission on Human Rights have the appropriate agency of government. (Simon, Jr. v. Commission
power to adjudicate? on Human Rights, 229 SCRA 117, 134-135, Jan. 5, 1994, En
Banc [Vitug, J.])
Held: In its Order x x x denying petitioners’ motion to
dismiss, the CHR theorizes that the intention of the members of 303. Does the petition for annulment of proclamation of
the Constitutional Commission is to make CHR a quasi-judicial a candidate merely involve the exercise by the
body. This view, however, has not heretofore been shared by COMELEC of its administrative power to review,
this Court. In Carino v. Commission on Human Rights, the revise and reverse the actions of the board of
Court x x x has observed that it is “only the first of the canvassers and, therefore, justifies non-observance
enumerated powers and functions that bears any resemblance to of procedural due process, or does it involve the
adjudication of adjudgment,” but that resemblance can in no exercise of the COMELEC's quasi-judicial
way be synonymous to the adjudicatory power itself. The Court function?
explained:
Held: Taking cognizance of private respondent's
“x x x [T]he Commission on Human Rights x petitions for annulment of petitioner's proclamation, COMELEC
x x was not meant by the fundamental law to be was not merely performing an administrative function. The
another court or quasi-judicial agency in this country, administrative powers of the COMELEC include the power to
or duplicate much less take over the functions of the determine the number and location of polling places, appoint
latter. election officials and inspectors, conduct registration of voters,
deputize law enforcement agencies and governmental
“The most that may be conceded to the instrumentalities to ensure free, orderly, honest, peaceful and
Commission in the way of adjudicative power is that it may credible elections, register political parties, organizations or
investigate, i.e., receive evidence and make findings of fact as coalition, accredit citizen's arms of the Commission, prosecute
regards claimed human rights violations involving civil and election offenses, and recommend to the President the removal
political rights. But fact finding is not adjudication, and cannot of or imposition of any other disciplinary action upon any
be likened to the judicial function of a court of justice, or even officer or employee it has deputized for violation or disregard of
a quasi-judicial agency or official. The function of receiving its directive, order or decision. In addition, the Commission
evidence and ascertaining therefrom the facts of a controversy also has direct control and supervision over all personnel
is not a judicial function, properly speaking. To be considered involved in the conduct of election. However, the resolution of
such, the faculty of receiving evidence and making factual the adverse claims of private respondent and petitioner as
conclusions in a controversy must be accompanied by the regards the existence of a manifest error in the questioned
authority of applying the law to those factual conclusions to the certificate of canvass requires the COMELEC to act as an
end that the controversy may be decided or determined arbiter. It behooves the Commission to hear both parties to
authoritatively, finally and definitively, subject to such appeals determine the veracity of their allegations and to decide whether
or modes of review as may be provided by law. This function, the alleged error is a manifest error. Hence, the resolution of
to repeat, the Commission does not have. this issue calls for the exercise by the COMELEC of its quasi-
(Simon, Jr. v. Commission on Human Rights, 229 SCRA 117, judicial power. It has been said that where a power rests in
125, Jan. 5, 1994, En Banc [Vitug, J.]) judgment or discretion, so that it is of judicial nature or
character, but does not involve the exercise of functions of a
302. Does the Commission on Human Rights have judge, or is conferred upon an officer other than a judicial
jurisdiction to issue TRO or writ of preliminary officer, it is deemed quasi-judicial. The COMELEC therefore,
injunction? acting as quasi-judicial tribunal, cannot ignore the requirements
of procedural due process in resolving the petitions filed by
Held: In Export Processing Zone Authority v. private respondent. (Federico S. Sandoval v. COMELEC, G.R.
Commission on Human Rights, the Court x x x explained: No. 133842, Jan. 26, 2000 [Puno])

“The constitutional provision directing the 304. Discuss the contempt power of the Commission on
CHR to ‘provide for preventive measures and legal aid Human Rights (CHR). When may it be validly
services to the underprivileged whose human rights exercised?
have been violated or need protection’ may not be
construed to confer jurisdiction on the Commission to Held: On its contempt powers, the CHR is
issue a restraining order or writ of injunction for, if that constitutionally authorized to “adopt its operational guidelines
were the intention, the Constitution would have and rules of procedure, and cite for contempt for violations
expressly said so. ‘Jurisdiction is conferred only by the thereof in accordance with the Rules of Court.” Accordingly,
Constitution or by law.’ It is never derived by the CHR acted within its authority in providing in its revised
implication.” rules, its power “to cite or hold any person in direct or indirect
123
contempt, and to impose the appropriate penalties in accordance when there is estoppel on the part of the administrative
with the procedure and sanctions provided for in the Rules of agency concerned;
Court.” That power to cite for contempt, however, should be when there is irreparable injury;
understood to apply only to violations of its adopted operational when the respondent is a department secretary whose
guidelines and rules of procedure essential to carry out its acts as an alter ego of the President bears the
investigatorial powers. To exemplify, the power to cite for implied and assumed approval of the latter;
contempt could be exercised against persons who refuse to when to require exhaustion of administrative remedies
cooperate with the said body, or who unduly withhold relevant would be unreasonable;
information, or who decline to honor summons, and the like, in when it would amount to a nullification of a claim;
pursuing its investigative work. The “order to desist” (a when the subject matter is a private land in land case
semantic interplay for a restraining order) in the instance before proceeding;
us, however, is not investigatorial in character but prescinds when the rule does not provide a plain, speedy and
from an adjudicative power that it does not possess. x x x adequate remedy, and
(Simon, Jr. v. Commission on Human Rights, 229 SCRA 117, when there are circumstances indicating the urgency of
134, Jan. 5, 1994, En Banc [Vitug, J.]) judicial intervention.
(Paat v. CA, 266 SCRA 167 [1997])
305. Discuss the Doctrine of Primary Jurisdiction (or
Prior Resort). 2. Non-exhaustion of administrative remedies is not
jurisdictional. It only renders the action premature, i.e., claimed
Held: Courts cannot and will not resolve a controversy cause of action is not ripe for judicial determination and for that
involving a question which is within the jurisdiction of an reason a party has no cause of action to ventilate in court.
administrative tribunal, especially where the question demands (Carale v. Abarintos, 269 SCRA 132, March 3, 1997, 3 rd Div.
the exercise of sound administrative discretion requiring the [Davide])
special knowledge, experience and services of the
administrative tribunal to determine technical and intricate
matters of fact. D. THE LAW OF PUBLIC OFFICERS

In recent years, it has been the jurisprudential trend to 307. Define Appointment. Discuss its nature.
apply this doctrine to cases involving matters that demand the
special competence of administrative agencies even if the Held: An “appointment” to a public office is the
question involved is also judicial in character. It applies “where unequivocal act of designating or selecting by one having the
a claim is originally cognizable in the courts, and comes into authority therefor of an individual to discharge and perform the
play whenever enforcement of the claim requires the resolution duties and functions of an office or trust. The appointment is
of issues which, under a regulatory scheme, have been placed deemed complete once the last act required of the appointing
within the special competence of an administrative body; in authority has been complied with and its acceptance thereafter
such case, the judicial process is suspended pending referral of by the appointee in order to render it effective. Appointment
such issues to the administrative body for its view.” necessarily calls for an exercise of discretion on the part of the
appointing authority. In Pamantasan ng Lungsod ng Maynila v.
In cases where the doctrine of primary jurisdiction is Intermediate Appellate Court, reiterated in Flores v. Drilon, this
clearly applicable, the court cannot arrogate unto itself the Court has held:
authority to resolve a controversy, the jurisdiction over which is
lodged with an administrative body of special competence. “The power to appoint is, in essence,
(Villaflor v. CA, 280 SCRA 297, Oct. 9, 1992, 3 rd Div. discretionary. The appointing power has the right of
[Panganiban]) choice which he may exercise freely according to his
judgment, deciding for himself who is best qualified
306. Discuss the Doctrine of Exhaustion of among those who have the necessary qualifications and
Administrative Remedies. What are the exceptions eligibilities. It is a prerogative of the appointing power
thereto. x x x.”

Held: 1. Before a party is allowed to seek the Indeed, it may rightly be said that the right of choice is the heart
intervention of the court, it is a pre-condition that he should of the power to appoint. In the exercise of the power of
have availed of all the means of administrative processes appointment, discretion is an integral thereof. (Bermudez v.
afforded him. Hence, if a remedy within the administrative Torres, 311 SCRA 733, Aug. 4, 1999, 3 rd Div. [Vitug])
machinery can still be resorted to by giving the administrative
officer concerned every opportunity to decide on a matter that 308. May the Civil Service Commission, or the Supreme
comes within his jurisdiction then such remedy should be Court, validly nullify an appointment on the
exhausted first before the court’s judicial power can be sought. ground that somebody else is better qualified?
The premature invocation of court’s jurisdiction is fatal to one’s
cause of action. Accordingly, absent any finding of waiver or Held: The head of an agency who is the appointing
estoppel the case is susceptible of dismissal for lack of cause of power is the one most knowledgeable to decide who can best
action. This doctrine of exhaustion of administrative remedies perform the functions of the office. Appointment is an
was not without its practical and legal reasons, for one thing, essentially discretionary power and must be performed by the
availment of administrative remedy entails lesser expenses and officer vested with such power according to his best lights, the
provides for a speedier disposition of controversies. It is no less only condition being that the appointee should possess the
true to state that the courts of justice for reasons of comity and qualifications required by law. If he does, then the appointment
convenience will shy away from a dispute until the system of cannot be faulted on the ground that there are others better
administrative redress has been completed and complied with so qualified who should have been preferred. Indeed, this is a
as to give the administrative agency concerned every prerogative of the appointing authority which he alone can
opportunity to correct its error and to dispose of the case. decide. The choice of an appointee from among those who
This doctrine is disregarded: possess the required qualifications is a political and
administrative decision calling for considerations of wisdom,
when there is a violation of due process; convenience, utility and the interests of the service which can
when the issue involved is purely a legal question; best be made by the head of the office concerned, the person
when the administrative action is patently illegal most familiar with the organizational structure and
amounting to lack or excess of jurisdiction;
124
environmental circumstances within which the appointee must appropriate eligibility prescribed.” Achacoso did not. At best,
function. therefore, his appointment could be regarded only as
temporary. And being so, it could be withdrawn at will by the
As long as the appointee is qualified the Civil Service appointing authority and “at a moment’s notice,” conformably
Commission has no choice but to attest to and respect the to established jurisprudence.
appointment even if it be proved that there are others with
superior credentials. The law limits the Commission’s authority The Court, having considered these
only to whether or not the appointees possess the legal submissions and the additional arguments of the parties
qualifications and the appropriate civil service eligibility, in the petitioner’s Reply and of the Solicitor-General’s
nothing else. If they do then the appointments are approved Rejoinder, must find for the respondents.
because the Commission cannot exceed its power by
substituting its will for that of the appointing authority. Neither The mere fact that a position belongs to the
can we. (Rimonte v. CSC, 244 SCRA 504-505, May 29, 1995, Career Service does not automatically confer security of tenure
En Banc [Bellosillo, J.]) in its occupant even if he does not possess the required
qualifications. Such right will have to depend on the nature of
309. Does the “next-in-rank” rule import any his appointment, which in turn depends on his eligibility or
mandatory or peremptory requirement that the lack of it. A person who does not have the requisite
person next-in-rank must be appointed to the qualifications for the position cannot be appointed to it in the
vacancy? first place or, only as an exception to the rule, may be
appointed to it merely in an acting capacity in the absence of
Held: The “next-in-rank rule is not absolute; it only appropriate eligibles. The appointment extended to him cannot
applies in cases of promotion, a process which denotes a scalar be regarded as permanent even if it may be so designated.
ascent of an officer to another position higher either in rank or
salary. And even in promotions, it can be disregarded for Evidently, private respondent’s appointment did not
sound reasons made known to the next-in-rank, as the concept attain permanency. Not having taken the necessary Career
does not import any mandatory or peremptory requirement that Executive Service examination to obtain the requisite eligibility,
the person next-in-rank must be appointed to the vacancy. The he did not at the time of his appointment and up to the present,
appointing authority, under the Civil Service Law, is allowed to possess the needed eligibility for a position in the Career
fill vacancies by promotion, transfer of present employees, Executive Service. Consequently, his appointment as Ministry
reinstatement, reemployment, and appointment of outsiders who Legal Counsel-CESO IV/Department Legal Counsel and/or
have appropriate civil service eligibility, not necessarily in that Director III, was merely temporary. Such being the case, he
order. There is no legal fiat that a vacancy must be filled only could be transferred or reassigned without violating the
by promotion; the appointing authority is given wide discretion constitutionally guaranteed right to security of tenure.
to fill a vacancy from among the several alternatives provided
by law. Private respondent capitalizes on his lack of CES
eligibility by adamantly contending that the mobility and
What the Civil Service Law provides is that if a flexibility concepts in the assignment of personnels under the
vacancy is filled by promotion, the person holding the position Career Executive Service do not apply to him because he s not a
next in rank thereto “shall be considered for promotion.” Career Executive Service Officer. Obviously, the contention is
without merit. As correctly pointed out by the Solicitor
In Taduran v. Civil Service Commission, the Court General, non-eligibles holding permanent appointments to CES
construed that phrase to mean that the person next-in-rank positions were never meant to remain immobile in their status.
“would be among the first to be considered for the vacancy, if Otherwise, their lack of eligibility would be a premium vesting
qualified.” In Santiago, Jr. v. Civil Service Commission, the them with permanency in the CES positions, a privilege even
Court elaborated the import of the rule in the following manner: their eligible counterparts do not enjoy.

“One who is next-in-rank is entitled to Then too, the cases on unconsented transfer invoked by
preferential consideration for promotion to the higher private respondent find no application in the present case. To
vacancy but it does not necessarily follow that he and reiterate, private respondent’s appointment is merely temporary;
no one else can be appointed. The rule neither grants a hence, he could be transferred or reassigned to other positions
vested right to the holder nor imposes a ministerial without violating his right to security of tenure. (De Leon v.
duty on the appointing authority to promote such Court of Appeals, 350 SCRA 1, Jan. 22, 2001, En Banc
person to the next higher position x x x” [Ynares-Santiago])
(Abila v. CSC, 198 SCRA 102, June 3, 1991, En Banc
[Feliciano]) 311. In the career executive service, is a career
executive service (CES) eligibility all that an
310. Can a person who lacks the necessary employee needs to acquire security of tenure? Is
qualifications for a public position be appointed to appointment to a CES rank necessary for the
it in a permanent capacity? Illustrative case. acquisition of such security of tenure?

Held: At the outset, it must be stressed that the Held: In the career executive service, the acquisition
position of Ministry Legal Counsel-CESO IV is embraced in the of security of tenure which presupposes a permanent
Career Executive Service. X x x appointment is governed by the rules and regulations
promulgated by the CES Board x x x.
In the case at bar, there is no question that private
respondent does not have the required CES eligibility. As As clearly set forth in the foregoing provisions, two
admitted by private respondent in his Comment, he is “not a requisites must concur in order that an employee in the career
CESO or a member of the Career Executive Service.” executive service may attain security of tenure, to wit:

In the case of Achacoso v. Macaraig, et al., the Court CES eligibility; and
held: Appointment to the appropriate CES rank.

It is settled that a permanent appointment can In addition, it must be stressed that the security of
be issued only “to a person who meets all the requirements for tenure of employees in the career executive service (except first
the position to which he s being appointed, including the and second level employees in the civil service), pertains only to
125
rank and not to the office or to the position to which they may freedom from misgivings or betrayals of personal trust or
be appointed. Thus, a career executive service officer may be confidential matters of state. (De los Santos v. Mallare, 87 Phil.
transferred or reassigned from one position to another without 289 [1950])
losing his rank which follows him wherever he is transferred or
reassigned. In fact, a CESO suffers no diminution of salary Under the proximity rule, the occupant of a particular
even if assigned to a CES position with lower salary grade, as he position could be considered a confidential employee if the
is compensated according to his CES rank and not on the basis predominant reason why he was chosen by the appointing
of the position or office he occupies. authority was the latter’s belief that he can share a close intimate
relationship with the occupant which ensures freedom of
In the case at bar, there is no question that respondent discussion without fear or embarrassment or misgivings of
Ramon S. Roco, though a CES eligible, does not possess the possible betrayal of personal trust or confidential matters of
appropriate CES rank, which is – CES rank level V, for the state. Withal, where the position occupied is more remote from
position of Regional Director of the LTO (Region V). Falling that of the appointing authority, the element of trust between
short of one of the qualifications that would complete his them is no longer predominant. (CSC v. Salas, 274 SCRA 414,
membership in the CES, respondent cannot successfully June 19, 1997)
interpose violation of security of tenure. Accordingly, he could
be validly reassigned to other positions in the career executive 314. Does the Civil Service Law contemplate a review of
service. x x x decisions exonerating officers or employees from
administrative charges?
Moreover, under the mobility and flexibility principles
of the Integrated Reorganization Plan, CES personnel may be Held: By this ruling, we now expressly abandon and
reassigned or transferred from one position to another x x x. overrule extant jurisprudence that “the phrase ‘party adversely
affected by the decision’ refers to the government employee
One last point. Respondent capitalizes on the fact that against whom the administrative case is filed for the purpose of
petitioner Luis Mario M. General is not a CES eligible. The disciplinary action which may take the form of suspension,
absence, however, of such CES eligibility is of no moment. As demotion in rank or salary, transfer, removal or dismissal from
stated in Part III, Chapter I, Article IV, paragraph 5(c), of the office” and not included are “cases where the penalty imposed is
Integrated Reorganization Plan – suspension for not more than thirty (30) days or fine in an
amount not exceeding thirty days salary” (Paredes v. Civil
“x x x the President may, in exceptional cases, Service Commission, 192 SCRA 84, 85) or “when respondent is
appoint any person who is not a Career Executive exonerated of the charges, there is no occasion for appeal.”
Service eligible; provided that such appointee shall (Mendez v. Civil Service Commission, 204 SCRA 965, 968) In
subsequently take the required Career Executive other words, we overrule prior decisions holding that the Civil
Service examination and that he shall not be promoted Service Law “does not contemplate a review of decisions
to a higher class until he qualified in such exonerating officers or employees from administrative charges”
examination.” enunciated in Paredes v. Civil Service Commission (192 SCRA
84); Mendez v. Civil Service Commission (204 SCRA 965);
Evidently, the law allows appointment of those who are Magpale v. Civil Service Commission (215 SCRA 398); Navarro
not CES eligible, subject to the obtention of said eligibility, in v. Civil Service Commission and Export Processing Zone
the same manner that the appointment of respondent who does Authority (226 SCRA 207) and more recently Del Castillo v.
not possess the required CES rank (CES rank level V) for the Civil Service Commission (237 SCRA 184). (CSC v. Pedro O.
position of Regional Director of the LTO, is permitted in a Dacoycoy, G.R. No. 135805, April 29, 1999, En Banc [Pardo])
temporary capacity. (General v. Roco, 350 SCRA 528, Jan. 29,
2001, 1st Div. [Ynares-Santiago]) 315. What is preventive suspension? Discuss its nature.

312. How are positions in the Civil Service classified? Held: Imposed during the pendency of an
Discuss the characteristics of each. administrative investigation, preventive suspension is not a
penalty in itself. It is merely a measure of precaution so that the
Ans.: Positions in the Civil Service may be classified employee who is charged may be separated, for obvious
into: 1) Career Positions, and 2) Non-Career Positions. reasons, from the scene of his alleged misfeasance while the
same is being investigated. Thus preventive suspension is
Career Positions are characterized by (1) entrance distinct from the administrative penalty of removal from office
based on merit and fitness to be determined as far as practicable such as the one mentioned in Sec. 8(d) of P.D. No. 807. While
by competitive examination, or based on highly technical the former may be imposed on a respondent during the
qualifications; (2) opportunity for advancement to higher career investigation of the charges against him, the latter is the penalty
positions; and (3) security of tenure (Sec. 7, Chap. 2, Subtitle A, which may only be meted upon him at the termination of the
Title I, Bk. V, E.O. No. 292). investigation or the final disposition of the case. (Beja, Sr. v.
CA, 207 SCRA 689, March 31, 1992 [Romero])
The Non-Career Service shall be characterized by (1)
entrance on bases other than of the usual tests of merit or fitness 316. Discuss the kinds of preventive suspension under
utilized for the career service; and (2) tenure which is limited to the Civil Service Law. When may a civil service
a period specified by law, or which is coterminous with that of employee placed under preventive suspension be
the appointing authority or subject to his pleasure, or which is entitled to compensation?
limited to the duration of a particular project for which purpose
employment was made (Sec. 9, Chap. 2, Subtitle A, Title I, Bk. Held: There are two kinds of preventive suspension of
V, E.O. No. 292). civil service employees who are charged with offenses
punishable by removal or suspension: (1) preventive suspension
313. What is a primarily confidential position? What is pending investigation (Sec. 51, Civil Service Law, EO No. 292)
the test to determine whether a position is primarily and (2) preventive suspension pending appeal if the penalty
confidential or not? imposed by the disciplining authority is suspension or dismissal
and, after review, the respondent is exonerated (Section 47, par.
Held: A primarily confidential position is one which 4, Civil Service Law, EO No. 292).
denotes not only confidence in the aptitude of the appointee for
the duties of the office but primarily close intimacy which Preventive suspension pending investigation is not a
ensures freedom from intercourse without embarrassment or penalty. It is a measure intended to enable the disciplining
126
authority to investigate charges against respondent by
preventing the latter from intimidating or in any way influencing There can be no question in this case as to the power
witnesses against him. If the investigation is not finished and a and authority of respondent Deputy Ombudsman to issue an
decision is not rendered within that period, the suspension will order of preventive suspension against an official like the
be lifted and the respondent will automatically be reinstated. If petitioner, to prevent that official from using his office to
after investigation respondent is found innocent of the charges intimidate or influence witnesses (Gloria v. CA, et al., G.R. No.
and is exonerated, he should be reinstated. However, no 131012, April 21, 1999, p. 7, 306 SCRA 287) or to tamper with
compensation was due for the period of preventive suspension records that might be vital to the prosecution of the case against
pending investigation. The Civil Service Act of 1959 (R.A. No. him (Yasay, Jr. v. Desierto, et al., G.R. No. 134495, December
2260) providing for compensation in such a case once the 28, 1998, p. 9, 300 SCRA 494). In our view, the present
respondent was exonerated was revised in 1975 and the controversy simply boils down to this pivotal question: Given
provision on the payment of salaries during suspension was the purpose of preventive suspension and the circumstances of
deleted. this case, did respondent Deputy Ombudsman commit a grave
abuse of discretion when he set the period of preventive
But although it is held that employees who are suspension at six months?
preventively suspended pending investigation are not entitled to
the payment of their salaries even if they are exonerated, they Preventive suspension under Sec. 24, R.A. 6770 x x x
are entitled to compensation for the period of their suspension may be imposed when, among other factors, the evidence of
pending appeal if eventually they are found innocent. guilt is strong. The period for which an official may be
preventively suspended must not exceed six months. In this
Preventive suspension pending investigation x x x is case, petitioner was preventively suspended and ordered to
not a penalty but only a means of enabling the disciplining cease and desist from holding office for the entire period of six
authority to conduct an unhampered investigation. On the other months, which is the maximum provided by law.
hand, preventive suspension pending appeal is actually punitive
although it is in effect subsequently considered illegal if The determination of whether or not the evidence of
respondent is exonerated and the administrative decision finding guilt is strong as to warrant preventive suspension rests with the
him guilty is reversed. Hence, he should be reinstated with full Ombudsman. The discretion as regards the period of such
pay for the period of the suspension. (Gloria v. CA, G.R. No. suspension also necessarily belongs to the Ombudsman, except
131012, April 21, 1999, En Banc [Mendoza]) that he cannot extend the period of suspension beyond that
provided by law. But, in our view, both the strength of the
317. Discuss the power of Ombudsman to conduct evidence to warrant said suspension and the propriety of the
administrative investigations, and to impose length or period of suspension imposed on petitioner are
preventive suspension. properly raised in this petition for certiorari and prohibition. X
xx
Held: Worth stressing, to resolve the present
controversy, we must recall that the authority of the Xxx
Ombudsman to conduct administrative investigations is
mandated by no less than the Constitution. x x x Given these findings, we cannot say now that there is
no evidence sufficiently strong to justify the imposition of
R.A. 6770, the Ombudsman Law, further grants the preventive suspension against petitioner. But considering its
Office of the Ombudsman the statutory power to conduct purpose and the circumstances in the case brought before us, it
administrative investigations. x x x does appear to us that the imposition of the maximum period of
six months is unwarranted.
Section 21 of R.A. 6770 names the officials subject to
the Ombudsman’s disciplinary authority x x x. X x x [G]ranting that now the evidence against
petitioner is already strong, even without conceding that initially
Petitioner is an elective local official accused of grave it was weak, it is clear to us that the maximum six-month period
misconduct and dishonesty. That the Office of the Ombudsman is excessive and definitely longer than necessary for the
may conduct an administrative investigation into the acts Ombudsman to make its legitimate case against petitioner. We
complained of, appears clear from the foregoing provisions of must conclude that the period during which petitioner was
R.A. 6770. already preventively suspended, has been sufficient for the
lawful purpose of preventing petitioner from hiding and
However, the question of whether or not the destroying needed documents, or harassing and preventing
Ombudsman may conduct an investigation over a particular act witnesses who wish to appear against him. (Garcia v. Mojica,
or omission is different from the question of whether or not 314 SCRA 207, Sept. 10, 1999, 2nd Div. [Quisumbing])
petitioner, after investigation, may be held administratively
liable. This distinction ought here to be kept in mind even as we 318. Distinguish preventive suspension under the Local
must also take note that the power to investigate is distinct from Government Code from preventive suspension
the power to suspend preventively an erring public officer. under the Ombudsman Act.
Held: We reach the foregoing conclusion, however,
Likewise worthy of note, the power of the Office of the without necessarily subscribing to petitioner’s claim that the
Ombudsman to preventively suspend an official subject to its Local Government Code, which he averred should apply to this
administrative investigation is provided by specific provision of case of an elective local official, has been violated. True, under
law. x x x said Code, preventive suspension may only be imposed after the
issues are joined, and only for a maximum period of sixty days.
We have previously interpreted the phrase “under his Here, petitioner was suspended without having had the chance
authority” to mean that the Ombudsman can preventively to refute first the charges against him, and for the maximum
suspend all officials under investigation by his office, regardless period of six months provided by the Ombudsman Law. But as
of the branch of government in which they are employed, respondents argue, administrative complaints commenced under
excepting of course those removable by impeachment, members the Ombudsman Law are distinct from those initiated under the
of Congress and the Judiciary. Local Government Code. Respondents point out that the shorter
period of suspension under the Local Government Code is
The power to preventively suspend is available not intended to limit the period of suspension that may be imposed
only to the Ombudsman but also to the Deputy Ombudsman. by a mayor, a governor, or the President, who may be motivated
This is the clear import of Section 24 of R.A. 6770 abovecited. by partisan political considerations. In contrast the
127
Ombudsman, who can impose a longer period of preventive Miriam Defensor-Santiago? Will the order of
suspension, is not likely to be similarly motivated because it is a suspension prescribed by Republic Act No. 3019
constitutional body. The distinction is valid but not decisive, in not encroach on the power of Congress to
our view, of whether there has been grave abuse of discretion in discipline its own ranks under the Constitution?
a specific case of preventive suspension.
Held: The petition assails the authority of the
Xxx Sandiganbayan to decree a ninety-day preventive suspension of
Mme. Miriam Defensor-Santiago, a Senator of the Republic of
Respondents may be correct in pointing out the reason the Philippines, from any government position, and furnishing a
for the shorter period of preventive suspension imposable under copy thereof to the Senate of the Philippines for the
the Local Government Code. Political color could taint the implementation of the suspension order.
exercise of the power to suspend local officials by the mayor,
governor, or President’s office. In contrast the Ombudsman, The authority of the Sandiganbayan to order the
considering the constitutional origin of his Office, always ought preventive suspension of an incumbent public official charged
to be insulated from the vagaries of politics, as respondents with violation of the provisions of Republic Act No. 3019 has
would have us believe. both legal and jurisprudential support. X x x

In Hagad v. Gozo-Dadole, on the matter of whether or In the relatively recent case of Segovia v.
not the Ombudsman has been stripped of his power to Sandiganbayan, the Court reiterated:
investigate local elective officials by virtue of the Local
Government Code, we said: “The validity of Section 13, R.A. 3019, as
amended – treating of the suspension pendente lite of
“Indeed, there is nothing in the Local Government an accused public officer – may no longer be put at
Code to indicate that it has repealed, whether expressly or issue, having been repeatedly upheld by this Court.
impliedly, the pertinent provisions of the Ombudsman Act.
The two statutes on the specific matter in question are not “X x x
so inconsistent, let alone irreconcilable, as to compel us to
only uphold one and strike down the other.” “The provision of suspension pendente lite
applies to all persons indicted upon a valid information
It was also argued in Hagad, that the six-month under the Act, whether they be appointive or elective
preventive suspension under the Ombudsman Law is “much too officials; or permanent or temporary employees, or
repugnant” to the 60-day period that may be imposed under the pertaining to the career or non-career service.” (At pp.
Local Government Code. But per J. Vitug, “the two provisions 336-337)
govern differently.”
It would appear, indeed, to be a ministerial duty of the
However, petitioner now contends that Hagad did not court to issue an order of suspension upon determination of the
settle the question of whether a local elective official may be validity of the information filed before it. Once the information
preventively suspended even before the issues could be joined. is found to be sufficient in form and substance, the court is
Indeed it did not, but we have held in other cases that there bound to issue an order of suspension as a matter of course, and
could be preventive suspension even before the charges against there seems to be “no ifs and buts about it.” Explaining the
the official are heard, or before the official is given an nature of the preventive suspension, the Court in the case of
opportunity to prove his innocence. Preventive suspension is Bayot v. Sandiganbayan:
merely a preliminary step in an administrative investigation and
is not in any way the final determination of the guilt of the “x x x It is not a penalty because it is not
official concerned. imposed as a result of judicial proceedings. In fact, if
acquitted, the official concerned shall be entitled to
Petitioner also avers that the suspension order against reinstatement and to the salaries and benefits which he
him was issued in violation of Section 26[2] of the Ombudsman failed to receive during suspension.”
Law x x x.
In issuing the preventive suspension of petitioner, the
Petitioner argues that before an inquiry may be Sandiganbayan merely adhered to the clear and unequivocal
converted into a full-blown administrative investigation, the mandate of the law, as well as the jurisprudence in which the
official concerned must be given 72 hours to answer the charges Court has, more than once, upheld Sandiganbayan’s authority to
against him. In his case, petitioner says the inquiry was decree the suspension of public officials and employees indicted
converted into an administrative investigation without him being before it.
given the required number of hours to answer.
Section 13 of Republic Act No. 3019 does not state that
Indeed, it does not appear that petitioner was given the the public officer concerned must be suspended only in the
requisite 72 hours to submit a written answer to the complaint office where he is alleged to have committed the acts with
against him. This, however, does not make invalid the which he has been charged. Thus, it has been held that the use
preventive suspension order issued against him. As we have of the word “office” would indicate that it applies to any office
earlier stated, a preventive suspension order may be issued even which the officer charged may be holding, and not only the
before the charges against the official concerned is heard. particular office under which he stands accused. (Bayot v.
Sandiganbayan, supra; Segovia v. Sandiganbayan, supra.)
Moreover, respondents state that petitioner was given
10 days to submit his counter-affidavit to the complaint filed by En passant, while the imposition of suspension is not
respondent Tagaan. We find this 10-day period is in keeping automatic or self-operative as the validity of the information
with Section 5[a] of the Rules of Procedure of the Office of the must be determined in a pre-suspension hearing, there is no hard
Ombudsman x x x. (Garcia v. Mojica, 314 SCRA 207, Sept. 10, and fast rule as to the conduct thereof. It has been said that –
1999, 2nd Div. [Quisumbing])
“ ‘x x x No specific rules need be laid down
319. Does Section 13, Republic Act No. 3019 exclude for such pre-suspension hearing. Suffice it to state that the
from its coverage the members of Congress and, accused should be given a fair and adequate opportunity to
therefore, the Sandiganbayan erred in decreeing challenge the VALIDITY OF THE CRIMINAL
the preventive suspension order against Senator PROCEEDINGS against him, e.g., that he has not been afforded
128
the right of due preliminary investigation; that the acts for which The suspension contemplated in the above
he stands charged do not constitute a violation of the provisions constitutional provision is a punitive measure that is imposed
of Republic Act 3019 or the bribery provisions of the Revised upon determination by the Senate or the House of
Penal Code which would warrant his mandatory suspension Representatives, as the case may be, upon an erring member.
from office under Section 13 of the Act; or he may present a Thus, in its resolution in the case of Ceferino Paredes, Jr. v.
motion to quash the information on any of the grounds provided Sandiganbayan, et al., the Court affirmed the order of
for in Rule 117 of the Rules of Court x x x.’ suspension of Congressman Paredes by the Sandiganbayan,
despite his protestations on the encroachment by the court on
“x x x the prerogatives of Congress. The Court ruled:

“Likewise, he is accorded the right to “x x x Petitioner’s invocation of Section 16


challenge the propriety of his prosecution on the ground that the (3), Article VI of the Constitution – which deals with the power
acts for which he is charged do not constitute a violation of Rep. of each House of Congress inter alia to ‘punish its Members for
Act 3019, or of the provisions on bribery of the Revised Penal disorderly behavior,’ and ‘suspend or expel a Member’ by a vote
Code, and the right to present a motion to quash the information of two-thirds of all its Members subject to the qualification that
on any other grounds provided in Rule 117 of the Rules of the penalty of suspension, when imposed, should not exceed sixty
Court. days – in unavailing, as it appears to be quite distinct from the
suspension spoken of in Section 13 of RA 3019, which is not a
“However, a challenge to the validity of the penalty but a preliminary, preventive measure, prescinding from
criminal proceedings on the ground that the acts for the fact that the latter is not being imposed on petitioner for
which the accused is charged do not constitute a misbehavior as a Member of the House of Representatives.”
violation of the provisions of Rep. Act No. 3019, or of The doctrine of separation of powers by itself may not
the provisions on bribery of the Revised Penal Code, be deemed to have effectively excluded Members of Congress
should be treated only in the same manner as a from Republic Act No. 3019 nor from its sanctions. The maxim
challenge to the criminal proceeding by way of a simply recognizes each of the three co-equal and independent,
motion to quash on the ground provided in Paragraph albeit coordinate, branches of the government – the Legislative,
(a), Section 2 of Rule 117 of the Rules of Court, i.e., the Executive and the Judiciary – has exclusive prerogatives and
that the facts charged do not constitute an offense. In cognizance within its own sphere of influence and effectively
other words, a resolution of the challenge to the prevents one branch from unduly intruding into the internal
validity of the criminal proceeding, on such ground, affairs of either branch.
should be limited to an inquiry whether the facts
alleged in the information, if hypothetically admitted, Parenthetically, it might be well to elaborate a bit.
constitute the elements of an offense punishable under Section 1, Article VIII, of the 1987 Constitution, empowers the
Rep. Act 3019 or the provisions on bribery of the Court to act not only in the settlement of “actual controversies
Revised Penal Code.” (Luciano v. Mariano, 40 SCRA involving rights which are legally demandable and enforceable,”
187 [1971]; People v. Albano, 163 SCRA 511, 517-519 but also in the determination of “whether or not there has been a
[1988]) grave abuse of discretion amounting to lack or excess of
jurisdiction on the part of any branch or instrumentality of the
The law does not require that the guilt of the accused government.” The provision allowing the Court to look into any
must be established in a pre-suspension proceeding before trial possible grave abuse of discretion committed by any
on the merits proceeds. Neither does it contemplate a government instrumentality has evidently been couched in
proceeding to determine (1) the strength of the evidence of general terms in order to make it malleable to judicial
culpability against him, (2) the gravity of the offense charged, or interpretation in the light of any emerging milieu. In its normal
(3) whether or not his continuance in office could influence the concept, the term has been said to imply an arbitrary, despotic,
witnesses or pose a threat to the safety and integrity of the capricious or whimsical exercise of judgment amounting to lack
records and other evidence before the court could have a valid or excess of jurisdiction. When the question, however, pertains
basis in decreeing preventive suspension pending the trial of the to an affair internal to either of Congress or the Executive, the
case. All it secures to the accused is adequate opportunity to Court subscribes to the view that unless an infringement of any
challenge the validity or regularity of the proceedings against specific Constitutional proscription thereby inheres the Court
him, such as, that he has not been afforded the right to due should not deign substitute its own judgment over that of any of
preliminary investigation, that the acts imputed to him do not the other two branches of government. It is an impairment or a
constitute a specific crime warranting his mandatory suspension clear disregard of a specific constitutional precept or provision
from office under Section 13 of Republic Act No. 3019, or that that can unbolt the steel door for judicial intervention. If any
the information is subject to quashal on any of the grounds set part of the Constitution is not, or ceases to be, responsive to
out in Section 3, Rule 117, of the Revised Rules on Criminal contemporary needs, it is the people, not the Court, who must
Procedure. promptly react in the manner prescribed by the Charter itself.

Xxx Republic Act No. 3019 does not exclude from its
coverage the members of Congress and that, therefore, the
The pronouncement, upholding the validity of the Sandiganbayan did not err in thus decreeing the assailed
information filed against petitioner, behooved Sandiganbayan to preventive suspension order.
discharge its mandated duty to forthwith issue the order of
preventive suspension. Attention might be called to the fact that Criminal Case
No. 16698 has been decided by the First Division of the
The order of suspension prescribed by Republic Act Sandiganbayan on 06 December 1999, acquitting herein
No. 3019 is distinct from the power of Congress to discipline its petitioner. The Court, nevertheless, deems it appropriate to
own ranks under the Constitution which provides that each – render this decision for future guidance on the significant issue
raised by petitioner. (Santiago v. Sandiganbayan, 356 SCRA
“x x x house may determine the rules of its 636, April 18, 2001, En Banc [Vitug])
proceedings, punish its Members for disorderly behavior, and,
with the concurrence of two-thirds of all its Members, suspend 320. May an elective public official be validly appointed
or expel a Member. A penalty of suspension, when imposed, or designated to any public office or position
shall not exceed sixty days.” (Section 16[3], Article VI, 1987 during his tenure?
Constitution)

129
Ans.: No elective official shall be eligible for (13); Ruben B. Ancheta and Jose A. Rono of twelve (12) each;
appointment or designation in any capacity to any public office Manuel P. Alba, Gilberto O. Teodoro, and Edgardo Tordesillas
or position during his tenure. (Sec. 7, 1st par., Art. IX-B, 1987 of eleven (11) each; and Lilia Bautista and Teodoro Q. Pena of
Constitution) ten (10) each.

321. May an appointive public official hold any other The blatant betrayal of public trust evolved into one of
office or employment? the serious causes of discontent with the Marcos regime. It was
therefore quite inevitable and in consonance with the
Ans.: Unless otherwise allowed by law or by the overwhelming sentiment of the people that the 1986
primary functions of his position, no appointive official shall Constitutional Commission, convened as it was after the people
hold any other office or employment in the Government or any successfully unseated former President Marcos, should draft
subdivision, agency or instrumentality thereof, including into its proposed Constitution the provisions under consideration
government-owned or controlled corporation. (Sec. 7, 2nd par., which are envisioned to remedy, if not correct, the evils that
Art. IX-B, 1987 Constitution) flow from the holding of multiple governmental offices and
employment. X x x
322. May the President, Vice-President, Members of the
Cabinet, their deputies or assistants hold any other But what is indeed significant is the fact that although
office or employment? Section 7, Article IX-B already contains a blanket prohibition
against the holding of multiple offices or employment in the
Ans.: The President, Vice-President, the Members of government subsuming both elective and appointive public
the Cabinet, and their deputies or assistants shall not, unless officials, the Constitutional Commission should see it fit to
otherwise provided in this Constitution, hold any other office or formulate another provision, Sec. 13, Article VII, specifically
employment during their tenure. (Sec. 13, Art. VII, 1987 prohibiting the President, Vice-President, members of the
Constitution) Cabinet, their deputies and assistants from holding any other
office or employment during their tenure, unless otherwise
323. Does the prohibition in Section 13, Article VII of provided in the Constitution itself.
the 1987 Constitution insofar as Cabinet members,
their deputies or assistants are concerned admit of Evidently, from this move as well as in the different
the broad exceptions made for appointive officials phraseologies of the constitutional provisions in question, the
in general under Section 7, par. (2), Article IX-B? intent of the framers of the Constitution was to impose a stricter
prohibition on the President and his official family in so far as
Held: The threshold question therefore is: does the holding other offices or employment in the government or
prohibition in Section 13, Article VII of the 1987 Constitution elsewhere is concerned.
insofar as Cabinet members, their deputies or assistants are
concerned admit of the broad exceptions made for appointive Moreover, such intent is underscored by a comparison
officials in general under Section 7, par. (2), Article IX-B of Section 13, Article VII with other provisions of the
which, for easy reference is quoted anew, thus: “Unless Constitution on the disqualifications of certain public officials
otherwise allowed by law or by the primary functions of his or employees from holding other offices or employment. Under
position, no appointive official shall hold any other office or Section 13, Article VI, “[N]o Senator or Member of the House
employment in the government or any subdivision, agency or of Representatives may hold any other office or employment in
instrumentality thereof, including government-owned or the Government x x x.” Under section 5(4), Article XVI, “[N]o
controlled corporation or their subsidiaries.” member of the armed forces in the active service shall, at any
time, be appointed in any capacity to a civilian position in the
We rule in the negative. Government, including government-owned or controlled
corporations or any of their subsidiaries.” Even Section 7(2),
Xxx Article IX-B, relied upon by respondents provides “[U]nless
otherwise allowed by law or by the primary functions of his
The practice of designating members of the Cabinet, position, no appointive official shall hold any other office or
their deputies and assistants as members of the governing bodies employment in the Government.”
or boards of various government agencies and instrumentalities,
including government-owned and controlled corporations, It is quite notable that in all these provisions on
became prevalent during the time legislative powers in this disqualifications to hold other office or employment, the
country were exercised by former President Ferdinand E. prohibition pertains to an office or employment in the
Marcos pursuant to his martial law authority. There was a government and government-owned or controlled corporations
proliferation of newly-created agencies, instrumentalities and or their subsidiaries. In striking contrast is the wording of
government-owned and controlled corporations created by Section 13, Article VII which states that “[T]he President, Vice-
presidential decrees and other modes of presidential issuances President, the Members of the Cabinet, and their deputies or
where Cabinet members, their deputies or assistants were assistants shall not, unless otherwise provided in this
designated to head or sit as members of the board with the Constitution, hold any other office or employment during their
corresponding salaries, emoluments, per diems, allowances and tenure.” In the latter provision, the disqualification is absolute,
other perquisites of office. X x x not being qualified by the phrase “in the Government.” The
prohibition imposed on the President and his official family is
This practice of holding multiple offices or positions in therefore all-embracing and covers both public and private
the government soon led to abuses by unscrupulous public office or employment.
officials who took advantage of this scheme for purposes of
self-enrichment. X x x Going further into Section 13, Article VII, the second
sentence provides: “They shall not, during said tenure, directly
Particularly odious and revolting to the people’s sense or indirectly, practice any other profession, participate in any
of propriety and morality in government service were the data business, or be financially interested in any contract with, or in
contained therein that Roberto v. Ongpin was a member of the any franchise, or special privilege granted by the Government or
governing boards of twenty-nine (29) governmental agencies, any subdivision, agency or instrumentality thereof, including
instrumentalities and corporations; Imelda R. Marcos of twenty- government-owned or controlled corporations or their
three (23); Cesar E.A. Virata of twenty-two (22); Arturo R. subsidiaries.” These sweeping, all-embracing prohibitions
Tanco, Jr. of fifteen (15); Jesus S. Hipolito and Geronimo Z. imposed on the President and his official family, which
Velasco, of fourteen each (14); Cesar C. Zalamea of thirteen prohibitions are not similarly imposed on other public officials
130
or employees such as the Members of Congress, members of the understood as intended to be a positive and unequivocal
civil service in general and members of the armed forces, are negation (Varney v. Justice, 86 Ky 596; 6 S.W. 457; Hunt v.
proof of the intent of the 1987 Constitution to treat the President State, 22 Tex. App. 396, 3 S.W. 233). The phrase “unless
and his official family as a class by itself and to impose upon otherwise provided in this Constitution” must be given a literal
said class stricter prohibitions. interpretation to refer only to those particular instances cited in
the Constitution itself, to wit: the Vice-President being
Xxx appointed as a member of the Cabinet under Section 3, par. (2),
Article VII; or acting as President in those instances provided
Thus, while all other appointive officials in the civil under Section 7, pars. (2) and (3), Article VII; and, the Secretary
service are allowed to hold other office or employment in the of Justice being ex-officio member of the Judicial and Bar
government during their tenure when such is allowed by law or Council by virtue of Section 8 (1), Article VIII.
by the primary functions of their positions, members of the
Cabinet, their deputies and assistants may do so only when Xxx
expressly authorized by the Constitution itself. In other words,
Section 7, Article IX-B is meant to lay down the general rule It being clear x x x that the 1987 Constitution seeks to
applicable to all elective and appointive public officials and prohibit the President, Vice-President, members of the Cabinet,
employees, while Section 13, Article VII is meant to be the their deputies or assistants from holding during their tenure
exception applicable only to the President, the Vice-President, multiple offices or employment in the government, except in
Members of the Cabinet, their deputies and assistants. those cases specified in the Constitution itself and as above
clarified with respect to posts held without additional
This being the case, the qualifying phrase “unless compensation in an ex-officio capacity as provided by law and
otherwise provided in this Constitution” in Section 13, Article as required by the primary functions of their office, the citation
VII cannot possibly refer to the broad exceptions provided under of Cabinet members (then called Ministers) as examples during
Section 7, Article IX-B of the 1987 Constitution. To construe the debate and deliberation on the general rule laid down for all
said qualifying phrase as respondents would have us to do, appointive officials should be considered as mere personal
would render nugatory and meaningless the manifest intent and opinions which cannot override the constitution’s manifest
purpose of the framers of the Constitution to impose a stricter intent and the people’s understanding thereof.
prohibition on the President, Vice-President, Members of the
Cabinet, their deputies and assistants with respect to holding In the light of the construction given to Section 13,
other offices or employment in the government during their Article VII in relation to Section 7, par. (2), Article IX-B of the
tenure. Respondents’ interpretation that Section 13 of Article 1987 Constitution, Executive Order No. 284 dated July 23, 1987
VII admits of the exceptions found in Section 7, par. (2) of is unconstitutional. Ostensibly restricting the number of
Article IX-B would obliterate the distinction so carefully set by positions that Cabinet members, undersecretaries or assistant
the framers of the Constitution as to when the high-ranking secretaries may hold in addition to their primary position to not
officials of the Executive Branch from the President to assistant more than two (2) positions in the government and government
Secretary, on the one hand, and the generality of civil servants corporations, Executive Order No. 284 actually allows them to
from the rank immediately below Assistant Secretary hold multiple offices or employment in direct contravention of
downwards, on the other, may hold any other office or position the express mandate of Section 13, Article VII of the 1987
in the government during their tenure. Constitution prohibiting them from doing so, unless otherwise
provided in the 1987 Constitution itself.
Moreover, respondents’ reading of the provisions in
question would render certain parts of the Constitution The Court is alerted by respondents to the impractical
inoperative. This observation applies particularly to the Vice- consequences that will result from a strict application of the
President who, under Section 13 of Article VII is allowed to prohibition mandated under Section 13, Article VII on the
hold other office or employment when so authorized by the operations of the Government, considering that Cabinet
Constitution, but who as an elective public official under Sec. 7, members would be stripped of their offices held in an ex-officio
par. (1) of Article IX-B is absolutely ineligible “for appointment capacity, by reason of their primary positions or by virtue of
or designation in any capacity to any public office or position legislation. As earlier clarified in this decision, ex-officio posts
during his tenure.” Surely, to say that the phrase “unless held by the executive official concerned without additional
otherwise provided in this Constitution” found in Section 13, compensation as provided by law and as required by the primary
Article VII has reference to Section 7, par. (1) of Article IX-B functions of his office do not fall under the definition of “any
would render meaningless the specific provisions of the other office” within the contemplation of the constitutional
Constitution authorizing the Vice-President to become a prohibition. With respect to other offices or employment held
member of the Cabinet (Sec. 3, Ibid.), and to act as President by virtue of legislation, including chairmanships or directorships
without relinquishing the Vice-Presidency where the President in government-owned or controlled corporations and their
shall not have been chosen or fails to qualify (Sec. 7, Article subsidiaries, suffice it to say that the feared impractical
VII). Such absurd consequence can be avoided only by consequences are more apparent than real. Being head of an
interpreting the two provisions under consideration as one, i.e., executive department is no mean job. It is more than a full-time
Section 7, par. (1) of Article IX-B providing the general rule and job, requiring full attention, specialized knowledge, skills and
the other, i.e., Section 13, Article VII as constituting the expertise. If maximum benefits are to be derived from a
exception thereto. In the same manner must Section 7, par. (2) department head’s ability and expertise, he should be allowed to
of Article IX-B be construed vis-à-vis Section 13, Article VII. attend to his duties and responsibilities without the distraction of
other governmental offices or employment. He should be
Xxx precluded from dissipating his efforts, attention and energy
Since the evident purpose of the framers of the 1987 among too many positions and responsibility, which may result
Constitution is to impose a stricter prohibition on the President, in haphazardness and inefficiency. Surely the advantages to be
Vice-President, members of the Cabinet, their deputies and derived from this concentration of attention, knowledge and
assistants with respect to holding multiple offices or expertise, particularly at this stage of our national and economic
employment in the government during their tenure, the development, far outweigh the benefits, if any, that may be
exception to this prohibition must be read with equal severity. gained from a department head spreading himself too thin and
On its face, the language of Section 13, Article VII is taking in more than what he can handle.
prohibitory so that it must be understood as intended to be a
positive and unequivocal negation of the privilege of holding Finding Executive Order No. 284 to be constitutionally
multiple government offices and employment. Verily, wherever infirm, the Court hereby orders respondents x x x to
the language used in the constitution is prohibitory, it is to be immediately relinquish their other offices or employment, as
131
herein defined, in the government, including government-owned as it should be, because the representatives so designated merely
or controlled corporations and their subsidiaries. (Civil perform duties in the Board in addition to those already
Liberties Union v. Executive Secretary, 194 SCRA 317, Feb. performed under their original appointments.”
22, 1991, En Banc [Fernan, CJ])
The term “primary” used to describe “functions” refers
324. Does the prohibition against holding dual or to the order of importance and thus means chief or principal
multiple offices or employment under Section 13, function. The term is not restricted to the singular but may refer
Article VII of the Constitution apply to posts to the plural (33A Words and Phrases, p. 210). The additional
occupied by the Executive officials specified duties must not only be closely related to, but must be required
therein without additional compensation in an ex- by the official’s primary functions. Examples of designations to
officio capacity as provided by law and as required positions by virtue of one’s primary functions are the Secretaries
by the primary functions of said officials’ office? of Finance and Budget sitting as members of the Monetary
Board, and the Secretary of Transportation and Communications
Held: The prohibition against holding dual or multiple acting as Chairman of the Maritime Industry Authority and the
offices or employment under Section 13, Article VII of the Civil Aeronautics Board.
Constitution must not, however, be construed as applying to
posts occupied by the Executive officials specified therein If the functions to be performed are merely incidental,
without additional compensation in an ex-officio capacity as remotely related, inconsistent, incompatible, or otherwise alien
provided by law and as required (As opposed to the term to the primary function of a cabinet official, such additional
“allowed” used in Section 7, par. (2), Article IX-B of the functions would fall under the purview of “any other office”
Constitution, which is permissive. “Required” suggests an prohibited by the Constitution. An example would be the Press
imposition, and therefore, obligatory in nature) by the primary Undersecretary sitting as a member of the Board of the
functions of said officials’ office. The reason is that these posts Philippine Amusement and Gaming Corporation. The same rule
do not comprise “any other office” within the contemplation of applies to such positions which confer on the cabinet official
the constitutional prohibition but are properly an imposition of management functions and/or monetary compensation, such as
additional duties and functions on said officials. To characterize but not limited to chairmanships or directorships in government-
these posts otherwise would lead to absurd consequences, owned or controlled corporations and their subsidiaries.
among which are: The President of the Philippines cannot chair
the National Security Council reorganized under Executive Mandating additional duties and functions to the
Order No. 115. Neither can the Vice-President, the Executive President, Vice-President, Cabinet Members, their deputies or
Secretary, and the Secretaries of National Defense, Justice, assistants which are not inconsistent with those already
Labor and Employment and Local Government sit in this prescribed by their offices or appointments by virtue of their
Council, which would then have no reason to exist for lack of a special knowledge, expertise and skill in their respective
chairperson and members. The respective undersecretaries and executive offices is a practice long-recognized in many
assistant secretaries, would also be prohibited. jurisdictions. It is a practice justified by the demands of
efficiency, policy direction, continuity and coordination among
Xxx the different offices in the Executive Branch in the discharge of
Indeed, the framers of our Constitution could not have its multifarious tasks of executing and implementing laws
intended such absurd consequences. A Constitution, viewed as affecting national interest and general welfare and delivering
a continuously operative charter of government, is not to be basic services to the people. It is consistent with the power
interpreted as demanding the impossible or the impracticable; vested on the President and his alter egos, the Cabinet members,
and unreasonable or absurd consequences, if possible, should be to have control of all the executive departments, bureaus and
avoided. offices and to ensure that the laws are faithfully executed.
Without these additional duties and functions being assigned to
To reiterate, the prohibition under Section 13, Article the President and his official family to sit in the governing
VII is not to be interpreted as covering positions held without bodies or boards of governmental agencies or instrumentalities
additional compensation in ex-officio capacities as provided by in an ex-officio capacity as provided by law and as required by
law and as required by the primary functions of the concerned their primary functions, they would be deprived of the means for
official’s office. The term ex-officio means “from office; by control and supervision, thereby resulting in an unwieldy and
virtue of office.” It refers to an “authority derived from official confused bureaucracy.
character merely, not expressly conferred upon the individual
character, but rather annexed to the official position.” Ex officio It bears repeating though that in order that such
likewise denotes an “act done in an official character, or as a additional duties or functions may not transgress the prohibition
consequence of office, and without any other appointment or embodied in Section 13, Article VII of the 1987 Constitution,
authority than that conferred by the office.” An ex-officio such additional duties or functions must be required by the
member of a board is one who is a member by virtue of his title primary functions of the official concerned, who is to perform
to a certain office, and without further warrant or appointment. the same in an ex-officio capacity as provided by law, without
To illustrate, by express provision of law, the Secretary of receiving any additional compensation therefor.
Transportation and Communications is the ex-officio Chairman
of the Board of the Philippine Ports Authority (Sec. 7, E.O. The ex-officio position being actually and in legal
778), and the Light Rail Transit Authority (Sec. 1, E.O. 210). contemplation part of the principal office, it follows that the
official concerned has no right to receive additional
The Court had occasion to explain the meaning of an compensation for his services in the said position. The reason is
ex-officio position in Rafael v. Embroidery and Apparel Control that these services are already paid for and covered by the
and Inspection Board, thus: “An examination of Section 2 of the compensation attached to his principal office. It should be
questioned statute (R.A. 3137) reveals that for the chairman and obvious that if, say, the Secretary of Finance attends a meeting
members of the Board to qualify they need only be designated of the Monetary Board as an ex-officio member thereof, he is
by the respective department heads. With the exception of the actually and in legal contemplation performing the primary
representative from the private sector, they sit ex-officio. I order function of his principal office in defining policy in monetary
to be designated they must already be holding positions in the and banking matters, which come under the jurisdiction of his
offices mentioned in the law. Thus, for instance, one who does department. For such attendance, therefore, he is not entitled to
not hold a previous appointment in the Bureau of Customs, collect any extra compensation, whether it be in the form of a
cannot, under the act, be designated a representative from that per diem or an honorarium or an allowance, or some other such
office. The same is true with respect to the representatives from euphemism. By whatever name it is designated, such additional
the other offices. No new appointments are necessary. This is compensation is prohibited by the Constitution. (Civil Liberties
132
Union v. Executive Secretary, 194 SCRA 317, Feb. 22, 1991, The rules on nepotism shall likewise not be applicable
En Banc [Fernan, CJ]) to the case of a member of any family who, after his or her
appointment to any position in an office or bureau, contracts
325. Should members of the Cabinet appointed to other marriage with someone in the same office or bureau, in which
positions in the government pursuant to Executive event the employment or retention therein of both husband and
Order No. 284 which later was declared wife may be allowed. (Sec. 59, Chap. 7, Subtitle A, Title I, Bk.
unconstitutional by the SC for being violative of V, E.O. No. 292)
Section 13, Article VII of the Constitution be made
to reimburse the government for whatever pay and 329. What is the doctrine of forgiveness or
emoluments they received from holding such other condonation? Does it apply to pending criminal
positions? cases?

Held: During their tenure in the questioned positions, Held: 1. A public official cannot be removed for
respondents may be considered de facto officers and as such administrative misconduct committed during a prior term, since
entitled to emoluments for actual services rendered. It has been his re-election to office operates as a condonation of the
held that “in cases where there is no de jure officer, a de facto officer’s previous misconduct to the extent of cutting off the
officer, who, in good faith has had possession of the office and right to remove him therefor. The foregoing rule, however,
has discharged the duties pertaining thereto, is legally entitled to finds no application to criminal cases pending against petitioner.
the emoluments of the office, and may in an appropriate action (Aguinaldo v. Santos, 212 SCRA 768, 773 [1992])
recover the salary, fees and other compensations attached to the
office. This doctrine is, undoubtedly, supported on equitable 2. A reelected local official may not be held
grounds since it seems unjust that the public should benefit by administratively accountable for misconduct committed during
the services of an officer de facto and then be freed from all his prior term of office. The rationale for this holding is that
liability to pay any one for such services. Any per diem, when the electorate put him back into office, it is presumed that
allowances or other emoluments received by the respondents by it did so with full knowledge of his life and character, including
virtue of actual services rendered in the questioned positions his past misconduct. If, armed with such knowledge, it still
may therefore be retained by them. (Civil Liberties Union v. reelects him, then such reelection is considered a condonation of
Executive Secretary, 194 SCRA 317, Feb. 22, 1991, En Banc his past misdeeds. (Mayor Alvin B. Garcia v. Hon. Arturo C.
[Fernan, CJ]) Mojica, et al., G.R. No. 139043, Sept. 10, 1999 [Quisumbing])

326. May a Senator or Congressman hold any other 330. What is the Doctrine of Condonation? Illustrative
office or employment? case.

Ans.: No Senator or Member of the House of Held: Petitioner contends that, per our ruling in
Representatives may hold any other office or employment in the Aguinaldo v. Santos, his reelection has rendered the
government, or any subdivision, agency, or instrumentality administrative case filed against him moot and academic. This
thereof, including government-owned or controlled corporations is because his reelection operates as a condonation by the
or their subsidiaries, during his term without forfeiting his seat. electorate of the misconduct committed by an elective official
Neither shall he be appointed to any office which may have during his previous term. Petitioner further cites the ruling of
been created or the emoluments thereof increased during the this Court in Pascual v. Hon. Provincial Board of Nueva Ecija,
term for which he was elected. (Sec. 13, Art. VI, 1987 citing Conant v. Brogan, that
Constitution).
“x x x When the people have elected a man to
327. What are the situations covered by the law on office, it must be assumed that they did this with
nepotism? knowledge of his life and character, and that they
disregarded or forgave his faults or misconduct, if he
Held: Under the definition of nepotism, one is guilty had been guilty of any. It is not for the court, by reason
of nepotism if an appointment is issued in favor of a relative of such faults or misconduct to practically overrule the
within the third civil degree of consanguinity or affinity of any will of the people.”
of the following:
Respondents, on the other hand, contend that while the
a) appointing authority; contract in question was signed during the previous term of
b) recommending authority; petitioner, it was to commence or be effective only on
c) chief of the bureau or office; and September 1998 or during his current term. It is the
d) person exercising immediate supervision over the respondents’ submission that petitioner “went beyond the
appointee. protective confines” of jurisprudence when he “agreed to extend
his act to his current term of office.” Aguinaldo cannot apply,
Clearly, there are four situations covered. In the last according to respondents, because what is involved in this case
two mentioned situations, it is immaterial who the appointing or is a misconduct committed during a previous term but to be
recommending authority is. To constitute a violation of the law, effective during the current term.
it suffices that an appointment is extended or issued in favor of a
relative within the third civil degree of consanguinity or affinity Respondents maintain that,
of the chief of the bureau or office, or the person exercising
immediate supervision over the appointee. (CSC v. Pedro O. “x x x petitioner performed two acts with respect to the
Dacoycoy, G.R. No. 135805, April 29, 1999, En Banc [Pardo]) contract: he provided for a suspensive period making
the supply contract commence or be effective during
328. What are the exemptions from the operation of the his succeeding or current term and during his current
rules on nepotism? term of office he acceded to the suspensive period
making the contract effective during his current term
Ans.: The following are exempted from the operation by causing the implementation of the contract.”
of the rules on nepotism: (a) persons employed in a confidential
capacity, (b) teachers, (c) physicians, and (d) members of the Hence, petitioner cannot take refuge in the fact of his
Armed Forces of the Philippines. reelection, according to respondents.

133
Further, respondents point out that the contract in “The rule adopted in Pascual, qualified in
question was signed just four days before the date of the 1998 Aguinaldo insofar as criminal cases are concerned, is still a good
election and so it could not be presumed that when the people of law. Such a rule is not only founded on the theory that an
Cebu City voted petitioner to office, they did so with full official’s reelection expresses the sovereign will of the electorate
knowledge of petitioner’s character. to forgive or condone any act or omission constituting a ground
for administrative discipline which was committed during his
On this point, petitioner responds that knowledge of an previous term. We may add that sound policy dictates it. To rule
official’s previous acts is presumed and the court need not otherwise would open the floodgates to exacerbating endless
inquire whether, in reelecting him, the electorate was actually partisan contests between the reelected official and his political
aware of his prior misdeeds. enemies, who may not stop to hound the former during his new
term with administrative cases for acts alleged to have been
Petitioner cites our ruling in Salalima v. Guingona, committed during his previous term. His second term may thus
wherein we absolved Albay governor Ramon R. Salalima of his be devoted to defending himself in the said cases to the detriment
administrative liability as regards a retainer agreement he signed of public service x x x.”
in favor of a law firm during his previous term, although
disbursements of public funds to cover payments under the The above ruling in Salalima applies to this case.
agreement were still being done during his subsequent term. Petitioner cannot anymore be held administratively liable for an
Petitioner argues that, following Salalima, the doctrine of act done during his previous term, that is, his signing of the
Aguinaldo applies even where the effects of the acts complained contract with F.E. Zuellig.
of are still evident during the subsequent term of the reelected
official. The implementation of the contract is a mere incident The assailed retainer agreement in Salalima was
of its execution. Besides, according to petitioner, the “sole act” executed sometime in 1990. Governor Salalima was reelected
for which he has been administratively charged is the signing of in 1992 and payments for the retainer continued to be made
the contract with F.E. Zuellig. The charge, in his view, excludes during his succeeding term. This situation is no different from
the contract’s execution or implementation, or any act the one in the present case, wherein deliveries of the asphalt
subsequent to the perfection of the contract. under the contract with F.E. Zuellig and the payments therefor
were supposed to have commenced on September 1998, during
In Salalima, we recall that the Solicitor General petitioner’s second term.
maintained that Aguinaldo did not apply to that case because the
administrative case against Governor Rodolfo Aguinaldo of However, respondents argue that the contract, although
Cagayan was already pending when he filed his certificate of signed on May 7, 1998, during petitioner’s prior term, is to be
candidacy for his reelection bid. Nevertheless, in Salalima, the made effective only during his present term.
Court applied the Aguinaldo doctrine, even if the administrative
case against Governor Salalima was filed after his reelection. We fail to see any difference to justify a valid
distinction in the result. The agreement between petitioner
Xxx (representing Cebu City) and F.E. Zuellig was perfected on the
date the contract was signed, during petitioner’s prior term. At
We now come to the concluding inquiry. Granting that that moment, petitioner already acceded to the terms of the
the Office of the Ombudsman may investigate, for purposes contract, including stipulations now alleged to be prejudicial to
provided for by law, the acts of petitioner committed prior to his the city government. Thus, any culpability petitioner may have
present term of office; and that it may preventively suspend him in signing the contract already became extant on the day the
for a reasonable period, can that office hold him contract was signed. It hardly matters that the deliveries under
administratively liable for said acts? the contract are supposed to have been made months later.

In a number of cases, we have repeatedly held that a While petitioner can no longer be held administratively
reelected local official may not be held administratively liable for signing the contract with F.E. Zuellig, however, this
accountable for misconduct committed during his prior term of should not prejudice the filing of any case other than
office. The rationale for this holding is that when the electorate administrative against petitioner. Our ruling in this case, may
put him back into office, it is resumed that it did so with full not be taken to mean the total exoneration of petitioner for
knowledge of his life and character, including his past whatever wrongdoing, if any, might have been committed in
misconduct. If, armed with such knowledge, it still reelects signing the subject contract. The ruling now is limited to the
him, then such reelection is considered a condonation of his past question of whether or not he may be held administratively
misdeeds. liable therefor, and it is our considered view that he may not.
(Garcia v. Mojica, 314 SCRA 207, Sept. 10, 1999, 2 nd Div.
However, in the present case, respondents point out [Quisumbing])
that the contract entered into by petitioner with F.E. Zuellig was
signed just four days before the date of the elections. It was not 331. Petitioner claims that Benipayo has no authority to
made an issue during the election, and so the electorate could remove her as Director IV of the EID and reassign
not be said to have voted for petitioner with knowledge of this her to the Law Department. Petitioner further
particular aspect of his life and character. argues that only the COMELEC, acting as a
collegial body, can authorize such reappointment.
For his part, petitioner contends that “the only Moreover, petitioner maintains that a reassignment
conclusive determining factor” as regards the people’s thinking without her consent amounts to removal from
on the matter is an election. On this point we agree with office without due process and therefore illegal.
petitioner. That the people voted for an official with knowledge
of his character is presumed, precisely to eliminate the need to Held: Petitioner’s posturing will hold water if
determine, in factual terms, the extent of this knowledge. Such Benipayo does not possess any color of title to the office of
an undertaking will obviously be impossible. Our rulings on the Chairman of the COMELEC. We have ruled, however, that
matter do not distinguish the precise timing or period when the Benipayo is the de jure COMELEC Chairman, and
misconduct was committed, reckoned from the date of the consequently he has full authority to exercise all the powers of
official’s reelection, except that it must be prior to said date. that office for so long as his ad interim appointment remains
effective. X x x. The Chairman, as the Chief Executive of the
As held in Salalima, COMELEC, is expressly empowered on his own authority to
transfer or reassign COMELEC personnel in accordance with
the Civil Service Law. In the exercise of this power, the
134
Chairman is not required by law to secure the approval of the the same reason, Benipayo’s order designating Cinco Officer-in-
COMELEC en banc. Charge of the EID is legally unassailable. (Matibag v.
Benipayo, 380 SCRA 49, April 2, 2002, En Banc [Carpio])
Petitioner’s appointment papers x x x indisputably
show that she held her Director IV position in the EID only in 332. May the appointment of a person assuming a
an acting or temporary capacity. Petitioner is not a Career position in the civil service under a completed
Executive Service (CES), and neither does she hold Career appointment be validly recalled or revoked?
Executive Service Eligibility, which are necessary qualifications
for holding the position of Director IV as prescribed in the Held: It has been held that upon the issuance of an
Qualifications Standards (Revised 1987) issued by the Civil appointment and the appointee’s assumption of the position in
Service Commission. Obviously, petitioner does not enjoy the civil service, “he acquires a legal right which cannot be
security of tenure as Director IV. X x x taken away either by revocation of the appointment or by
removal except for cause and with previous notice and hearing.”
Xxx Moreover, it is well-settled that the person assuming a position
in the civil service under a completed appointment acquires a
Having been appointed merely in a temporary or acting legal, not just an equitable, right to the position. This right is
capacity, and not possessed of the necessary qualifications to protected not only by statute, but by the Constitution as well,
hold the position of Director IV, petitioner has no legal basis in which right cannot be taken away by either revocation of the
claiming that her reassignment was contrary to the Civil Service appointment, or by removal, unless there is valid cause to do so,
Law. X x x provided that there is previous notice and hearing.

Still, petitioner assails her reassignment, carried out Petitioner admits that his very first official act upon
during the election period, as a prohibited act under Section 261 assuming the position of town mayor was to issue Office Order
(h) of the Omnibus Election Code x x x. No. 95-01 which recalled the appointments of the private
respondents. There was no previous notice, much less a hearing
Xxx accorded to the latter. Clearly, it was petitioner who acted in
undue haste to remove the private respondents without regard
Petitioner claims that Benipayo failed to secure the approval of for the simple requirements of due process of law. While he
the COMELEC en banc to effect transfers or reassignments of argues that the appointing power has the sole authority to revoke
COMELEC personnel during the election period. Moreover, said appointments, there is no debate that he does not have
petitioner insists that the COMELEC en banc must concur to blanket authority to do so. Neither can he question the CSC’s
every transfer or reassignment of COMELEC personnel during jurisdiction to affirm or revoke the recall.
the election period.
Rule V, Section 9 of the Omnibus Implementing
Contrary to petitioner’s allegation, the COMELEC did Regulations of the Revised Administrative Code specifically
in fact issue COMELEC Resolution No. 3300 dated November provides that “an appointment accepted by the appointee cannot
6, 2000, exempting the COMELEC from Section 261 (h) of the be withdrawn or revoked by the appointing authority and shall
Omnibus Election Code. X x x remain in force and in effect until disapproved by the
Commission.” Thus, it is the CSC that is authorized to recall an
Xxx appointment initially approved, but only when such appointment
and approval are proven to be in disregard of applicable
The proviso in COMELEC Resolution No. 3300, requiring due provisions of the civil service law and regulations.
notice and hearing before any transfer or reassignment can be
made within thirty days prior to election day, refers only to Moreover, Section 10 of the same rule provides:
COMELEC field personnel and not to head office personnel like
the petitioner. Under the Revised Administrative Code, the Sec. 10. An appointment issued in accordance
COMELEC Chairman is the sole officer specifically vested with with pertinent laws and rules shall take effect immediately
the power to transfer or reassign COMELEC personnel. The upon its issuance by the appointing authority, and if the
COMELEC Chairman will logically exercise the authority to appointee has assumed the duties of the position, he shall be
transfer or reassign COMELEC personnel pursuant to entitled to receive his salary at once without awaiting the
COMELEC Resolution No. 3300. The COMELEC en banc approval of his appointment by the Commission. The
cannot arrogate unto itself this power because that will mean appointment shall remain effective until disapproved by the
amending the Revised Administrative Code, an act the Commission. In no case shall an appointment take effect
COMELEC en banc cannot legally do. earlier than the date of its issuance.

COMELEC Resolution No. 3300 does not require that Section 20 of Rule VI also provides:
every transfer or reassignment of COMELEC personnel should
carry the concurrence of the COMELEC as a collegial body. Sec. 20. Notwithstanding the initial approval of an
Interpreting Resolution No. 3300 to require such concurrence appointment, the same may be recalled on any of the
will render the resolution meaningless since the COMELEC en following grounds:
banc will have to approve every personnel transfer or
reassignment, making the resolution utterly useless. Resolution Non-compliance with the procedures/criteria
No. 3300 should be interpreted for what it is, an approval to provided in the agency’s Merit
effect transfers and reassignments of personnel, without need of Promotion Plan;
securing a second approval from the COMELEC en banc to Failure to pass through the agency’s
actually implement such transfer or reassignment. Selection/Promotion Board;
Violation of the existing collective agreement
The COMELEC Chairman is the official expressly between management and employees
authorized by law to transfer or reassign COMELEC personnel. relative to promotion; or
The person holding that office, in a de jure capacity, is Violation of other existing civil service law,
Benipayo. The COMELEC en banc, in COMELEC Resolution rules and regulations.
No. 3300, approved the transfer or reassignment of COMELEC
personnel during the election period. Thus, Benipayo’s order Accordingly, the appointments of the private
reassigning petitioner from the EID to the Law Department does respondents may only be recalled on the above-cited grounds.
not violate Section 261 (h) of the Omnibus Election Code. For And yet, the only reason advanced by the petitioner to justify
135
the recall was that these were “midnight appointments.” The Xxx
CSC correctly ruled, however, that the constitutional prohibition
on so-called “midnight appointments,” specifically those made As may be gleaned from the pleadings of the parties,
within two (2) months immediately prior to the next presidential the issues are: (1) whether private respondent Eusebia R.
elections, applies only to the President or Acting President. (De Galzote may be considered absent without leave; (b) whether
Rama v. Court of Appeals, 353 SCRA 94, Feb. 28, 2001, En due process had been observed before she was dropped from the
Banc [Ynares-Santiago]) rolls; and, (3) whether she may be deemed to have abandoned
her position, hence, not entitled to reinstatement with back
333. Is a government employee who has been ordered salaries for not having filed a formal application for leave.
arrested and detained for a non-bailable offense Encapsulated, the issues may be reduced to whether private
and for which he was suspended for his inability to respondent may be considered absent without leave or whether
report for work until the termination of his case, she abandoned her job as to justify being dropped from the
still required to file a formal application for leave service for not filing a formal application for leave.
of absence to ensure his reinstatement upon his
acquittal and thus protect his security of tenure? Petitioner would have private respondent declared on
Concomitantly, will his prolonged absence from AWOL and faults her for failing to file an application for leave
office for more than one (1) year automatically of absence under Secs. 20 (Now Sec. 52 of Rule XVI, Leave of
justify his being dropped from the rolls without Absence, of Res. No. 91-1631 dated 27 December 1991, as
prior notice despite his being allegedly placed amended by CSC MC No. 41, s. 1998) and 35 (Now Sec. 63 of
under suspension by his employer until the Rule XVI, Leave of Absence, of Res. No. 91-1631 dated 27
termination of his case, which finally resulted in December 1991, as amended by CSC MC Nos. 41, s. 1998 and
his acquittal for lack of evidence? 14, s. 1999) of the CSC Rules and rejects the CSC’s ruling of an
“automatic leave of absence for the period of her detention”
Held: EUSEBIA R. GALZOTE was employed as a since the “current Civil Service Law and Rules do not contain
lowly clerk in the service of the City Government of Makati any specific provision on automatic leave of absence.”
City. With her meager income she was the lone provider for her
children. But her simple life was disrupted abruptly when she The Court believes that private respondent cannot be
was arrested without warrant and detained for more than three faulted for failing to file prior to her detention an application for
(3) years for a crime she did not commit. Throughout her ordeal leave and obtain approval thereof. The records clearly show
she trusted the city government that the suspension imposed on that she had been advised three (3) days after her arrest, or on 9
her was only until the final disposition of her case. As she drew September 1991, that petitioner City government of Makati City
near her vindication she never did expect the worst to come to had placed her under suspension until the final disposition of
her. On the third year of her detention the city government her criminal case. This act of petitioner indubitably recognized
lifted her suspension, dropped her from the rolls without prior private respondent’s predicament and thus allowed her to forego
notice and without her knowledge, much less gave her an reporting for work during the pendency of her criminal case
opportunity to forthwith correct the omission of an application without the needless exercise of strict formalities. At the very
for leave of absence belatedly laid on her. least, this official communication should be taken as an
equivalent of a prior approved leave of absence since it was her
Upon her acquittal for lack of evidence and her release employer itself which placed her under suspension and thus
from detention she was denied reinstatement to her position. excused her from further formalities in applying for such leave.
She was forced to seek recourse in the Civil Service Moreover, the arrangement bound the City Government to allow
Commission which ordered her immediate reinstatement with private respondent to return to her work after the termination of
back wages from 19 October 1994, the date when she presented her case, i.e., if acquitted of the criminal charge. This pledge
herself for reassumption of duties but was turned back by the sufficiently served as legitimate reason for her to altogether
city government, up to the time of her actual reinstatement. dispense with the formal application for leave; there was no
reason to, as in fact it was not required, since she was for all
Xxx practical purposes incapacitated or disabled to do so.

Plainly, the case of petitioner City Government of Indeed, private respondent did not have the least
Makati City revolves around a rotunda of doubt, a dilemma intention to go on AWOL from her post as Clerk III of
concerning the legal status and implications of its suspension of petitioner, for AWOL means the employee leaving or
private respondent Eusebia R. Galzote and the automatic leave abandoning his post without justifiable reason and without
of absence espoused by the Civil Service Commission. Against notifying his employer. In the instant case, private respondent
this concern is the punctilious adherence to technicality, the had a valid reason for failing to report for work as she was
requirement that private respondent should have filed an detained without bail. Hence, right after her release from
application for leave of absence in proper form. The instant detention, and when finally able to do so, she presented herself
case is therefore a dispute between, at its worst, private to the Municipal Personnel Officer of petitioner City
respondent’s substantial compliance with the standing rules, and Government to report for work. Certainly, had she been told
the City Government’s insistence that the lowly clerk should that it was still necessary for her to file an application for leave
have still gone through the formalities of applying for leave despite the 9 September 1991 assurance from petitioner, private
despite her detention, of which petitioner had actual notice, and respondent would have lost no time in filing such piece of
the suspension order couched in simple language that she was document. But the situation momentarily suspending her from
being suspended until the final disposition of her criminal case. work persisted: petitioner City Government did not alter the
modus vivendi with private respondent and lulled her into
The meaning of suspension until the final disposition of believing that its commitment that her suspension was only until
her case is that should her case be dismissed she should be the termination of her case was true and reliable. Under the
reinstated to her position with payment of back wages. She did circumstances private respondent was in, prudence would have
not have to apply for leave of absence since she was already dictated petitioner, more particularly the incumbent city
suspended by her employer until her case would be terminated. executive, in patria potestas, to advise her that it was still
We have done justice to the workingman in the past; today we necessary – although indeed unnecessary and a useless
will do no less by resolving all doubts in favor of the humble ceremony – to file such application despite the suspension order,
employee in faithful obeisance to the constitutional mandate to before depriving her of her legitimate right to return to her
afford full protection to labor (Const., Art. XIII, Sec. 3, par. 1; position. Patria potestas in piatate debet, non in atrocitate,
Art. II, Sec. 18) consistere. Paternal power should consist or be exercised in
affection, not in atrocity.
136
and rewards system, integrate all human resources development
It is clear from the records that private respondent programs for all levels and ranks, and institutionalize a
Galzote was arrested and detained without a warrant on 6 management climate conducive to public accountability.”
September 1991 for which reason she and her co-accused were Besides, the Administrative Code of 1987 further empowers the
subjected immediately to inquest proceedings. This fact is CSC to “prescribe, amend, and enforce rules and regulations for
evident from the instant petition itself and its attachments x x x. carrying into effect the provisions of the Civil Service Law and
Hence, her ordeal in jail began on 6 September 1991 and ended other pertinent laws,” and for matters concerning leaves of
only after her acquittal, thus leaving her no time to attend to the absence, the Code specifically vests the CSC to ordain –
formality of filing a leave of absence.
Sec. 60. Leave of absence. – Officers and
But petitioner City Government would employees in the Civil Service shall be entitled to leave
unceremoniously set aside its 9 September 1991 suspension of absence, with or without pay, as may be provided by
order claiming that it was superseded three (3) years later by a law and the rules and regulations of the Civil Service
memorandum dropping her from the rolls effective 21 January Commission in the interest of the service.
1993 for absence “for more than one (1) year without official
leave.” Hence, the suspension order was void since there was Pursuant thereto the CSC promulgated Resolution No.
no pending administrative charge against private respondent so 91-1631 dated 27 December 1991 entitled Rules Implementing
that she was not excused from filing an application for leave. Book V of Executive Order No. 292 and Other Pertinent Civil
Service Laws which it has several times amended through
We do not agree. In placing private respondent under memorandum circulars. It devotes Rule XVI to leaves of
suspension until the final disposition of her criminal case, the absence. Petitioner City Government relies upon Secs. 20 and
Municipal Personnel Officer acted with competence, so he 35 to debunk the CSC ruling of an automatic leave of absence.
presumably knew that his order of suspension was not akin to Significantly, these provisions have been amended so that Sec.
either suspension as penalty or preventive suspension since 20 of the Civil Service Rules is now Sec. 52 of Rule XVI, on
there was no administrative case against private respondent. As Leave of Absence, of Resolution No. 91-1631 dated 27
competence on the part of the MPO is presumed, any error on December 1991 as amended by CSC MC No. 41, s. 1998, and
his part should not prejudice private respondent, and that what Sec. 35 is now Sec. 63 as amended by CSC MC Nos. 41, s. 1998
he had in mind was to consider her as being on leave of absence and 14, s. 1999.
without pay and their employer-employee relationship being
merely suspended, not severed, in the meantime. This Xxx
construction of the order of suspension is actually more
consistent with logic as well as fairness and kindness to its As a general rule, Secs. 20 and 52, as well as Secs. 35
author, the MPO. Significantly, the idea of a suspended and 63, require an approved leave of absence to avoid being an
employer-employee relationship is widely accepted in labor law AWOL. However, these provisions cannot be interpreted as
to account for situations wherein laborers would have no work exclusive and referring only to one mode of securing the
to perform for causes not attributable to them. We find no basis approval of a leave of absence which would require an
for denying the application of this principle to the instant case employee to apply for it, formalities and all, before exceeding
which also involves a lowly worker in the public service. thirty (30) days of absence in order to avoid from being dropped
from the rolls. There are, after all, other means of seeking and
Moreover, we certainly cannot nullify the City granting an approved leave of absence, one of which is the CSC
Government’s order of suspension, as we have no reason to do recognized rule of automatic leave of absence under specified
so, much less retroactively apply such nullification to deprive circumstances. x x x
private respondent of a compelling and valid reason for not
filing the leave application. For as we have held, a void act Xxx
though in law a mere scrap of paper nonetheless confers
legitimacy upon past acts or omissions done in reliance thereof. As properly noted, the CSC was only interpreting its
Consequently, the existence of a statute or executive order prior own rules on leave of absence and not a statutory provision (As
to its being adjudged void is an operative fact to which legal a matter of fact, Sec. 60 of the Administrative Code does not
consequences are attached. It would indeed be ghastly unfair to provide for any rule on leave of absence other than that civil
prevent private respondent from relying upon the order of servants are entitled to leave of absence) in coming up with this
suspension in lieu of a formal leave application. uniform rule. Undoubtedly, the CSC like any other agency has
the power to interpret its own rules and any phrase contained in
At any rate, statements are, or should be, construed them with its interpretation significantly becoming part of the
against the one responsible for the confusion; otherwise stated, rules themselves. x x x
petitioner must assume full responsibility for the consequences
of its own act, hence, he should be made to answer for the mix- Xxx
up of private respondent as regards the leave application. At the
very least, it should be considered estopped from claiming that Under RA 6656 (An Act to Protect the Security of
its order of suspension is void or that it did not excuse private Tenure of Civil Service Officers and Employees in the
respondent from filing an application for leave on account of her Implementation of Government Reorganization) and RA 7160
incarceration. It is a fact that she relied upon this order, issued (The Local Government Code of 1991), civil servants who are
barely three (3) days from the date of her arrest, and assumed found illegally dismissed or retrenched are entitled to full pay
that when the criminal case would be settled she could return to for the period of their separation.
work without need of any prior act. x x x
Our final point. An efficient and honest bureaucracy is
Xxx never inconsistent with the emphasis on and the recognition of
the basic rights and privileges of our civil servants or, for that
The holding of the Civil Service Commission that matter, the constitutional mandates of the Civil Service
private respondent was on automatic leave of absence during the Commission. In fact only from an enlightened corps of
period of her detention must be sustained. The CSC is the government workers and an effective CSC grows the
constitutionally mandated central personnel agency of the professionalization of the bureaucracy. Indeed the government
Government tasked to “establish a career service and adopt cannot be left in the lurch; but neither could we decree that
measures to promote morale, efficiency, integrity, government personnel be separated from their jobs
responsiveness, progressiveness and courtesy in the civil indiscriminately regardless of fault. The fine line between these
service” (Const., Art. IX-B, Sec. 3) and “strengthen the merit concerns may be difficult to clearly draw but if we only exerted
137
extra effort to rebel against the allure of legal over- petitioners instituted the current action x x x assailing the
simplification, justice would have been done where it is truly constitutionality of certain provisions of said law. The removal
due. (City Government of Makati City v. Civil Service of petitioners from their positions by virtue of a constitutionally
Commission, 376 SCRA 248, Feb. 6, 2002, En Banc infirm act necessarily negates a finding of voluntary
[Bellosillo]) relinquishment. (Canonizado v. Aguirre, 351 SCRA 659, 665-
668, Feb. 15, 2001, En Banc [Gonzaga-Reyes])
334. What is abandonment of office? What are its
essential elements? 336. Distinguish “term” of office from “tenure” of the
incumbent.
Held: Abandonment of an office is the voluntary
relinquishment of an office by the holder, with the intention of Held: In the law of public officers, there is a settled
terminating his possession and control thereof. In order to distinction between “term” and “tenure.” “[T]he term of an
constitute abandonment of an office, it must be total and under office must be distinguished from the tenure of the incumbent.
such circumstances as clearly to indicate an absolute The term means the time during which the officer may claim to
relinquishment. There must be a complete abandonment of hold office as of right, and fixes the interval after which the
duties of such continuance that the law will infer a several incumbents shall succeed one another. The tenure
relinquishment. Abandonment of duties is a voluntary act; it represents the term during which the incumbent actually holds
springs from and is accompanied by deliberation and freedom of the office. The term of office is not affected by the hold-over.
choice. There are, therefore, two essential elements of The tenure may be shorter than the term for reasons within or
abandonment: first, an intention to abandon and second, an overt beyond the power of the incumbent.” (Thelma P. Gaminde v.
or “external” act by which the intention is carried into effect. COA, G.R. No. 140335, Dec. 13, 2000, En Banc [Pardo])

Generally speaking, a person holding a public office 337. Discuss the operation of the rotational plan insofar
may abandon such office by non-user or acquiescence. Non- as the term of office of the Chairman and Members
user refers to a neglect to use a right or privilege or to exercise of the Constitutional Commissions is concerned.
an office. However, non-performance of the duties of an office
does not constitute abandonment where such non-performance Held: In Republic v. Imperial, we said that “the
results from temporary disability or from involuntary failure to operation of the rotational plan requires two conditions, both
perform. Abandonment may also result from an acquiescence indispensable to its workability: (1) that the terms of the first
by the officer in his wrongful removal or discharge, for instance, three (3) Commissioners should start on a common date, and (2)
after a summary removal, an unreasonable delay by an officer that any vacancy due to death, resignation or disability before
illegally removed in taking steps to vindicate his rights may the expiration of the term should only be filled only for the
constitute an abandonment of the office. Where, while desiring unexpired balance of the term.”
and intending to hold the office, and with no willful desire or
intention to abandon it, the public officer vacates it in deference Consequently, the terms of the first Chairmen and
to the requirements of a statute which is afterwards declared Commissioners of the Constitutional Commissions under the
unconstitutional, such a surrender will not be deemed an 1987 Constitution must start on a common date, irrespective of
abandonment and the officer may recover the effect. the variations in the dates of appointments and qualifications of
(Canonizado v. Aguirre, 351 SCRA 659, 665-668, Feb. 15, the appointees, in order that the expiration of the first terms of
2001, En Banc [Gonzaga-Reyes]) seven, five and three years should lead to the regular recurrence
of the two-year interval between the expiration of the terms.
335. By accepting another position in the government
during the pendency of a case – brought precisely Applying the foregoing conditions x x x, we rule that
to assail the constitutionality of his removal - may a the appropriate starting point of the terms of office of the first
person be deemed to have abandoned his claim for appointees to the Constitutional Commissions under the 1987
reinstatement? Constitution must be on February 2, 1987, the date of the
adoption of the 1987 Constitution. In case of a belated
Held: Although petitioners do not deny the appointment or qualification, the interval between the start of
appointment of Canonizado as Inspector General, they maintain the term and the actual qualification of the appointee must be
that Canonizado’s initiation and tenacious pursuance of the counted against the latter. (Thelma P. Gaminde v. COA, G.R.
present case would belie any intention to abandon his former No. 140335, Dec. 13, 2000, En Banc [Pardo])
office. Petitioners assert that Canonizado should not be faulted
for seeking gainful employment during the pendency of this 338. What is the hold-over doctrine? What is its
case. Furthermore, petitioners point out that from the time purpose?
Canonizado assumed office as Inspector General he never
received the salary pertaining to such position x x x. Held: 1. The concept of holdover when applied to a
public officer implies that the office has a fixed term and the
Xxx incumbent is holding onto the succeeding term. It is usually
provided by law that officers elected or appointed for a fixed
By accepting the position of Inspector General during term shall remain in office not only for that term but until their
the pendency of the present case – brought precisely to assail the successors have been elected and qualified. Where this
constitutionality of his removal from the NAPOLCOM – provision is found, the office does not become vacant upon the
Canonizado cannot be deemed to have abandoned his claim for expiration of the term if there is no successor elected and
reinstatement to the latter position. First of all, Canonizado did qualified to assume it, but the present incumbent will carry over
not voluntarily leave his post as Commissioner, but was until his successor is elected and qualified, even though it be
compelled to do so on the strength of Section 8 of RA 8551 x x beyond the term fixed by law.
x
Absent an express or implied constitutional or statutory
In our decision of 25 January 2000, we struck down the provision to the contrary, an officer is entitled to stay in office
abovequoted provision for being violative of petitioner’s until his successor is appointed or chosen and has qualified.
constitutionally guaranteed right to security of tenure. Thus, The legislative intent of not allowing holdover must be clearly
Canonizado harbored no willful desire or intention to abandon expressed or at least implied in the legislative enactment,
his official duties. In fact, Canonizado, together with petitioners otherwise it is reasonable to assume that the law-making body
x x x lost no time disputing what they perceived to be an illegal favors the same.
removal; a few weeks after RA 8551 took effect x x x
138
Indeed, the law abhors a vacuum in public offices, and there are two essential elements of abandonment; first, an
courts generally indulge in the strong presumption against a intention to abandon and, second, an overt or “external” act by
legislative intent to create, by statute, a condition which may which the intention is carried into effect. (Sangguniang Bayan
result in an executive or administrative office becoming, for any of San Andres, Catanduanes v. CA, 284 SCRA 276, Jan. 16,
period of time, wholly vacant or unoccupied by one lawfully 1998)
authorized to exercise its functions. This is founded on obvious
considerations of public policy, for the principle of holdover is 341. What is the effect of acceptance of an incompatible
specifically intended to prevent public convenience from office to a claim for reinstatement?
suffering because of a vacancy and to avoid a hiatus in the
performance of government functions. (Lecaroz v. Held: The next issue is whether Canonizado’s
Sandiganbayan, 305 SCRA 397, March 25, 1999, 2 nd Div. appointment to and acceptance of the position of Inspector
[Bellosillo]) General should result in an abandonment of his claim for
reinstatement to the NAPOLCOM. It is a well-settled rule that
2. The rule is settled that unless “holding over be he who, while occupying one office, accepts another
expressly or impliedly prohibited, the incumbent may continue incompatible with the first, ipso facto vacates the first office and
to hold over until someone else is elected and qualified to his title is thereby terminated without any other act or
assume the office.” This rule is demanded by the “most obvious proceeding. Public policy considerations dictate against
requirements of public policy, for without it there must allowing the same individual to perform inconsistent and
frequently be cases where, from a failure to elect or a refusal or incompatible duties. The incompatibility contemplated is not
neglect to qualify, the office would be vacant and the public the mere physical impossibility of one person’s performing the
service entirely suspended.” Otherwise stated, the purpose is to duties of the two offices due to a lack of time or the inability to
prevent a hiatus in the government pending the time when the be in two places at the same moment, but that which proceeds
successor may be chosen and inducted into office. (Galarosa v. from the nature and relations of the two positions to each other
Valencia, 227 SCRA 728, Nov. 11, 1993, En Banc [Davide, as to give rise to contrariety and antagonism should one person
Jr.]) attempt to faithfully and impartially discharge the duties of one
toward the incumbent of the other.
339. What is resignation? What are the requisites of a
valid resignation? There is no question that the positions of NAPOLCOM
Commissioner and Inspector General of the IAS are
Held: 1. It is the act of giving up or the act of an incompatible with each other. As pointed out by respondents,
officer by which he declines his office and renounces the further RA 8551 prohibits any personnel of the IAS from sitting in a
right to use it. It is an expression of the incumbent in some committee charged with the task of deliberating on the
form, express or implied, of the intention to surrender, renounce, appointment, promotion, or assignment of any PNP personnel,
and relinquish the office and the acceptance by competent and whereas the NAPOLCOM has the power of control and
lawful authority. To constitute a complete and operative supervision over the PNP. However, the rule on
resignation from public office, there must be: (a) an intention to incompatibility of duties will not apply to the case at bar
relinquish a part of the term; (b) an act of relinquishment; and because at no point did Canonizado discharge the functions of
(c) an acceptance by the proper authority. The last one is the two offices simultaneously. Canonizado was forced out of
required by reason of Article 238 of the Revised Penal Code. his first office by the enactment of Section 8 of RA 8551. Thus,
(Sangguniang Bayan of San Andres, Catanduanes v. CA, 284 when Canonizado was appointed as Inspector General x x x he
SCRA 276, Jan. 16, 1998) had ceased to discharge his official functions as NAPOLCOM
Commissioner. x x x Thus, to reiterate, the incompatibility of
2. Resignation x x x is a factual question and its duties rule never had a chance to come into play for petitioner
elements are beyond quibble: there must be an intent to resign never occupied the two positions, of Commissioner and
and the intent must be coupled by acts of relinquishment. The Inspector General, nor discharged their respective functions,
validity of a resignation is not governed by any formal concurrently.
requirement as to form. It can be oral. It can be written. It can
be express. It can be implied. As long as the resignation is Xxx
clear, it must be given legal effect. (Estrada v. Desierto, G.R.
Nos. 146710-15, March 2, 2001, en Banc [Puno]) As in the Tan v. Gimenez and Gonzales v. Hernandez
cases, Canonizado was compelled to leave his position as
340. What is abandonment of an office? What are its Commissioner, not by an erroneous decision, but by an
requisites? How is it distinguished from unconstitutional provision of law. Canonizado, like the
resignation? petitioners in the above mentioned cases, held a second office
during the period that his appeal was pending. As stated in the
Held: Abandonment of an office has been defined as Comment filed by petitioners, Canonizado was impelled to
the voluntary relinquishment of an office by the holder, with the accept this subsequent position by a desire to continue serving
intention of terminating his possession and control thereof. the country, in whatever capacity. Surely, this selfless and
Indeed, abandonment of office is a species of resignation; while noble aspiration deserves to be placed on at least equal footing
resignation in general is a formal relinquishment, abandonment with the worthy goal of providing for oneself and one’s family,
is a voluntary relinquishment through nonuser. either of which are sufficient to justify Canonizado’s acceptance
of the position of Inspector General. A Contrary ruling would
Abandonment springs from and is accompanied by deprive petitioner of his right to live, which contemplates not
deliberation and freedom of choice. Its concomitant effect is only a right to earn a living, as held in previous cases, but also a
that the former holder of an office can no longer legally right to lead a useful and productive life. Furthermore,
repossess it even by forcible reoccupancy. prohibiting Canonizado from accepting a second position during
the pendency of his petition would be to unjustly compel him to
Clear intention to abandon should be manifested by the bear the consequences of an unconstitutional act which under no
officer concerned. Such intention may be express or inferred circumstance can be attributed to him. However, before
from his own conduct. Thus, the failure to perform the duties Canonizado can re-assume his post as Commissioner, he should
pertaining to the office must be with the officer’s actual or first resign as Inspector General of the IAS-PNP. (Canonizado
imputed intention to abandon and relinquish the office. v. Aguirre, 351 SCRA 659, Feb. 15, 2001, En Banc [Gonzaga-
Abandonment of an office is not wholly a matter of intention; it Reyes])
results from a complete abandonment of duties of such
continuance that the law will infer a relinquishment. Therefore,
139
342. When may unconsented transfers be considered Canonizado, et al. v. Hon. Alexander P. Aguirre, et al., G.R.
anathema to security of tenure? No. 133132, Jan. 25, 2000, En Banc [Gonzaga-Reyes])

Held: As held in Sta. Maria v. Lopez: 344. What is reorganization? When is it valid? When is
it invalid?
"x x x the rule that outlaws unconsented
transfers as anathema to security of tenure applies only Held: 1. Reorganization takes place when there is an
to an officer who is appointed - not merely assigned - alteration of the existing structure of government offices or units
to a particular station. Such a rule does not pr[o]scribe therein, including the lines of control, authority and
a transfer carried out under a specific statute that responsibility between them. It involves a reduction of
empowers the head of an agency to periodically personnel, consolidation of offices, or abolition thereof by
reassign the employees and officers in order to improve reason of economy or redundancy of functions. Naturally, it
the service of the agency. x x x" may result in the loss of one's position through removal or
abolition of an office. However, for a reorganization to be
The guarantee of security of tenure under the Constitution is not valid, it must also pass the test of good faith, laid down in Dario
a guarantee of perpetual employment. It only means that an v. Mison:
employee cannot be dismissed (or transferred) from the service
for causes other than those provided by law and after due x x x As a general rule, a reorganization is
process is accorded the employee. What it seeks to prevent is carried out in "good faith" if it is for the purpose of
capricious exercise of the power to dismiss. But where it is the economy or to make bureaucracy more efficient. In
law-making authority itself which furnishes the ground for the that event, no dismissal (in case of dismissal) or
transfer of a class of employees, no such capriciousness can be separation actually occurs because the position itself
raised for so long as the remedy proposed to cure a perceived ceases to exist. And in that case, security of tenure
evil is germane to the purposes of the law. (Agripino A. De would not be a Chinese wall. Be that as it may, if the
Guzman, Jr., et al. v. COMELEC, G.R. No. 129118, July 19, "abolition" which is nothing else but a separation or
2000, En Banc [Purisima]) removal, is done for political reasons or purposely to
defeat security of tenure, or otherwise not in good faith,
343. Discuss Abolition of Office? no valid "abolition" takes place and whatever
"abolition" is done, is void ab initio. There is an
Held: The creation and abolition of public offices is invalid "abolition" as where there is merely a change of
primarily a legislative function. It is acknowledged that nomenclature of positions, or where claims of economy
Congress may abolish any office it creates without impairing the are belied by the existence of ample funds.
officer's right to continue in the position held and that such (Alexis C. Canonizado, et al. v. Hon. Alexander P. Aguirre, et
power may be exercised for various reasons, such as the lack of al., G.R. No. 133132, Jan. 25, 2000, En Banc [Gonzaga-
funds or in the interest of economy. However, in order for the Reyes])
abolition to be valid, it must be made in good faith, not for
political or personal reasons, or in order to circumvent the 2. While the President’s power to reorganize can not
constitutional security of tenure of civil service employees. be denied, this does not mean however that the reorganization
itself is properly made in accordance with law. Well-settled is
An abolition of office connotes an intention to do away the rule that reorganization is regarded as valid provided it is
with such office wholly and permanently, as the word pursued in good faith. Thus, in Dario v. Mison, this Court has
"abolished" denotes. Where one office is abolished and had the occasion to clarify that:
replaced with another office vested with similar functions, the
abolition is a legal nullity. Thus, in U.P. Board of Regents v. “As a general rule, a reorganization is carried out in
Rasul we said: ‘good faith’ if it is for the purpose of economy or to
make the bureaucracy more efficient. In that event no
It is true that a valid and bona fide abolition of dismissal or separation actually occurs because the
an office denies to the incumbent the right to security position itself ceases to exist. And in that case the
of tenure (De la Llana v. Alba, 112 SCRA 294 [1982]). security of tenure would not be a Chinese wall. Be that
However, in this case, the renaming and restructuring as it may, if the abolition which is nothing else but a
of the PGH and its component units cannot give rise to separation or removal, is done for political reasons or
a valid and bona fide abolition of the position of PGH purposely to defeat security of tenure, or otherwise not
Director. This is because where the abolished office in good faith, no valid abolition takes place and
and the offices created in its place have similar whatever abolition done is void ab initio. There is an
functions, the abolition lacks good faith (Jose L. invalid abolition as where there is merely a change of
Guerrero v. Hon. Antonio V. Arizabal, G.R. No. 81928, nomenclature of positions or where claims of economy
June 4, 1990, 186 SCRA 108 [1990]). We hereby are belied by the existence of ample funds.”
apply the principle enunciated in Cezar Z. Dario v. (Larin v. Executive Secretary, 280 SCRA 713, Oct. 16, 1997)
Hon. Salvador M. Mison (176 SCRA 84 [1989]) that
abolition which merely changes the nomenclature of 345. What are the circumstances evidencing bad faith in
positions is invalid and does not result in the removal the removal of employees as a result of
of the incumbent. reorganization and which may give rise to a claim
for reinstatement or reappointment)?
The above notwithstanding, and assuming that
the abolition of the position of the PGH Director and Held:
the creation of a UP-PGH Medical Center Director are
valid, the removal of the incumbent is still not justified Where there is a significant increase in the number of
for the reason that the duties and functions of the two positions in the new staffing pattern of the
positions are basically the same. department or agency concerned;
Where an office is abolished and another performing
This was also our ruling in Guerrero v. Arizabal, substantially the same functions is created;
wherein we declared that the substantial identity in the functions Where incumbents are replaced by those less qualified
between the two offices was indicia of bad faith in the removal in terms of status of appointment, performance and
of petitioner pursuant to a reorganization. (Alexis C. merit;

140
Where there is a reclassification of offices in the circumstances of the times. (Akbayan-Youth v. COMELEC,
department or agency concerned and the 355 SCRA 318, Mar. 26, 2001, En Banc [Buena])
reclassified offices perform substantially the same
functions as the original offices; 348. Discuss the reason behind the principle of ballot
Where the removal violates the order of separation secrecy. May the conduct of exit polls transgress
provided in Section 3 hereof. the sanctity and the secrecy of the ballot to justify
(Sec. 2, R.A. No. 6656; Larin v. Executive Secretary, 280 its prohibition?
SCRA 713, Oct. 16, 1997)
Held: The reason behind the principle of ballot
secrecy is to avoid vote buying through voter identification.
E. ELECTION LAWS Thus, voters are prohibited from exhibiting the contents of their
official ballots to other persons, from making copies thereof, or
346. Discuss the Right of Suffrage, and its substantive from putting distinguishing marks thereon so as to be identified.
and procedural requirements. Also proscribed is finding out the contents of the ballots cast by
particular voters or disclosing those of disabled or illiterate
Held: In a representative democracy such as ours, the voters who have been assisted. Clearly, what is forbidden is the
right of suffrage, although accorded a prime niche in the association of voters with their respective votes, for the purpose
hierarchy of rights embodied in the fundamental law, ought to of assuring that the votes have been cast in accordance with the
be exercised within the proper bounds and framework of the instructions of a third party. This result cannot, however, be
Constitution and must properly yield to pertinent laws skillfully achieved merely through the voters’ verbal and confidential
enacted by the Legislature, which statutes for all intents and disclosure to a pollster of whom they have voted for.
purposes, are crafted to effectively insulate such so cherished
right from ravishment and preserve the democratic institutions In exit polls, the contents of the official ballot are not
our people have, for so long, guarded against the spoils of actually exposed. Furthermore, the revelation of whom an
opportunism, debauchery and abuse. elector has voted for is not compulsory, but voluntary. Voters
may also choose not to reveal their identities. Indeed, narrowly
To be sure, the right of suffrage x x x is not at all tailored countermeasures may be prescribed by the Comelec, so
absolute. Needless to say, the exercise of the right of suffrage, as to minimize or suppress incidental problems in the conduct of
as in the enjoyment of all other rights, is subject to existing exit polls, without transgressing the fundamental rights of our
substantive and procedural requirements embodied in our people. (ABS-CBN Broadcasting Corporation v. COMELEC,
Constitution, statute books and other repositories of law. Thus, G.R. No. 133486, Jan. 28, 2000, En Banc [Panganiban])
as to the substantive aspect, Section 1, Article V of the
Constitution provides: 349. Does Section 5(d) of Rep. Act No. 9189 violate
Section 1, Article V of the 1987 Constitution of the
“SECTION 1. SUFFRAGE MAY BE Republic of the Philippines?
EXERCISED BY ALL CITIZENS OF THE
PHILIPPINES NOT OTHERWISE DISQUALIFIED Held: [P]etitioner posits that Section 5(d) is
BY LAW, WHO ARE AT LEAST EIGHTEEN unconstitutional because it violates Section 1, Article V of the
YEARS OF AGE, AND WHO SHALL HAVE 1987 Constitution which requires that the voter must be a
RESIDED IN THE PHILIPPINES FOR AT LEAST resident in the Philippines for at least one year and in the place
ONE YEAR AND IN THE PLACE WHEREIN THEY where he proposes to vote for at least six months immediately
PROPOSE TO VOTE FOR AT LAST SIX MONTHS preceding an election. Petitioner cites the ruling of the Court in
IMMEDIATELY PRECEDING THE ELECTION. Caasi v. Court of Appeals (G.R. No. 88831, 8 November 1990,
NO LITERACY, PROPERTY, OR OTHER 191 SCRA 229) to support his claim. In that case, the Court held
SUBSTANTIVE REQUIREMENT SHALL BE that a “green card” holder immigrant to the United States is
IMPOSED ON THE EXERCISE OF SUFFRAGE.” deemed to have abandoned his domicile and residence in the
Philippines.
As to the procedural limitation, the right of a citizen to
vote is necessarily conditioned upon certain procedural Petitioner further argues that Section 1, Article V of the
requirements he must undergo: among others, the process of Constitution does not allow provisional registration or a promise
registration. Specifically, a citizen in order to be qualified to by a voter to perform a condition to be qualified to vote in a
exercise his right to vote, in addition to the minimum political exercise; that the legislature should not be allowed to
requirements set by the fundamental charter, is obliged by law circumvent the requirement of the Constitution on the right of
to register, at present, under the provisions of Republic Act No. suffrage by providing a condition thereon which in effect
8189, otherwise known as the “Voter’s Registration Act of amends or alters the aforesaid residence requirement to qualify a
1996.” (Akbayan-Youth v. COMELEC, 355 SCRA 318, Mar. Filipino abroad to vote. He claims that the right of suffrage
26, 2001, En Banc [Buena]) should not be granted to anyone who, on the date of the election,
does not possess the qualifications provided for by Section 1,
347. Discuss the nature of Voter’s Registration. Article V of the Constitution.

Held: Stated differently, the act of registration is an Xxx


indispensable precondition to the right of suffrage. For
registration is part and parcel of the right to vote and an The seed of the present controversy is the interpretation
indispensable element in the election process. Thus, x x x that is given to the phrase, “qualified citizens of the Philippines
registration cannot and should not be denigrated to the lowly abroad” as it appears in R.A. No. 9189 x x x.
stature of a mere statutory requirement. Proceeding from the
significance of registration as a necessary requisite to the right X x x Under Section 5(d) of R.A. No. 9189, one of
to vote, the State undoubtedly, in the exercise of its inherent those disqualified from voting is an immigrant or permanent
police power, may then enact laws to safeguard and regulate the resident who is recognized as such in the host country unless
act of voter’s registration for the ultimate purpose of conducting he/she executes an affidavit declaring that he/she shall resume
honest, orderly and peaceful election, to the incidental yet actual physical permanent residence in the Philippines not later
generally important end, that even pre-election activities could than three years from approval of his/her registration under said
be performed by the duly constituted authorities in a realistic Act.
and orderly manner – one which is not indifferent and so far
removed from the pressing order of the day and the prevalent
141
Petitioner questions the rightness of the mere act of residency requirement of voters x x x. Thus, Section 2, Article
execution of an affidavit to qualify the Filipinos abroad who are V of the Constitution came into being to remove any doubt as to
immigrants or permanent residents, to vote. He focuses solely the inapplicability of the residency requirement in Section 1. It
on Section 1, Article V of the Constitution in ascribing is precisely to avoid any problems that could impede the
constitutional infirmity to Section 5(d) of R.A. No. 9189, totally implementation of its pursuit to enfranchise the largest number
ignoring the provisions of Section 2 empowering Congress to of qualified Filipinos who are not in the Philippines that the
provide a system for absentee voting by qualified Filipinos Constitutional Commission explicitly mandated Congress to
abroad. provide a system for overseas absentee voting.

A simple, cursory reading of Section 5(d) of R.A. No. The discussion of the Constitutional Commission on
9189 may indeed give the impression that it contravenes Section the effect of the residency requirement prescribed by Section 1,
1, Article V of the Constitution. Filipino immigrants and Article V of the Constitution on the proposed system for
permanent residents overseas are perceived as having left and absentee voting for qualified Filipinos abroad is enlightening x x
abandoned the Philippines to live permanently in their host x.
countries and therefore, a provision in the law enfranchising
those who do not possess the residency requirement of the Clearly therefrom, the intent of the Constitutional
Constitution by the mere act of executing an affidavit expressing Commission is to entrust to Congress the responsibility of
their intent to return to the Philippines within a given period, devising a system of absentee voting. The qualifications of
risks a declaration of unconstitutionality. However, the risk is voters as stated in Section 1 shall remain except for the
more apparent than real. residency requirement. This is in fact the reason why the
Constitutional Commission opted for the term qualified
Xxx Filipinos abroad with respect to the system of absentee voting
that Congress should draw up. As stressed by Commissioner
As the essence of R.A. No. 9189 is to enfranchise Monsod, by the use of the adjective qualified with respect to
overseas qualified Filipinos, it behooves the Court to take a Filipinos abroad, the assumption is that they have the
holistic view of the pertinent provisions of both the Constitution “qualifications and none of the disqualifications to vote.” In
and R.A. No. 9189. It is a basic rule in constitutional fine-tuning the provision on absentee voting, the Constitutional
construction that the Constitution should be construed as a Commission discussed how the system should work x x x.
whole. X x x
It is clear from these discussions of the members of the
R.A. No. 9189 was enacted in obeisance to the Constitutional Commission that they intended to enfranchise as
mandate of the first paragraph of Section 2, Article V of the much as possible all Filipino citizens abroad who have not
Constitution that Congress shall provide a system for voting by abandoned their domicile of origin. The Commission even
qualified Filipinos abroad. It must be stressed that Section 2 intended to extend to young Filipinos who reach voting age
does not provide for the parameters of the exercise of legislative abroad whose parents’ domicile of origin is in the Philippines,
authority in enacting said law. Hence, in the absence of and consider them qualified as voters for the first time.
restrictions, Congress is presumed to have duly exercised its
function as defined in Article VI (the Legislative Department) of It is in pursuance of that intention that the Commission
the Constitution. provided for Section 2 immediately after the residency
requirement of Section 1. By the doctrine of necessary
To put matters in their right perspective, it is necessary implication in statutory construction, which may be applied in
to dwell first on the significance of absentee voting. The construing constitutional provisions (Marcelino v. Cruz, 121
concept of absentee voting is relatively new. X x x SCRA 51, 56), the strategic location of Section 2 indicates that
the Constitutional Commission provided for an exception to the
Ordinarily, an absentee is not a resident and vice versa; actual residency requirement of Section 1 with respect to
a person cannot be at the same time, both a resident and an qualified Filipinos abroad. The same Commission has in effect
absentee (1 WORDS AND PHRASES 264 citing Savant v. declared that qualified Filipinos who are not in the Philippines
Mercadal, 66 So. 961, 962, 136 La., 248). However, under our may be allowed to vote though they do not satisfy the residency
election laws and the countless pronouncements of the Court requirement in Section 1, Article V of the Constitution.
pertaining to elections, an absentee remains attached to his
residence in the Philippines as residence is considered That Section 2 of Article V of the Constitution is an
synonymous with domicile. exception to the residency requirement found in Section 1 of the
same Article was in fact the subject of debate when Senate Bill
Xxx No. 2104, which became R.A. No. 9189, was deliberated upon
on the Senate floor x x x.
Aware of the domiciliary legal tie that links an
overseas Filipino to his residence in this country, the framers of Xxx
the Constitution considered the circumstances that impelled
them to require Congress to establish a system for overseas Accordingly, Section 4 of R.A. No. 9189 provides for
absentee voting x x x. the coverage of the absentee voting process x x x which does
not require physical residency in the Philippines; and Section 5
Xxx of the assailed law which enumerates those who are disqualified
x x x.
Thus, the Constitutional Commission recognized the
fact that while millions of Filipinos reside abroad principally for As finally approved into law, Section 5(d) of R.A. No.
economic reasons and hence they contribute in no small 9189 specifically disqualifies an immigrant or permanent
measure to the economic uplift of this country, their voices are resident who is “recognized as such in the host country”
marginal insofar as the choice of this country’s leaders is because immigration or permanent residence in another country
concerned. implies renunciation of one’s residence in his country of origin.
However, same Section allows an immigrant and permanent
The Constitutional Commission realized that under the resident abroad to register as voter for as long as he/she executes
laws then existing and considering the novelty of the system of an affidavit to show that he/she has not abandoned his domicile
absentee voting in this jurisdiction, vesting overseas Filipinos in pursuance of the constitutional intent expressed in Sections 1
with the right to vote would spawn constitutional problems and 2 of Article V that “all citizens of the Philippines not
especially because the Constitution itself provides for the otherwise disqualified by law” must be entitled to exercise the
142
right of suffrage and, that Congress must establish a system for presumed not to have lost his domicile by his physical absence
absentee voting; for otherwise, if actual, physical residence in from this country. His having become an immigrant or
the Philippines is required, there is no sense for the framers of permanent resident of his host country does not necessarily
the Constitution to mandate Congress to establish a system for imply an abandonment of his intention to return to his domicile
absentee voting. of origin, the Philippines. Therefore, under the law, he must be
given the opportunity to express that he has not actually
Contrary to the claim of petitioner, the execution of the abandoned his domicile in the Philippines by executing the
affidavit itself is not the enabling or enfranchising act. The affidavit required by Sections 5(d) and 8(c) of the law.
affidavit required in Section 5(d) is not only proof of the
intention of the immigrant or permanent resident to go back and Petitioner’s speculative apprehension that the
resume residency in the Philippines, but more significantly, it implementation of Section 5(d) would affect the credibility of
serves as an explicit expression that he had not in fact the elections is insignificant as what is important is to ensure
abandoned his domicile of origin. Thus, it is not correct to say that all those who possess the qualifications to vote on the date
that the execution of the affidavit under Section 5(d) violates the of the election are given the opportunity and permitted to freely
Constitution that proscribes “provisional registration or a do so. The COMELEC and the Department of Foreign Affairs
promise by a voter to perform a condition to be qualified to vote gave enough resources and talents to ensure the integrity and
in a political exercise.” credibility of any election conducted pursuant to R.A. No. 9189.

To repeat, the affidavit is required of immigrants and As to the eventuality that the Filipino abroad would
permanent residents abroad because by their status in their host renege on his undertaking to return to the Philippines, the
countries, they are presumed to have relinquished their intent to penalty of perpetual disenfranchisement provided for by Section
return to this country; thus, without the affidavit, the 5(d) would suffice to serve as deterrence to non-compliance
presumption of abandonment of Philippine domicile shall with his/her undertaking under the affidavit.
remain.
Petitioner argues that should a sizable number of
Further perusal of the transcripts of the Senate “immigrants” renege on their promise to return, the result of the
proceedings discloses another reason why the Senate required elections would be affected and could even be a ground to
the execution of said affidavit. It wanted the affiant to exercise contest the proclamation of the winning candidates and cause
the option to return or to express his intention to return to his further confusion and doubt on the integrity of the results of the
domicile of origin and not to preempt that choice by legislation. election. Indeed, the probability that after an immigrant has
Xxx exercised the right to vote, he shall opt to remain in his host
country beyond the third year from the execution of the
Xxx affidavit, is not farfetched. However, it is not for this Court to
determine the wisdom of a legislative exercise. X x x
In the advent of The Overseas Absentee Voting Act of
2003 or R.A. No. 9189, they may still be considered as a Congress itself was conscious of said probability and in
“qualified citizen of the Philippines abroad” upon fulfillment of fact, it has addressed the expected problem. Section 5(d) itself
the requirements of registration under the new law for the provides for a deterrence which is that the Filipino who fails to
purpose of exercising their right of suffrage. return as promised stands to lose his right of suffrage. Under
Section 9, should a registered overseas absentee voter fail to
It must be emphasized that Section 5(d) does not only vote for two consecutive national elections, his name may be
require an affidavit or a promise to “resume actual physical ordered removed from the National Registry of Overseas
permanent residence in the Philippines not later than three years Absentee Voters.
from approval of his/her registration,” the Filipinos abroad must
also declare that they have not applied for citizenship in another Other serious legal questions that may be raised would
country. Thus, they must return to the Philippines otherwise, be: what happens to the votes cast by the qualified voters abroad
their failure to return “shall be cause for the removal” of their who were not able to return within three years as promised?
names “from the National Registry of absentee voters and What is the effect on the votes cast by the non-returnees in favor
his/her permanent disqualification to vote in absentia.” of the winning candidates? The votes cast by qualified Filipinos
abroad who failed to return within three years shall not be
Thus, Congress crafted a process of registration by invalidated because they were qualified to vote on the date of
which a Filipino voter permanently residing abroad who is at the elections, but their failure to return shall be cause for the
least eighteen years old, not otherwise disqualified by law, who removal of the names of the immigrants or permanent residents
has not relinquished Philippine citizenship and who has not from the National Registry of Absentee Voters and their
actually abandoned his/her intentions to return to his/her permanent disqualification to vote in absentia.
domicile of origin, the Philippines, is allowed to register and
vote in the Philippine embassy, consulate or other foreign In fine, considering the underlying intent of the
service establishments of the place which has jurisdiction over Constitution, the Court does not find Section 5(d) of R.A. No.
the country where he/she has indicated his/her address for 9189 as constitutionally defective. (Makalintal v. COMELEC,
purposes of the elections, while providing for safeguards to a G.R. No. 157013, July 10, 2003, En Banc [Austria-Martinez])
clean election.
350. Discuss the meaning and purpose of residency
Xxx requirement in Election Law.

Contrary to petitioner’s claim that Section 5(d) Held: 1. The meaning and purpose of the residency
circumvents the Constitution, Congress enacted the law requirement were explained recently in our decision in Aquino
prescribing a system of overseas absentee voting in compliance v. Comelec, as follows:
with the constitutional mandate. Such mandate expressly
requires that Congress provide a system of absentee voting that X x x [T]he place “where a party actually or
necessarily presupposes that the “qualified citizen of the constructively has his permanent home,” where he, no
Philippines abroad” is not physically present in the country. matter where he may be found at any given time,
The provisions of Sections 5(d) and 11 are components of the eventually intends to return and remain, i.e., his
system of overseas absentee voting established by R.A. No. domicile, is that to which the Constitution refers when
9189. The qualified Filipino abroad who executed the affidavit it speaks of residence for the purposes of election law.
is deemed to have retained his domicile in the Philippines. He is The manifest purpose of this deviation from the usual
143
conceptions of residency in law as explained in
Gallego v. Vera is “to exclude strangers or newcomers Domicile of origin is acquired by every person at birth
unfamiliar with the conditions and needs of the and continues until replaced by the acquisition of another
community” from taking advantage of favorable domicile. More specifically, it is the domicile of the child’s
circumstances existing in that community for electoral parents or of the persons upon whom the child is legally
gain. While there is nothing wrong with the practice of dependent at birth. Although referred to as domicile of birth,
establishing residence in a given area for meeting domicile of origin is actually the domicile of one’s parents at the
election law requirements, this nonetheless defeats the time of birth and may not necessarily be the actual place of
essence of representation, which is to place through the one’s birth (25 Am Jur 2d, Domicil Sec 11 at 13). Domicile of
assent of voters those most cognizant and sensitive to choice is a domicile chosen by a person to replace his or her
the needs of a particular district, if a candidate falls former domicile. An adult may change domicile at will. The
short of the period of residency mandated by law for choice involves an exercise of free will and presumes legal
him to qualify. That purpose could be obviously best capacity to make a choice. While intention is a principal feature
met by individuals who have either had actual of domicile of choice, a mere intention without the fact of actual
residence in the area for a given period or who have presence in the locality cannot bring about the acquisition of a
been domiciled in the same area either by origin or by new domicile. Domicile of choice generally consists of a bodily
choice. presence in a particular locality and a concurrent intent to
(Marcita Mamba Perez v. COMELEC, G.R. No. 133944, Oct. remain there permanently or at least indefinitely (Id. at Sec 12).
28, 1999, En Banc [Mendoza]) Domicle by operation of law is a domicile that the law
attributes to a person independent of a person’s residence or
2. The Constitution and the law requires residence as a intention. It applies to infants, incompetents, and other persons
qualification for seeking and holding elective public office, in under disabilities that prevent them from acquiring a domicile of
order to give candidates the opportunity to be familiar with the choice (Id. at sec 13). (Puno, Concurring and Dissenting
needs, difficulties, aspirations, potentials for growth and all Opinion in Makalintal v. COMELEC, G.R. No. 157013, July
matters vital to the welfare of their constituencies; likewise, it 10, 2003, En Banc [Austria-Martinez])
enables the electorate to evaluate the office seekers’
qualifications and fitness for the job they aspire for. Inasmuch 353. What is required to successfully effect a change of
as Vicente Y. Emano has proven that he, together with his domicile? Is a Filipino who becomes an
family, (1) had actually resided in a house he bought in 1973 in “immigrant” or “permanent resident” of a foreign
Cagayan de Oro City; (2) had actually held office there during country considered to have changed his domicile?
his three terms as provincial governor of Misamis Oriental, the
provincial capitol being located therein; and (3) has registered as Held: In Romualdez-Marcos v. COMELEC, we
voter in the city during the period required by law, he could not ruled that domicile of origin is not easily lost. To successfully
be deemed “a stranger or newcomer” when he ran for and was effect a change of domicile, one must demonstrate an actual
overwhelmingly voted as city mayor. Election laws must be removal or an actual change of domicile; a bona fide intention
liberally construed to give effect to the popular mandate. of abandoning the former place of residence and establishing a
(Torayno, Sr. v. COMELEC, 337 SCRA 574, Aug. 9, 2000, En new one; and acts which correspond with purpose. This change
Banc [Panganiban]) of domicile is effected by a Filipino who becomes an
“immigrant” or a “permanent resident” of a foreign
3. Generally, in requiring candidates to have a country. Thus, we held in Caasi v. Court of Appeals (Supra
minimum period of residence in the area in which they seek to note 4), viz:
be elected, the Constitution or the law intends to prevent the
possibility of a “stranger or newcomer unacquainted with the Miguel’s application for immigrant and
conditions and needs of a community and not identified with the permanent residence in the U.S. and his possession of a
latter from [seeking] an elective office to serve that green card attesting to such status are conclusive proof
community.” Such provision is aimed at excluding outsiders that he is a permanent resident of the U.S. despite his
“from taking advantage of favorable circumstances existing in occasional visits to the Philippines. The waiver of such
that community for electoral gain.” Establishing residence in a immigrant status should be as indubitable as his
community merely to meet an election law requirement defeats application for it. Absent clear evidence that he made
the purpose of representation: to elect through the assent of an irrevocable waiver of that status or that he
voters those most cognizant and sensitive to the needs of the surrendered his green card to the appropriate U.S.
community. This purpose is “best met by individuals who have authorities before he ran for mayor x x x (Id. at 237)
either had actual residence in the area for a given period or who
have been domiciled in the same area either by origin or by The doctrine in Caasi is by no means new. Our
choice.” (Torayno, Sr. v. COMELEC, 337 SCRA 574, Aug. 9, election laws have continuously regarded “immigrants” or
2000, En Banc [Panganiban]) “permanent residents” of a foreign country to have lost their
domiciles in the Philippines and hence are not qualified to run
351. Does the fact that a person is registered as a voter for public office (See for instance, Rep. Act No. 7160, section
in one district proof that he is not domiciled in 40(f); B.P. Blg. 52, sec. 4; B.P. Blg. 881, sec. 68). There is no
another district? reason not to apply the Caasi ruling in disputes involving the
qualification of voters. In essence, both cases concern the
Held: The fact that a person is registered as a voter in fulfillment of the residence requirements. (Puno, Concurring
one district is not proof that he is not domiciled in another and Dissenting Opinion in Makalintal v. COMELEC, G.R.
district. Thus, in Faypon v. Quirino, this Court held that the No. 157013, July 10, 2003, En Banc [Austria-Martinez])
registration of a voter in a place other than his residence of
origin is not sufficient to consider him to have abandoned or lost 354. What is the Lone Candidate Law? What are its
his residence. (Marcita Mamba Perez v. COMELEC, G.R. No. salient provisions?
133944, Oct. 28, 1999, En Banc [Mendoza])
Ans.: The Lone Candidate Law is Republic Act No.
352. What are the three classes of domicile? Discuss. 8295, enacted on June 6, 1997. Section 2 thereof provides that
“Upon the expiration of the deadline for the filing of the
Held: There are three classes of domicile, namely: certificate of candidacy in a special election called to fill a
domicile of origin, domicile of choice, and domicile by vacancy in an elective position other than for President and
operation of law. At any given point, a person can only have Vice-President, when there is only one (1) qualified candidate
one domicile. for such position, the lone candidate shall be proclaimed elected
144
to the position by proper proclaiming body of the Commission substituted under Section 77 of the Code. (Miranda v. Abaya,
on Elections without holding the special election upon G.R. No. 136351, July 28, 1999, en Banc [Melo])
certification by the Commission on Elections that he is the only
candidate for the office and is thereby deemed elected.” 358. Should the votes cast for the substituted candidate
be considered votes for the substitute candidate?
Section 3 thereof provides that “the lone candidate so
proclaimed shall assume office not earlier than the scheduled Ans.: Republic Act No. 9006, otherwise known as the
election day, in the absence of any lawful ground to deny due Fair Election Act, provides in Section 12 thereof: “In case of
course or cancel the certificate of candidacy in order to prevent valid substitutions after the official ballots have been printed,
such proclamation, as provided for under Sections 69 and 78 of the votes cast for the substituted candidates shall be considered
Batas Pambansa Bilang 881 also known as the Omnibus as stray votes but shall not invalidate the whole ballot. For this
Election Code.” purpose, the official ballots shall provide spaces where the
voters may write the name of the substitute candidates if they
355. Who are disqualified to run in a special election are voting for the latter: Provided, however, That if the
under the Lone Candidate Law? substitute candidate is of the same family name, this provision
shall not apply.”
Ans.: Section 4 of the Lone Candidate Law provides
that “In addition to the disqualifications mentioned in Sections 359. What is the effect of the filing of certificate of
12 and 68 of the Omnibus Election Code and Section 40 of candidacy by elective officials?
Republic Act No. 7160, otherwise known as the Local
Government Code, whenever the evidence of guilt is strong, the Ans.: COMELEC Resolution No. 3636, promulgated
following persons are disqualified to run in a special election March 1, 2001, implementing the Fair Election Act (R.A. No.
called to fill the vacancy in an elective office, to wit: 9006) provides in Section 26 thereof: “any elective official,
whether national or local, who has filed a certificate of
a) Any elective official who has resigned from his candidacy for the same or any other office shall not be
office by accepting an appointive office or for considered resigned from his office.”
whatever reason which he previously occupied
but has caused to become vacant due to his Note that Section 67 of the Omnibus Election Code and
resignation; and the first proviso in the third paragraph of Section 11 of Republic
b) Any person who, directly or indirectly, coerces, Act No. 8436 which modified said Section 67, were expressly
bribes, threatens, harasses, intimidates or actually repealed and rendered ineffective, respectively, by Section 14
causes, inflicts or produces any violence, injury, (Repealing Clause) of The Fair Election Act (R.A. No. 9006).
punishment, torture, damage, loss or disadvantage
to any person or persons aspiring to become a 360. What kind of “material misrepresentation” is
candidate or that of the immediate member of his contemplated by Section 78 of the Omnibus
family, his honor or property that is meant to Election Code as a ground for disqualification of a
eliminate all other potential candidate.” candidate? Does it include the use of surname?

356. What is the purpose of the law in requiring the Held: Therefore, it may be concluded that the material
filing of certificate of candidacy and in fixing the misrepresentation contemplated by Section 78 of the (Omnibus
time limit therefor? Election) Code refers to qualifications for elective office. This
conclusion is strengthened by the fact that the consequences
Held: The evident purpose of the law in requiring the imposed upon a candidate guilty of having made a false
filing of certificate of candidacy and in fixing the time limit representation in his certificate of candidacy are grave – to
therefor are: (a) to enable the voters to know, at least sixty days prevent the candidate from running or, if elected, from serving,
before the regular election, the candidates among whom they are or to prosecute him for violation of the election laws. It could
to make the choice, and (b) to avoid confusion and not have been the intention of the law to deprive a person of
inconvenience in the tabulation of the votes cast. For if the law such a basic and substantial political right to be voted for a
did not confine the choice or election by the voters to the duly public office upon just any innocuous mistake.
registered candidates, there might be as many persons voted for
as there are voters, and votes might be cast even for unknown or [A]side from the requirement of materiality, a false
fictitious persons as a mark to identify the votes in favor of a representation under Section 78 must consist of a “deliberate
candidate for another office in the same election. (Miranda v. attempt to mislead, misinform, or hide a fact which would
Abaya, G.R. No. 136351, July 28, 1999) otherwise render a candidate ineligible.” In other words, it must
be made with an intention to deceive the electorate as to one’s
357. May a disqualified candidate and whose certificate qualifications for public office. The use of a surname, when not
of candidacy was denied due course and/or intended to mislead or deceive the public as to one’s identity, is
canceled by the Comelec be validly substituted? not within the scope of the provision. (Victorino Salcedo II v.
COMELEC, G.R. No. 135886, Aug. 16, 1999, En Banc
Held: Even on the most basic and fundamental [Gonzaga-Reyes])
principles, it is readily understood that the concept of a
substitute presupposes the existence of the person to be 361. Who has authority to declare failure of elections
substituted, for how can a person take the place of somebody and the calling of special election? What are the
who does not exist or who never was. The Court has no other three instances where a failure of election may be
choice but to rule that in all instances enumerated in Section 77 declared?
of the Omnibus Election Code, the existence of a valid
certificate of candidacy seasonably filed is a requisite sine qua Held: The COMELEC’s authority to declare failure of
non. elections is provided in our election laws. Section 4 of RA 7166
provides that the Comelec sitting en banc by a majority vote of
All told, a disqualified candidate may only be its members may decide, among others, the declaration of
substituted if he had a valid certificate of candidacy in the first failure of election and the calling of special election as provided
place because, if the disqualified candidate did not have a valid in Section 6 of the Omnibus Election Code. X x x
and seasonably filed certificate of candidacy, he is and was not a
candidate at all. If a person was not a candidate, he cannot be There are three instances where a failure of election
may be declared, namely, (a) the election in any polling place
145
has not been held on the date fixed on account of force majeure, candidate. The board then decided to use the copies of election
violence, terrorism, fraud or other analogous causes; (b) the returns furnished to the municipal trial court. Petitioner therein
election in any polling place has been suspended before the hour filed a petition to stop the proceedings of the board of
fixed by law for the closing of the voting on account of force canvassers on the ground that it had no authority to use said
majeure, violence, terrorism, fraud or other analogous causes; or election returns obtained from the municipal trial court. The
(c) after the voting and during the preparation and transmission petition was denied. Next, he filed a petition assailing the
of the election returns or in the custody or canvass thereof, such composition of the board of canvassers. Despite that petition,
election results in a failure to elect on account of force majeure, the board of canvassers proclaimed the winning candidates.
violence, terrorism, fraud or other analogous causes. In these Later on, petitioner filed a petition to declare a failure of
instances, there is a resulting failure to elect. This is obvious in election alleging that the attendant facts would justify
the first two scenarios, where the election was not held and declaration of such failure. On review, we ruled that
where the election was suspended. As to the third scenario, petitioner’s first two actions involved pre-proclamation
where the preparation and the transmission of the election controversies which can no longer be entertained after the
returns give rise to the consequence of failure to elect, it must x winning candidates have been proclaimed. Regarding the
x x, be interpreted to mean that nobody emerged as a winner. petition to declare a failure of election, we held that the
(Banaga, Jr. v. COMELEC, 336 SCRA 701, July 31, 2000, En destruction and loss of copies of election returns intended for
Banc [Quisumbing]) the municipal board of canvassers on account of violence is not
one of the causes that would warrant the declaration of failure of
362. What are the two conditions that must concur election. The reason is that voting actually took place as
before the COMELEC can act on a verified scheduled and other valid election returns still existed.
petition seeking to declare a afailure of election? Moreover, the destruction or loss did not affect the result of the
election. We also declared that there is failure of elections only
Held: Before the COMELEC can act on a verified when the will of the electorate has been muted and cannot be
petition seeking to declare a failure of election two conditions ascertained. If the will of the people is determinable, the same
must concur, namely: (1) no voting took place in the precinct or must as far as possible be respected.
precincts on the date fixed by law, or even if there was voting,
the election resulted in a failure to elect; and (2) the votes not Xxx
cast would have affected the result of the election. Note that the
cause of such failure of election could only be any of the In Loong v. COMELEC, the petition for annulment of
following: force majeure, violence, terrorism, fraud or other election results or to declare failure of elections in Parang, Sulu,
analogous causes. on the ground of statistical improbability and massive fraud was
granted by the COMELEC. Even before the technical
Thus, in Banaga, Jr. v. COMELEC, the SC held: examination of election documents was conducted, the Comelec
already observed badges of fraud just by looking at the election
“We have painstakingly examined the petition results in Parang. Nevertheless, the Comelec dismissed the
filed by petitioner Banaga before the Comelec. But we petition for annulment of election results or to declare failure of
found that petitioner did not allege at all that elections were elections in the municipalities of Tapul, Panglima Estino, Pata,
either not held or suspended. Neither did he aver that Siasi and Kalinggalang Calauag. The COMELEC dismissed the
although there was voting, nobody was elected. On the latter action on ground of untimeliness of the petition, despite a
contrary, he conceded that an election took place for the finding that the same badges of fraud evident from the results of
office of vice-mayor of Paranaque City, and that private the election based on the certificates of canvass of votes in
respondent was, in fact, proclaimed elected to that post. Parang, are also evident in the election results of the five
While petitioner contends that the election was tainted with mentioned municipalities. We ruled that Comelec committed
widespread anomalies, it must be noted that to warrant a grave abuse of discretion in dismissing the petition as there is no
declaration of failure of election the commission of fraud law which provides for a reglementary period to file annulment
must be such that it prevented or suspended the holding of of elections when there is yet no proclamation. The election
an election, or marred fatally the preparation and resulted in a failure to elect on account of fraud. Accordingly,
transmission, custody and canvass of the election returns. we ordered the Comelec to reinstate the aforesaid petition.
These essential facts ought to have been alleged clearly by Those circumstances, however, are not present in this case, so
the petitioner below, but he did not.” that reliance on Loong by petitioner Banaga is misplaced.
(Banaga, Jr. v. COMELEC, 336 SCRA 701, July 31, 2000, En
363. Cite instances when Comelec may or may not Banc [Quisumbing])
validly declare failure of elections.
364. Is a petition to declare failure of election different
Held: In Mitmug v. COMELEC, petitioner instituted from a petition to annul the election results?
with the COMELEC an action to declare failure of election in
forty-nine precincts where less than a quarter of the electorate Held: A prayer to declare failure of elections and a
were able to cast their votes. He also lodged an election protest prayer to annul the election results x x x are actually of the same
with the Regional Trial Court disputing the result of the election nature. Whether an action is for declaration of failure of
in all precincts in his municipality. The Comelec denied motu elections or for annulment of election results, based on
proprio and without due notice and hearing the petition to allegations of fraud, terrorism, violence or analogous causes, the
declare failure of election despite petitioner’s argument that he Omnibus Election Code denominates them similarly. (Banaga,
has meritorious grounds in support thereto, that is, massive Jr. v. COMELEC, 336 SCRA 701, July 31, 2000, En Banc
disenfranchisement of voters due to terrorism. On review, we [Quisumbing])
ruled that the Comelec did not gravely abuse its discretion in
denying the petition. It was not proven that no actual voting 365. What conditions must concur before the Comelec
took place. Neither was it shown that even if there was voting, can act on a verified petition seeking to declare a
the results thereon would be tantamount to failure to elect. failure of election? Is low turn-out of voters
Considering that there is no concurrence of the conditions enough basis to grant the petition?
seeking to declare failure of election, there is no longer need to
receive evidence on alleged election irregularities. Held: Before COMELEC can act on a verified petition
seeking to declare a failure of election, two (2) conditions must
In Sardea v. COMELEC, all election materials and concur: first, no voting has taken place in the precinct or
paraphernalia with the municipal board of canvassers were precincts on the date fixed by law or, even if there was voting,
destroyed by the sympathizers of the losing mayoralty
146
the election nevertheless results in failure to elect; and, second, petition to declare failure of elections or to annul election
the votes not cast would affect the result of the election. results. (Banaga, Jr. v. COMELEC, 336 SCRA 701, July 31,
2000, En Banc [Quisumbing])
There can be failure of election in a political unit only
if the will of the majority has been defiled and cannot be 367. What are pre-proclamation cases, and exceptions
ascertained. But, if it can be determined, it must be accorded thereto? What Court has jurisdiction over pre-
respect. After all, there is no provision in our election laws proclamation cases?
which requires that a majority of registered voters must cast
their votes. All the law requires is that a winning candidate Held: Pre-proclamation cases refer to any question
must be elected by a plurality of valid votes, regardless of the pertaining to or affecting the proceedings of the board of
actual number of ballots cast. Thus, even if less than 25% of the canvassers which may be raised by any candidate or by any
electorate in the questioned precincts cast their votes, the same registered political party or coalition of political parties before
must still be respected. (Mitmug v. COMELEC, 230 SCRA 54, the board or directly with the Commission, or any matter raised
Feb. 10, 1994, En Banc [Bellosillo]) under Sections 233, 234, 235 and 236 in relation to the
preparation, transmission, receipt, custody and appreciation of
366. Distinguish a petition to declare failure of elections election returns (Section 241, Omnibus Election Code). The
from an election protest. Comelec has exclusive jurisdiction over all pre-proclamation
controversies (Section 242, supra). As an exception, however,
Held: While petitioner may have intended to institute to the general rule, Section 15 of Republic Act 7166 prohibits
an election protest by praying that said action may also be candidates in the presidential, vice-presidential, senatorial and
considered an election protest, in our view, petitioner’s action is congressional elections from filing pre-proclamation cases. It
a petition to declare a failure of elections or annul election states:
results. It is not an election protest.
“Sec. 15. Pre-Proclamation Cases Not
First, his petition before the Comelec was instituted Allowed in Elections for President, Vice-President,
pursuant to Section 4 of Republic Act No. 7166 in relation to Senator, and Members of the House of Representatives.
Section 6 of the Omnibus Election Code. Section 4 of RA 7166 - For purposes of the elections for President, Vice-
refers to “postponement, failure of election and special President, Senator and Member of the House of
elections” while Section 6 of the Omnibus Election Code relates Representatives, no pre-proclamation cases shall be
to “failure of election.” It is simply captioned as “Petition to allowed on matters relating to the preparation,
Declare Failure of Elections and/or For Annulment of transmission, receipt, custody and appreciation of
Elections.” election returns or the certificates of canvass, as the
case may be. However, this does not preclude the
Second, an election protest is an ordinary action while authority of the appropriate canvassing body motu
a petition to declare a failure of elections is a special action proprio or upon written complaint of an interested
under the 1993 Comelec Rules of Procedure as amended. An person to correct manifest errors in the certificate of
election protest is governed by Rule 20 on ordinary actions, canvass or election returns before it.”
while a petition to declare failure of elections is covered by Rule
26 under special actions. The prohibition aims to avoid delay in the proclamation of the
winner in the election, which delay might result in a vacuum in
In this case, petitioner filed his petition as a special these sensitive posts. The law, nonetheless, provides an
action and paid the corresponding fee therefor. Thus, the exception to the exception. The second sentence of Section 15
petition was docketed as SPA-98-383. This conforms to allows the filing of petitions for correction of manifest errors in
petitioner’s categorization of his petition as one to declare a the certificate of canvass or election returns even in elections for
failure of elections or annul election results. In contrast, an president, vice-president and members of the House of
election protest is assigned a docket number starting with Representatives for the simple reason that the correction of
“EPC,” meaning election protest case. manifest error will not prolong the process of canvassing nor
delay the proclamation of the winner in the election. The rule is
Third, petitioner did not comply with the requirements consistent with and complements the authority of the Comelec
for filing an election protest. He failed to pay the required filing under the Constitution to “enforce and administer all laws and
fee and cash deposits for an election protest. Failure to pay regulations relative to the conduct of an election, plebiscite,
filing fees will not vest the election tribunal jurisdiction over the initiative, referendum and recall” (Section 2[1], Article IX-C,
case. Such procedural lapse on the part of a petitioner would 1987 Constitution) and its power to “decide, except those
clearly warrant the outright dismissal of his action. involving the right to vote, all questions affecting elections.”
(Section 2[3], Article IX-C, supra) (Federico S. Sandoval v.
Fourth, an en banc decision of Comelec in an ordinary COMELEC, G.R. No. 133842, Jan. 26, 2000 [Puno])
action becomes final and executory after thirty (30) days from
its promulgation, while an en banc decision in a special action 368. Who has authority to rule on petitions for
becomes final and executory after five (5) days from correction of manifest error in the certificate of
promulgation, unless restrained by the Supreme Court (Comelec canvass or election returns?
Rules of Procedure, Rule 18, Section 13 [a], [b]). For that
reason, a petition cannot be treated as both an election protest Held: The authority to rule on petitions for correction
and a petition to declare failure of elections. of manifest error is vested in the Comelec en banc. Section 7 of
Rule 27 of the 1993 COMELEC Rules of Procedure provides
Fifth, the allegations in the petition decisively that if the error is discovered before proclamation, the board of
determine its nature. Petitioner alleged that the local elections canvassers may motu proprio, or upon verified petition by any
for the office of vice-mayor in Paranaque City held on May 11, candidate, political party, organization or coalition of political
1998, denigrates the true will of the people as it was marred parties, after due notice and hearing, correct the errors
with widespread anomalies on account of vote buying, flying committed. The aggrieved party may appeal the decision of the
voters and glaring discrepancies in the election returns. He board to the Commission and said appeal shall be heard and
averred that those incidents warrant the declaration of a failure decided by the Commission en banc. Section 5, however, of the
of elections. same rule states that a petition for correction of manifest error
may be filed directly with the Commission en banc provided
Given these circumstances, public respondent cannot that such errors could not have been discovered during the
be said to have gravely erred in treating petitioner’s action as a canvassing despite the exercise of due diligence and
147
proclamation of the winning candidate had already been made. Una Kibad v. Comelec, the SC warned that the doctrine of
(Federico S. Sandoval v. COMELEC, G.R. No. 133842, Jan. statistical improbability must be viewed restrictively, the utmost
26, 2000 [Puno]) care being taken lest in penalizing the fraudulent and corrupt
practices, innocent voters become disenfranchised, a result
369. Distinguish Election Protest from Petition for Quo which hardly commends itself. (Arthur V. Velayo v.
Warranto. COMELEC, G.R. No. 135613, March 9, 2000, En Banc
[Puno])
Held: In Samad v. COMELEC, we explained that a
petition for quo warranto under the Omnibus Election Code 373. What Court has jurisdiction over election protests
raises in issue the disloyalty or ineligibility of the winning and quo warranto proceedings involving
candidate. It is a proceeding to unseat the respondent from Sangguniang Kabataan (SK) elections?
office but not necessarily to install the petitioner in his place.
An election protest is a contest between the defeated and Held: Any contest relating to the election of members
winning candidates on the ground of frauds or irregularities in of the Sangguniang Kabataan (including the chairman) –
the casting and counting of the ballots, or in the preparation of whether pertaining to their eligibility or the manner of their
the returns. It raises the question of who actually obtained the election – is cognizable by MTCs, MCTCs, and MeTCs.
plurality of the legal votes and therefore is entitled to hold the Section 6 of Comelec Resolution No. 2824 which provides that
office. (Dumayas, Jr. v. COMELEC, G.R. Nos. 141952-53, cases involving the eligibility or qualification of SK candidates
April 20, 2001, En Banc [Quisumbing]) shall be decided by the City/Municipal Election Officer whose
decision shall be final, applies only to proceedings before the
370. What is a counter-protest? When should it be election. Before proclamation, cases concerning eligibility of
filed? SK officers and members are cognizable by the Election
Officer. But after the election and proclamation, the same cases
Held: Under the Comelec Rules of Procedure, the become quo warranto cases cognizable by MTCs, MCTCs, and
protestee may incorporate in his answer a counter-protest. It has MeTCs. The distinction is based on the principle that it is the
been said that a counter-protest is tantamount to a counterclaim proclamation which marks off the jurisdiction of the courts from
in a civil action and may be presented as a part of the answer the jurisdiction of election officials.
within the time he is required to answer the protest, i.e., within
five (5) days upon receipt of the protest, unless a motion for The case of Jose M. Mercado v. Board of Election
extension is granted, in which case it must be filed before the Supervisors, in which this Court ruled that election protests
expiration of the extended time. involving SK elections are to be determined by the Board of
Election Supervisors was decided under the aegis of Comelec
As early as in the case of Arrieta v. Rodriguez, the SC Resolution No. 2499, which took effect on August 27, 1992.
had firmly settled the rule that the counter-protest must be filed However, Comelec Resolution No. 2824, which took effect on
within the period provided by law, otherwise, the forum loses its February 6, 1996 and was passed pursuant to R.A. 7808, in
jurisdiction to entertain the belatedly filed counter-protest. (Kho relation to Arts. 252-253 of the Omnibus Election Code, has
v. COMELEC, 279 SCRA 463, Sept. 25, 1997, En Banc since transferred the cognizance of such cases from the Board of
[Torres]) Election Supervisors to the MTCs, MCTCs and MeTCs. Thus,
the doctrine of Mercado is no longer controlling. (Francis King
371. What is the effect of death of a party in an election L. Marquez v. COMELEC, G.R. No. 127318, Aug. 25, 1999,
protest? Should it warrant the dismissal of the En Banc [Purisima])
protest?
374. What acts of a Division of the COMELEC may be
Held: An election protest involves both the private subject of a motion for reconsideration of the
interests of the rival candidates and the public interest in the COMELEC en banc?
final determination of the real choice of the electorate, and for
this reason, an election contest necessarily survives the death of Held: Section 5, Rule 19 of the COMELEC Rules of
the protestant or the protestee. X x x. But while the right to a Procedure, provides:
public office is personal and exclusive to the public officer, an
election protest is not purely personal and exclusive to the “SEC. 5. How Motion for Reconsideration
protestant or to the protestee such that after the death of either Disposed of. - Upon the filing of a motion to
would oust the court of all authority to continue the protest reconsider a decision, resolution, order or ruling of a
proceedings. An election contest, after all, involves not merely Division, the Clerk of Court concerned shall, within
conflicting private aspirations but is imbued with paramount twenty-four (24) hours from the filing thereof, notify
public interests. The death of the protestant neither constitutes a the presiding Commissioner. The latter shall within
ground for the dismissal of the contest nor ousts the trial court two (2) days thereafter certify the case to the
of its jurisdiction to decide the election contest. (De Castro v. Commission en banc.”
COMELEC, 267 SCRA 806, Feb. 7, 1997)
Under the above-quoted rule, the acts of a Division that
372. Does the fact that one or a few candidates in an are subject of a motion for reconsideration must have a
election got zero votes in one or a few precincts character of finality before the same can be elevated to the
adequately support a finding that the election COMELEC en banc. The elementary rule is that an order is
returns are statistically improbable? final in nature if it completely disposes of the entire case. But if
there is something more to be done in the case after its issuance,
Held: From experiences in past elections, it is possible that order is interlocutory.
for one candidate or even a few candidates to get zero votes in
one or a few precincts. As correctly pointed out by public respondent in its
assailed order of November 29, 1999, the October 11, 1999 did
Standing alone and without more, the bare fact that a not dispose of the case completely as there is something more to
candidate for public office received zero votes in one or two be done which is to decide the election protest. As such, it is the
precincts can not adequately support a finding that the subject herein public respondent (Second Division of the COMELEC)
election returns are statistically improbable. A no-vote for a which issued the interlocutory order of October 11, 1999 that
particular candidate in election returns is but one strand in the should resolve petitioner’s motion for reconsideration, not the
web of circumstantial evidence that those election returns were COMELEC en banc. Accordingly, the applicable rule on the
prepared under “duress, force and intimidation.” In the case of
148
subject is Section 5(c), Rule 3 of the COMELEC Rules of 378. What is the Metropolitan Manila Development
Procedure, which states: Authority (MMDA)? Is it a local government unit
or public corporation endowed with legislative
“Rule 3, Section 5(c). Any motion to power? May it validly exercise police power? How
reconsider a decision, resolution, order or ruling of a is it distinguished from the former Metro Manila
Division shall be resolved by the Commission en banc Council (MMC) created under PD No. 824?
except motions on interlocutory orders of the division,
which shall be resolved by the divisions which issued Held: Metropolitan or Metro Manila is a body
the order.” composed of several local government units – i.e., twelve (12)
cities and five (5) municipalities x x x. With the passage of
That only final orders of a Division may be raised Republic Act No. 7924 in 1995, Metropolitan Manila was
before the COMELEC en banc is in accordance with Article IX- declared as a “special development and administrative region”
C, Section 3 of the Constitution which mandates that only and the Administration of “metrowide” basic services affecting
motions for reconsideration of final decisions shall be decided the region placed under “a development authority” referred to
by the Commission on Elections en banc, thus: as the MMDA.

“Sec. 3. The Commission on Elections may The governing board of the MMDA is the Metro
sit en banc or in two divisions, and shall promulgate its Manila Council. The Council is composed of the mayors of the
rules of procedure in order to expedite disposition of component 12 cities and 5 municipalities, the president of the
election cases, including pre-proclamation Metro Manila Vice-Mayors’ League and the president of the
controversies. All such election cases shall be heard Metro Manila Councilors’ League. The Council is headed by a
and decided in division, provided that motions for Chairman who is appointed by the President and vested with the
reconsideration of decisions shall be decided by the rank of cabinet member. As the policy-making body of the
Commission en banc.” MMDA, the Metro Manila Council approves metro-wide plans,
programs and projects, and issues the necessary rules and
It bears stressing that under this constitutional regulations for the implementation of said plans; it approves the
provision, the COMELEC en banc shall decide motions for annual budget of the MMDA and promulgates the rules and
reconsideration only of “decisions” of a Division, meaning regulations for the delivery of basic services, collection of
those acts of final character. Clearly, the assailed order denying service and regulatory fees, fines and penalties. X x x
petitioner's demurrer to evidence, being interlocutory, may not,
be resolved by the COMELEC en banc. (Gementiza v. Clearly, the scope of the MMDA’s function is limited
Commission on Elections, 353 SCRA 724, March 6, 2001, En to the delivery of the seven (7) basic services. One of these is
Banc [Sandoval-Gutierrez]) transport and traffic management x x x.

Xxx
F. THE LAW OF PUBLIC CORPORATIONS
Clearly, the MMDA is not a political unit of
375. What is an autonomous region? government. The power delegated to the MMDA is that given
to the Metro Manila Council to promulgate administrative rules
Ans.: An autonomous region consists of provinces, and regulations in the implementation of the MMDA’s
cities, municipalities, and geographical areas sharing common functions. There is no grant of authority to enact ordinances
and distinctive historical and cultural heritage, economic and and regulations for the general welfare of the inhabitants of the
social structures, and other relevant characteristics within the metropolis. This was explicitly stated in the last Committee
framework of the Constitution and the national sovereignty as deliberations prior to the bill’s presentation to Congress. X x x
well as the territorial integrity of the Republic of the Philippines.
(Sec. 15, Art. X, 1987 Constitution) It is thus beyond doubt that the MMDA is not a local
government unit or a public corporation endowed with
376. What are administrative regions? Are they legislative power. It is not even a “special metropolitan political
considered territorial and political subdivisions of subdivision” as contemplated in Section 11, Article X of the
the State? Who has the power to create Constitution. The creation of a “special metropolitan political
administrative regions? subdivision” requires the approval by a majority of the votes
cast in a plebiscite in the political units directly affected. R.A.
Held: Administrative regions are mere groupings of No. 7924 was not submitted to the inhabitants of Metro Manila
contiguous provinces for administrative purposes. They are not in a plebiscite. The Chairman of the MMDA is not an official
territorial and political subdivisions like provinces, cities, elected by the people, but appointed by the President with the
municipalities and barangays. While the power to merge rank and privileges of a cabinet member. In fact, part of his
administrative regions is not expressly provided for in the function is to perform such other duties as may be assigned to
Constitution, it is a power which has traditionally been lodged him by the President, whereas in local government units, the
with the President to facilitate the exercise of the power of President merely exercises supervisory authority. This
general supervision over local governments. (Abbas v. emphasizes the administrative character of the MMDA.
COMELEC, 179 SCRA 287, Nov. 10, 1989, En Banc [Cortes])
Clearly then, the MMC under P.D. No. 824 is not the
377. Is there a conflict between the power of the same entity as the MMDA under R.A. No. 7924. Unlike the
President to merge administrative regions with the MMC, the MMDA has no power to enact ordinances for the
constitutional provision requiring a plebiscite in welfare of the community. It is the local government units,
the merger of local government units? acting through their respective legislative councils, that possess
legislative power and police power. In the case at bar, the
Held: There is no conflict between the power of the Sangguniang Panlungsod of Makati City did not pass any
President to merge administrative regions with the constitutional ordinance or resolution ordering the opening of Neptune Street,
provision requiring a plebiscite in the merger of local hence, its proposed opening by petitioner MMDA is illegal x x
government units because the requirement of a plebiscite in a x. (MMDA v. Bel-Air Village Association, Inc., 328 SCRA
merger expressly applies only to provinces, cities, municipalities 836, March 27, 2000, 1st Div. [Puno])
or barangays, not to administrative regions. (Abbas v.
COMELEC, 179 SCRA 287, Nov. 10, 1989, En Banc [Cortes]) 379. Discuss the concept of local autonomy.

149
Held: Autonomy is either decentralization of Code. Was the City of Butuan correct in its
administration or decentralization of power. There is assertion?
decentralization of administration when the central government
delegates administrative powers to political subdivisions in Held: Only the powers of the Land Transportation
order to broaden the base of government and in the process to Franchising Regulatory Board (LTFRB) to regulate the
make local governments more responsive and accountable, and operation of tricycles-for-hire and to grant franchises for the
ensure their fullest development as self-reliant communities and operation thereof had been devolved to local governments under
make them more effective partners in the pursuit of national the Local Government Code. Clearly unaffected by the Local
development and social progress. At the same time, it relieves Government Code are the powers of the LTO under R.A. No.
the central government of the burden of managing local affairs 4136 requiring the registration of all kinds of motor vehicles
and enables it to concentrate on national concerns. The “used or operated on or upon any public highway” in the
President exercises general supervision over them, but only to country. This can be gleaned from the explicit language of the
ensure that local affairs are administered according to law. He statute itself, as well as the corresponding guidelines issued by
has no control over their acts in the sense that he can substitute the DOTC. In fact, even the power of LGUs to regulate the
their judgments with his own. operation of tricycles and to grant franchises for the operation
thereof are still subject to the guidelines prescribed by the
Decentralization of power, on the other hand, involves DOTC. (LTO v. City of Butuan, G.R. No. 131512, Jan. 20,
an abdication of political power in favor of local government 2000, 3rd Div. [Vitug])
units declared autonomous. In that case, the autonomous
government is free to chart its own destiny and shape its own 383. The City of Pasig created Barangays Karangalan
future with minimum intervention from central authorities. and Napico and plebiscites were scheduled to ratify
According to a constitutional author, decentralization of power said creation. It was found, however, that the two
amounts to “self-immolation,” since in that event, the proposed barangays were subject of a pending
autonomous government becomes accountable not to the central boundary dispute between the City of Pasig and the
authorities but to its constituency. (Limbona v. Mangelin, 170 Municipality of Cainta in the RTC of Antipolo.
SCRA 786, Feb. 28, 1989, En Banc [Sarmiento]) Whether or not the plebiscites scheduled should be
suspended or cancelled in view of the pending
380. What kind of local autonomy is contemplated by boundary dispute between the two local
the Constitution? What about the autonomy governments and, if one had already been held,
contemplated insofar as the autonomous regions whether it should be nullified.
are concerned?
Held: To begin with, we agree with the position of the
Held: 1. The principle of local autonomy under the COMELEC that Civil Case No. 94-3006 involving the boundary
1987 Constitution simply means “decentralization.” It does not dispute between the Municipality of Cainta and the City of
make local governments sovereign within the state or an Pasig presents a prejudicial question which must first be decided
“imperium in imperio.” Remaining to be an intra sovereign before the plebiscites for the creation of the proposed barangays
subdivision of one sovereign nation, but not intended, however, may be held.
to be an imperium in imperio,” the local government unit is
autonomous in the sense that it is given more powers, authority, Xxx
responsibilities and resources. Power which used to be highly
centralized in Manila, is thereby deconcentrated, enabling In the case at bar, while the City of Pasig vigorously
especially the peripheral local government units to develop not claims that the areas covered by the proposed Barangays
only at their own pace and discretion but also with their own Karangalan and Napico are within its territory, it can not deny
resources and assets. (Alvarez v. Guingona, Jr., 252 SCRA 695, that portions of the same area are included in the boundary
Jan. 31, 1996, En Banc [Hermosisima]) dispute case pending before the Regional Trial Court of
Antipolo. Surely, whether the areas in controversy shall be
2. The constitutional guarantee of local autonomy in decided as within the territorial jurisdiction of the Municipality
the Constitution refers to the administrative autonomy of local of Cainta or the City of Pasig has material bearing to the
government units or, cast in more technical language, the creation of the proposed Barangays Karangalan and Napico.
decentralization of government authority. Indeed, a requisite for the creation of a barangay is for its
territorial jurisdiction to be properly identified by metes and
On the other hand, the creation of autonomous regions bounds or by more or less permanent natural boundaries (Sec.
in Muslim Mindanao and the Cordilleras, which is peculiar to 386[b], R.A. No. 7160). Precisely because territorial jurisdiction
the 1987 Constitution, contemplates the grant of political is an issue raised in the pending civil case, until and unless such
autonomy and not just administrative autonomy to these regions. issue is resolved with finality, to define the territorial
Thus, the provision in the Constitution for an autonomous jurisdiction of the proposed barangays would only be an
regional government with a basic structure consisting of an exercise in futility. Not only that, we would be paving the way
executive department and a legislative assembly and special for potentially ultra vires acts of such barangays. X x x
courts with personal, family and property law jurisdiction in
each of the autonomous regions. (Cordillera Broad Coalition v. Moreover, considering the expenses entailed in the
COA, 181 SCRA 495, Jan. 29, 1990, En Banc [Cortes]) holding of plebiscites, it is far more prudent to hold in abeyance
the conduct of the same, pending final determination of whether
381. What is the meaning of "devolution"? or not the entire area of the proposed barangays are truly within
the territorial jurisdiction of the City of Pasig.
Ans: The term “devolution” refers to the act by which
the National government confers power and authority upon the Neither do we agree that merely because a plebiscite
various local government units to perform specific functions and had already been held in the case of the proposed Barangay
responsibilities. (Sec. 17[e], 2nd par., Local Government Code) Napico, the petition of the Municipality of Cainta has already
been rendered moot and academic. The issue raised by the
382. The City of Butuan enacted an ordinance Municipality of Cainta in its petition before the COMELEC
prohibiting the Land Transportation Office (LTO) against the holding of the plebiscite for the creation of Barangay
to register motor vehicles, tricycles in particular, as Napico are still pending determination before the Antipolo
well as to issue licenses for the driving thereof, Regional Trial Court.
contending that these powers have been devolved to
local governments under the Local Government Xxx
150
thereof did not provide for a cadastral type of
Therefore, the plebiscite on the creation of Barangay description of its boundary but merely provided
Karangalan should be held in abeyance pending final resolution that the boundary of the new city of Makati shall
of the boundary dispute between the City of Pasig and the be the boundary of the present municipality of
Municipality of Cainta by the Regional Trial Court of Antipolo Makati. Petitioners contended in a petition
City. In the same vein, the plebiscite held on March 15, 1997 to brought the SC that R.A. 7854 was defective
ratify the creation of Barangay Napico, Pasig City, should be because it did not comply with the requirement in
annulled and set aside. (City of Pasig v. COMELEC, 314 the Local Government Code that “the territorial
SCRA 179, Sept. 10, 1999, En Banc [Ynares-Santiago]) jurisdiction of newly created or converted cities
should be described by metes and bounds, with
384. Are the Internal Revenue Allotments (IRAs) technical descriptions.” Note that at the time the
considered income and, therefore, to be included in law was enacted, there was a pending boundary
the computation of the average annual income of a dispute between Makati and one of its neighbors,
municipality for purposes of its conversion into an Taguig, before the regular court. Should the
independent component city? contention be upheld?

Held: Yes. The IRAs are items of income because Held: Given the facts of the cases at bench, we cannot
they form part of the gross accretion of the funds of the local perceive how this evil (uncertainty in the boundaries of local
government unit. The IRAs regularly and automatically accrue government units will sow costly conflicts in the exercise of
to the local treasury without need of any further action on the government powers which ultimately will prejudice the people’s
part of the local government unit. They thus constitute income welfare) can be brought about by the description made in
which the local government can invariably rely upon as the Section 2 of R.A. No. 7854. Petitioners have not demonstrated
source of much needed funds. that the delineation of the land area of the proposed City of
Makati will cause confusion as to its boundaries. We note that
Xxx said delineation did not change even by an inch the land area
previously covered by Makati as a municipality. Section 2 did
[T]o reiterate, IRAs are a regular, recurring item of not add, subtract, divide, or multiply the established land area of
income; nil is there a basis, too, to classify the same as a special Makati. In language that cannot be any clearer, Section 2 stated
fund or transfer, since IRAs have a technical definition and that the city’s land area “shall comprise the present territory of
meaning all its own as used in the Local Government Code that the municipality.”
unequivocally makes it distinct from special funds or transfers
referred to when the Code speaks of “funding support from the The deliberations of Congress will reveal that there is a
national government, its instrumentalities and government- legitimate reason why the land area of the proposed City of
owned or controlled corporations.” Makati was not defined by metes and bounds, with technical
descriptions. At the time of the consideration of R.A. No. 7854,
Thus, Department of Finance Order No. 35-93 the territorial dispute between the municipalities of Makati and
correctly encapsulizes the full import of the above disquisition Taguig over Fort Bonifacio was under court litigation. Out of a
when it defined ANNUAL INCOME to be “revenues and becoming sense of respect to a co-equal department of
receipts realized by provinces, cities and municipalities from government, the legislators felt that the dispute should be left to
regular sources of the Local General Fund including the internal the courts to decide. They did not want to foreclose the dispute
revenue allotment and other shares provided for in Sections by making a legislative finding of fact which could decide the
284, 290 and 291 of the Code, but exclusive of non-recurring issue. This would have ensued if they defined the land area of
receipts, such as other national aids, grants, financial assistance, the proposed city by its exact metes and bounds, with technical
loan proceeds, sales of fixed assets, and similar others”. Such descriptions. We take judicial notice of the fact that Congress
order, constituting executive or contemporaneous construction has also refrained from using the metes and bounds description
of a statute by an administrative agency charged with the task of of the land area of other local government units with unsettled
interpreting and applying the same, is entitled to full respect and boundary disputes.
should be accorded great weight by the courts, unless such
construction is clearly shown to be in sharp conflict with the We hold that the existence of a boundary dispute does
Constitution, the governing statute, or other laws. (Alvarez v. not per se present an insurmountable difficulty which will
Guingona, Jr., 252 SCRA 695, Jan. 31, 1996, En Banc prevent Congress from defining with reasonable certitude the
[Hermosisima, Jr., J.]) territorial jurisdiction of a local government unit. In the cases at
bench, Congress maintained the existing boundaries of the
385. State the importance of drawing with precise proposed City of Makati but as an act of fairness, made them
strokes the territorial boundaries of a local subject to the ultimate resolution by the courts. Considering
government unit. these peculiar circumstances, we are not prepared to hold that
Section 2 of R.A. No. 7854 is unconstitutional. We sustain the
Held: The importance of drawing with precise strokes submission of the Solicitor General in this regard, viz:
the territorial boundaries of a local unit of government cannot be
overemphasized. The boundaries must be clear for they define “Going now to Sections 7 and 450 of the
the limits of the territorial jurisdiction of a local government Local Government Code, it is beyond cavil that the
unit. It can legitimately exercise powers of government only requirement stated therein, viz: ‘the territorial
within the limits of its territorial jurisdiction. Beyond these jurisdiction of newly created or converted cities should
limits, its acts are ultra vires. Needless to state, any uncertainty be described by metes and bounds, with technical
in the boundaries of local government units will sow costly descriptions” – was made in order to provide a means
conflicts in the exercise of governmental powers which by which the area of said cities may be reasonably
ultimately will prejudice the people’s welfare. This is the evil ascertained. In other words, the requirement on metes
sought to be avoided by the Local Government Code in and bounds was meant merely as a tool in the
requiring that the land area of a local government unit must be establishment of local government units. It is not an
spelled out in metes and bounds, with technical descriptions. end in itself. Ergo, so long as the territorial jurisdiction
(Mariano, Jr. v. COMELEC, 242 SCRA 211, 217-219, Mar. 7, of a city may be reasonably ascertained, i.e., by
1995, En Banc [Puno]) referring to common boundaries with neighboring
municipalities, as in this case, then, it may be
386. R.A. 7854 was enacted converting the Municipality concluded that the legislative intent behind the law has
of Makati into a highly urbanized city. Section 2 been sufficiently served.
151
Certainly, Congress did not intend that laws 389. Acebedo Optical Company, Inc. applied for a
creating new cities must contain therein detailed permit to engage in the business of running an
technical descriptions similar to those appearing in optical shop. Its application was granted with
Torrens titles, as petitioners seem to imply. To require several conditions. The conditions, in essence,
such description in the law as a condition sine qua non prohibit it from engaging in the practice of
for its validity would be to defeat the very purpose optometry as a corporate body or entity. Later, the
which the Local Government Code seeks to serve. The grant was revoked by the Mayor on the alleged
manifest intent of the Code is to empower local ground that it violated all the conditions of its
government units and to give them their rightful due. It business permit. Was the revocation valid?
seeks to make local governments more responsive to
the needs of their constituents while at the same time Held: In the case at bar, what is sought by petitioner
serving as a vital cog in national development. To (Acebedo Optical Company, Inc.) from respondent City Mayor
invalidate R.A. No. 7854 on the mere ground that no is a permit to engage in the business of running an optical shop.
cadastral type of description was used in the law would It does not purport to seek a license to engage in the practice of
serve the letter but defeat the spirit of the Code. It then optometry as a corporate body or entity, although it does have in
becomes a case of the master serving the slave, instead its employ, persons who are duly licensed to practice optometry
of the other way around. This could not be the by the Board of Examiners in Optometry.
intendment of the law.” X x x
(Mariano, Jr. v. COMELEC, 242 SCRA 211, 217-219, Mar. 7, The case of Samahan ng Optometrists sa Pilipinas v.
1995, En Banc [Puno]) Acebedo International Corporation, G.R. No. 117097,
promulgated by this Court on March 21, 1997, is in point. X x x
387. Discuss the authority of mayors to issue or grant
licenses and business permits, and how should it be The First Division of this Court x x x ruled in favor of
exercised. respondent Acebedo International Corporation, holding that “the
fact that private respondent hires optometrists who practice their
Held: The authority of city mayors to issue or grant profession in the course of their employment in private
licenses and business permits is beyond cavil. It is provided for respondent’s optical shops, does not translate into a practice of
by law. optometry by private respondent itself.” The Court further
elucidated that in both the old and new Optometry Law, R.A.
Xxx No. 1998, it is significant to note that there is no prohibition
against the hiring by corporations of optometrists. The Court
However, the power to grant or issue licenses or concluded thus:
business permits must always be exercised in accordance with
law, with utmost observance of the rights of all concerned to “All told, there is no law that prohibits the
due process and equal protection of the law. hiring by corporations of optometrists or considers the
hiring by corporations of optometrists as a practice by
Succinct and in point is the ruling of this Court, that: the corporation itself of the profession of optometry.”

“x x x While a business may be regulated, In the present case, the objective of the imposition of
such regulation must, however, be within the bounds of subject conditions on petitioner’s business permit could be
reason, i.e., the regulatory ordinance must be attained by requiring the optometrists in petitioner’s employ to
reasonable, and its provision cannot be oppressive produce a valid certificate of registration as optometrists, from
amounting to an arbitrary interference with the the Board of Examiners in Optometry. A business permit is
business or calling subject of regulation. A lawful issued primarily to regulate the conduct of business and the City
business or calling may not, under the guise of Mayor cannot, through the issuance of such permit, regulate the
regulation, be unreasonably interfered with even by the practice of a profession, like that of optometry. Such a function
exercise of police power. X x x is within the exclusive domain of the administrative agency
specifically empowered by law to supervise the profession, in
X x x The exercise of police power by the this case the Professional Regulations Commission and the
local government is valid unless it contravenes the Board of Examiners in Optometry.
fundamental law of the land or an act of the legislature,
or unless it is against public policy or is unreasonable, It is significant to note that during the deliberations of
oppressive, partial, discriminating or in derogation of a the bicameral conference committee of the Senate and the
common right.” (Balacuit v. CFI of Agusan del Norte, House of Representatives on R.A. 8050 x x x the committee
163 SCRA 182) (Acebedo Optical Company, Inc. v. failed to reach a consensus as to the prohibition on indirect
CA, 329 SCRA 314, 326-327, March 31, 2000, En practice of optometry by corporations. (Acebedo Optical
Banc [Purisima]) Company, Inc. v. CA, 329 SCRA 314, 328-330, March 31,
2000, En Banc [Purisima])
388. Distinguish the power to grant a license or permit
to do business and the power to issue a license to 390. May a local government unit validly authorize an
engage in the practice of a particular profession. expropriation of private property through a mere
resolution of its lawmaking body?
Held: Distinction must be made between the grant of a
license or permit to do business and the issuance of a license to Held: The Local Government Code expressly and
engage in the practice of a particular profession. The first is clearly requires an ordinance or a local law for that purpose. A
usually granted by the local authorities and the second is issued resolution that merely expresses the sentiment or opinion of the
by the Board or Commission tasked to regulate the particular Municipal Council will not suffice. The case of Province of
profession. A business permit authorizes the person, natural or Camarines Sur v. Court of Appeals which held that a mere
otherwise, to engage in business or some form of commercial resolution may suffice to support the exercise of eminent
activity. A professional license, on the other hand, is the grant domain by a local government unit is not in point because the
of authority to a natural person to engage in the practice or applicable law at that time was B.P. 337, the previous Local
exercise of his or her profession. (Acebedo Optical Company, Government Code, which had provided that a mere resolution
Inc. v. CA, 329 SCRA 314, 328, March 31, 2000, En Banc would enable an LGU to exercise eminent domain. In contrast,
[Purisima]) R.A. 7160, the present Local Government Code, explicitly
152
required an ordinance for this purpose. (Municipality of However, the very same Presidential Decree No. 1445,
Paranaque v. V.M. Realty Corp., 292 SCRA 678, July 20, 1998 which is the cornerstone of petitioner’s arguments, does not
[Panganiban]) provide that the absence of an appropriation law ipso facto
makes a contract entered into by a local government unit null
391. What are the requisites before a Local Government and void. Section 84 of the statute specifically provides:
Unit can validly exercise the power of eminent
domain? Revenue funds shall not be paid out of any
public treasury or depository except in pursuance of an
Held: In Municipality of Paranaque v. V.M. Realty appropriation law or other specific statutory authority.
Corp. (292 SCRA 678, July 20, 1998 [Panganiban]), it was
clarified that the requisites before a local government unit can Consequently, public funds may be disbursed not only
validly exercise the power of eminent domain are: pursuant to an appropriation law, but also in pursuance of other
specific statutory authority, i.e., Section 84 of PD 1445. Thus,
An ordinance is enacted by the local legislative council when a contract is entered into by a city mayor pursuant to
authorizing the local chief executive, in behalf of specific statutory authority, the law, i.e., PD 1445 allows the
the LGU, to exercise the power of eminent domain disbursement of funds from any public treasury or depository
or pursue expropriation proceedings over a therefor. It can thus be plainly seen that the law invoked by
particular private property; petitioner Quezon City itself provides that an appropriation law
The power of eminent domain is exercised for public is not the only authority upon which public funds shall be
use, purpose or welfare, or for the benefit of the disbursed.
poor and the landless;
There is payment of just compensation, as required Furthermore, then Mayor Brigido Simon, Jr. did not
under Section 9, Article III of the Constitution, and enter into the subject contract without legal authority. The
other pertinent laws; Local Government Code of 1983, or B.P. Blg. 337, which was
A valid and definite offer has been previously made to then in force, specifically and exclusively empowered the city
the owner of the property sought to be mayor to “represent the city in its business transactions, and sign
expropriated, but said offer was not accepted. all warrants drawn on the city treasury and all bonds, contracts
and obligations of the city.” Such power granted to the city
392. May the Sangguniang Panlalawigan validly mayor by B.P. Blg. 337 was not qualified nor restricted by any
disapprove a resolution or ordinance of a prior action or authority of the city council. We note that while
municipality calling for the expropriation of the subsequent Local Government Code of 1991, which took
private property to be made site of a Farmers effect after the execution of the subject contracts, provides that
Center and Other Government Sports Facilities on the mayor’s representation must be “upon authority of the
the ground that said “expropriation is unnecessary sangguniang panlungsod or pursuant to law or ordinance,” there
considering that there are still available lots of the was no such qualification under the old code. (Citations
municipality for the establishment of a government omitted)
center”?
We must differentiate the provisions of the old Local
Held: Under the Local Government Code, the Government Code of 1983, B.P. Blg. 337, which was then in
Sangguniang Panlalawigan is granted the power to declare a force, from that of the Local Government Code of 1991, R.A.
municipal resolution invalid on the sole ground that it is beyond No. 7160, which now requires that the mayor’s representation of
the power of the Sangguniang Bayan or Mayor to issue. As held the city in its business transactions must be “upon authority of
in Velazco v. Blas, “The only ground upon which a provincial the sangguniang panlungsod or pursuant to law or ordinance”
board may declare any municipal resolution, ordinance or order (Section 455 [vi]. No such prior authority was required under
invalid is when such resolution, ordinance, or order is ‘beyond B.P. Blg. 337. This restriction, therefore, cannot be imposed on
the powers conferred upon the council or president making the the city mayor then since the two contracts were entered into
same.’ A strictly legal question is before the provincial board in before R.A. No. 7160 was even enacted.
its consideration of a municipal resolution, ordinance, or order.
The provincial board’s disapproval of any resolution, ordinance, Under B.P. Blg. 337, while the city mayor has no
or order must be premised specifically upon the fact that such power to appropriate funds to support the contracts, neither does
resolution, ordinance, or order is outside the scope of the legal said law prohibit him from entering into contracts unless and
powers conferred by law. If a provincial board passes these until funds are appropriated therefor. In fact, it is his bounden
limits, it usurps the legislative functions of the municipal duty to so represent the city in all its business transactions. On
council or president. Such has been the consistent course of the other hand, the city council must provide for the “depositing,
executive authority.” (Moday v. CA, 268 SCRA 586, Feb. 20, leaving or throwing of garbage” and to appropriate funds for
1997) such expenses. (Section 177 [b]). It cannot refuse to so provide
and appropriate public funds for such services which are very
393. Is a contract entered into by the city mayor vital to the maintenance of cleanliness of the city and the good
involving the expenditure of public funds by the health of its inhabitants.
local government without prior appropriation by
the city council valid and binding? By entering into the two contracts, Mayor Simon did
not usurp the city council’s power to provide for the proper
Held: If we are to limit our disquisition to the cited disposal of garbage and to appropriate funds therefor. The
provisions of Presidential Decree No. 1445, or the Auditing execution of contracts to address such a need is his statutory
Code of the Philippines, in conjunction with Section 177 (b) of duty, just as it is the city council’s duty to provide for said
Batas Pambansa Blg. 337, or the Local Government Code of services. There is no provision in B.P. Blg. 337, however, that
1983, which empowered the Sangguniang Panlungsod to prohibits the city mayor from entering into contracts for the
“appropriate funds for expenses of the city government, and fix public welfare, unless and until there is prior authority from the
the salaries of its officers and employees according to law,” city council. This requirement was imposed much later by R.A.
there would be no debate that prior appropriation by the city No. 7160, long after the contracts had already been executed
council and a certification that funds are available therefore is and implemented.
indeed mandatorily required.
Even the very Charter of Quezon City, more
Xxx particularly Section 9(f), Section 12(a)and Section 12(m)
thereof, simply provide that the mayor shall exercise general
153
powers and duties, such as signing “all warrants drawn on the expenses of hiring a private lawyer” and that “the interests of
city treasurer and all bonds, contracts, and obligations of the the municipality would be best protected if a government lawyer
city,” even as it grants the City Council the power, by ordinance handles its litigations.”
or resolution, “to make all appropriations for the expenses of
the government of the city,” as well as “to prohibit the throwing Private lawyers may not represent municipalities on
or depositing of offal, garbage, refuse, or other offensive matter their own. Neither may they do so even in collaboration with
in the same, and to provide for its collection and disposition x x authorized government lawyers. This is anchored on the
x.” (Citations omitted) principle that only accountable public officers may act for and
in behalf of public entities and that public funds should not be
While the powers and duties of the Mayor and the City expended to hire private lawyers. (Ramos v. CA, 269 SCRA 34,
Council are clearly delineated, there is nothing in the cited March 3, 1997)
provisions, nor even in the statute itself, that requires “prior
authorization by the city council by proper enactment of an 398. May a municipality adopt the work already
ordinance” before the City Mayor can enter into contracts. performed in good faith by a private lawyer, which
work proved beneficial to it?
Private respondent Lexber asserts that the subject
contract was entered into by Mayor Simon in behalf of the Held: Although a municipality may not hire a private
Quezon City government pursuant to specific statutory lawyer to represent it in litigations, in the interest of substantial
authority, more particularly the provisions of Executive Order justice, however, it was held that a municipality may adopt the
No. 392 (Constituting the Metro Manila Authority [MMA]). work already performed in good faith by such private lawyer,
City of Quezon v. Lexber Incorporated, 354 SCRA 493, Mar. which work is beneficial to it (1) provided that no injustice is
15, 2001, 1st Div. [Ynares-Santiago]) thereby heaped on the adverse party and (2) provided further
that no compensation in any guise is paid therefor by said
394. Who has the legal authority to represent a municipality to the private lawyer. Unless so expressly adopted,
municipality in lawsuits? the private lawyer’s work cannot bind the municipality. (Ramos
v. CA, 269 SCRA 34, March 3, 1997)
Held: Only the provincial fiscal, provincial attorney,
and municipal attorney should represent a municipality in its 399. Does the President’s power of general supervision
lawsuits. Only in exceptional instances may a private attorney extend to the liga ng mga barangay, which is not a
be hired by a municipality to represent it in lawsuits. (Ramos v. local government unit?
CA, 269 SCRA 34, March 3, 1997)
Held: We rule in the affirmative. In Opinion No. 41,
395. What are the exceptional instances when a private Series of 1995, the Department of Justice ruled that the liga ng
attorney may be validly hired by a municipality in mga barangay is a government organization, being an
its lawsuits? association, federation, league or union created by law or by
authority of law, whose members are either appointed or elected
Held: In Alinsug v. RTC Br. 58, San Carlos City, government officials. The Local Government Code defines the
Negros Occidental, it was held that “the law allows a private liga ng mga barangay as an organization of all barangays for the
counsel to be hired by a municipality only when the primary purpose of determining the representation of the liga in
municipality is an adverse party in a case involving the the sanggunians, and for ventilating, articulating and
provincial government or another municipality or city within the crystallizing issues affecting barangay government
province. This provision has its apparent origin in De Guia v. administration and securing, through proper and legal means,
The Auditor General where the Court held that the solutions thereto (Sec. 491, Local Government Code). X x x
municipality’s authority to employ a private attorney is
expressly limited only to situations where the provincial fiscal Xxx
would be disqualified to serve and represent it.” (Ramos v. CA,
269 SCRA 34, March 3, 1997) The ligas are primarily governed by the provisions of
the Local Government Code (Book III, Title VI, Local
396. Cite instances when the provincial fiscal may be Government Code). However, their respective constitution and
disqualified to represent in court a particular by-laws shall govern other matters affecting internal
municipality. organization of the liga not otherwise provided for in the Local
Government Code provided that the constitution and by-laws
Held: As held in Enriquez, Sr. v. Gimenez, the shall be suppletory to the provisions of Book III, Title VI of the
provincial fiscal may be disqualified to represent in court a Local Government Code and shall always conform to the
particular municipality in the following instances: provisions of the Constitution and existing laws (Sec. 507, Local
Government Code).
If and when original jurisdiction of case involving the
municipality is vested in the Supreme Court; Having in mind the foregoing principles, we rule that
When the municipality is a party adverse to the Memorandum Circular No. 97-193 of the DILG insofar as it
provincial government or to some other authorizes the filing a Petition for Review of the BES with the
municipality in the same province; and regular courts in a post proclamation electoral protest is of
When, in a case involving the municipality, he, or his doubtful constitutionality. We agree with both the petitioner
wife, or child, is pecuniarily involved, as heir, and the Solicitor General that in authorizing the filing of the
legatee, creditor or otherwise. petition for review of the decision of the BES with the regular
(Ramos v. CA, 269 SCRA 34, March 3, 1997) courts, the DILG Secretary in effect amended and modified the
GUIDELINES promulgated by the National Liga Board and
397. May a municipality be represented by a private law adopted by the LIGA which provides that the decision of the
firm which had volunteered its services gratis, in BES shall be subject to review by the National Liga Board. The
collaboration with the municipal attorney and the amendment of the GUIDELINES is more than an exercise of the
fiscal? power of supervision but is an exercise of the power of control,
which the President does not have over the LIGA. Although the
Held: No. Such representation will be violative of DILG is given the power to prescribe rules, regulations and
Section 1983 of the old Administrative Code. This strict other issuances, the Administrative Code limits its authority to
coherence to the letter of the law appears to have been dictated merely “monitoring compliance” by local government units of
by the fact that “the municipality should not be burdened with such issuances. To monitor means “to watch, observe or check”
154
and is compatible with the power of supervision of the DILG Held: Under existing law, local government units, in
Secretary over local governments, which is limited to checking addition to having administrative autonomy in the exercise of
whether the local government unit concerned or the officers their functions, enjoy fiscal autonomy as well. Fiscal autonomy
thereof perform their duties as per statutory enactments. means that local governments have the power to create their
Besides, any doubt as to the power of the DILG Secretary to own sources of revenue in addition to their equitable share in the
interfere with local affairs should be resolved in favor of the national taxes released by the national government, as well as
greater autonomy of the local government. the power to allocate their resources in accordance with their
own priorities. It extends to the preparation of their budgets,
The public respondent judge therefore committed grave and local officials in turn have to work within the constraints
abuse of discretion amounting to lack or excess of jurisdiction in thereof. They are not formulated at the national level and
not dismissing the respondent’s Petition for Review for failure imposed on local governments, whether they are relevant to
to exhaust all administrative remedies and for lack of local needs and resources or not. Hence, the necessity of a
jurisdiction. (Bito-Onon v. Fernandez, 350 SCRA 732, Jan. 31, balancing of viewpoints and the harmonization of proposals
2001, 3rd Div. [Gonzaga-Reyes]) from both local and national officials, who in any case are
partners in the attainment of national goals.
400. May the President validly withhold a portion of the
internal revenue allotments of Local Government Local fiscal autonomy does not, however, rule out any
Units legally due them by administrative fiat? manner of national government intervention by way of
supervision, in order to ensure that local programs, fiscal and
Held: The Constitution vests the President with the otherwise, are consistent with national goals. Significantly, the
power of supervision, not control, over local government units President, by constitutional fiat, is the head of the economic and
(LGUs). Such power enables him to see to it that LGUs and planning agency of the government (Section 9, Article XII of the
their officials execute their tasks in accordance with law. While Constitution), primarily responsible for formulating and
he may issue advisories and seek their cooperation in solving implementing continuing, coordinated and integrated social and
economic difficulties, he cannot prevent them from performing economic policies, plans and programs (Section 3, Chapter 1,
their tasks and using available resources to achieve their goals. Subtitle C, Title II, Book V, EO 292 [Administrative Code of
He may not withhold or alter any authority or power given them 1987]) for the entire country. However, under the Constitution,
by the law. Thus, the withholding of a portion of internal the formulation and the implementation of such policies and
revenue allotments legally due them cannot be directed by programs are subject to “consultations with the appropriate
administrative fiat. public agencies, various private sectors, and local government
units.” The President cannot do so unilaterally. (Pimentel, Jr. v.
Xxx Aguirre, 336 SCRA 201, July 19, 2000, En Banc
[Panganiban])
Section 4 of AO 372 cannot x x x be upheld. A basic
feature of local fiscal autonomy is the automatic release of the 402. What are the requisites before the President may
shares of LGUs in the National internal revenue. This is interfere in local fiscal matters?
mandated by no less than the Constitution. The Local
Government Code (Sec. 286[a]) specifies further that the release Held: x x x [T]he Local Government Code provides
shall be made directly to the LGU concerned within five (5) (Sec. 284. See also Art. 379 of the Rules and Regulations
days after every quarter of the year and “shall not be subject to Implementing the Local Government Code of 1991):
any lien or holdback that may be imposed by the national
government for whatever purpose.” As a rule, the term “shall” “x x x [I]n the event the national government
is a word of command that must be given a compulsory incurs an unmanaged public sector deficit, the
meaning.” The provision is, therefore, imperative. President of the Philippines is hereby authorized, upon
the recommendation of [the] Secretary of Finance,
Section 4 of AO 372, however, orders the withholding, Secretary of the Interior and Local Government and
effective January 1, 1998, of 10 percent of the LGUs’ IRA Secretary of Budget and Management, and subject to
“pending the assessment and evaluation by the Development consultation with the presiding officers of both Houses
Budget Coordinating Committee of the emerging fiscal of Congress and the presidents of the liga, to make the
situation” in the country. Such withholding clearly contravenes necessary adjustments in the internal revenue allotment
the Constitution and the law. Although, temporary, it is of local government units but in no case shall the
equivalent to a holdback, which means “something held back or allotment be less than thirty percent (30%) of the
withheld. Often temporarily.” Hence, the “temporary” nature of collection of national internal revenue taxes of the third
the retention by the national government does not matter. Any fiscal year preceding the current fiscal year x x x”
retention is prohibited.
There are therefore several requisites before the
In sum, while Section 1 of AO 372 may be upheld as President may interfere in local fiscal matters: (1) an unmanaged
an advisory effected in times of national crisis, Section 4 thereof public sector deficit of the national government; (2)
has no color of validity at all. The latter provision effectively consultations with the presiding officers of the Senate and the
encroaches on the fiscal autonomy of local governments. House of Representatives and the presidents of the various local
Concededly, the President was well-intentioned in issuing his leagues; and (3) the corresponding recommendation of the
Order to withhold the LGUs’ IRA, but the rule of law requires secretaries of the Department of Finance, Interior and Local
that even the best intentions must be carried out within the Government, and Budget and Management. Furthermore, any
parameters of the Constitution and the law. Verily, laudable adjustment in the allotment shall in no case be less than thirty
purposes must be carried out by legal methods. (Pimentel, Jr. v. percent (30%) of the collection of national internal revenue
Aguirre, G.R. No. 132988, 336 SCRA 201, July 19, 2000, En taxes of the third fiscal year preceding the current one.
Banc [Panganiban]) (Pimentel, Jr. v. Aguirre, 336 SCRA 201, July 19, 2000, En
Banc [Panganiban])
401. What is meant by fiscal autonomy of Local
Governments? Does it rule out in any manner 403. On May 3, 2001, petitioner filed with the Provincial
national government intervention by way of Election Supervisor in Pagadian City a petition for
supervision in order to ensure that local programs the disqualification of respondent Sulong,
are consistent with national goals? pursuant to Sec. 40[b] of Republic Act No. 7160
(Local Government Code), which disqualifies from
running for any elective local position “those
155
removed from office as a result of an pursuant to Sec. 68 of the Local Government Code, which
administrative case.” It appears that respondent makes decisions in administrative cases immediately executory.
Sulong had previously won as mayor of Lapuyan
on January 18, 1988. In the May 11, 1992, and Indeed, considering the failure of the Sangguniang
again in the May 8, 1995 elections, he was Panlalawigan to resolve respondent’s motion, it is unfair to the
reelected. In a petition for disqualification, electorate to be told after they have voted for respondent Sulong
petitioner alleged that in 1991, during his first term that after all he is disqualified, especially since at the time of the
as mayor of Lapuyan, respondent Sulong, along elections on May 14, 2001, the decision of the Sangguniang
with a municipal councilor of Lapuyan and several Panlalawigan had been rendered nearly ten years ago. (Atty.
other individuals, was administratively charged Miguel M. Lingating v. Commission on Elections and Cesar
(AC No. 12-91) with various offenses, and that, on B. Sulong, G.R. No. 153475, Nov. 13, 2002, En Banc
February 4, 1992, the Sangguniang Panlalawigan [Mendoza])
of Zamboanga del Sur found him guilty of the
charges and ordered his removal from office. 404. Under Section 8, Article X of the Constitution,
Petitioner claimed that this decision had become "[T]he term of office of elective local officials x x x
final and executory, and consequently the then shall be three years and no such official shall serve
vice-mayor of Lapuyan, Vicente Imbing, took his for more than three consecutive terms." How is
oath as mayor vice respondent Sulong on March 3, this term limit for elective local officials to be
1992. interpreted?

Respondent Sulong denied that the Held: The term limit for elective local officials must
decision in AC No. 12-91 had become final and be taken to refer to the right to be elected as well as the right to
executory. He averred that after receiving a copy serve in the same elective position. Consequently, it is not
of the decision on February 17, 1992, he filed a enough that an individual has served three consecutive terms in
motion for reconsideration and/or notice of appeal an elective local office, he must also have been elected to the
thereof on February 18, 1992; that on February same position for the same number of times before the
27, 1992, the Sangguniang Panlalawigan required disqualification can apply. (Borja, Jr. v. COMELEC and
Jim Lingating, the complainant in AC No. 12-91, Capco, Jr., G.R. No. 133495, Sept. 3, 1998, 295 SCRA 157, En
to comment on respondent Sulong’s motion for Banc [Mendoza])
reconsideration and/or notice of appeal; that the
said complainant had not yet complied therewith 405. Case No. 1. Suppose A is a vice-mayor who
and his (respondent Sulong’s) motion had becomes mayor by reason of the death of the
consequently remained pending. Respondent incumbent. Six months before the next election, he
Sulong denied he had been removed from office by resigns and is twice elected thereafter. Can he run
virtue of the decision in AC No. 12-91. again for mayor in the next election?

Held: Petitioner contends that the COMELEC en banc Ans.: Yes, because although he has already first
erred in applying the ruling in Aguinaldo v. Commission on served as mayor by succession and subsequently resigned from
Elections in holding that the reelection of respondent Sulong in office before the full term expired, he has not actually served
1992 and 1995 as mayor of Lapuyan had the effect of condoning three full terms in all for the purpose of applying the term limit.
the misconduct for which he was ordered dismissed by the Under Art. X, Sec. 8, voluntary renunciation of the office is not
Sangguniang Panlalawigan of Zamboanga del Sur. Petitioner considered as an interruption in the continuity of his service for
cites Reyes v. Commission on Elections in which we held that an the full term only if the term is one “for which he was elected.”
elective local executive officer, who is removed before the Since A is only completing the service of the term for which the
expiration of the term for which he was elected, is disqualified deceased and not he was elected, A cannot be considered to
from being a candidate for a local elective position under have completed one term. His resignation constitutes an
Section 40[b] of the Local Government Code. interruption of the full term.

Xxx 406. Case No. 2. Suppose B is elected Mayor and,


during his first term, he is twice suspended for
However, Reyes cannot be applied to this case because misconduct for a total of 1 year. If he is twice
it appears that the 1992 decision of the Sangguniang reelected after that, can he run for one more term
Panlalawigan, finding respondent Sulong guilty of dishonesty, in the next election?
falsification and malversation of public funds, has not until now
become final. x x x The filing of his motion for reconsideration Ans.: Yes, because he has served only two full terms
prevented the decision of Sangguniang Panlalawigan from successively.
becoming final.
In both cases, the mayor is entitled to run for reelection
While R.A. No. 7160 on disciplinary actions is silent because the two conditions for the application of the
on the filing of a motion for reconsideration, the same cannot be disqualification provisions have not concurred, namely, that the
interpreted as a prohibition against the filing of a motion for local official concerned has been elected three consecutive times
reconsideration. x x x. and that he has fully served three consecutive terms. In the first
case, even if the local official is considered to have served three
There is thus no decision finding respondent guilty to full terms notwithstanding his resignation before the end of the
speak of. As Provincial Secretary of Zamboanga del Sur first term, the fact remains that he has not been elected three
Wilfredo Cimafranca attested, the Sangguniang Panlalawigan times. In the second case, the local official has been elected
simply considered the matter as having become moot and three consecutive times, but he has not fully served three
academic because it was “overtaken by the local elections of consecutive terms.
May [11], 1992.”
407. Case No. 3. The case of vice-mayor C who
Neither can the succession of the then vice-mayor of becomes mayor by succession involves a total
Lapuyan x x x and the highest ranking municipal councilor of failure of the two conditions to concur for the
Lapuyan x x x to the offices of mayor and vice-mayor, purpose of applying Art. X, Sec. 8. Suppose he is
respectively, be considered proof that the decision in AC No. twice elected after that term, is he qualified to run
12-91 had become final because it appears to have been made again in the next election?
156
second sentence of the constitutional provision under scrutiny
Ans.: Yes, because he was not elected to the office of states, "Voluntary renunciation of office for any length of time
mayor in the first term but simply found himself thrust into it by shall not be considered as an interruption in the continuity of
operation of law. Neither had he served the full term because he service for the full term for which he was elected." The clear
only continued the service, interrupted by the death, of the intent of the framers of the Constitution to bar any attempt to
deceased mayor. (Borja, Jr. v. COMELEC and Capco, Jr., circumvent the three-term limit by a voluntary renunciation of
G.R. No. 133495, Sept. 3, 1998, 295 SCRA 157, En Banc office and at the same time respect the people's choice and grant
[Mendoza]) their elected official full service of a term is evident in this
provision. Voluntary renunciation of a term does not cancel the
408. What are the policies embodied in the renounced term in the computation of the three term limit;
constitutional provision barring elective local conversely, involuntary severance from office for any length of
officials, with the exception of barangay officials, time short of the full term provided by law amounts to an
from serving more than three consecutive terms? interruption of continuity of service. Lonzanida vacated his post
a few months before the next mayoral elections, not by
Held: To prevent the establishment of political voluntary renunciation but in compliance with the legal process
dynasties is not the only policy embodied in the constitutional of writ of execution issued by the COMELEC to that effect.
provision in question (barring elective local officials, with the Such involuntary severance from office is an interruption of
exception of barangay officials, from serving more than three continuity of service and thus, Lonzanida did not fully serve the
consecutive terms). The other policy is that of enhancing the 1995-1998 mayoral term.
freedom of choice of the people. To consider, therefore, only
stay in office regardless of how the official concerned came to In sum, Lonzanida was not the duly elected mayor and
that office – whether by election or by succession by operation that he did not hold office for the full term; hence, his
of law – would be to disregard one of the purposes of the assumption of office from May 1995 to March 1998 cannot be
constitutional provision in question. (Borja, Jr. v. COMELEC counted as a term for purposes of computing the three-term
and Capco, Jr., G.R. No. 133495, Sept. 3, 1998, 295 SCRA limit. (Lonzanida v. COMELEC, 311 SCRA 602, July 28,
157, En Banc [Mendoza]) 1999, En Banc [Gonzaga-Reyes])

409. Lonzanida was previously elected and served two 410. Mayor Edward S. Hagedorn of Puerto Princesa
consecutive terms as mayor of San Antonio, City was elected for three consecutive times in the
Zambales prior to the May 1995 mayoral elections. 1992, 1995 and 1998 elections and served in full
In the May 1995 elections he again ran for mayor his three consecutive terms as Mayor. In the 2001
of San Antonio, Zambales and was proclaimed elections, he ran for Governor of the Province of
winner. He assumed office and discharged the Palawan and lost. Socrates ran and won as Mayor
rights and duties of mayor until March 1998 when of Puerto Princesa in that election. On July 2,
he was ordered to vacate the post by reason of the 2002, the Preparatory Recall Assembly (PRA) of
COMELEC decision on the election protest against Puerto Princesa City adopted a Resolution calling
him which declared his opponent Juan Alvez the for the recall of incumbent Mayor Socrates. The
duly elected mayor. Alvez served the remaining COMELEC scheduled a Special Recall Election
portion of the 1995-1998 mayoral term. Is for Mayor of that City on September 24, 2002. Is
Lonzanida still qualified to run for mayor of San Mayor Hagedorn qualified to run again for Mayor
Antonio, Zambales in the May 1998 local in that Special Recall Election considering the
elections? circumstances?

Held: The two requisites for the application of the Held: The three-term limit rule for elective local
three-term rule was absent. First, Lonzanida cannot be officials is found in Section 8, Article X of the Constitution x x
considered as having been duly elected to the post in the May x.
1995 elections, and second, he did not fully serve the 1995-1998
mayoral term by reason of involuntary relinquishment of office. This three-term limit rule is reiterated in Section 43 (b)
After a re-appreciation and revision of the contested ballots the of RA No. 7160, otherwise known as the Local Government
COMELEC itself declared by final judgment that Lonzanida Code x x x.
lost in the May 1995 mayoral elections and his previous
proclamation as winner was declared null and void. His These constitutional and statutory provisions have two
assumption of office as mayor cannot be deemed to have been parts. The first part provides that an elective local official
by reason of a valid election but by reason of a void cannot serve for more than three consecutive terms. The clear
proclamation. It has been repeatedly held by the SC that a intent is that only consecutive terms count in determining the
proclamation subsequently declared void is no proclamation at three-term limit rule. The second part states that voluntary
all and while a proclaimed candidate may assume office on the renunciation of office for any length of time does not interrupt
strength of the proclamation of the Board of Canvassers he is the continuity of service. The clear intent is that involuntary
only a presumptive winner who assumes office subject to the severance from office for any length of time interrupts
final outcome of the election protest. Lonzanida did not serve a continuity of service and prevents the service before and after
term as mayor of San Antonio, Zambales from May 1995 to the interruption from being joined together to form a continuous
March 1998 because he was not duly elected to the post; he service or consecutive terms.
merely assumed office as presumptive winner, which
presumption was later overturned by the COMELEC when it After three consecutive terms, an elective local official
decided with finality that Lonzanida lost in the May 1995 cannot seek immediate reelection for a fourth term. The
mayoral elections. prohibited election refers to the next regular election for the
same office following the end of the third consecutive term.
Second, Lonzanida cannot be deemed to have served Any subsequent election, like a recall election, is no longer
the May 1995 to 1998 term because he was ordered to vacate his covered by the prohibition for two reasons. First, a subsequent
post before the expiration of the term. His opponents' election like a recall election is no longer an immediate
contention that Lonzanida should be deemed to have served one reelection after three consecutive terms. Second, the
full term from May 1995-1998 because he served the greater intervening period constitutes an involuntary interruption in the
portion of that term has no legal basis to support it; it disregards continuity of service.
the second requisite for the application of the disqualification,
i.e., that he has fully served three consecutive terms. The Xxx
157
in determining the three-consecutive term limit
Clearly, what the Constitution prohibits is an fixed by law?
immediate reelection for a fourth term following three
consecutive terms. The Constitution, however, does not prohibit Held: After due deliberation, the Court voted 8 to 7 to
a subsequent reelection for a fourth term as long as the DISMISS the petition.
reelection is not immediately after the end of the third
consecutive term. A recall election mid-way in the term VITUG, J., joined by YNARES-SANTIAGO, J., voted
following the third consecutive term is a subsequent election but to dismiss the petition. He contended that as revealed by the
not an immediate reelection after the third term. records of the Constitutional Commission, the Constitution
envisions a continuous and an uninterrupted service for three
Neither does the Constitution prohibit one barred from full terms before the proscription applies. Therefore, not being
seeking immediate reelection to run in any other subsequent a full term, a recall term should not be counted or used as a basis
election involving the same term of office. What the for the disqualification whether served prior (as in this case) or
Constitution prohibits is a consecutive fourth term. The debates subsequent (as in the Socrates case) to the nine-year, full three-
in the Constitutional Commission evidently show that the term limit.
prohibited election referred to by the framers of the Constitution
is the immediate reelection after the third term, not any other MENDOZA, J., in whose opinion QUISUMBING, J.,
subsequent election. joined, voted to dismiss the petition on the ground that, in
accordance with the ruling in Borja, Jr. v. COMELEC; Arcos v.
Xxx COMELEC; Lonzanida v. COMELEC; and Adormeo v.
COMELEC, a term during which succession to a local elective
In the case of Hagedorn, his candidacy in the recall office takes place or a recall election is held should not be
election on September 24, 2002 is not an immediate reelection counted in determining whether an elective local official has
after his third consecutive term which ended on June 30, 2001. served more than three consecutive terms. He argued that the
The immediate reelection that the Constitution barred Hagedorn Constitution does not prohibit elective local officials from
from seeking referred to the regular elections in 2001. serving for more than three consecutive terms because, in fact, it
Hagedorn did not seek reelection in the 2001 elections. excludes from the three-term limit interruptions in the continuity
of service, so long as such interruptions are not due to the
Xxx voluntary renunciation of the office by the incumbent. Hence,
the period from June 28, 1994 to June 30, 1995, during which
From June 30, 2001 until the recall election on respondent Leonardo B. Roman served as governor of Bataan
September 24, 2002, the mayor of Puerto Princesa was Socrates. by virtue of a recall election held in 1993, should not be
This period is clearly an interruption in the continuity of counted. Since on May 14, 2001 respondent had previously
Hagedorn’s service as mayor, not because of his voluntary served as governor of Bataan for only two consecutive terms
renunciation, but because of a legal prohibition. Hagedorn’s (1995-1998 and 1998-2001), his election on that day was
three consecutive terms ended on June 30, 2001. Hagedorn’s actually only his third term for the same position.
new recall term from September 24, 2002 to June 30, 2004 is
not a seamless continuation of his previous three consecutive PANGANIBAN, J., joined by PUNO, J., also voted to
terms as mayor. One cannot stitch together Hagedorn’s dismiss the petition. He argued that a recall term should not be
previous three-terms with his new recall term to make the recall considered as one full term, because a contrary interpretation
term a fourth consecutive term because factually it is not. An would in effect cut short the elected official’s service to less
involuntary interruption occurred from June 30, 2001 to than nine years and shortchange his constituents. The desire to
September 24, 2002 which broke the continuity or consecutive prevent monopoly of political power should be balanced against
character of Hagedorn’s service as mayor. the need to uphold the voters’ obvious preference who, in the
present case, is Roman who received 97 percent of the votes
X x x In Hagedorn’s case, the nearly 15-month period cast. He explained that, in Socrates, he also voted to affirm the
he was out of office, although short of a full term of three years, clear choice of the electorate, because in a democracy the people
constituted an interruption in the continuity of his service as should, as much as legally possible, be governed by leaders
mayor. The Constitution does not require the interruption or freely chosen by them in credible elections. He concluded that,
hiatus to be a full term of three years. The clear intent is that in election cases, when two conflicting legal positions are of
interruption “for any length of time,” as long as the cause is almost equal weight, the scales of justice should be tilted in
involuntary, is sufficient to break an elective local official’s favor of the people’s overwhelming choice.
continuity of service. (Victorino Dennis M. Socrates v. The
Commission on Elections, G.R. No. 154512, Nov. 12, 2002, En AZCUNA, J., joined by BELLOSILLO, J., also voted
Banc [Carpio]) to dismiss, arguing that it is clear from the constitutional
provision that the disqualification applies only if the terms are
411. Petitioners would seek the disqualification of consecutive and the service is full and continuous. Hence,
respondent Leonardo B. Roman on the ground of service for less than a term, except only in case of voluntary
his having transgressed the three-term limit under renunciation, should not count to disqualify an elective local
Section 8, Article X, of the 1987 Constitution and official from running for the same position. This case is
Section 43 of Republic Act No. 7160 (Local different from Socrates, where the full three consecutive terms
Government Code). The focal issue presented had been continuously served so that disqualification had clearly
before the Court x x x would revolve on the attached.
question of whether or not private respondent
Roman exceeded the three-term limit for elective On the other hand, SANDOVAL-GUTIERREZ, J.,
local officials, expressed in the Constitution and with whom DAVIDE, C.J., and AUSTRIA-MARTINEZ,
the Local Government Code, when he again ran CORONA, and CALLEJO, SR., JJ., concurred, holds the view
for the position of Governor in the 14th of May that the recall term served by respondent Roman, comprising the
2001 elections, having occupied and served in that period June 28, 1994 to June 30, 1995, should be considered as
position following the 1993 recall elections, as well one term. Since he thereafter served for two consecutive terms
as the 1995 and 1998 regular elections, from 1995 to 1998 and from 1998 to 2001, his election on May
immediately prior to the 2001 elections. In fine, 14, 2001 was actually his fourth term and contravenes Art. X,
should respondent’s incumbency to the post of Sec. 8 of the Constitution. For this reason, she voted to grant
Governor following the recall elections be included the petition and to declare respondent’s election on May 14,
2002 as null and void.
158
This same issue has been passed and ruled upon by the
CARPIO, J., joined by CARPI0-MORALES, J., also Commission on Elections no less than five times. Consistently,
dissented and voted to grant the petition. He held that a recall it has held that the term of a newcomer in recall elections cannot
term constitutes one term and that to totally ignore a recall term be counted as a full term and may not thus be included in
in determining the three-term limit would allow local officials to counting the three-term limit prescribed under the law. The
serve for more than nine consecutive years contrary to the Commission on Elections, with its fact-finding facilities, its
manifest intent of the framers of the Constitution. He contended familiarity with political realities, and its peculiar expertise in
that respondent Roman’s election in 2001 cannot exempt him dealing with election controversies, should be in a good vantage
from the three-term limit imposed by the Constitution. point to resolve issues of this nature. Concededly, no ready
made formulae are always extant to address occasional complex
In his Separate Opinion, Justice Vitug voted to issues, allowing time and experience to merely evolve and
dismiss the petition on the following considerations: ultimately provide acceptable solutions. In the administration of
election laws, it would be unsound by an excessive zeal to
In order that the three-consecutive term limit can apply, remove from the Commission on Elections the initiative it takes
two conditions must concur, i.e., (1) that the elective local on such questions which, in fact, by legal mandate properly
official concerned has been elected for three consecutive terms belong to it.
to the same local government position, and (2) that he has
served three consecutive full terms, albeit a voluntary Nor should it be ignored that the law here involved is a
renunciation of the office for any length of time shall not be limitation on the right of suffrage not only on the candidate for
deemed to be an interruption in the continuity of the service for office but also, and most importantly, on the electorate.
the full term for which he is elected. The constitutional Respondent Roman has won the election to the post of Governor
provision does not appear to be all that imprecise for and in its of Bataan with a comfortable margin against his closest
application. Section 8, Article X, of the Constitution is explicit opponent. Where a candidate appears to be the clear choice of
that the “term of office of elective local officials x x x shall be the people, doubts on the candidate’s eligibility, even only as a
three years” which phrase is forthwith followed by its mandate practical matter, must be so resolved as to respect and carry out,
that “no such official shall serve for more than three consecutive not defeat, the paramount will of the electorate. While the
terms,” and that “[v]oluntary renunciation of the office for any Constitution would attempt to prevent the monopolization of
length of time shall not be considered as an interruption in the political power, indeed a wise rule, the precept of preserving the
continuity of his service for the full term for which he [is] freedom of choice of the people on who shall rightfully hold the
elected.” The law evidently contemplates a continuous full reins of government for them is no less than fundamental in
three-year term before the proscription can apply. looking at its overriding intent. (Melanio L. Mendoza and
Mario E. Ibarra v. Commission on Elections and Leonardo B.
The Constitutional Commission, in its deliberations, Roman, G.R. No. 149736, Dec. 17, 2002, En Banc)
referred to a full nine (9) years of service for each elective local
government official in the application of the prohibition, 412. When may a permanent vacancy arise under
envisioning at the same time a continuous and uninterrupted Section 44 of the Local Government Code?
period of nine years by providing for only one exception, i.e.,
when an incumbent voluntarily gives up the office. Held: Under Section 44, a permanent vacancy arises
when an elective official fills a higher vacant office, refuses to
Xxx assume office, fails to qualify, dies, is removed from office,
voluntarily resigns, or is otherwise permanently incapacitated to
A winner who dislodges in a recall election an discharge the functions of his office. (Navarro v. Court of
incumbent elective local official merely serves the balance of Appeals, 355 SCRA 672, Mar. 28, 2001, 1 st Div. [Kapunan])
the latter’s term of office; it is not a full three-year term. It also
goes without saying that an incumbent elective local official 413. How is Section 45(b) of the Local Government
against whom a recall election is initiated and who nevertheless Code to be interpreted? What is the reason behind
wins in a recall election must be viewed as being a continuing the right given to a political party to nominate a
term of office and not as a break in reckoning his three replacement where a permanent vacancy occurs in
consecutive terms. X x x the Sanggunian?

If involuntary severance from the service which Held: What is crucial is the interpretation of Section
results in the incumbent’s being unable to finish his term of 45(b) providing that “x x x only the nominee of the political
office because of his ouster through valid recall proceedings party under which the Sanggunian member concerned has been
negates “one term” for purposes of applying the three-term elected and whose elevation to the position next higher in rank
limit, as so intimated in Lonzanida, it stands to reason that the created the last vacancy in the Sanggunian shall be appointed in
balance of the term assumed by the newly elected local official the manner hereinabove provided. The appointee shall come
in a recall election should not also be held to be one term in from the political party as that of the Sanggunian member who
reckoning the three-term limit. In both situations, neither the caused the vacancy x x x.”
elective local official who is unable to finish his term nor the
elected local official who only assumes the balance of the term The reason behind the right given to a political party to
of the ousted local official following the recall election could be nominate a replacement where a permanent vacancy occurs in
considered to have served a full three-year term set by the the Sanggunian is to maintain the party representation as willed
Constitution. by the people in the election.

This view is not inconsistent, but indeed in line, with With the elevation of petitioner Tamayo, who belonged
the conclusion ultimately reached in Socrates v. Commission to REFORMA-LM, to the position of Vice-Mayor, a vacancy
on Elections, where the Court has considered Hagedorn, occurred in the Sanggunian that should be filled up with
following his three full terms of nine years, still qualified to run someone who should belong to the political party of petitioner
in a recall election conducted about a year and a half after the Tamayo. Otherwise, REFORMA-LM’s representation in the
most recent regular local elections. A recall term then, not Sanggunian would be diminished. To argue that the vacancy
being a full three-year term, is not to be counted or used as a created was that formerly held by Rolando Lalas, a LAKAS-
basis for disqualification whether it is held prior or subsequent NUCD-Kampi member, would result in the increase of that
to the nine year full three-term limit. party’s representation in the Sanggunian at the expense of the
REFORMA-LM. This interpretation is contrary to the letter and
spirit of the law and thus violative of a fundamental rule in
159
statutory construction which is to ascertain and give effect to the done so in clear and unequivocal terms. But as it is, there is no
intent and purpose of the law. As earlier pointed out, the reason such intent.
behind par. (b), section 44 of the Local Government Code is the
maintenance of party representation in the Sanggunian in Moreover, adopting or updating of house rules would
accordance with the will of the electorate. necessarily entail work beyond the day of the first regular
session. Does this mean that prior thereto, the local council's
The “last vacancy” in the Sanggunian refers to that hands were tied and could not act on any other matter? That
created by the elevation of the member formerly occupying the would certainly be absurd for it would result in a hiatus and a
next higher in rank which in turn also had become vacant by any paralysis in the local legislature's work which could not have
of the causes already enumerated. The term “last vacancy” is been intended by the law. (Malonzo v. Zamora, 311 SCRA
thus used in Sec. 45 (b) to differentiate it from the other vacancy 224, July 27, 1999, En Banc [Romero])
previously created. The term by no means refers to the vacancy
in the No. 8 position which occurred with the elevation of 417. May local elective officials practice their profession
Rolando Lalas to the seventh position in the Sanggunian. Such or engage in any occupation?
construction will result in absurdity. (Navarro v. Court of
Appeals, 355 SCRA 672, Mar. 28, 2001, 1 st Div. [Kapunan]) Ans.: Sec. 90, Local Government Code, provides:

414. May an incumbent Vice-Governor, while SEC. 90. Practice of Profession. – (a) All governors,
concurrently the Acting Governor, continue to city and municipal mayors are prohibited from practicing their
preside over the sessions of the Sangguniang profession or engaging in any occupation other than the exercise
Panlalawigan (SP)? If no, who may preside in the of their functions as local chief executives.
meantime?
(b) Sanggunian members may practice their
Held: Being the acting governor, the Vice-governor professions, engage in any occupation, or teach in schools
cannot continue to simultaneously exercise the duties of the except during session hours: Provided, That sanggunian
latter office, since the nature of the duties of the Provincial members who are also members of the Bar shall not:
Governor calls for a full-time occupant to discharge them. Such
is not only consistent with but also appears to be the clear Appear as counsel before any court in any civil
rationale of the new (Local Government) Code wherein the case wherein a local government unit or any
policy of performing dual functions in both offices has already office, agency, or instrumentality of the
been abandoned. To repeat, the creation of a temporary vacancy government is the adverse party;
in the office of the Governor creates a corresponding vacancy in Appear as counsel in any criminal case wherein an
the office of the Vice-Governor whenever the latter acts as officer or employee of the national or local
Governor by virtue of such temporary vacancy. This event government is accused of an offense
constitutes an “inability” on the part of the regular presiding committed in relation to his office;
officer (Vice-Governor) to preside during the SP sessions, Collect any fee for their appearance in
which thus calls for the operation of the remedy set in Article administrative proceedings involving the local
49(b) of the Local Government Code – concerning the election government unit of which he is an official;
of a temporary presiding officer. The continuity of the Acting and
Governor’s (Vice-Governor) powers as presiding officer of the Use property and personnel of the government
SP is suspended so long as he is in such capacity. Under except when the sanggunian member
Section 49(b), “in the event of the inability of the regular concerned is defending the interest of the
presiding officer to preside at the sanggunian session, the Government.
members present and constituting a quorum shall elect from
among themselves a temporary presiding officer.” (Gamboa, (c) Doctors of medicine may practice their profession
Jr. v. Aguirre, Jr., G.R. No. 134213, July 20, 1999, En Banc even during official hours of work only on occasions of
[Ynares-Santiago]) emergency: Provided, that the officials concerned do not derive
monetary compensation therefrom.
415. Distinguish an ordinance from a mere resolution.
418. What is recall?
Held: A municipal ordinance is different from a
resolution. An ordinance is a law, but a resolution is merely a Held: Recall is a mode of removal of a public officer
declaration of the sentiment or opinion of a lawmaking body on by the people before the end of his term of office. The people's
a specific matter. An ordinance possesses a general and prerogative to remove a public officer is an incident of their
permanent character, but a resolution is temporary in nature. sovereign power and in the absence of constitutional restraint,
Additionally, the two are enacted differently – a third reading is the power is implied in all governmental operations. Such
necessary for an ordinance, but not for a resolution, unless power has been held to be indispensable for the proper
decided otherwise by a majority of all the Sanggunian members. administration of public affairs. Not undeservedly, it is
(Municipality of Paranaque v. V.M. Realty Corporation, 292 frequently described as a fundamental right of the people in a
SCRA 678, July 20, 1998 [Panganiban]) representative democracy. (Garcia v. COMELEC, 227 SCRA
108, Oct. 5, 1993, En Banc [Puno])
416. On its first regular session, may the Sanggunian
transact business other than the matter of adopting 419. What is the ground for recall? Is this subject to
or updating its existing rules or procedure? judicial inquiry?

Held: We cannot infer the mandate of the (Local Held: Former Senator Aquilino Pimentel, Jr., a major
Government) Code that no other business may be transacted on author of the subject law in his book The Local Government
the first regular session except to take up the matter of adopting Code of 1991: The Key to National Development, stressed the
or updating rules. All that the law requires is that “on the first same reason why the substantive content of a vote of lack of
regular session x x x the sanggunian concerned shall adopt or confidence is beyond any inquiry, thus:
update its existing rules or procedures.” There is nothing in the
language thereof that restricts the matters to be taken up during “There is only one ground for recall of local
the first regular session merely to the adoption or updating of government officials: loss of confidence. This means
the house rules. If it were the intent of Congress to limit the that the people may petition or the Preparatory Recall
business of the local council to such matters, then it would have Assembly may resolve to recall any local elective
160
official without specifying any particular ground
except loss of confidence. There is no need for them to (b) No recall shall take place within one (1) year from
bring up any charge of abuse or corruption against the the date of the official’s assumption to office or one (1) year
local elective officials who are subject of any recall immediately preceding a regular local election.
petition.
423. Section 74 of the Local Government Code provides
In the case of Evardone v. Commission on that “no recall shall take place within one year x x
Elections, et al., 204 SCRA 464, 472 (1991), the Court x immediately preceding a regular local election.”
ruled that ‘loss of confidence’ as a ground for recall is What does the term “regular local election,” as
a political question. In the words of the Court, used in this section, mean?
'whether or not the electorate of the municipality of
Sulat has lost confidence in the incumbent mayor is a Held: The term “regular local election” under Sec. 74
political question.’” of the Local Government Code of 1991 which provides that “no
(Garcia v. COMELEC, 227 SCRA 108, Oct. 5, 1993, En Banc recall shall take place within one (1) year x x x immediately
[Puno]) preceding a regular local election” refers to one where the
position of the official sought to be recalled is to be actually
420. The members of the Preparatory Recall Assembly contested and filled by the electorate (Paras v. Comelec, G.R.
(PRA) of the province of Bataan adopted a No. 123169, Nov. 4, 1996). The one-year time bar will not
resolution calling for the recall of Governor apply where the local official sought to be recalled is a Mayor
Garcia. It was admitted, however, by the and the approaching election is a barangay election. (Angobung
proponents of the recall resolution that only those v. COMELEC, G.R. No. 126576, March 5, 1997)
members of the assembly inclined to agree were
notified of the meeting where said resolution was 424. Does the word “Recall” in paragraph (b) of Section
adopted “as a matter of strategy and security.” 74 of the Local Government Code include the
They justified these selective notices on the ground convening of the Preparatory Recall Assembly and
that the law (Local Government Code) does not the filing by it of a recall resolution? Discuss.
specifically mandate the giving of notice. Should
this submission be sustained? Held: We can agree that recall is a process which
begins with the convening of the preparatory recall assembly or
Held: We reject this submission of the respondents. the gathering of the signatures at least 25% of the registered
The due process clause of the Constitution requiring notice as an voters of a local government unit, and then proceeds to the filing
element of fairness is inviolable and should always be of a recall resolution or petition with the COMELEC, the
considered part and parcel of every law in case of its silence. verification of such resolution or petition, the fixing of the date
The need for notice to all the members of the assembly is also of the recall election, and the holding of the election on the
imperative for these members represent the different sectors of scheduled date. However, as used in paragraph (b) of Sec. 74,
the electorate of Bataan. To the extent that they are not notified “recall” refers to the election itself by means of which voters
of the meeting of the assembly, to that extent is the sovereign decide whether they should retain their local official or elect his
voice of the people they represent nullified. The resolution to replacement.
recall should articulate the majority will of the members of the
assembly but the majority will can be genuinely determined Xxx
only after all the members of the assembly have been given a
fair opportunity to express the will of their constituents. To sum up, the term “recall” in paragraph (b) refers to
Needless to stress, the requirement of notice is mandatory for it the recall election and not to the preliminary proceedings to
is indispensable in determining the collective wisdom of the initiate recall –
members of the Preparatory Recall Assembly. Its non-
observance is fatal to the validity of the resolution to recall Because Sec. 74 speaks of limitations on “recall”
petitioner Garcia as Governor of the province of Bataan. which, according to Sec. 69, is a power which
(Garcia v. COMELEC, G.R. No. 111511, Sept. 21, 1993; 227 shall be exercised by the registered voters of a
SCRA 100, Oct. 5, 1993, En Banc [Puno]) local government unit. Since the voters do not
exercise such right except in an election, it is clear
421. Will it be proper for the Commission on Elections that the initiation of recall proceedings is not
to act on a petition for recall signed by just one prohibited within the one-year period provided in
person? paragraph (b);
Because the purpose of the first limitation in paragraph
Held: A petition for recall signed by just one person is (b) is to provide voters a sufficient basis for
in violation of the statutory 25% minimum requirement as to the judging an elective local official, and final judging
number of signatures supporting any petition for recall. Sec. is not done until the day of the election; and
69(d) of the Local Government Code of 1991 expressly provides Because to construe the limitation in paragraph (b) as
that 'recall of any elective x x x municipal x x x official may including the initiation of recall proceedings would
also be validly initiated upon petition of at least twenty-five unduly curtail freedom of speech and of assembly
percent (25%) of the total number of registered voters in the guaranteed in the Constitution.
local government unit concerned during the election in which (Jovito O. Claudio v. COMELEC, G.R. No. 140560, May 4,
the local official sought to be recalled was elected.' The law is 2000, En Banc [Mendoza])
plain and unequivocal as to what constitutes recall proceedings:
only a petition of at least 25% of the total number of registered 425. The members of the Preparatory Recall Assembly
voters may validly initiate recall proceedings. (Angobung v. (PRA) of Puerto Princesa City met and adopted a
COMELEC, G.R. No. 126576, March 5, 1997) resolution calling for the recall of incumbent
Mayor Dennis Victorino M. Socrates on the
422. What are the limitations on recall? ground of loss of confidence on July 2, 2002.
Mayor Socrates argued that they have no authority
Ans.: Section 74, Local Government Code, provides: to adopt said Recall Resolution because a majority
of PRA members were seeking a new electoral
SEC. 74. Limitations on Recall. – (a) Any elective mandate in the barangay elections scheduled on
local official may be the subject of a recall election only once July 15, 2002. Should his contention be sustained?
during his term of office for loss of confidence.
161
Held: This argument deserves scant consideration
considering that when the PRA members adopted the Recall
Resolution their terms of office had not yet expired. They were G. PUBLIC INTERNATIONAL LAW
all de jure sangguniang barangay members with no legal
disqualification to participate in the recall assembly under 428. What is the Doctrine of Incorporation? How is it
Section 70 of the Local Government Code. (Victorino Dennis applied by local courts?
M. Socrates v. The Commission on Elections, G.R. No.
154512, Nov. 12, 2002, En Banc [Carpio]) Held: Under the doctrine of incorporation, rules of
international law form part of the law of the land and no further
426. Whether or not a local elective official who became legislative action is needed to make such rules applicable in the
City Mayor by legal succession can be the subject domestic sphere.
of a recall election by virtue of a Preparatory
Recall Assembly Resolution which was passed or The doctrine of incorporation is applied whenever
adopted when the she was still the Vice-Mayor. municipal tribunals (or local courts) are confronted with
situations in which there appears to be a conflict between a rule
Held: The specific purpose of the Preparatory Recall of international law and the provisions of the Constitution or
Assembly was to remove Amelita S. Navarro as the elected statute of the local state. Efforts should first be exerted to
Vice-Mayor of Santiago City since PRA Resolution No. 1 dated harmonize them, so as to give effect to both since it is to be
July 12, 1999 expressly states that “ x x x it is hereby resolved presumed that municipal law was enacted with proper regard for
to invoke the rescission of the electoral mandate of the the generally accepted principles of international law in
incumbent City Vice-Mayor Amelita S. Navarro for loss of observance of the Incorporation Clause in Section 2, Article II
confidence through a recall election to be set by the of the Constitution. In a situation however, where the conflict is
Commission on Election as provided for under Section 71 of the irreconcilable and a choice has to be made between a rule of
Local Government Code of 1991.” However, the said PRA international law and municipal law, jurisprudence dictates that
Resolution No. 1 is no longer applicable to her inasmuch as she municipal law should be upheld by the municipal courts for the
had already vacated the office of Vice-Mayor on October 11, reason that such courts are organs of municipal law and are
1999 when she assumed the position of City Mayor of Santiago accordingly bound by it in all circumstances. The fact that
City. international law has been made part of the law of the land does
not pertain to or imply the primacy of international law over
Even if the Preparatory Recall Assembly were to national or municipal law in the municipal sphere. The doctrine
reconvene to adopt another resolution for the recall of Amelita of incorporation, as applied in most countries, decrees that rules
Navarro, this time as Mayor of Santiago City, the same would of international law are given equal standing with, but are not
still not prosper in view of Section 74 (b) of the Local superior to, national legislative enactments. Accordingly, the
Government Code of 1991 which provides that “No recall shall principle of lex posterior derogat priori takes effect – a treaty
take place within one (1) year from the date of the official’s may repeal a statute and a statute may repeal a treaty. In states
assumption of office or one (1) year immediately preceding a where the Constitution is the highest law of the land, such as the
regular election.” There is no more allowable time in the light Republic of the Philippines, both statutes and treaties may be
of that law within which to hold recall elections for that invalidated if they are in conflict with the Constitution.
purpose. The then Vice-Mayor Amelita S. Navarro assumed (Secretary of Justice v. Hon. Ralph C. Lantion, G.R. No.
office as Mayor of Santiago City on October 11, 1999. One 139465, Jan. 18, 2000, En Banc [Melo])
year after her assumption of office as Mayor will be October 11,
2000 which is already within the one (1) year prohibited period 429. Discuss the contemporary view on the rightful
immediately preceding the next regular election in May 2001. place of an Individual in International Law? Does
(Afiado v. Commission on Elections, 340 SCRA 600, Sept. 18, he remain a mere “object” of International Law, or
2000, En Banc [De Leon] is he now a proper “subject” of International Law?

427. May the Punong Barangay validly appoint or Held: Then came the long and still ongoing debate on
remove the barangay treasurer, the barangay what should be the subject of international law. The 20th
secretary, and other appointive barangay officials century saw the dramatic rise and fall of different types and hues
without the concurrence of the majority of all the of authoritarianism – the fascism of Italy’s Mussolini and
members of the Sangguniang Barangay? Germany’s Hitler, the militarism of Japan’s Hirohito and the
communism of Russia’s Stalin, etc. The sinking of these isms
Held: The Local Government Code explicitly vests on led to the elevation of the rights of the individual against the
the punong barangay, upon approval by a majority of all the state. Indeed, some species of human rights have already been
members of the sangguniang barangay, the power to appoint or accorded universal recognition. Today, the drive to
replace the barangay treasurer, the barangay secretary, and internationalize rights of women and children is also on high
other appointive barangay officials. Verily, the power of gear. The higher rating given to human rights on the hierarchy
appointment is to be exercised conjointly by the punong of values necessarily led to the re-examination of the rightful
barangay and a majority of all the members of the sangguniang place of the individual in international law. Given the harshest
barangay. Without such conjoint action, neither an appointment eye is the moss-covered doctrine that international law deals
nor a replacement can be effectual. only with States and that individuals are not its subject. For its
undesirable corollary is that sub-doctrine that an individual’s
Applying the rule that the power to appoint includes right in international law is a near cipher. Translated in
the power to remove x x x the questioned dismissal from office extradition law, the view that once commanded a consensus is
of the barangay officials by the punong barangay without the that since a fugitive is a mere object and not a subject of
concurrence of the majority of all the members of the international law, he is bereft of rights. An extraditee, so it was
Sangguniang Barangay cannot be legally justified. To rule held, is a mere “object transported from one state to the other as
otherwise could also create an absurd situation of the an exercise of the sovereign will of the two states involved.”
Sangguniang Barangay members refusing to give their approval The re-examination consigned this pernicious doctrine to the
to the replacements selected by the punong barangay who has museum of ideas. The new thinkers of international law then
unilaterally terminated the services of the incumbents. It is gave a significant shape to the role and rights of the individual
likely that the legislature did not intend this absurdity to follow in state-concluded treaties and other international agreements. x
from its enactment of the law. (Ramon Alquizola, Sr. v. x x (Concurring Opinion, Puno J., in Jeffrey Liang [Huefeng]
Gallardo Ocol, G.R. No. 132413, Aug. 27, 1999, 3 rd Div. v. People, G.R. No. 125865, Mar. 26, 2001, 1 st Div. [Motion for
[Vitug]) Reconsideration])
162
International organizations and bodies have realized
430. What must a person who feels aggrieved by the acts the necessity of applying policies, programs and specific rules
of a foreign sovereign do to espouse his cause? concerning IPs in some nations. The World Bank, for example,
first adopted a policy on IPs as a result of the dismal experience
Held: Private respondent is not left without any legal of projects in Latin America. The World Bank now seeks to
remedy for the redress of its grievances. Under both Public apply its current policy on IPs to some of its projects in Asia.
International Law and Transnational Law, a person who feels This policy has provided an influential model for the projects of
aggrieved by the acts of a foreign sovereign can ask his own the Asian Development Bank.
government to espouse his cause through diplomatic channels.
The 1987 Philippine Constitution formally recognizes
Private respondent can ask the Philippine government, the existence of ICCs/IPs and declares as a State policy the
through the Foreign Office, to espouse its claims against the promotion of their rights within the framework of national unity
Holy See. Its first task is to persuade the Philippine government and development (Section 22, Article II, 1987 Constitution).
to take up with the Holy See the validity of its claim. Of course, The IPRA amalgamates the Philippine category of ICCs with
the Foreign Office shall first make a determination of the impact the international category of IPs, and is heavily influenced by
of its espousal on the relations between the Philippine both the International Labor Organization (ILO) Convention
government and the Holy See. Once the Philippine government 169 and the United Nations (UN) Draft Declaration on the
decides to espouse the claim, the latter ceases to be a private Rights of Indigenous Peoples.
cause.
ILO Convention No. 169 is entitled the “Convention
According to the Permanent Court of International Concerning Indigenous and Tribal Peoples in Independent
Justice, the forerunner of the International Court of Justice: Countries” (also referred to as the “Indigenous and Tribal
Peoples Convention, 1989”) and was adopted on June 27, 1989.
“By taking up the case of one of its subjects It is based on the Universal Declaration of Human Rights, the
and by resorting to diplomatic action or international International Covenant on Economic, Social and Cultural
judicial proceedings on his behalf, a State is in reality Rights, the International Covenant on Civil and Political Rights,
asserting its own rights - its right to ensure, in the and many other international instruments on the prevention of
person of its subjects, respect for the rules of discrimination. ILO Convention No. 169 revised the
international law” (The Mavrommatis Palestine “Convention Concerning the Protection and Integration of
Concessions, 1 Hudson, World Court Reports 293, 302 Indigenous and Other Tribal and Semi-Tribal Populations in
[1924]). Independent Countries” passed on June 26, 1957.
(Holy See, The v. Rosario, Jr., 238 SCRA 524, 538-539, Dec. Developments in international law made it appropriate to adopt
1, 1994, En Banc [Quiason]) new international standards on indigenous peoples “with a view
to removing the assimilationist orientation of the earlier
431. Discuss the Indigenous International Movement. standards,” and “recognizing the aspirations of these peoples to
Is the Philippines an active participant in the exercise control over their own institutions, ways of life and
Indigenous International Movement? economic development.” (Separate Opinion, Puno, J., in Cruz
v. Secretary of Environment and Natural Resources, 347
Held: The Indigenous Peoples Rights Act (IPRA) is a SCRA 128, 238-241, Dec. 6, 2000, En Banc)
recognition of our active participation in the indigenous
international movement. 432. Is sovereignty really absolute and all-
encompassing? If no, what are its restrictions and
The indigenous movement can be seen as the heir to a limitations?
history of anti-imperialism stretching back to prehistoric times.
The movement received a massive impetus during the 1960’s Held: While sovereignty has traditionally been
from two sources. First, the decolonization of Asia and Africa deemed absolute and all-encompassing on the domestic level, it
brought into the limelight the possibility of peoples controlling is however subject to restrictions and limitations voluntarily
their own destinies. Second, the right of self-determination was agreed to by the Philippines, expressly or impliedly, as a
enshrined in the UN Declaration on Human Rights. The rise of member of the family of nations. By the doctrine of
the civil rights movement and anti-racism brought to the incorporation, the country is bound by generally accepted
attention of North American Indians, Aborigines in Australia, principles of international law, which are considered to be
and Maori in New Zealand the possibility of fighting for automatically part of our own laws. One of the oldest and most
fundamental rights and freedoms. fundamental rules in international law is pacta sunt servanda –
international agreements must be performed in good faith. A
In 1974 and 1975, international indigenous state which has contracted valid international obligations is
organizations were founded, and during the 1980’s, indigenous bound to make in its legislations such modifications as may be
affairs were on the international agenda. The people of the necessary to ensure the fulfillment of the obligations.
Philippine Cordillera were the first Asians to take part in the
international indigenous movement. It was the Cordillera By their inherent nature, treaties really limit or restrict
People’s Alliance that carried out successful campaigns against the absoluteness of sovereignty. By their voluntary act, nations
the building of the Chico River Dam in 1981-82 and they have may surrender some aspects of their state power in exchange for
since become one of the best-organized indigenous bodies in the greater benefits granted by or derived from a convention or pact.
world. After all, states, like individuals, live with coequals, and in
pursuit of mutually covenanted objectives and benefits, they
Presently, there is a growing concern for indigenous also commonly agree to limit the exercise of their otherwise
rights in the international scene. This came as a result of the absolute rights. Thus, treaties have been used to record
increased publicity focused on the continuing disrespect for agreements between States concerning such widely diverse
indigenous human rights and the destruction of the indigenous matters as, for example, the lease of naval bases, the sale or
peoples’ environment, together with the national governments’ cession of territory, the termination of war, the regulation of
inability to deal with the situation. Indigenous rights came as a conduct of hostilities, the formation of alliances, the regulation
result of both human rights and environmental protection, and of commercial relations, the settling of claims, the laying down
have become a part of today’s priorities for the international of rules governing conduct in peace and the establishment of
agenda. international organizations. The sovereignty of a state therefore
cannot in fact and in reality be considered absolute. Certain
restrictions enter into the picture: (1) limitations imposed by the
163
very nature of membership in the family of nations and (2) An organized civil government that has control and
limitations imposed by treaty stipulations. (Tanada v. Angara, direction over the armed struggle launched by the
272 SCRA 18, May 2, 1997 [Panganiban]) rebels;
Occupation of a substantial portion of the national
433. Discuss the Status of the Vatican and the Holy See territory;
in International Law. Seriousness of the struggle, which must be so
widespread thereby leaving no doubt as to the
Held: Before the annexation of the Papal States by outcome;
Italy in 1870, the Pope was the monarch and he, as the Holy Willingness on the part of the rebels to observe the
See, was considered a subject of International Law. With the rules and customs of war.
loss of the Papal States and the limitation of the territory under
the Holy See to an area of 108.7 acres, the position of the Holy 435. Discuss the legal consequences that follow
See in International Law became controversial. recognition of belligerency.

In 1929, Italy and the Holy See entered into the Lateran Ans.: Before recognition as such, it is the legitimate
Treaty, where Italy recognized the exclusive dominion and government that is responsible for the acts of the rebels
sovereign jurisdiction of the Holy See over the Vatican City. It affecting foreign nationals and their properties. Once
also recognized the right of the Holy See to receive foreign recognition is given, the legitimate government may no longer
diplomats, to send its own diplomats to foreign countries, and to be held responsible for their acts; responsibility is shifted to the
enter into treaties according to International Law. rebel government;

The Lateran Treaty established the statehood of the The legitimate government, once it recognizes the
Vatican City “for the purpose of assuring to the Holy See rebels as belligerents, is bound to observe the laws and customs
absolute and visible independence and of guaranteeing to it of war in conducting the hostilities;
indisputable sovereignty also in the field of international
relations.” From the point of view of third States, the effect of
recognition of belligerency is to put them under obligation to
In view of the wordings of the Lateran Treaty, it is observe strict neutrality and abide by the consequences arising
difficult to determine whether the statehood is vested in the from that position;
Holy See or in the Vatican City. Some writers even suggested
that the treaty created two international persons - the Holy See On the side of the rebels, recognition of belligerency
and Vatican City. puts them under responsibility to third States and to the
legitimate government for all their acts which do not conform to
The Vatican City fits into none of the established the laws and customs of war. (Salonga & Yap, Public
categories of states, and the attribution to it of “sovereignty” International Law, 5th Ed. [1992], p. 33)
must be made in a sense different from that in which it is
applied to other states. In a community of national states, the 436. Discuss the occasions when the use of force may be
Vatican City represents an entity organized not for political but allowed under the UN Charter.
for ecclesiastical purposes and international objects. Despite its
size and object, the Vatican City has an independent Ans.: There are only two occasions when the use of
government of its own, with the Pope, who is also head of the force is allowed under the UN Charter. The first is when it is
Roman Catholic Church, as the Holy See or Head of State, in authorized in pursuance of the enforcement action that may be
conformity with its traditions, and the demands of its mission in decreed by the Security Council under Art. 42. The second is
the world. Indeed, the worldwide interests and activities of the when it is employed in the exercise of the inherent right of self-
Vatican City are such as to make it in a sense an “international defense under conditions prescribed in Art. 51. (Justice Isagani
state.” A. Cruz, in an article entitled “A New World Order” written in
his column “Separate Opinion” published in the March 30,
One authority wrote that the recognition of the Vatican 2003 issue of the Philippines Daily Inquirer)
City as a state has significant implication - that it is possible for
any entity pursuing objects essentially different from those 437. Is the United States justified in invading Iraq
pursued by states to be invested with international personality. invoking its right to defend itself against an
expected attack by Iraq with the use of its
Inasmuch as the Pope prefers to conduct foreign biological and chemical weapons of mass
relations and enter into transactions as the Holy See and not in destruction?
the name of the Vatican City, one can conclude that in the
Pope's own view, it is the Holy See that is the international Ans.: The United States is invoking its right to defend
person. itself against an expected attack by Iraq with the use of its
biological and chemical weapons of mass destruction. There is
The Republic of the Philippines has accorded the Holy no evidence of such a threat, but Bush is probably invoking the
See the status of a foreign sovereign. The Holy See, through its modern view that a state does not have to wait until the potential
Ambassador, the Papal Nuncio, has had diplomatic enemy fires first. The cowboy from Texas says that outdrawing
representations with the Philippine government since 1957. the foe who is about to shoot is an act of self-defense.
This appears to be the universal practice in international
relations. (Holy See, The v. Rosario, Jr., 238 SCRA 524, 533- Art. 51 says, however, that there must first be an
534, Dec. 1, 1994, En Banc [Quiason]) “armed attack” before a state can exercise its inherent right of
self-defense, and only until the Security Council, to which the
434. What are the conditions before the rights of aggression should be reported, shall have taken the necessary
belligerency may be accorded the rebels? measures to maintain international peace and security. It was
the United States that made the “armed attack” first, thus
Ans.: As a matter of legal theory, the rebels have to becoming the aggressor, not Iraq. Iraq is now not only
fulfill certain conditions before the rights of belligerency are exercising its inherent right of self-defense as recognized by the
accorded them, namely: UN Charter. (Justice Isagani A. Cruz, in an article entitled “A
New World Order” written in his column “Separate Opinion”
published in the March 30, 2003 issue of the Philippines Daily
Inquirer)
164
Imprisonment or other severe deprivation of physical
438. Will the subsequent discovery of weapons of mass liberty in violation of fundamental rules of
destruction in Iraq after its invasion by the US international law;
justify the attack initiated by the latter? Torture;
Rape, sexual slavery, enforced prostitution, forced
Ans.: Even if Iraq’s hidden arsenal is discovered – or pregnancy, enforced sterilization, or any other
actually used – and the United States is justified in its form of sexual violence of comparable gravity;
suspicions, that circumstance will not validate the procedure Persecution against any identifiable group or
taken against Iraq. It is like searching a person without warrant collectivity on political, racial, national, ethnic,
and curing the irregularity with the discovery of prohibited cultural, religious, gender as defined in paragraph
drugs in his possession. The process cannot be reversed. The 3, or other grounds that are universally recognized
warrant must first be issued before the search and seizure can be as impermissible under international law, in
made. connection with any act referred to in this
paragraph or any crime within the jurisdiction of
The American invasion was made without permission the Court;
from the Security Council as required by the UN Charter. Any Enforced disappearance of persons;
subsequent discovery of the prohibited biological and chemical The crime of apartheid;
weapons will not retroactively legalize that invasion, which was, Other inhumane acts of a similar character intentionally
legally speaking, null and void ab initio. (Justice Isagani A. causing great suffering, or serious injury to body
Cruz, in an article entitled “A New World Order” written in or to mental or physical health.
his column “Separate Opinion” published in the March 30,
2003 issue of the Philippines Daily Inquirer) 2. For the purpose of paragraph 1:

439. What Crimes come within the jurisdiction of the “Attack directed against any civilian population”
Rome Statute of the International Criminal Court? means a course of conduct involving the multiple
commission of acts referred to in paragraph 1
Ans.: 1. The jurisdiction of the Court shall be limited against any civilian population, pursuant to or in
to the most serious crimes of concern to the international furtherance of a State or organizational policy to
community as a whole. The Court has jurisdiction in commit such attack;
accordance with this Statute with respect to the following “Extermination” includes the intentional infliction of
crimes: conditions of life, inter alia the deprivation of
access to food and medicine, calculated to bring
The crime of genocide; about the destruction of part of a population;
Crimes against humanity; “Enslavement” means the exercise of any or all of the
War crimes; powers attaching to the right of ownership over a
The crime of aggression. person and includes the exercise of such power in
the course of trafficking in persons, in particular
2. The Court shall exercise jurisdiction over the crime women and children;
of aggression once a provision is adopted in accordance with “Deportation or forcible transfer of population” means
articles 121 and 123 defining the crime and setting out the forced displacement of the persons concerned by
conditions under which the Court shall exercise jurisdiction with expulsion or other coercive acts from the area in
respect to this crime. Such a provision shall be consistent with which they are lawfully present, without grounds
the relevant provisions of the Charter of the United Nations. permitted under international law;
(Art. 5, Rome Statute of the International Criminal Court) “Torture” means the intentional infliction of severe
pain or suffering, whether physical or mental,
440. What is Genocide? upon a person in the custody or under the control
of the accused; except that torture shall not include
Ans.: For the purpose of this Statute, “genocide” pain or suffering arising only from, inherent in or
means any of the following acts committed with intent to incidental to lawful, sanctions;
destroy, in whole or in part, a national, ethnical, racial or “Forced pregnancy” means the unlawful confinement,
religious group, such as: of a woman forcibly made pregnant, with the
intent of affecting the ethnic composition of any
Killing members of the group; population or carrying out other grave violations
Causing serious bodily or mental harm to members of of international law. This definition shall not in
the group; any way be interpreted as affecting national laws
Deliberately inflicting on the group conditions of life relating to pregnancy;
calculated to bring about its physical destruction in “Persecution” means the intentional and severe
whole or in part; deprivation of fundamental rights contrary to
Imposing measures intended to prevent births within international law by reason of the identity of the
the group; group or collectivity;
Forcibly transferring children of the group to another “The crime of apartheid” means inhumane acts of a
group. character similar to those referred to in paragraph
(Art. 6, Rome Statute of the International Criminal Court) 1, committed in the context of an institutionalized
regime of systematic oppression and domination
441. What are Crimes against Humanity? by one racial group over any other racial group or
groups and committed with the intention of
Ans.: 1. For the purpose of this Statute, “crime against maintaining that regime;
humanity” means any of the following acts when committed as “Enforced disappearance of persons” means the arrest,
part of a widespread or systematic attack directed against any detention or abduction of persons by, or with the
civilian population, with knowledge of the attack: authorization, support or acquiescence of, a State
or a political organization, followed by a refusal to
Murder; acknowledge the deprivation of freedom or to give
Extermination; information on the fate or whereabouts of those
Enslavement; persons, with the intention of removing them from
Deportation or forcible transfer of population;
165
the protection of the law for a prolonged period of Department of Foreign Affairs. (Holy See, The v. Rosario, Jr.,
time. 238 SCRA 524, Dec. 1, 1994, En Banc [Quiason])

3. For the purpose of this Statute, it is understood that 445. Should Courts blindly adhere and take on its face
the term “gender” refers to the two sexes, male and female, the communication from the Department of
within the context of society. The term “gender” does not Foreign Affairs (DFA) that a person is covered by
indicate any meaning different from the above. any immunity?
(Art. 7, Rome Statute of the International Criminal Court)
Held: Courts cannot blindly adhere and take on its
442. What are International Organizations? Discuss face the communication from the DFA that petitioner is covered
their nature. by any immunity. The DFA’s determination that a certain
person is covered by immunity is only preliminary which has no
Held: International organizations are institutions binding effect in courts. In receiving ex parte the DFA’s advice
constituted by international agreement between two or more and in motu proprio dismissing the two criminal cases without
States to accomplish common goals. The legal personality of notice to the prosecution, the latter’s right to due process was
these international organizations has been recognized not only violated. It should be noted that due process is a right of the
in municipal law, but in international law as well. accused as much as it is of the prosecution. The needed inquiry
in what capacity petitioner was acting at the time of the alleged
Permanent international commissions and utterances requires for its resolution evidentiary basis that has
administrative bodies have been created by the agreement of a yet to be presented at the proper time. At any rate, it has been
considerable number of States for a variety of international ruled that the mere invocation of the immunity clause does not
purposes, economic or social and mainly non-political. In so far ipso facto result in the dropping of the charges. (Liang v.
as they are autonomous and beyond the control of any one State, People, 323 SCRA 692, Jan. 28, 2000, 1 st Div. [Ynares-
they have distinct juridical personality independent of the Santiago])
municipal law of the State where they are situated. As such,
they are deemed to possess a species of international personality 446. Discuss the basis of the argument that a
of their own. (SEAFDEC-AQD v. NLRC, 206 SCRA 283, Feb. determination by the DFA that a person is entitled
14, 1992) to diplomatic immunity is a political question
binding on the courts.
443. Discuss the basic immunities of international
organizations and the reason for affording them Held: Petitioner’s argument that a determination by
such immunities. the Department of Foreign Affairs that he is entitled to
diplomatic immunity is a political question binding on the
Held: One of the basic immunities of an international courts, is anchored on the ruling enunciated in the case of WHO,
organization is immunity from local jurisdiction, i.e., that it is et al. v. Aquino, et al., viz:
immune from legal writs and processes issued by the tribunals
of the country where it is found. The obvious reason for this is “It is a recognized principle of international
that the subjection of such an organization to the authority of the law and under our system of separation of powers that
local courts would afford a convenient medium through which diplomatic immunity is essentially a political question
the host government may interfere in their operations or even and courts should refuse to look beyond a
influence or control its policies and decisions; besides, such determination by the executive branch of the
subjection to local jurisdiction would impair the capacity of government, and where the plea of diplomatic
such body to discharge its responsibilities impartially on behalf immunity is recognized and affirmed by the executive
of its member-states. (SEAFDEC-AQD v. NLRC, 206 SCRA branch of the government as in the case at bar, it is then
283, Feb. 4, 1992) the duty of the courts to accept the claim of immunity
upon appropriate suggestion by the principal law
444. Is the determination of the executive branch of the officer of the government, the Solicitor General in this
government that a state or instrumentality is case, or other officer acting under his direction. Hence,
entitled to sovereign or diplomatic immunity in adherence to the settled principle that courts may not
subject to judicial review, or is it a political so exercise their jurisdiction by seizure and detention
question and therefore, conclusive upon the of property, as to embarrass the executive arm of the
courts? government in conducting foreign relations, it is
accepted doctrine that in such cases the judicial
Held: The issue of petitioner’s (The Holy See) non- department of the government follows the action of the
suability can be determined by the trial court without going to political branch and will not embarrass the latter by
trial in light of the pleadings x x x. Besides, the privilege of assuming an antagonistic jurisdiction.”
sovereign immunity in this case was sufficiently established by
the Memorandum and Certification of the Department of This ruling was reiterated in the subsequent cases of
Foreign Affairs. As the department tasked with the conduct of International Catholic Migration Commission v. Calleja; The
the Philippines’ foreign relations, the Department of Foreign Holy See v. Rosario, Jr.; Lasco v. United Nations; and DFA v.
Affairs has formally intervened in this case and officially NLRC.
certified that the Embassy of the Holy See is a duly accredited
diplomatic mission to the Republic of the Philippines exempt The case of WHO v. Aquino involved the search and
from local jurisdiction and entitled to all the rights, privileges seizure of personal effects of petitioner Leonce Verstuyft, an
and immunities of a diplomatic mission or embassy in this official of the WHO. Verstuyft was certified to be entitled to
country. The determination of the executive arm of government diplomatic immunity pursuant to the Host Agreement executed
that a state or instrumentality is entitled to sovereign or between the Philippines and the WHO.
diplomatic immunity is a political question that is conclusive
upon the courts. Where the plea of immunity is recognized and ICMC v. Calleja concerned a petition for certification
affirmed by the executive branch, it is the duty of the courts to election filed against ICMC and IRRI. As international
accept this claim so as not to embarrass the executive arm of the organizations, ICMC and IRRI were declared to possess
government in conducting the country’s foreign relations. As in diplomatic immunity. It was held that they are not subject to
International Catholic Migration Commission and in World local jurisdictions. It was ruled that the exercise of jurisdiction
Health Organization, we abide by the certification of the by the Department of Labor over the case would defeat the very
purpose of immunity, which is to shield the affairs of
166
international organizations from political pressure or control by and immunities as may be granted by the receiving State.
the host country and to ensure the unhampered performance of International immunities may be specially important in relation
their functions. to the State of which the official is a national. Secondly, the
immunity of a diplomatic agent from the jurisdiction of the
Holy See v. Rosario, Jr. involved an action for receiving State does not exempt him from the jurisdiction of the
annulment of sale of land against the Holy See, as represented sending State; in the case of international immunities there is no
by the Papal Nuncio. The Court upheld the petitioner’s defense sending State and an equivalent for the jurisdiction of the
of sovereign immunity. It ruled that where a diplomatic envoy Sending State therefore has to be found either in waiver of
is granted immunity from the civil and administrative immunity or in some international disciplinary or judicial
jurisdiction of the receiving state over any real action relating to procedure. Thirdly, the effective sanctions which secure respect
private immovable property situated in the territory of the for diplomatic immunity are the principle of reciprocity and the
receiving state, which the envoy holds on behalf of the sending danger of retaliation by the aggrieved State; international
state for the purposes of the mission, with all the more reason immunities enjoy no similar protection. (Concurring Opinion,
should immunity be recognized as regards the sovereign itself, Puno J., in Jeffrey Liang [Huefeng] v. People, G.R. No.
which in that case is the Holy See. 125865, Mar. 26, 2001, 1st Div. [Motion for Reconsideration])

In Lasco v. United Nations, the United Nations 448. Discuss the immunity of International Officials.
Revolving Fund for Natural Resources Exploration was sued
before the NLRC for illegal dismissal. The Court again upheld Held: The generally accepted principles which are
the doctrine of diplomatic immunity invoked by the Fund. now regarded as the foundation of international immunities are
contained in the ILO Memorandum, which reduced them in
Finally, DFA v. NLRC involved an illegal dismissal three basic propositions, namely: (1) that international
case filed against the Asian Development Bank. Pursuant to its institutions should have a status which protects them against
Charter and the Headquarters Agreement, the diplomatic control or interference by any one government in the
immunity of the Asian Development Bank was recognized by performance of functions for the effective discharge of which
the Court. they are responsible to democratically constituted international
bodies in which all the nations concerned are represented; (2)
It bears to stress that all of these cases pertain to the that no country should derive any financial advantage by
diplomatic immunity enjoyed by international organizations. levying fiscal charges on common international funds; and (3)
Petitioner asserts that he is entitled to the same diplomatic that the international organization should, as a collectivity of
immunity and he cannot be prosecuted for acts allegedly done in States Members, be accorded the facilities for the conduct of its
the exercise of his official functions. official business customarily extended to each other by its
individual member States. The thinking underlying these
The term “international organizations” – propositions is essentially institutional in character. It is not
concerned with the status, dignity or privileges of individuals,
“is generally used to describe an organization but with the elements of functional independence necessary to
set up by agreement between two or more states. free international institutions from national control and to
Under contemporary international law, such enable them to discharge their responsibilities impartially on
organizations are endowed with some degree of behalf of all their members. (Concurring Opinion, Puno J., in
international legal personality such that they are Jeffrey Liang [Huefeng] v. People, G.R. No. 125865, Mar. 26,
capable of exercising specific rights, duties and 2001, 1st Div. [Motion for Reconsideration])
powers. They are organized mainly as a means for
conducting general international business in which the 449. What are the three methods of granting privileges
member states have an interest.” (ICMC v. Calleja) and immunities to the personnel of international
organizations? Under what category does the
International public officials have been defined as: Asian Development Bank and its Personnel fall?

“x x x persons who, on the basis of an Held: Positive international law has devised three
international treaty constituting a particular methods of granting privileges and immunities to the personnel
international community, are appointed by this of international organizations. The first is by simple
international community, or by an organ of it, and are conventional stipulation, as was the case in the Hague
under its control to exercise, in a continuous way, Conventions of 1899 and 1907. The second is by internal
functions in the interest of this particular international legislation whereby the government of a state, upon whose
community, and who are subject to a particular territory the international organization is to carry out its
personal status.” functions, recognizes the international character of the
organization and grants, by unilateral measures, certain
“Specialized agencies” are international organizations privileges and immunities to better assure the successful
having functions in particular fields, such as posts, functioning of the organization and its personnel. In this
telecommunications, railways, canals, rivers, sea transport, civil situation, treaty obligation for the state in question to grant
aviation, meteorology, atomic energy, finance, trade, education concessions is lacking. Such was the case with the Central
and culture, health and refugees. (Concurring Opinion, Puno Commission of the Rhine at Strasbourg and the International
J., in Jeffrey Liang [Huefeng] v. People, G.R. No. 125865, Institute of Agriculture at Rome. The third is a combination of
Mar. 26, 2001, 1st Div. [Motion for Reconsideration]) the first two. In this third method, one finds a conventional
obligation to recognize a certain status of an international
447. What are the differences between Diplomatic and organization and its personnel, but the status is described in
International Immunities? Discuss. broad and general terms. The specific definition and application
of those general terms are determined by an accord between the
Held: There are three major differences between organization itself and the state wherein it is located. This is the
diplomatic and international immunities. Firstly, one of the case with the League of Nations, the Permanent Court of
recognized limitations of diplomatic immunity is that members Justice, and the United Nations.
of the diplomatic staff of a mission may be appointed from
among the nationals of the receiving State only with the express The Asian Development Bank and its Personnel fall
consent of that State; apart from inviolability and immunity under this third category.
from jurisdiction in respect of official acts performed in the
exercise of their functions, nationals enjoy only such privileges
167
There is a connection between diplomatic privileges may be judicial or executive, but they are rarely political or
and immunities and those extended to international officials. functions of representation, such as those of the diplomat.
The connection consists in the granting, by contractual
provisions, of the relatively well-established body of diplomatic There is a difference of degree as well as of kind. The
privileges and immunities to international functionaries. This interruption of the activities of a diplomatic agent is likely to
connection is purely historical. Both types of officials find the produce serious harm to the purposes for which his immunities
basis of their special status in the necessity of retaining were granted. But the interruption of the activities of the
functional independence and freedom from interference by the international official does not, usually, cause serious dislocation
state of residence. However, the legal relationship between an of the functions of an international secretariat.
ambassador and the state to which he is accredited is entirely
different from the relationship between the international official On the other hand, they are similar in the sense that
and those states upon whose territory he might carry out its acts performed in an official capacity by either a diplomatic
functions. envoy or an international official are not attributable to him as
an individual but are imputed to the entity he represents, the
The privileges and immunities of diplomats and those state in the case of the diplomat, and the organization in the case
of international officials rest upon different legal foundations. of the international official. (Concurring Opinion, Puno J., in
Whereas those immunities awarded to diplomatic agents are a Jeffrey Liang [Huefeng] v. People, G.R. No. 125865, Mar. 26,
right of the sending state based on customary international law, 2001, 1st Div. [Motion for Reconsideration])
those granted to international officials are based on treaty or
conventional law. Customary international law places no 450. What is the reason behind the current tendency of
obligation on a state to recognize a special status of an reducing privileges and immunities of personnel of
international official or to grant him jurisdictional immunities. international organizations to a minimum?
Such an obligation can only result from specific treaty
provisions. Held: Looking back over 150 years of privileges and
immunities granted to the personnel of international
The special status of the diplomatic envoy is regulated organizations, it is clear that they were accorded a wide scope of
by the principle of reciprocity by which a state is free to treat the protection in the exercise of their functions – The Rhine Treaty
envoy of another state as its envoys are treated by that state. of 1804 between the German Empire and France which
The juridical basis of the diplomat’s position is firmly provided “all the rights of neutrality” to persons employed in
established in customary international law. The diplomatic regulating navigation in the international interest; The Treaty of
envoy is appointed by the sending State but it has to make Berlin of 1878 which granted the European Commission of the
certain that the agreement of the receiving State has been given Danube “complete independence of territorial authorities” in the
for the person it proposes to accredit as head of the mission to exercise of its functions; The Convention of the League which
that State (Article 4, Vienna Convention on Diplomatic granted “diplomatic immunities and privileges.” Today, the age
Relations). of the United Nations finds the scope of protection narrowed.
The current tendency is to reduce privileges and immunities of
The staff personnel of an international organization – personnel of international organizations to a minimum. The
the international officials – assume a different position as tendency cannot be considered as a lowering of the standard but
regards their special status. They are appointed or elected to rather as a recognition that the problem on the privileges and
their position by the organization itself, or by a competent organ immunities of international officials is new. The solution to the
of it; they are responsible to the organization and their official problem presented by the extension of diplomatic prerogatives
acts are imputed to it. The juridical basis of their special to international functionaries lies in the general reduction of the
position is found in conventional law, since there is no special position of both types of agents in that the special status
established basis of usage or custom in the case of the of each agent is granted in the interest of function. The wide
international official. Moreover, the relationship between an grant of diplomatic prerogatives was curtailed because of
international organization and a member-state does not admit of practical necessity and because the proper functioning of the
the principle of reciprocity, for it is contradictory to the basic organization did not require such extensive immunity for its
principle of equality of states. An international organization officials. While the current direction of the law seems to be to
carries out functions in the interest of every member state narrow the prerogatives of the personnel of international
equally. The international official does not carry out his organizations, the reverse is true with respect to the prerogatives
functions in the interest of any state, but in serving the of the organizations themselves, considered as legal entities.
organization he serves, indirectly, each state equally. He cannot Historically, states have been more generous in granting
be, legally, the object of the operation of the principle of privileges and immunities to organizations than they have to the
reciprocity between states under such circumstances. It is personnel of these organizations.
contrary to the principle of equality of states for one state
member of an international organization to assert a capacity to Thus, Section 2 of the General Convention on the
extract special privileges for its nationals from other member Privileges and Immunities of the United Nations states that the
states on the basis of a status awarded by it to an international UN shall enjoy immunity from every form of legal process
organization. It is upon this principle of sovereign equality that except insofar as in any particular case it has expressly waived
international organizations are built. its immunity. Section 4 of the Convention on the Privileges and
Immunities of the Specialized Agencies likewise provides that
It follows from this same legal circumstance that a state the specialized agencies shall enjoy immunity from every form
called upon to admit an official of an international organization of legal process subject to the same exception. Finally, Article
does not have a capacity to declare him persona non grata. 50[1] of the ADB Charter and Section 5 of the Headquarters
Agreement similarly provide that the bank shall enjoy immunity
The functions of the diplomat and those of the from every form of legal process, except in cases arising out of
international official are quite different. Those of the diplomat or in connection with the exercise of its powers to borrow
are functions in the national interest. The task of the money, to guarantee obligations, or to buy and sell or
ambassador is to represent his state, and its specific interest, at underwrite the sale of securities.
the capital of another state. The functions of the international
official are carried out in the international interest. He does not The phrase “immunity from every form of legal
represent a state or the interest of any specific state. He does process” as used in the UN General Convention has been
not usually “represent” the organization in the true sense of that interpreted to mean absolute immunity from a state’s
term. His functions normally are administrative, although they jurisdiction to adjudicate or enforce its law by legal process, and
it is said that states have not sought to restrict that immunity of
168
the United Nations by interpretation or amendment. Similar Liang [Huefeng] v. People, G.R. No. 125865, Mar. 26, 2001,
provisions are contained in the Special Agencies Convention as 1st Div. [Motion for Reconsideration])
well as in the ADB Charter and Headquarters Agreement.
These organizations were accorded privileges and immunities in 452. Who is competent to determine whether a given act
their charters by language similar to that applicable to the of international officials and representatives is
United Nations. It is clear therefore that these organizations private or official?
were intended to have similar privileges and immunities. From
this, it can be easily deduced that international organizations Held: In connection with this question, the current
enjoy absolute immunity similar to the diplomatic prerogatives tendency to narrow the scope of privileges ad immunities of
granted to diplomatic envoys. international officials and representatives is most apparent.
Prior to the regime of the United Nations, the determination of
Even in the United States this seems to be the this question rested with the organization and its decision was
prevailing rule x x x. final. By the new formula, the state itself tends to assume this
competence. If the organization is dissatisfied with the decision,
On the other hand, international officials are governed under the provisions of the General Convention of the United
by a different rule. Section 18[a] of the General Convention on Nations, or the Special Convention for Specialized Agencies,
Privileges and Immunities of the United Nations states that the Swiss Arrangement, and other current dominant instruments,
officials of the United Nations shall be immune from legal it may appeal to an international tribunal by procedures outlined
process in respect of words spoken or written and all acts in these instruments. Thus, the state assumes this competence in
performed by them in their official capacity. The Convention the first instance. It means that, if a local court assumes
on Specialized Agencies carries exactly the same provision. jurisdiction over an act without the necessity of waiver from the
The Charter of the ADB provides under Article 55[i] that organization, the determination of the nature of the act is made
officers and employees of the bank shall be immune from legal at the national level.
process with respect to acts performed by them in their official
capacity except when the Bank waives immunity. Section 45 It appears that the inclination is to place the
[a] of the ADB Headquarters Agreement accords the same competence to determine the nature of an act as private or
immunity to the officers and staff of the bank. There can be no official in the courts of the state concerned. That the practical
dispute that international officials are entitled to immunity only notion seems to be to leave to the local courts determination of
with respect to acts performed in their official capacity, unlike whether or not a given act is official or private does not
international organizations which enjoy absolute immunity. necessarily mean that such determination is final. If the United
Nations questions the decision of the Court, it may invoke
Clearly, the most important immunity to an proceedings for settlement of disputes between the organization
international official, in the discharge of his international and the member states as provided in Section 30 of the General
functions, is immunity from local jurisdiction. There is no Convention. Thus, the decision as to whether a given act is
argument in doctrine or practice with the principle that an official or private is made by the national courts in the first
international official is independent of the jurisdiction of the instance, but it may be subjected to review in the international
local authorities for his official acts. Those acts are not his, but level if questioned by the United Nations.
are imputed to the organization, and without waiver the local
courts cannot hold him liable for them. In strict law, it would xxx
seem that even the organization itself could have no right to
waive an official’s immunity for his official acts. This permits Under the Third Restatement of the Law, it is
local authorities to assume jurisdiction over an individual for an suggested that since an international official does not enjoy
act which is not, in the wider sense of the term, his act al all. It personal inviolability from arrest or detention and has immunity
is the organization itself, as a juristic person, which should only with respect to official acts, he is subject to judicial or
waive its own immunity and appear in court, not the individual, administrative process and must claim his immunity in the
except insofar as he appears in the name of the organization. proceedings by showing that the act in question was an official
Provisions for immunity from jurisdiction for official acts act. Whether an act was performed in the individual’s official
appear, aside from the aforementioned treatises, in the capacity is a question for the court in which a proceeding is
constitution of most modern international organizations. The brought, but if the international organization disputes the court’s
acceptance of the principle is sufficiently widespread to be finding, the dispute between that organization and the state of
regarded as declaratory of international law. (Concurring the forum is to be resolved by negotiation, by an agreed mode of
Opinion, Puno J., in Jeffrey Liang [Huefeng] v. People, G.R. settlement or by advisory opinion of the International Court of
No. 125865, Mar. 26, 2001, 1 st Div. [Motion for Justice.
Reconsideration])
Recognizing the difficulty that by reason of the right of
451. What is the status of the international official with a national court to assume jurisdiction over private acts without
respect to his private acts? a waiver of immunity, the determination of the official or
private character of a particular act may pass from international
Held: Section 18 [a] of the General Convention has to national, Jenks proposes three ways of avoiding difficulty in
been interpreted to mean that officials of the specified categories the matter. The first would be for a municipal court before
are denied immunity from local jurisdiction for acts of their which a question of the official or private character of a
private life and empowers local courts to assume jurisdiction in particular act arose to accept as conclusive in the matter any
such cases without the necessity of waiver. It has earlier been claim by the international organization that the act was official
mentioned that historically, international officials were granted in character, such a claim being regarded as equivalent to a
diplomatic privileges and immunities and were thus considered governmental claim that a particular act is an act of State. Such
immune for both private and official acts. In practice, this wide a claim would be in effect a claim by the organization that the
grant of diplomatic prerogatives was curtailed because of proceedings against the official were a violation of the
practical necessity and because the proper functioning of the jurisdictional immunity of the organization itself which is
organization did not require such exclusive immunity for its unqualified and therefore not subject to delimitation in the
officials. Thus, the current status of the law does not maintain discretion of the municipal court. The second would be for a
that states grant jurisdictional immunity to international court to accept as conclusive in the matter a statement by the
officials for acts of their private lives. This much is explicit executive government of the country where the matter arises
from the charter and Headquarters Agreement of the ADB certifying the official character of the act. The third would be to
which contain substantially similar provisions to that of the have recourse to the procedure of international arbitration.
General Convention. (Concurring Opinion, Puno J., in Jeffrey Jenks opines that it is possible that none of these three solutions
169
would be applicable in all cases; the first might be readily 455. Cite some transactions by a foreign state with
acceptable only in the clearest cases and the second is available private parties that were considered by the Supreme
only if the executive government of the country where the Court as acts “jure imperii” and acts “jure
matter arises concurs in the view of the international gestionis.”
organization concerning the official character of the act.
However, he surmises that taken in combination, these various Held: This Court has considered the following
possibilities may afford the elements of a solution to the transactions by a foreign state with private parties as acts jure
problem. (Concurring Opinion, Puno J., in Jeffrey Liang imperii: (1) the lease by a foreign government of apartment
[Huefeng] v. People, G.R. No. 125865, Mar. 26, 2001, 1 st Div. buildings for use of its military officers; (2) the conduct of
[Motion for Reconsideration]) public bidding for the repair of a wharf at a United States Naval
Station; and (3) the change of employment status of base
453. Discuss the extent of the international official’s employees.
immunity for official acts.
On the other hand, this Court has considered the
Held: The international official’s immunity for official following transactions by a foreign state with private parties as
acts may be likened to a consular official’s immunity from acts jure gestionis: (1) the hiring of a cook in the recreation
arrest, detention, and criminal or civil process which is not center, consisting of three restaurants, a cafeteria, a bakery, a
absolute but applies only to acts or omissions in the store, and a coffee and pastry shop at the John Hay Air Station
performance of his official functions, in the absence of special in Baguio City, to cater to American servicemen and the general
agreement. Since a consular officer is not immune from all public; and (2) the bidding for the operation of barber shops in
legal processes, he must respond to any process and plead and Clark Air Base in Angeles City. The operation of the
prove immunity on the ground that the act or omission restaurants and other facilities open to the general public is
underlying the process was in the performance of his official undoubtedly for profit as a commercial and not a governmental
functions. The issue has not been authoritatively determined, activity. By entering into the employment contract with the
but apparently the burden is on the consular official to prove his cook in the discharge of its proprietary function, the United
status as well as his exemption in the circumstances. In the States government impliedly divested itself of it sovereign
United States, the US Department of State generally has left it to immunity from suit. (Holy See, The v. Rosario, Jr., 238 SCRA
the courts to determine whether a particular act was within a 524, Dec. 1, 1994, En Banc [Quiason])
consular officer’s official duties. (Concurring Opinion, Puno
J., in Jeffrey Liang [Huefeng] v. People, G.R. No. 125865, 456. What should be the guidelines to determine what
Mar. 26, 2001, 1st Div. [Motion for Reconsideration]) activities and transactions shall be considered
“commercial” and as constituting acts “jure
454. Discuss the two conflicting concepts of sovereign gestionis” by a foreign state?
immunity from suit.
Held: In the absence of legislation defining what
Held: There are two conflicting concepts of sovereign activities and transactions shall be considered “commercial” and
immunity, each widely held and firmly established. According as constituting acts jure gestionis, we have to come out with our
to the classical or absolute theory, a sovereign cannot, without own guidelines, tentative they may be.
its consent, be made a respondent in the courts of another
sovereign. According to the newer or restrictive theory, the Certainly, the mere entering into a contract by a foreign
immunity of the sovereign is recognized only with regard to state with a private party cannot be the ultimate test. Such an
public acts or acts jure imperii of a state, but not with regard to act can only be the start of the inquiry. The logical question is
private acts or acts jure gestionis. whether the foreign state is engaged in the activity in the regular
course of business. If the foreign state is not engaged regularly
Some states passed legislation to serve as guidelines in a business or trade, the particular act or transaction must then
for the executive or judicial determination when an act may be be tested by its nature. If the act is in pursuit of a sovereign
considered as jure gestionis. The United States passed the activity, or an incident thereof, then it is an act jure imperii,
Foreign Sovereign Immunities Act of 1976, which defines a especially when it is not undertaken for gain or profit.
commercial activity as “either a regular course of commercial
conduct or a particular commercial transaction or act.” As held in United States of America v. Guinto:
Furthermore, the law declared that the “commercial character of
the activity shall be determined by reference to the nature of the “There is no question that the United States of
course of conduct or particular transaction or act, rather than by America, like any other state, will be deemed to have
reference to its purpose.” The Canadian Parliament enacted in impliedly waived its non-suability if it has entered into
1982 an Act to Provide For State Immunity in Canadian Courts. a contract in its proprietary or private capacity. It is
The Act defines a “commercial activity” as any particular only when the contract involves its sovereign or
transaction, act or conduct or any regular course of conduct that governmental capacity that no such waiver may be
by reason of its nature, is of a “commercial character.” implied.”
(Holy See, The v. Rosario, Jr., 238 SCRA 524, Dec. 1, 1994,
The restrictive theory, which is intended to be a En Banc [Quiason])
solution to the host of problems involving the issue of sovereign
immunity, has created problems of its own. Legal treatises and 457. May the Holy See be sued for selling the land it
the decisions in countries which follow the restrictive theory acquired by donation from the Archdiocese of
have difficulty in characterizing whether a contract of a Manila to be made site of its mission or the
sovereign state with a private party is an act jure gestionis or an Apostolic Nunciature in the Philippines but which
act jure imperii. purpose cannot be accomplished as the land was
occupied by squatters who refused to vacate the
The restrictive theory came about because of the entry area?
of sovereign states into purely commercial activities remotely
connected with the discharge of governmental functions. This is Held: In the case at bench, if petitioner (Holy See) has
particularly true with respect to the Communist states which bought and sold lands in the ordinary course of a real estate
took control of nationalized business activities and international business, surely the said transaction can be categorized as an act
trading. (Holy See, The v. Rosario, Jr., 238 SCRA 524, Dec. 1, jure gestionis. However, petitioner has denied that the
1994, En Banc [Quiason]) acquisition and subsequent disposal of Lot 5-A were made for

170
profit but claimed that it acquired said property for the site of its Court allowed the said Department to file its memorandum in
mission or the Apostolic Nunciature in the Philippines. x x x support of petitioner’s claim of sovereign immunity.

Lot 5-A was acquired by petitioner as a donation from In some cases, the defense of sovereign immunity was
the Archdiocese of Manila. The donation was made not for submitted directly to the local courts by the respondents through
commercial purpose, but for the use of petitioner to construct their private counsels. In cases where the foreign states bypass
thereon the official place of residence of the Papal Nuncio. The the Foreign Office, the courts can inquire into the facts and
right of a foreign sovereign to acquire property, real or personal, make their own determination as to the nature of the acts and
in a receiving state, necessary for the creation and maintenance transactions involved. (Holy See, The v. Rosario, Jr., 238
of its diplomatic mission, is recognized in the 1961 Vienna SCRA 524, Dec. 1, 1994, En Banc [Quiason])
Convention on Diplomatic Relations. This treaty was concurred
in by the Philippine Senate and entered into force in the 459. What is extradition? To whom does it apply?
Philippines on November 15, 1965.
Held: It is the “process by which persons charged with
In Article 31(a) of the Convention, a diplomatic envoy or convicted of crime against the law of a State and found in a
is granted immunity from the civil and administrative foreign State are returned by the latter to the former for trial or
jurisdiction of the receiving state over any real action relating to punishment. It applies to those who are merely charged with an
private immovable property situated in the territory of the offense but have not been brought to trial; to those who have
receiving state which the envoy holds on behalf of the sending been tried and convicted and have subsequently escaped from
state for the purposes of the mission. If this immunity is custody; and those who have been convicted in absentia. It does
provided for a diplomatic envoy, with all the more reason not apply to persons merely suspected of having committed an
should immunity be recognized as regards the sovereign itself, offense but against whom no charge has been laid or to a person
which in this case is the Holy See. whose presence is desired as a witness or for obtaining or
enforcing a civil judgment.” (Weston, Falk, D' Amato,
The decision to transfer the property and the International Law and Order, 2nd ed., p. 630 [1990], cited in
subsequent disposal thereof are likewise clothed with a Dissenting Opinion, Puno, J., in Secretary of Justice v. Hon.
governmental character. Petitioner did not sell Lot 5-A for Ralph C. Lantion, G.R. No. 139465, Jan. 18, 2000, En Banc)
profit or gain. It merely wanted to dispose off the same because
the squatters living thereon made it almost impossible for 460. Discuss the basis for allowing extradition.
petitioner to use it for the purpose of the donation. (Holy See,
The v. Rosario, Jr., 238 SCRA 524, Dec. 1, 1994, En Banc Held: Extradition was first practiced by the Egyptians,
[Quiason]) Chinese, Chaldeans and Assyro-Babylonians but their basis for
allowing extradition was unclear. Sometimes, it was granted
458. How is sovereign or diplomatic immunity pleaded due to pacts; at other times, due to plain good will. The
in a foreign court? classical commentators on international law thus focused their
early views on the nature of the duty to surrender an extraditee -
Held: In Public International Law, when a state or -- whether the duty is legal or moral in character. Grotius and
international agency wishes to plead sovereign or diplomatic Vattel led the school of thought that international law imposed a
immunity in a foreign court, it requests the Foreign Office of the legal duty called civitas maxima to extradite criminals. In sharp
state where it is sued to convey to the court that said defendant contrast, Puffendorf and Billot led the school of thought that the
is entitled to immunity. so-called duty was but an "imperfect obligation which could
become enforceable only by a contract or agreement between
In the United States, the procedure followed is the states.
process of “suggestion,” where the foreign state or the
international organization sued in an American court requests Modern nations tilted towards the view of Puffendorf
the Secretary of State to make a determination as to whether it is and Billot that under international law there is no duty to
entitled to immunity. If the Secretary of State finds that the extradite in the absence of treaty, whether bilateral or
defendant is immune from suit, he, in turn, asks the Attorney multilateral. Thus, the US Supreme Court in US v. Rauscher,
General to submit to the court a “suggestion” that the defendant held: “x x x it is only in modern times that the nations of the
is entitled to immunity. In England, a similar procedure is earth have imposed upon themselves the obligation of delivering
followed, only the Foreign Office issues a certification to that up these fugitives from justice to the states where their crimes
effect instead of submitting a “suggestion”. were committed, for trial and punishment. This has been done
generally by treaties x x x. Prior to these treaties, and apart
In the Philippines, the practice is for the foreign from them there was no well-defined obligation on one country
government or the international organization to first secure an to deliver up such fugitives to another; and though such delivery
executive endorsement of its claim of sovereign or diplomatic was often made it was upon the principle of comity x x x.”
immunity. But how the Philippine Foreign Office conveys its (Dissenting Opinion, Puno, J., in Secretary of Justice v. Hon.
endorsement to the courts varies. In International Catholic Ralph C. Lantion, G.R. No. 139465, Jan. 18, 2000, En Banc)
Migration Commission v. Calleja, the Secretary of Foreign
Affairs just sent a letter directly to the Secretary of Labor and 461. What is the nature of an extradition proceeding?
Employment, informing the latter that the respondent-employer Is it akin to a criminal proceeding?
could not be sued because it enjoyed diplomatic immunity. In
World Health Organization v. Aquino, the Secretary of Foreign Held: [A]n extradition proceeding is sui generis. It is
Affairs sent the trial court a telegram to that effect. In Baer v. not a criminal proceeding which will call into operation all the
Tizon, the U.S. Embassy asked the Secretary of Foreign Affairs rights of an accused as guaranteed by the Bill of Rights. To
to request the Solicitor General to make, in behalf of the begin with, the process of extradition does not involve the
commander of the United States Naval Base at Olongapo City, determination of the guilt or innocence of an accused. His guilt
Zambales, a “suggestion” to respondent Judge. The Solicitor or innocence will be adjudged in the court of the state where he
General embodied the “suggestion” in a Manifestation and will be extradited. Hence, as a rule, constitutional rights that are
Memorandum as amicus curiae. only relevant to determine the guilt or innocence of an accused
cannot be invoked by an extraditee especially by one whose
In the case at bench, the Department of Foreign extradition papers are still undergoing evaluation. As held by
Affairs, through the Office of Legal Affairs moved with this the US Supreme Court in United States v. Galanis:
Court to be allowed to intervene on the side of petitioner. The

171
“An extradition proceeding is not a criminal It ought to follow that the RP-US Extradition Treaty
prosecution, and the constitutional safeguards that calls for an interpretation that will minimize if not prevent the
accompany a criminal trial in this country do not shield escape of extraditees from the long arm of the law and expedite
an accused from extradition pursuant to a valid treaty.” their trial. x x x
(Wiehl, Extradition Law at the Crossroads: The Trend
Toward Extending Greater Constitutional Procedural [A]n equally compelling factor to consider is the
Protections To Fugitives Fighting Extradition from the understanding of the parties themselves to the RP-US
United States, 19 Michigan Journal of International Extradition Treaty as well as the general interpretation of the
Law 729, 741 [1998], citing United States v. Galanis, issue in question by other countries with similar treaties with
429 F. Supp. 1215 [D. Conn. 1977]) the Philippines. The rule is recognized that while courts have
the power to interpret treaties, the meaning given them by the
There are other differences between an extradition proceeding departments of government particularly charged with their
and a criminal proceeding. An extradition proceeding is negotiation and enforcement is accorded great weight. The
summary in nature while criminal proceedings involve a full- reason for the rule is laid down in Santos III v. Northwest Orient
blown trial. In contradistinction to a criminal proceeding, the Airlines, et al., where we stressed that a treaty is a joint
rules of evidence in an extradition proceeding allow admission executive-legislative act which enjoys the presumption that “it
of evidence under less stringent standards. In terms of the was first carefully studied and determined to be constitutional
quantum of evidence to be satisfied, a criminal case requires before it was adopted and given the force of law in the country.”
proof beyond reasonable doubt for conviction while a fugitive (Secretary of Justice v. Hon. Ralph C. Lantion, G.R. No.
may be ordered extradited “upon showing of the existence of a 139465, Oct. 17, 2000, En Banc [Puno])
prima facie case.” Finally, unlike in a criminal case where
judgment becomes executory upon being rendered final, in an 464. Discuss the Five Postulates of Extradition.
extradition proceeding, our courts may adjudge an individual
extraditable but the President has the final discretion to extradite Held:
him. The United States adheres to a similar practice whereby
the Secretary of State exercises wide discretion in balancing the 1. Extradition Is a Major Instrument for the
equities of the case and the demands of the nation's foreign Suppression of Crime.
relations before making the ultimate decision to extradite.
First, extradition treaties are entered into for the
As an extradition proceeding is not criminal in purpose of suppressing crime by facilitating the arrest and
character and the evaluation stage in an extradition proceeding custodial transfer of a fugitive from one state to the other.
is not akin to a preliminary investigation, the due process
safeguards in the latter do not necessarily apply to the former. With the advent of easier and faster means of
This we hold for the procedural due process required by a given international travel, the flight of affluent criminals from one
set of circumstances “must begin with a determination of the country to another for the purpose of committing crime and
precise nature of the government function involved as well as evading prosecution has become more frequent. Accordingly,
the private interest that has been affected by governmental governments are adjusting their methods of dealing with
action.” The concept of due process is flexible for “not all criminals and crimes that transcend international boundaries.
situations calling for procedural safeguards call for the same
kind of procedure.” (Secretary of Justice v. Hon. Ralph C. Today, “a majority of nations in the world community
Lantion, G.R. No. 139465, Oct. 17, 2000, En Banc [Puno]) have come to look upon extradition as the major effective
instrument of international co-operation in the suppression of
462. Will the retroactive application of an extradition crime.” It is the only regular system that has been devised to
treaty violate the constitutional prohibition against return fugitives to the jurisdiction of a court competent to try
"ex post facto" laws? them in accordance with municipal and international law.

Held: The prohibition against ex post facto law applies Xxx


only to criminal legislation which affects the substantial rights
of the accused. This being so, there is no merit in the contention Indeed, in this era of globalization, easier and faster
that the ruling sustaining an extradition treaty’s retroactive international travel, and an expanding ring of international
application violates the constitutional prohibition against ex post crimes and criminals, we cannot afford to be an isolationist
facto laws. The treaty is neither a piece of criminal legislation state. We need to cooperate with other states in order to
nor a criminal procedural statute. (Wright v. CA, 235 SCRA improve our chances of suppressing crime in our country.
341, Aug. 15, 1994 [Kapunan])
2. The Requesting State Will Accord Due Process to
463. Discuss the rules in the interpretation of the Accused.
extradition treaties.
Second, an extradition treaty presupposes that both
Held: [A]ll treaties, including the RP-US Extradition parties thereto have examined, and that both accept and trust,
Treaty, should be interpreted in light of their intent. Nothing each other’s legal system and judicial process. More pointedly,
less than the Vienna Convention on the Law of Treaties to which our duly authorized representative’s signature on an extradition
the Philippines is a signatory provides that “a treaty shall be treaty signifies our confidence in the capacity and willingness of
interpreted in good faith in accordance with the ordinary the other state to protect the basic rights of the person sought to
meaning to be given to the terms of the treaty in their context be extradited. That signature signifies our full faith that the
and in light of its object and purpose.” x x x. It cannot be accused will be given, upon extradition to the requesting state,
gainsaid that today, countries like the Philippines forge all relevant and basic rights in the criminal proceedings that will
extradition treaties to arrest the dramatic rise of international take place therein; otherwise, the treaty would not have been
and transnational crimes like terrorism and drug trafficking. signed, or would have been directly attacked for its
Extradition treaties provide the assurance that the punishment of unconstitutionality.
these crimes will not be frustrated by the frontiers of territorial
sovereignty. Implicit in the treaties should be the unbending 3. The Proceedings Are Sui Generis.
commitment that the perpetrators of these crimes will not be
coddled by any signatory state. Third, as pointed out in Secretary of Justice v. Lantion,
extradition proceedings are not criminal in nature. In criminal
proceedings, the constitutional rights of the accused are at fore;
172
in extradition which is sui generis – in a class by itself – they partner; as well as in the ability and the willingness of the latter
are not. to grant basic rights to the accused in the pending criminal case
therein.
Xxx
3. By nature then, extradition proceedings are not
Given the foregoing, it is evident that the extradition equivalent to a criminal case in which guilt or innocence is
court is not called upon to ascertain the guilt or the innocence of determined. Consequently, an extradition case is not one in
the person sought to be extradited. Such determination during which the constitutional rights of the accused are necessarily
the extradition proceedings will only result in needless available. It is more akin, if at all, to a court’s request to police
duplication and delay. Extradition is merely a measure of authorities for the arrest of the accused who is at large or has
international judicial assistance through which a person charged escaped detention or jumped bail. Having once escaped the
with or convicted of a crime is restored to a jurisdiction with the jurisdiction of the requesting state, the reasonable prima facie
best claim to try that person. It is not part of the function of the presumption is that the person would escape again if given the
assisting authorities to enter into questions that are the opportunity.
prerogative of that jurisdiction. The ultimate purpose of
extradition proceedings in court is only to determine whether 4. Immediately upon receipt of the petition for
the extradition request complies with the Extradition Treaty, extradition and its supporting documents, the judge shall make a
and whether the person sought is extraditable. prima facie finding whether the petition is sufficient in form and
substance, whether it complies with the Extradition Treaty and
Compliance Shall Be in Good Faith. Law, and whether the person sought is extraditable. The
magistrate has discretion to require the petitioner to submit
Fourth, our executive branch of government further documentation, or to personally examine the affiants or
voluntarily entered into the Extradition Treaty, and our witnesses. If convinced that a prima facie case exists, the judge
legislative branch ratified it. Hence, the Treaty carries the immediately issues a warrant for the arrest of the potential
presumption that its implementation will serve the national extraditee and summons him or her to answer and to appear at
interest. scheduled hearings on the petition.

Fulfilling our obligations under the Extradition Treaty 5. After being taken into custody, potential extraditees
promotes comity (In line with the Philippine policy of may apply for bail. Since the applicants have a history of
cooperation and amity with all nations set forth in Article II, absconding, they have the burden of showing that (a) there is no
Section 2, Constitution). On the other hand, failure to fulfill our flight risk and no danger to the community; and (b) there exist
obligations thereunder paints at bad image of our country before special, humanitarian or compelling circumstances. The
the world community. Such failure would discourage other grounds used by the highest court in the requesting state for the
states from entering into treaties with us, particularly an grant of bail therein may be considered, under the principle of
extradition treaty that hinges on reciprocity. reciprocity as a special circumstance. In extradition cases, bail
is not a matter of right; it is subject to judicial discretion in the
Verily, we are bound by pacta sunt servanda to comply context of the peculiar facts of each case.
in good faith with our obligations under the Treaty. This
principle requires that we deliver the accused to the requesting 6. Potential extraditees are entitled to the rights to due
country if the conditions precedent to extradition, as set forth in process and to fundamental fairness. Due process does not
the Treaty, are satisfied. In other words, “[t]he demanding always call for a prior opportunity to be heard. A subsequent
government, when it has done all that the treaty and the law opportunity is sufficient due to the flight risk involved. Indeed,
require it to do, is entitled to the delivery of the accused on the available during the hearings on the petition and the answer is
issue of the proper warrant, and the other government is under the full chance to be heard and to enjoy fundamental fairness
obligation to make the surrender.” Accordingly, the Philippines that is compatible with the summary nature of extradition.
must be ready and in a position to deliver the accused, should it
be found proper. 7. This Court will always remain a protector of human
rights, a bastion of liberty, a bulwark of democracy and the
There Is an Underlying Risk of Flight. conscience of society. But it is also well aware of the
limitations of its authority and of the need for respect for the
Fifth, persons to be extradited are presumed to be flight prerogatives of the other co-equal and co-independent organs of
risks. This prima facie presumption finds reinforcement in the government.
experience of the executive branch: nothing short of
confinement can ensure that the accused will not flee the 8. We realize that extradition is essentially an
jurisdiction of the requested state in order to thwart their executive, not a judicial, responsibility arising out of the
extradition to the requesting state. (Government of the United presidential power to conduct foreign relations and to
States of America v. Hon. Guillermo Purganan, G.R. No. implement treaties. Thus, the Executive Department of
148571, Sept. 24, 2002, En Banc [Panganiban]) government has broad discretion in its duty and power of
implementation.
465. Discuss the Ten Points to consider in Extradition
Proceedings? 9. On the other hand, courts merely perform oversight
functions and exercise review authority to prevent or excise
Held: 1. The ultimate purpose of extradition grave abuse and tyranny. They should not allow contortions,
proceedings is to determine whether the request expressed in the delays and “over-due process” every little step of the way, lest
petition, supported by its annexes and the evidence that may be these summary extradition proceedings become not only inutile
adduced during the hearing of the petition, complies with the but also sources of international embarrassment due to our
Extradition Treaty and Law; and whether the person sought is inability to comply in good faith with a treaty partner’s simple
extraditable. The proceedings are intended merely to assist the request to return a fugitive. Worse, our country should not be
requesting state in bringing the accused – or the fugitive who converted into a dubious haven where fugitives and escapees
has illegally escaped – back to its territory, so that the criminal can unreasonably delay, mummify, mock, frustrate, checkmate
process may proceed therein. and defeat the quest for bilateral justice and international
cooperation.
2. By entering into an extradition treaty, the
Philippines is deemed to have reposed its trust in the reliability 10. At bottom, extradition proceedings should be
or soundness of the legal and judicial system of its treaty conducted with all deliberate speed to determine compliance
173
with the Extradition Treaty and Law; and, while safeguarding recommendations and other acts agreed upon and signed by the
basic individual rights, to avoid the legalistic contortions, delays plenipotentiaries attending the conference. It is not the treaty
and technicalities that may negate that purpose. (Government itself. It is rather a summary of the proceedings of a protracted
of the United States of America v. Hon. Guillermo Purganan, conference which may have taken place over several years. It
G.R. No. 148571, Sept. 24, 2002, En Banc [Panganiban]) will not require the concurrence of the Senate. The documents
contained therein are deemed adopted without need for
466. What is a Treaty? Discuss. ratification. (Tanada v. Angara, 272 SCRA 18, May 2, 1997
[Panganiban])
Held: A treaty, as defined by the Vienna Convention
on the Law of Treaties, is “an international instrument 470. What is the “most-favored-nation” clause? What
concluded between States in written form and governed by is its purpose?
international law, whether embodied in a single instrument or in
two or more related instruments, and whatever its particular Answer: 1. The most-favored-nation clause may be
designation.” There are many other terms used for a treaty or defined, in general, as a pledge by a contracting party to a treaty
international agreement, some of which are: act, protocol, to grant to the other party treatment not less favorable than that
agreement, compromis d' arbitrage, concordat, convention, which has been or may be granted to the “most favored” among
declaration, exchange of notes, pact, statute, charter and modus other countries. The clause has been commonly included in
vivendi. All writers, from Hugo Grotius onward, have pointed treaties of commercial nature. (Salonga & Yap, Public
out that the names or titles of international agreements included International Law, 5th Edition, 1992, pp. 141-142)
under the general term treaty have little or no significance.
Certain terms are useful, but they furnish little more than mere 2. The purpose of a most favored nation clause is to
description grant to the contracting party treatment not less favorable than
that which has been or may be granted to the "most favored"
Article 2[2] of the Vienna Convention provides that among other countries. The most favored nation clause is
“the provisions of paragraph 1 regarding the use of terms in the intended to establish the principle of equality of international
present Convention are without prejudice to the use of those treatment by providing that the citizens or subjects of the
terms, or to the meanings which may be given to them in the contracting nations may enjoy the privileges accorded by either
internal law of the State.” (BAYAN [Bagong Alyansang party to those of the most favored nation (Commissioner of
Makabayan] v. Executive Secretary Ronaldo Zamora, G.R. Internal Revenue v. S.C. Johnson and Son, Inc., 309 SCRA
No. 138570, Oct. 10, 2000, En Banc [Buena]) 87, 107-108, June 25, 1999, 3rd Div. [Gonzaga-Reyes])

467. Discuss the binding effect of treaties and executive 471. What are the two types of most-favored nation
agreements in international law. clause?

Held: [I]n international law, there is no difference Held: There are generally two types of most-favored-
between treaties and executive agreements in their binding nation clause, namely, conditional and unconditional.
effect upon states concerned, as long as the functionaries have According to the clause in its unconditional form, any advantage
remained within their powers. International law continues to of whatever kind which has been or may in future be granted by
make no distinction between treaties and executive agreements: either of the contracting parties to a third State shall
they are equally binding obligations upon nations. (BAYAN simultaneously and unconditionally be extended to the other
[Bagong Alyansang Makabayan] v. Executive Secretary under the same or equivalent conditions as those under which it
Ronaldo Zamora, G.R. No. 138570, Oct. 10, 2000, En Banc has been granted to the third State. (Salonga & Yap, Public
[Buena]) International Law, 5th Edition, 1992, pp. 141-142)

468. Do the Philippines recognize the binding effect of 472. Discuss the essence of the principle behind the
executive agreements even without the concurrence "most-favored-nation" clause as applied to tax
of the Senate or Congress? treaties?

Held: In our jurisdiction, we have recognized the Held: The essence of the principle is to allow the
binding effect of executive agreements even without the taxpayer in one state to avail of more liberal provisions granted
concurrence of the Senate or Congress. In Commissioner of in another tax treaty to which the country of residence of such
Customs v. Eastern Sea Trading, we had occasion to pronounce: taxpayer is also a party provided that the subject matter of
taxation x x x is the same as that in the tax treaty under which
“x x x the right of the Executive to enter into the taxpayer is liable.
binding agreements without the necessity of subsequent
Congressional approval has been confirmed by long In Commissioner of Internal Revenue v. S.C. Johnson
usage. From the earliest days of our history we have and Son, Inc., 309 SCRA 87, June 25, 1999, the SC did not
entered into executive agreements covering such grant the claim filed by S.C. Johnson and Son, Inc., a non-
subjects as commercial and consular relations, most- resident foreign corporation based in the USA, with the BIR for
favored-nation rights, patent rights, trademark and refund of overpaid withholding tax on royalties pursuant to the
copyright protection, postal and navigation most-favored-nation clause of the RP-US Tax Treaty in relation
arrangements and the settlement of claims. The to the RP-West Germany Tax Treaty. It held:
validity of these has never been seriously questioned by
our courts." Given the purpose underlying tax treaties and
(BAYAN [Bagong Alyansang Makabayan] v. Executive the rationale for the most favored nation clause, the
Secretary Ronaldo Zamora, G.R. No. 138570, Oct. 10, 2000, concessional tax rate of 10 percent provided for in the
En Banc [Buena]) RP-Germany Tax Treaty should apply only if the taxes
imposed upon royalties in the RP-US Tax Treaty and in
469. What is a "protocol de cloture"? Will it require the RP-Germany Tax Treaty are paid under similar
concurrence by the Senate? circumstances. This would mean that private
respondent (S.C. Johnson and Son, Inc.) must prove
Held: A final act, sometimes called protocol de that the RP-US Tax Treaty grants similar tax reliefs to
cloture, is an instrument which records the winding up of the residents of the United States in respect of the taxes
proceedings of a diplomatic conference and usually includes a imposable upon royalties earned from sources within
reproduction of the texts of treaties, conventions,
174
the Philippines as those allowed to their German readily plead the Constitution as a convenient excuse for non-
counterparts under the RP-Germany Tax Treaty. compliance with our obligations, duties and responsibilities
under international law.
The RP-US and the RP-West Germany Tax
Treaties do not contain similar provisions on tax Beyond this, Article 13 of the Declaration of Rights
crediting. Article 24 of the RP-Germany Tax Treaty x and Duties of States adopted by the International Law
x x expressly allows crediting against German income Commission in 1949 provides: Every State has the duty to carry
and corporation tax of 20% of the gross amount of out in good faith its obligations arising from treaties and other
royalties paid under the law of the Philippines. On the sources of international law, and it may not invoke provisions in
other hand, Article 23 of the RP-US Tax Treaty, which its constitution or its laws as an excuse for failure to perform
is the counterpart provision with respect to relief for this duty.
double taxation, does not provide for similar crediting
of 20% of the gross amount of royalties paid. X x x Equally important is Article 26 of the Convention
which provides that “Every treaty in force is binding upon the
X x x The entitlement of the 10% rate by U.S. parties to it and must be performed by them in good faith.” This
firms despite the absence of matching credit (20% for is known as the principle of pacta sunt servanda which
royalties) would derogate from the design behind the preserves the sanctity of treaties and have been one of the most
most favored nation clause to grant equality of fundamental principles of positive international law, supported
international treatment since the tax burden laid upon by the jurisprudence of international tribunals. (BAYAN
the income of the investor is not the same in the two [Bagong Alyansang Makabayan] v. Executive Secretary
countries. The similarity in the circumstances of Ronaldo Zamora, G.R. No. 138570, Oct. 10, 2000, 342 SCRA
payment of taxes is a condition for the enjoyment of 449, 492-493, En Banc [Buena])
most favored nation treatment precisely to underscore
the need for equality of treatment. 476. Explain the “pacta sunt servanda” rule.

473. Discuss the nature of ratification in the treaty- Held: One of the oldest and most fundamental rules in
making process? international law is pacta sunt servanda – international
agreements must be performed in good faith. “A treaty
Held: Ratification is generally held to be an executive engagement is not a mere moral obligation but creates a legally
act, undertaken by the head of state or of the government, as the binding obligation on the parties x x x. A state which has
case may be, through which the formal acceptance of the treaty contracted valid international obligations is bound to make in its
is proclaimed. A State may provide in its domestic legislation legislations such modifications as may be necessary to ensure
the process of ratification of a treaty. (BAYAN [Bagong the fulfillment of the obligations undertaken.” (Tanada v.
Alyansang Makabayan] v. Executive Secretary Ronaldo Angara, 272 SCRA 18, May 2, 1997 [Panganiban])
Zamora, G.R. No. 138570, Oct. 10, 2000, En Banc [Buena])
477. Explain the "rebus sic stantibus" rule (i.e., things
474. How is the consent of the State to be bound by a remaining as they are).
treaty by ratification expressed?
Held: According to Jessup, the doctrine constitutes an
Held: The consent of the State to be bound by a treaty attempt to formulate a legal principle which would justify non-
is expressed by ratification when: (a) the treaty provides for performance of a treaty obligation if the conditions with rela
such ratification, (b) it is otherwise established that the tion to which the parties contracted have changed so materially
negotiating States agreed that ratification should be required, (c) and so unexpectedly as to create a situation in which the
the representative of the State has signed the treaty subject to exaction of performance would be unreasonable. The key
ratification, or (d) the intention of the State to sign the treaty element of this doctrine is the vital change in the condition of
subject to ratification appears from the full powers of its the contracting parties that they could not have foreseen at the
representative, or was expressed during the negotiation. time the treaty was concluded. (Santos III v. Northwest Orient
(BAYAN [Bagong Alyansang Makabayan] v. Executive Airlines, 210 SCRA 256, June 23, 1992)
Secretary Ronaldo Zamora, G.R. No. 138570, Oct. 10, 2000,
En Banc [Buena]) 478. Does the “rebus sic stantibus” rule operate
automatically to render a treaty inoperative?
475. Discuss the effect of the ratification of the Visiting
Forces Agreement (VFA). Held: The doctrine of rebus sic stantibus does not
operate automatically to render the treaty inoperative. There is a
Held: With the ratification of the VFA, which is necessity for a formal act of rejection, usually made by the head
equivalent to final acceptance, and with the exchange of notes of state, with a statement of the reasons why compliance with
between the Philippines and the United States of America, it the treaty is no longer required. (Santos III v. Northwest Orient
now becomes obligatory and incumbent on our part, under the Airlines, 210 SCRA 256, June 23, 1992)
principles of international law, to be bound by the terms of the
agreement. Thus, no less than Section 2, Article II of the 479. What is the “Doctrine of Effective Nationality”
Constitution, declares that the Philippines adopts the generally (Genuine Link Doctrine)?
accepted principles of international law as part of the law of the
land and adheres to the policy of peace, equality, justice, Held: This principle is expressed in Article 5 of the
freedom, cooperation and amity with all nations. Hague Convention of 1930 on the Conflict of Nationality Laws
as follows:
As a member of the family of nations, the Philippines
agrees to be bound by generally accepted rules for the conduct Art. 5. Within a third State a person having
of its international relations. While the international obligation more than one nationality shall be treated as if he had
devolves upon the state and not upon any particular branch, only one. Without prejudice to the application of its
institution, or individual member of its government, the law in matters of personal status and of any convention
Philippines is nonetheless responsible for violations committed in force, a third State shall, of the nationalities which
by any branch or subdivision of its government or any official any such person possesses, recognize exclusively in its
thereof. As an integral part of the community of nations, we are territory either the nationality of the country in which
responsible to assure that our government, Constitution and laws he is habitually and principally resident or the
will carry out our international obligation. Hence, we cannot nationality of the country with which in the
175
circumstances he appears to be in fact most closely
connected. (Frivaldo v. COMELEC, 174 SCRA 245,
June 23, 1989)

176

You might also like